You are on page 1of 562

Graduate Coursework

WES C. ERBSEN
September 2010 - December 2011
This document last updated on January 27, 2013
Contents
Contents i
1 Electrodynamics II 1
1.1 Homework #1 . . . . . . . . . . . . . . . . . . . . . . . . . . . . . . . . . . . . . . . . . . . . . . . . 1
1.2 Homework #2 . . . . . . . . . . . . . . . . . . . . . . . . . . . . . . . . . . . . . . . . . . . . . . . . 14
1.3 Homework #3 . . . . . . . . . . . . . . . . . . . . . . . . . . . . . . . . . . . . . . . . . . . . . . . . 25
1.4 Homework #4 . . . . . . . . . . . . . . . . . . . . . . . . . . . . . . . . . . . . . . . . . . . . . . . . 34
1.5 Homework #5 . . . . . . . . . . . . . . . . . . . . . . . . . . . . . . . . . . . . . . . . . . . . . . . . 40
1.6 Homework #6 . . . . . . . . . . . . . . . . . . . . . . . . . . . . . . . . . . . . . . . . . . . . . . . . 46
1.7 Homework #7 . . . . . . . . . . . . . . . . . . . . . . . . . . . . . . . . . . . . . . . . . . . . . . . . 54
1.8 Homework #8 . . . . . . . . . . . . . . . . . . . . . . . . . . . . . . . . . . . . . . . . . . . . . . . . 63
1.9 Homework #9 . . . . . . . . . . . . . . . . . . . . . . . . . . . . . . . . . . . . . . . . . . . . . . . . 78
2 Quantum Mechanics II 89
2.1 Homework #1 . . . . . . . . . . . . . . . . . . . . . . . . . . . . . . . . . . . . . . . . . . . . . . . . 89
2.2 Homework #2 . . . . . . . . . . . . . . . . . . . . . . . . . . . . . . . . . . . . . . . . . . . . . . . . 101
2.3 Homework #3 . . . . . . . . . . . . . . . . . . . . . . . . . . . . . . . . . . . . . . . . . . . . . . . . 124
2.4 Homework #4 . . . . . . . . . . . . . . . . . . . . . . . . . . . . . . . . . . . . . . . . . . . . . . . . 144
2.5 Homework #5 . . . . . . . . . . . . . . . . . . . . . . . . . . . . . . . . . . . . . . . . . . . . . . . . 161
2.6 Homework #6 . . . . . . . . . . . . . . . . . . . . . . . . . . . . . . . . . . . . . . . . . . . . . . . . 174
2.7 Homework #7 . . . . . . . . . . . . . . . . . . . . . . . . . . . . . . . . . . . . . . . . . . . . . . . . 187
2.8 Homework #9 . . . . . . . . . . . . . . . . . . . . . . . . . . . . . . . . . . . . . . . . . . . . . . . . 198
2.9 Homework #10 . . . . . . . . . . . . . . . . . . . . . . . . . . . . . . . . . . . . . . . . . . . . . . . 213
Appendix A * . . . . . . . . . . . . . . . . . . . . . . . . . . . . . . . . . . . . . . . . . . . . . . . . . . . . . . . 222
Appendix B * . . . . . . . . . . . . . . . . . . . . . . . . . . . . . . . . . . . . . . . . . . . . . . . . . . . . . . . 225
3 Statistical Mechanics 227
3.1 Homework #1 . . . . . . . . . . . . . . . . . . . . . . . . . . . . . . . . . . . . . . . . . . . . . . . . 227
3.2 Homework #2 . . . . . . . . . . . . . . . . . . . . . . . . . . . . . . . . . . . . . . . . . . . . . . . . 235
3.3 Homework #3 . . . . . . . . . . . . . . . . . . . . . . . . . . . . . . . . . . . . . . . . . . . . . . . . 247
3.4 Homework #4 . . . . . . . . . . . . . . . . . . . . . . . . . . . . . . . . . . . . . . . . . . . . . . . . 262
3.5 Homework #7 . . . . . . . . . . . . . . . . . . . . . . . . . . . . . . . . . . . . . . . . . . . . . . . . 275
4 Mathematical Methods 283
4.1 Homework #3 . . . . . . . . . . . . . . . . . . . . . . . . . . . . . . . . . . . . . . . . . . . . . . . . 283
4.2 Homework #4 . . . . . . . . . . . . . . . . . . . . . . . . . . . . . . . . . . . . . . . . . . . . . . . . 288
4.3 Homework #5 . . . . . . . . . . . . . . . . . . . . . . . . . . . . . . . . . . . . . . . . . . . . . . . . 294
4.4 Homework #6 . . . . . . . . . . . . . . . . . . . . . . . . . . . . . . . . . . . . . . . . . . . . . . . . 303
W. Erbsen CONTENTS
4.5 Homework #7 . . . . . . . . . . . . . . . . . . . . . . . . . . . . . . . . . . . . . . . . . . . . . . . . 310
4.6 Homework #8 . . . . . . . . . . . . . . . . . . . . . . . . . . . . . . . . . . . . . . . . . . . . . . . . 315
4.7 Homework #9 . . . . . . . . . . . . . . . . . . . . . . . . . . . . . . . . . . . . . . . . . . . . . . . . 324
4.8 Homework #10 . . . . . . . . . . . . . . . . . . . . . . . . . . . . . . . . . . . . . . . . . . . . . . . 332
4.9 Homework #11 . . . . . . . . . . . . . . . . . . . . . . . . . . . . . . . . . . . . . . . . . . . . . . . 342
4.10 Homework #12 . . . . . . . . . . . . . . . . . . . . . . . . . . . . . . . . . . . . . . . . . . . . . . . 355
5 Departmental Examinations 367
5.1 Quantum Mechanics . . . . . . . . . . . . . . . . . . . . . . . . . . . . . . . . . . . . . . . . . . . . 367
5.2 Electrodynamics . . . . . . . . . . . . . . . . . . . . . . . . . . . . . . . . . . . . . . . . . . . . . . 410
5.3 Modern Physics . . . . . . . . . . . . . . . . . . . . . . . . . . . . . . . . . . . . . . . . . . . . . . . 449
5.4 Statistical Mechanics . . . . . . . . . . . . . . . . . . . . . . . . . . . . . . . . . . . . . . . . . . . . 497
Chapter 1
Electrodynamics II
1.1 Homework #1
Problem 9.1
A copper ring of radius a is xed at a distance d (with a d) directly above an identical copper ring. Each ring
has a resistance R for circulating currents. An increasing current I = I
o
t
/

is applied in the lower ring. Neglect


the self-inductance of each ring, and make appropriate approximations.
a) Find the dipole moment in the lower ring.
b) Find the magnetic ux through the upper ring.
c) Find the induced EMF and the current in the upper ring.
d) Find the induced dipole moment of the upper ring.
e) Show that the force between the rings is

F =
12
4
a
8
I
2
o
t
c
3
Rd
7

2
(1.1.1)
Is the force repulsive or attractive?
Solution
a) An expression for the magnetic dipole moment for a circular loop can be realized by applying
multipole expansion of the magnetic scalar potential (
m
) and noting that the coecients of
P

(cos ) /r
+1
are in fact the magnetic dipole moments (from Franklin p. 209). ?

=
2Ia
+1
c
_

1
/
2
+1
/
2
_

1
=
Ia
2
c
(1.1.2)

Errata indicates that c


3
c
4
.
W. Erbsen HOMEWORK #1
Where
=1
represents the magnetic moment of a dipole. The elementary magnetic dipole is
found by taking the limit of (1.1.1) as a 0 keeping Ia
2
xed. We subsequently arrive at
=
a
2
I
o
t
c
z (1.1.3)
b) The magnetic ux is of course given by

B
=
_
s
B dA
While the magnetic eld of a magnetic dipole is given by (7.108) on p. 209 as
B =
_
r
r
3
_
=
3 ( r) r
r
3
(1.1.4)
The closed surface we choose to integrate over is the upper hemisphere closed on the bottom by a
disk, as inspired by Griths in Ex. 8.2 on p. 353. ?
The general idea is that since the source of the eld lies outside the surface, the total ux
through the volume is zero:
_
v
B dA = 0. We then note that the ux entering through the disk
is the same as that exiting through the top of the hemisphere:
disk
=
hemisphere
. It i easiest to
calculate the ux through the hemisphere.
We also recognize that since a d, we can use the following approximation: tan a/d
sin = a/d cos a/d, as per the small angle approximation. We can then proceed to nd the ux
through the hemisphere:

B
=
_

0
B 2r
2
sin d
=2
_

0
_
3 ( r) r
r
3
_
r
2
sin d z
=
2
r
_

0
_
Ia
2
c
z
_
sin d z
=
2
2
a
2
I
cr
_

0
sin d

2
2
a
2
I
cr
_

0
_
a
d
_
d (
a
/
d
)

2
2
a
2
I
cr
a
2
d
2
(1.1.5)
Since d a, we can approximate r =

d
2
+ a
2
d
2
, and if we recall that I = I
o
t
/

, (1.1.5)
becomes

B

2
2
a
4
I
o
t
cd
3

(1.1.6)
c) The induced EMF in the upper ring is given by
E =
1
c
d
B
dt
(1.1.7)
CHAPTER 1: ELECTRODYNAMICS II 3
Substituting (1.1.6) into (1.1.7), we have
E
1
c
d
dt
_
2
2
a
4
I
o
t
cd
3

_
E
2
2
a
4
I
o
c
2
d
3

(1.1.8)
And the current in the upper ring (I

) can be found from Ohms Law,


I


E
R
I


2
2
a
4
I
o
c
2
d
3
R
(1.1.9)
d) To nd the induced dipole moment in the upper ring (

), we proceed in the same spirit that led


us to (1.1.3), but this time we use I

from (1.1.9):

2
2
a
4
I
o
c
2
d
3
R
_
a
2
c
z


2
3
a
6
I
o
c
3
d
3
R
z (1.1.10)
e) The force on a dipole in a magnetic eld B is given by Franklin as (7.144) on p. 216:
F = (B ) (1.1.11)
Assuming that the charges are xed and that the force of the lower loop on the upper loop is the
same as that of the upper loop and the lower loop, we can say that the eld is provided by the lower
loop, and the opposing magnetic moment is that of the upper ring. Substituting the expression for
B and (1.1.10) into (1.1.11),
F
__
a
2
I
o
t
cd
3

z
_

2
3
a
6
I
o
c
3
d
3
R
z
__

_
2
4
a
8
I
2
o
t
c
4
d
6

2
R
_
(1.1.12)
And in our case the gradient reduces to = (/r) r where r d, and (1.1.12) becomes
F
12
4
a
8
I
2
o
t
c
4
d
7

2
R
z (1.1.13)
Which is identical to (1.1.1) (with correction). Since (1.1.13) is positive, the force is repulsive .
Problem 9.2
a) Find the mutual inductance of the two rings in Problem 9.1 (making the suitable approximations).
W. Erbsen HOMEWORK #1
b) Use their mutual inductance to nd the EMF induced in the upper ring for the current I = I
o
t
/

in the lower
ring.
c) Find the mutual interaction energy in this system.
Solution
a) We know from (9.18) on p. 245 of Franklin that

2
= cM
21
I (1.1.14)
Using (1.1.6) from the previous problem, we can solve (1.1.14) to nd the mutual inductance:
M
21

_
1
cI
__
2
2
a
4
I
cd
3
_
M
21

2
2
a
4
c
2
d
3
(1.1.15)
b) As before, the induced EMF is given by
E =
1
c
d
B
dt
(1.1.16)
The magnetic ux can be found from the mutual inductance by substituting (1.1.15) into (1.1.14),
and the induced EMF can then be found by substituting the result into (1.1.16):
E
1
c
d
dt
__
cI
o
t

__
2
2
a
4
c
2
d
3
__
E
2
2
a
4
I
o
c
2
d
3

(1.1.17)
which is identical to (1.1.8).
c) The mutual interaction energy is most easily calculated from (5.152) from Jackson on p. 215: ?
w =
1
2
N

i=1
L
i
I
2
i
+
N

i=1
N

j=i
M
ij
I
i
I
j
(1.1.18)
Ignoring the self inductance terms in (1.1.18), we nd that
w =
1
2
[M
12
I
1
I
2
+ M
21
I
2
I
1
] (1.1.19)
Where the mutual inductances are of course equal. Substituting (1.1.9) and (1.1.15) into (1.1.19),
w
_
2
2
a
4
c
2
d
3
__
I
o
t

__

2
2
a
4
I
o
c
2
d
3
R
_
w
4
4
a
8
I
2
o
t
c
4
d
6

2
R
Problem 9.3
A long straight copper wire of radius a and resistance R carries a constant current I.
CHAPTER 1: ELECTRODYNAMICS II 5
a) Find the electric and magnetic elds at the surface of the wire.
b) Integrate the Poynting power ux through the surface of a piece of the wire of length L to show that the
power through the surface equals I
2
R.
c) Find the electromagnetic energy and momentum in this piece of wire.
Solution
a) Assuming that the wire can be approximated as innite, the magnetic eld can be found using
Amperes Law:
_
c
B d =
4
c
I
enc
B =
2I
ca

(1.1.20)
To nd the electric eld,
V =
_
f
i
E d
Where in our case E = [E[z and d = [d[z = dz z. We also note that
V = EL = IR E =
IR
L
z (1.1.21)
b) The Poynting power ux is most likely referring to the Poynting vector, which is really the
Poynting power ux density. We must rst nd S using (9.48) on p. 249
S =
c
4
(EH) (1.1.22)
Where in our case H = B. First, we calculate EH using (1.1.20) and (1.1.21),
E H =
_
IR
L
z
_

_
2I
ca

_
=
2I
2
R
caL
_
z

_
=
2I
2
R
caL
r (1.1.23)
The power loss is dened by P =
_
S
S dA. It is to our advantage to integrate in cylindrical
coordinates, and we recall that in our case r z = 0. Therefore, we can nd the Poynting power ux
using (1.1.22) and (1.1.23):
P =
c
4
_
S
(E H) dA
=
c
4
_

2I
2
R
caL
__
2
0
_
L
0
r ar ddz
=
I
2
R
2L
(2) (L) (1.1.24)
W. Erbsen HOMEWORK #1
It is easy to see that (1.1.24) reduces to
P = I
2
R
c) The electromagnetic energy density is given by (9.47) on p. 249 as
U
EM
=
1
8
(E D+ B H) (1.1.25)
Where in our case D = E and H = B. Therefore, (1.1.25) becomes
U
EM
=
1
8
_
E
2
+B
2
_
(1.1.26)
Where E and B are the values within the conductor. If we assume that the electric eld is uniform,
then we can use our previous value. To nd the magnetic eld,
_
C
B d =
4
c
_
r
2
_
I
a
2
B =
2I
ca
2
r

(1.1.27)
Substituting (1.1.21) and (1.1.27) into (1.1.26),
U
EM
=
1
8
_
_
IR
L
_
2
+
_
2I
ca
2
r
_
2
_
(1.1.28)
To nd the electromagnetic energy, we integrate the electromagnetic energy density (1.1.28) over
the entire volume:
w =
_
V
U
EM
d
=
1
8
_
V
_
_
IR
L
_
2
+
_
2I
ca
2
r
_
2
_
d
=
1
8
_
I
2
R
2
L
2
_
r drddz +
4I
2
c
2
a
4
_
r
3
drddz
_
=
1
8
_
I
2
R
2
L
2
_
a
0
r dr
_
2
0
d
_
L
0
dz +
4I
2
c
2
a
4
_
a
0
r
3
dr
_
2
0
d
_
L
0
dz
_
=
1
8
_
I
2
R
2
L
2
_
a
2
2
_
(2) (L) +
4I
2
c
2
a
4
_
a
4
4
_
(2) (L)
_
Simplifying this expression leads to:
w =
I
2
R
2
a
2
8L
+
I
2
L
4c
2
The electromagnetic momentum stored in the eld is given by (9.80) on p. 255 in Franklin:
P
EM
=
1
4c
_
V
E B d (1.1.29)
First calculating the cross product,
E B =
_
IR
L
z
_

_
2I
ca
2
r

_
=
2I
2
R
ca
2
L
r r (1.1.30)
CHAPTER 1: ELECTRODYNAMICS II 7
Substituting (1.1.30) into (1.1.29),
P
EM
=
1
4c
_
V
_

2I
2
R
ca
2
L
r
_
d =
I
2
R
2c
2
a
2
L
_
L
0
_
2
0
_
a
0
r drddz P
EM
= 0
Problem 9.4
The upper and lower curves of the hysteresis loop of a hard ferromagnetic are given by
B
+
=B
o
[tanh (H/H
o
+.5) .1] , (1.1.31a)
B

=B
o
[tanh (H/H
o
.5) + .1] , (1.1.31b)
respectively, for 1.5 < H/H
o
< +1.5.
a) Find the energy lost from the magnetic eld in one cycle.
b) If B
o
= 3 kilogauss, H
o
= 1 gauss, and the frequency is 60 Hz, nd the power loss in watts.
Solution
a) The work done per unit volume in each cycle of a hysteresis loop is given by the total area,
w =
_
B dH (1.1.32)
Where B = (B
+
B

). Applying this to (1.1.32) over the specied intervals,


w =
_
+1.5
1.5
(B
+
B

) dH
=
_
+1.5
1.5
_
B
o
_
tanh
_
H
H
o
+.5
_
.1
_
B
o
_
tanh
_
H
H
o
.5
_
+ .1
__
dH (1.1.33)
If we dene = H/H
o
and d = (1/H
o
) dH, (1.1.33) becomes
w =H
o
_
+1.5
1.5
[B
o
(tanh ( +.5) .1) B
o
(tanh ( .5) + .1)] d
=B
o
H
o
__
+1.5
1.5
tanh ( + .5) d
_
+1.5
1.5
tanh ( .5) d .2
_
+1.5
1.5
d
_
=B
o
H
o
_
_
ln[cosh ( + .5)]

+1.5
1.5

_
ln[cosh ( .5)]

+1.5
1.5
.2
_

+1.5
1.5
d
_
=B
o
H
o
ln [cosh (2)] ln[cosh (1)] ln[cosh (1)] + ln[cosh (2)] .6 (1.1.34)
Evaluating (1.1.34) numerically leads to
w 1.18244 B
o
H
o
W. Erbsen HOMEWORK #1
b) To nd the energy loss according to the given parameters,
w
(1.18244)(1 gauss)(3 10
3
gauss)
1/60 Hz

w 212.832 10
3
erg s
1
(CGS)
w 212.832 10
4
Watts (SI)
Problem 9.5
Two point charges, each of charge q, are a distance 2d apart.
a) Find the Maxwell stress tensor [T] on the plane surface midway between the charges.
b) Find the force on either charge by integrating [T] dS over a plane surface, closing the surface with a large
hemisphere of radius R. (Show that the integral over the hemisphere vanishes in the limit R .)
c) Repeat parts (a) and (b) if the charges have opposite signs.
d) What would the force on either charge be if they were immersed in a simple dielectric of innite extent?
Solution
a) The Maxwell stress tensor is given in Griths as (8.19) on p. 352 as
[T] =
o
_
E
i
E
j

1
2

ij
E
2
_
+
1

o
_
B
i
B
j

1
2

ij
B
2
_
(1.1.35)
For the case in question, our charges are stationary so that the second portion of (1.1.35) vanishes.
We can then express [T] in matrix form as
[T] =
_
_
T
xx
T
xy
T
xz
T
yx
T
yy
T
yz
T
zx
T
zy
T
zz
_
_
=
o
_
_
E
2
x

1
/
2
E
2
E
x
E
y
E
x
E
z
E
x
E
y
E
2
y

1
/
2
E
2
E
y
E
z
E
z
E
x
E
y
E
z
E
2
z

1
/
2
E
2
_
_
In order to resolve the individual components of [T], we must nd the corresponding electric elds.
We note that the charges are located on the x-axis, and the plane between the charges is the
yz-plane. Furthermore, we recall that the electric eld of an electric dipole is given by
E =
1
4
o
p
r
3
(1.1.36)
And in our case r =
_
x
2
+y
2
+z
2
_1
/2

_
d
2
+ y
2
+ z
2
_1
/2
. With this in mind, we can now nd the
electric elds in the x, y, and z directions respectively:
E
x
=0
E
y
=
1
2
o
ey
(d
2
+ y
2
+z
2
)
3
/2
y
1
2
o
er cos
(d
2
+ r
2
)
3
/2
r
E
z
=
1
2
o
ez
(d
2
+ y
2
+z
2
)
3
/2
z
1
2
o
er sin
(d
2
+r
2
)
3
/2
r
CHAPTER 1: ELECTRODYNAMICS II 9
Where I shamelessly switched to polar coordinates. The total electric eld is then
E
2
=E
2
x
+E
2
y
+E
2
z
=
_
1
2
o
er cos
(d
2
+ r
2
)
3
/2
r
_
2
+
_
1
2
o
er sin
(d
2
+ r
2
)
3
/2
r
_
2
=
_
e
2
o
_
2
r
2
(d
2
+ r
2
)
3
_
cos
2
+ sin
2

_
=
e
2
4
2

2
o
r
2
(d
2
+ r
2
)
3
We now have all the tools required to calculate the components of [T]:
T
xx
=
e
2
8
2

2
o
r
2
(d
2
+ r
2
)
3
T
yy
=
e
2
4
2

2
o
r
2
(d
2
+r
2
)
3
_
cos
2

1
2
_
T
yz
=T
zy
=
e
2
4
2

2
o
r
2
(d
2
+ r
2
)
3
sin cos
T
zz
=
e
2
4
2

2
o
r
2
(d
2
+r
2
)
3
_
sin
2

1
2
_
T
xy
=T
xz
= T
yx
= T
zx
= 0
We now dene
=
e
2
4
2

2
o
r
2
(d
2
+r
2
)
3
(SI)
=4e
2
r
2
(d
2
+r
2
)
3
(CGS)
And the Maxwell stress tensor becomes
[T] =
o
_
_
/2 0 0
0
_
cos
2
1/2
_
sin cos
0 sin cos
_
sin
2
1/2
_
_
_
b) To integrate the Maxwell stress tensor, we take the route suggested in the prompt. We take a
hemisphere of radius R with the base coinciding with the plane surface halfway between the charges
(the yz-plane). If we let the hemisphere expand to very large values, then the eld at the boundary
of the hemisphere is seen as a dipole, R
3
, and the surface integral then varies like R
4
. If we
take R out to innity, then this portion of the surface integral vanishes since the components of [T]
go to zero. There remains then only the force across the plane boundary in the yz-plane.
To nd the force on either charge, we integrate [T] dS over the interstitial plane. We note
that the force is in the x-direction, and the transversal forces are cancelled. Therefore, the only
component of [T] that we need to integrate is T
xx
, as tabulated previously. We also recall that for
the equatorial disk dS = r drd x, and so the force is
F =
_
S
[T] dS
W. Erbsen HOMEWORK #1
= 2
_

0
T
xx
r dr x
= 2
o
_

0
_

e
2
8
2

2
o
r
2
(d
2
+r
2
)
3
_
r dr x
=
e
2
4
o
_

0
r
3
(d
2
+ r
2
)
3
dr x
_
partial fractions
=
e
2
4
o
_

0
_
r
(d
2
+r
2
)
2

d
2
r
(d
2
+r
2
)
3
_
dr x
=
e
2
4
o
_
_

0
r
(d
2
+ r
2
)
2
dr d
2
_

0
r
(d
2
+r
2
)
3
dr
_
x
_
u = d
2
+ r
2
, du = 2rdr
=
e
2
4
o
_
1
2
_

0
1
u
2
du
d
2
2
_

0
1
u
3
dr
_
x
=
e
2
4
o
_
1
2
_

1
(d
2
+ r
2
)

d
2
2
_

1
2 (d
2
+r
2
)

0
_
x
=
e
2
4
o
_
1
2
1
d
2

d
2
2
1
2d
4
_
x (1.1.37)
From (1.1.37), it is only a short leap to our answer:
F =
1
4
o
e
2
(2d)
2
x (SI), F =
e
2
(2d)
2
x (CGS)
Which is exactly what we expected; proceeding through Coulombs law yields the precisely same
result.
c) If the charges are opposite, then the symmetry changes, however the process is very much the same.
The electric elds in this case are
E
x
=
1
2
o
ed
(d
2
+ r
2
)
3
/2
x
E
y
=E
z
= 0
And the total electric eld is
E
2
= E
2
x
=
1
4
2

2
o
e
2
d
2
(d
2
+r
2
)
3
And now to calculate the components of [T],
T
xx
=
1
4
2

2
o
e
2
d
2
(d
2
+ r
2
)
3
T
yy
=
1
4
2

2
o
e
2
d
2
(d
2
+ r
2
)
3
T
zz
=
1
4
2

2
o
e
2
d
2
(d
2
+ r
2
)
3
T
xy
=T
xz
= T
yx
= T
yz
= T
zx
= T
zy
= 0
CHAPTER 1: ELECTRODYNAMICS II 11
And lets go ahead and dene
=
1
4
2

2
o
e
2
d
2
(d
2
+ r
2
)
3
Then the Maxwell stress tensor is
[T] =
o
_
_
0 0
0 0
0 0
_
_
To nd the force on either charge, we follow the same logic as before,
F =
_
S
[T] d S
= 2
o
_

0
T
xx
r dr x
= 2
o
_

0
_
1
4
2

2
o
e
2
d
2
(d
2
+ r
2
)
3
_
r dr x
=
e
2
d
2
2
o
_

0
r
(d
2
+ r
2
)
3
dr x
_
u = d
2
+ r
2
, du = 2rdr
=
e
2
d
2
4
o
_

0
1
u
3
du x
=
e
2
d
2
4
o
_

1
4 (d
2
+r
2
)
2

0
x
=
e
2
d
2
4
o
_
1
4d
4
_
x (1.1.38)
From (1.1.38) it is easy to see that the force is
F =
1
4
o
e
2
(2d)
2
x (SI), F =
e
2
(2d)
2
x (CGS)
Which is what we would expect.
d) If the system is immersed into a linear dielectric of permittivity , then this will eect our answers
only by including a factor of in the denominator of our expressions, which weakens the force of
attraction or repulsion.
vacuum dielectric
equal charges F =
e
2
(2d)
2
x F =
1

e
2
(2d)
2
x
opposite charges F =
e
2
(2d)
2
x F =
1

e
2
(2d)
2
x
(CGS)
Problem 9.7
W. Erbsen HOMEWORK #1
A magnetic dipole is located at the center of a uniform electric charge distribution of radius R, charge e,
and mass m.
a) Find the electromagnetic angular momentum of this conguration.
b) Find the value of the radius R for which the g-factor of this conguration equals 2.
Solution
a) We rst recall that the electromagnetic angular momentum is given by (9.113) on p. 260 as
L
EM
=
1
4c
_
V
r (E B) d (1.1.39)
Now, we nd the electric eld by
E =
e
_
4
/
3
r
3
_
4
/
3
R
3
r
2
r =
er
R
3
r (1.1.40)
The magnetic eld is given by (7.108) on p. 209 as
B =
3 ( r) r
r
3
(1.1.41)
We also recall that in our case = [[ z, and therefore (1.1.41) becomes
B =

r
3
z (1.1.42)
We now use (1.1.40) and (1.1.42) to examine the cross product in (1.1.39):
EB =
_
er
R
3
r
_

r
3
z
_
=
e
R
3
r
2
(r z) =
e
R
3
r
2
_
sin

_
=
e
R
3
r
2
sin

(1.1.43)
Now substituting (1.1.43) into (1.1.39),
L
EM
=
1
4c
_
V
r
_
e
R
3
r
2
sin

_
d
=
e
4cR
3
_
V
sin
r
_
r

_
d
=
e
4cR
3
_
V
sin
r

r
2
sin drdd
=
e
4cR
3
_
R
0
rdr
_

0
_
2
0
sin
2


dd (1.1.44)
We now recall from the back cover of Griths that

= cos cos x + cos sin y sin z (1.1.45)


Substituting (1.1.45) into (1.1.44),
L
EM
=
e
4cR
3
_
R
0
rdr
_

0
_
2
0
sin
2
[cos cos x + cos sin y sin z] dd
CHAPTER 1: ELECTRODYNAMICS II 13
=
e
8cR
_

0
_
2
0
[sin
2
cos cos x
. .
I
+sin
2
cos sin y
. .
II
sin
3
z
. .
III
] dd (1.1.46)
Taking the angular integrals of I, II and III separately yields
I =
__

0
sin
2
cos d
_
2
0
cos d
_
x = 0 (1.1.47a)
II =
__

0
sin
2
cos d
_
2
0
sin d
_
y = 0 (1.1.47b)
III =
__

0
sin
3
d
_
2
0
d
_
z
= 2
__

0
sin d
_

0
sin cos
2
d
_
z
_
u = cos , du = sin
=2
__

0
sin d +
_
1
1
u
2
du
_
z
=2
_
_
cos

0
+
_
u
3
3

1
1
_
z
=2
_
2
2
3
_
z
=
8
3
z (1.1.47c)
Substituting (1.1.47a)-(1.1.47c) into (1.1.46),
L
EM
=
e
8cR
_
0 + 0
8
3
z
_
L
EM
=
e
3cR
z (1.1.48)
b) We recall from (7.134) on p. 214 that = g
B
J, and also that L
EM
= J. Combining these, as
well as implementing (1.1.48), yields
3cRL
EM
e
=
g
B
L
EM


3cR
e
=
g
B

R =
g
B
e
3c
(1.1.49)
Recall that the Bohr Magneton is given by

B
=
e
2m
e
(1.1.50)
The appropriate values we require are
e =1.60217733 10
19
C
=1.05457162 10
34
Kg m
2
s
1
c =2.99792458 10
8
m s
1
m
e
=9.10938215 10
31
Kg
Now substituting (1.1.50) into (1.1.49) as well as the appropriate constants,
R =
(2)
_
1.60217733 10
19
C
_
2
(6) (2.99792458 10
8
m s
1
) (9.10938215 10
31
Kg)
R = 3.13321563 10
17
m
W. Erbsen HOMEWORK #2
Problem 9.8
a) Calculate e
2
/c in Gaussian units to verify Eq. (9.121).
b) Calculate e
2
/(4
o
c) in SI units.
Solution
a) The values needed in this calculation are
e =4.80320680 10
10
cm
3
/2
g
1
/2
s
1
=1.05457162 10
27
erg s
c =2.99792458 10
10
cm s
1
And so, we can now calculate :
e
2
c
=
_
4.80320680 10
10
cm
3
/2
g
1
/2
s
1
_
2
1.05457162 10
27
erg s 2.99792458 10
10
cm s
1

e
2
c
= 0.00729736
1
137.036
b) The necessary values to make the calculation in SI units are
e =1.60217733 10
19
C

o
=8.85418781 10
12
F m
1
=1.05457162 10
34
J s
c =2.99792458 10
8
m s
1
And now,
e
2
4
o
c
=
_
1.60217733 10
19
C
_
2
4 (8.85418781 10
12
F m
1
) (1.05457162 10
34
J) (s 2.99792458 10
8
m s
1
)

e
2
4
o
c
= 0.00729736
1
137.036
1.2 Homework #2
Problem 10.1
The intensity of sunlight at the Earths surface is 12 10
5
ergs/(cm
2
sec).
a) Find the electric eld of this radiation at the Earths surface. Express the answer in stavolts/cm and in
volts/m (1 statvolt = 300 volts). [Answer: 960 V/m]
CHAPTER 1: ELECTRODYNAMICS II 15
b) Find the radiation pressure (in dynes/cm
2
) if the sunlight is 100% reected at normal incidence. Compare
this to atmospheric pressure. [Answer: 2 10
5
dynes/cm
2
]
c) Find the radiation pressure if
i) the sunlight is absorbed (no reection).
ii) the sunlight is specularly reected from white sand. (The reected radiation has the same intensity
at any angle.)
d) What would the intensity, electric eld, and radiation pressure be at the surface of the sun?
Solution
a) The intensity is called the time averaged Poynting vector, and is given by (10.27) of Franklin: ?
S =
c

k
8
_

[E
0
[
2
(1.2.1)
In SI units, the intensity is given by (9.61) in Griths on p. 381: ?
S =
1
2
c
0
[E
0
[
2

k (1.2.2)
Since we are given the intensity, it is a trivial task to nd the electric eld. We solve (1.2.2) for E
o
;
S =
1
2
c
0
[E
o
[
2

k [E
0
[ =

2S

0
c
(1.2.3)
We recall that 1 erg = 10
7
J and
0
8.8542 A
2
s
4
/(Kg m
3
), (1.2.3) becomes
[E
0
[

(2)(1200 Kg m
2
/s)
8.8542 A
2
s
4
/(Kg m
3
) 3 10
8
m/s

[E
0
[ 950.54 V/m
[E
0
[ 3.1685 10
2
statvolt/cm
(1.2.4)
b) The radiation pressure is given by (10.33) in Franklin as
p
rad
=
2
c

S (1.2.5)
If we recall that 1 erg = 1 dyne cm, then we can solve (1.2.5) directly:
p
rad
=
(2)(12 10
5
dyne/(cm sec))
3 10
10
cm/sec
p
rad
= 8 10
5
dynes/cm
2
(1.2.6)
The atmospheric pressure is p
atm
10
6
dynes/cm
2
, which means that the radiation pressure mul-
tiplied by a factor of 1.25 10
10
is approximately equal to the atmospheric pressure at 1 atm.
c) The radiation pressure for the following unique cases is as follows:
i) If the radiation is completely absorbed, then the radiation pressure would be just half that as
if it were completely reected from (1.2.6):
p
rad
=
1
c

S p
rad
= 4 10
5
dynes/cm
2
W. Erbsen HOMEWORK #2
ii) If the radiation is specularly reected (evenly across all angles), then certainly the radiation
pressure will be less than if it were completely reected. In fact, for each angle, we are only
interested in the 0
o
projection, going back towards the sun. But wait! Things arent quite this
simple we must integrate over all possible angles. Doing this yields the force:
F =
_

0
_
/2
0
_
2S
c
cos
_
sin dd
=
2S
c
2
_
/2
0
cos sin d
=
2S
c
(1.2.7)
And to nd the force we must divide by the surface area (note that I have neglected the arbitrary
radial term, as it vanishes anyway):
p
rad
=
F
A
=
2S
c

1
2
p
rad
= 4 10
5
dynes/cm
2
d) To nd the intensity at the surface of the sun, we assume that the total radiation ux at an altitude
of Earth is equal to that at the surface of the sun. We know the intensity on the surface of the
Earth, and also the distance of Earth from the sun (R 1.5 10
11
m), so the total ux (as calculated
from Earths orbit) is
S
tot
= 4R
2
S
Earth
And we also know that the radius of the sun is r 7.0 10
8
m, so the intensity at the surface can
be found as
S
sun
=
R
2
r
2
S
Earth
S
sun
5.5 10
10
ergs/(cm
2
sec)
Problem 10.2
A beam of elliptically polarized light, propagating in the z-direction, passes through a polarizer in the xy-plane.
The maximumtransmitted intensity is 9I
o
when the polarizer is set at 30
o
to the x-axis. The minimumtransmitted
intensity is I
o
when the polarizer is set at 120
o
to the x-axis.
a) Find r, , E
+
, and E

in the circular basis.


b) Find E
x
and E
y
in the plane basis.
c) What would the transmitted intensities be if the polarizer were set along the x-axis, and then along the
y-axis?
Solution
I am going to solve this problem using Jones Calculus, starting in the linear basis. We note that the
electric eld of an (arbitrary) polarized wave can be expressed as
CHAPTER 1: ELECTRODYNAMICS II 17
E(z, t) =
_
[E
ox
[e
ix
x +[E
oy
[e
iy
y
_
e
i(kzt)
=
_
E
ox
x + E
oy
e
i
y
_
e
i(kzt)
(1.2.8)
And now (1.2.8) can be expanded and put into matrix form, ditching the explicit time dependence:
E(z, t) =
_
E
ox
E
oy
e
i
_
=
_
E
ox
E
oy
(cos + i sin )
_
(1.2.9)
We also recall that the arbitrary Jones Matrix for a linear polarizer at some angle is given by
M =
_
cos
2
sin cos
sin cos sin
2

_
We can apply the Jones Matrix to our incident elliptical wave given by (1.2.8), the result gives the
exiting eld.

Starting with the maximum eld when the polarizer is oriented at = 30


o
,
[E

30
[
2
=
_
3/4

3 /4

3 /4 1/4
__
E
ox
E
oy
(cos +i sin )
_
=
1
4
_
3E
2
ox
+E
2
oy
+ 2

3 E
ox
E
oy
cos
_
(1.2.10)
And similarly if the polarizer is oriented at 120
o
:
[E

120
[
2
=
_
1/4

3 /4

3 /4 3/4
__
E
ox
E
oy
(cos +i sin )
_
=
1
4
_
E
2
ox
+ 3E
2
oy
2

3 E
ox
E
oy
cos
_
(1.2.11)
The intensity, of course can be rewritten to allow us to rewrite (1.2.10) and (1.2.11):
9I
o
=
c
8
_
1
4
_
3E
2
ox
+ E
2
oy
+ 2

3 E
ox
E
oy
cos
_
_
(1.2.12a)
I
o
=
c
8
_
1
4
_
E
2
ox
+ 3E
2
oy
2

3 E
ox
E
oy
cos
_
_
(1.2.12b)
We introduce another equation, the derivation of which will not be shown here:
tan(2) =
2E
ox
E
oy
cos
E
2
ox
E
2
oy
(1.2.13)
Solving (1.2.12a)-(1.2.13) allows us to solve for our unknowns. We also note that = 2 30
o
, which is
deducible from the fact that the maximum intensity is observed at 30
o
. The tabulated results follow in
(a) and (b).
a) The phase angle is found to be = 40.89
o
, while E
+
and E

may be found from the results of


part (b):

The following steps have been abbreviated, and for that I apologize. I can provide scrap work/code if desired.
W. Erbsen HOMEWORK #2
E
+
= E
x
+ iE
y
=
_
I
o
c
(13.26 cos(t) +i8.86 cos (t 40.89
o
))
E

= E
x
iE
y
=
_
I
o
c
(13.26 cos(t) i8.86 cos (t 40.89
o
))
b) We found E
ox
and E
oy
directly after the prompt. The values for these are:
E
ox
= 13.26
_
I
o
c
, and E
oy
= 8.86
_
I
o
c
From which we may calculate E
x
and E
y
:
E
x
= 13.26
_
I
o
c
cos(t), and E
y
= 8.86
_
I
o
c
cos (t 40.89
o
)
c) If the polarizer is placed rst precisely along the x-axis, then the intensity is:
I
0
=
c
8
(13.26)
2
I
0
= 7.53 I
o
And if perfectly along the y-axis,
I
90
=
c
8
(8.86)
2
I
90
= 3.12 I
o
Problem 10.3
Consider a beam of partially plane polarized light with the same maximum and minimum intensities as in the
previous problem.
a) What is the percent polarization of this light?
b) What would the transmitted intensities be if the polarizer were set along the x-axis, and then along the
y-axis?
c) How could you tell whether an incident light beam were elliptically polarized or partially plane polarized?
Solution
a) The polarization can be found as follows
=
S
max
S
min
S
max
+ S
min
(1.2.14)
We know from the previous problem that the maximum intensity is S
max
= 9I
o
while the minimum
intensity is S
min
= I
o
, so (1.2.14) becomes
=
9I
o
I
o
9I
o
+I
o
=
8
10
( = 80.00%)
CHAPTER 1: ELECTRODYNAMICS II 19
b) If we imagine the partially-plane polarized light incident on the polarizer to be composed of orthog-
onal linear components along the x and y-directions, then the total intensity is
S

= S
2
x
+S
2
y
+ 2
_
S
x
S
y
cos
Where is the angle between S
x
and S
y
, which is most denitely 90
o
. We can nd S
max
and S
min
by taking these limits:
S
max
=
__
S
2
x
+ S
2
y
_
2
, and S
min
=
__
S
2
x
S
2
y
_
2
And now using (1.2.14) we can say that
S
max
S
min
=
1 +
1
(1.2.15)
From Malus Law, we can now say that
S

=
1
1 +
cos
2
()S
max
With the help of (1.2.15). We can now say that the transmitted intensity when the polarizer is
placed perfectly along the x and y-axis, respectively are
S

x
= 0.75 I
o
, and S

y
= 0.25 I
o
c) To tell if incident radiation were elliptically polarized or partial plane polarized, I would place a
linear polarizer in front of the incident beam followed by an energy meter. If the intensity changes
monotonically with polarization angle then the radiation is said to be more linearly polarized than
elliptically.
Problem 10.5
A horizontal light ray is incident on a 60
o
-60
o
-60
o
glass prism (n = 1.5) that is resting on a table. At what
angle with the horizontal does the ray leave the prism? (Assume that the ray does not strike the bottom of the
prism before exiting.)
Solution
The initial ray is coming in completely horizontal, however the incident angle to the medium is propor-
tional to the dimensions of the prism. In our case, the normal component of the front face of the prism
makes and angle that is precisely 30
o
from the incident ray. We now apply Snells Law,
n
a
sin
a
= n
g
sin
g

g
= sin
1
_
sin(30)
1.5
_
19.47
o
(1.2.16)
Now the ray is within the prism, and the next interface is that of the opposite surface of the prism. The
normal component of the exiting face is 60
o
in the positive direction to the rst, so that the angle between
the incident beam calculated in (1.2.16) and this normal angle is 60
o
19.47
o
= 40.53
o
. Applying Snells
Law once more,
W. Erbsen HOMEWORK #2
n
g
sin
g
= n
a
sin
a

a
= sin
1
[1.5 sin(40.53)]
a
77.10
o
Problem 10.6
For the prism in the preceding problem, what is the smallest angle of incidence for which a light ray will pass
directly through the prism without total internal reection?
Solution
Total internal reection occurs at
c
= sin
1
(1/1.5) 41.81
o
. This is, of course, relative to the normal
component of the interior of the exiting interface. The simplest way to nd the incident angle is to work
backwards from what we did in the preceding problem.
We then only must apply Snells Law once, for the rst interface. The critical angle calculated for
the outer face is oriented to the normal of the rst face as 60
o
41.81
o
= 18.19
o
. We can now apply
Snells Law:
n
g
sin
g
= n
a
sin
a

a
= sin
1
[1.5 sin(18.19
o
)]
a
27.92
o
Problem 10.7
You are standing in front of a rectangular sh tank lled with water (n = 1.33).
a) Show that you can always see through the back of the tank without total internal reection as you look
through the front face.
b) Show that if you look through the front face toward the right side of the tank, there is a maximum angle of
incidence for which there is total internal reection from the right face. What is this angle?
c) What is the minimum index of refraction of the liquid in the tank such there that would always be total
internal reection from the right face?
d) Show that the fact that there is a thickness of glass (n = 1.5) between the water and the air does not aect
this problem.
Solution
a) To show that you can always see through the back of the sh tank when viewed through the
front, it is sucient to show that when viewing the sh tank at the most extreme angle, that the
exiting angle is less than the critical angle. In our case, the critical angle for glass/air interface is

c
= sin
1
(1/1.5) 41.81
o
.
CHAPTER 1: ELECTRODYNAMICS II 21
There are 5 distinct zones in the problem with 4 interfaces. Starting from the front of the sh
tank, the air is region 1, then the glass is region 2 and so on. The critical angle refers to the glass/air
interface between regions 4 and 5, so to satisfy the requirement that light entering from the front
of the sh tank (region 1) will always exit through the back (region 2) we must show that
4
<
c
.
We imagine the incident angle to be as wide as possible, say 89.99
o
, which corresponds to
1
:
n
1
sin
1
=n2 sin
2

2
= sin
1
_
1
1.5
sin (89.99
o
)
_
41.81
o
n
2
sin
2
=n3 sin
3

3
= sin
1
_
1.5
1.33
sin (41.81
o
)
_
48.75
o
n
3
sin
3
=n4 sin
4

4
= sin
1
_
1.33
1.5
sin (48.75
o
)
_
41.80
o
(1.2.17)
From (1.2.17), we can see that
4
<
c
(41.80
o
< 41.81
o
) so that total internal reection is never
achieved, and you can always see through the back of the sh tank .
b) The process here is very much the same as in the preceding problem, except that we need to work
backwards. The critical angle has not changed,
4
=
c
41.81
o
. So, working backwards from
4
,
we have
n
3
sin

3
=n
4
sin
4

3
= sin
1
_
1.5
1.33
sin (41.81
o
)
_
48.75
o
n
2
sin
2
=n
3
sin
3

2
= sin
1
_
1.33
1.5
sin (90
o
48.75
o
)
_
35.77
o
n
1
sin
1
=n
2
sin
2

1
= sin
1
[1.5 sin(35.77
o
)] 61.27
o
(1.2.18)
Hence from (1.2.19) we have shown that the maximum angle of incidence to allow for total internal
reection on the right face of the sh tank is
1
= 61.27
o
.
c) To nd the minimum index of refraction of the liquid such that light incident to the front of the
sh tank will always satisfy total internal reection, we once again precede using Snells Law. The
only dierence is that this time we will leave the index of refraction for the liquid to be arbitrary.
n
1
sin
1
=n
2
sin
2

2
= sin
1
_
1
1.5
sin(
1
)
_
n
2
sin
2
=n
3
sin
3

3
= sin
1
_
1.5
n
sin
_
sin
1
_
1
1.5
sin (
1
)
___
= sin
1
_
1
n
sin (
1
)
_
n
3
sin
3
=n
4
sin
4
sin (
4
) =
n
1.5
sin
_
90
o
sin
1
_
1
n
sin (
1
)
__
(1.2.19)
It is possible to solve (1.2.19) for n, using the appropriate trig substitutions:
1.5 sin(
4
) =nsin
_
90
o
sin
1
_
1
n
sin (
1
)
__
=n
_
sin (90
o
) cos
_
sin
1
_
1
n
sin (
1
)
__
cos (90
o
) sin
_
sin
1
_
1
n
sin (
1
)
___
=n
_
1
sin
2
(
1
)
n
2
_
1
/2
=
_
n
2
sin
2
(
1
)

1
/2
(1.2.20)
W. Erbsen HOMEWORK #2
If we recall that
4
=
c
and imagine our incident angle to the most extreme position of
1
= 89.99
o
,
(1.2.20) can be solved to nd the minimum index of refraction:
n =
_
(1.5)
2
sin
2
(41.81
o
) + sin
2
(89.99
o
)
_1
/2
n 1.41 (1.2.21)
d) The fact that the width of the glass is negligible can be shown by recomputing the results of parts
(a) and (b) with assuming that there is no glass media. For part (a), where we are asked to show
that total internal reection is never satised if viewing through the front of the tank, we start with
the critical angle at the second interface, which is
c
= sin
1
(1/1.33) 48.75
o
, and applying Snells
Law:
n
1
sin
1
= n
2
sin
c

1
=sin
1
[1.33 sin(48.75
o
)] = 90
o
Since you can never see through the sh tank at exactly 90
o
, we have shown that the critical
condition cannot be satised, as previously shown in (1.2.17).
For part (b), we can similarly work backwards from the critical angle:
n
1
sin
1
= n
2
sin
c

1
=sin
1
[1.33 sin(90
o
48.75
o
)] 61.27
o
(1.2.22)
As can be seen, (1.2.22) matches (1.2.19), and therefore from these two instances we are forced to
admit that the width of the glass is irrelevant .
Problem 10.9
a) Solve Eqs. (10.117)-(10.120), to get the transmitted and reected electric elds given by Eqs. (10.121) and
(10.122).
b) Use these elds to get the transmission and reection coecients for the coated surface.
c) For an original air (n = 1) to glass (n = 1.5) interface, nd the index of refraction and thickness of a coating
that would have no reection at the incident wavelength of 5, 000

A.
Solution
a) Equations (10.117)-(10.120) read
E
1
+E

1
=E
2
+E

2
e
ik2d
(1.2.23a)
n
1
(E
1
E

1
) =n
2
(E
2
E

2
e
ik2d
) (1.2.23b)
E
2
e
ik2d
+E

2
=E
3
(1.2.23c)
n
2
(E
2
e
ik2d
E

2
) =n
3
E
3
(1.2.23d)
We start with (1.2.23a) and rearrange it:
E

1
= E
2
+ E

2
e
ik2d
E
1
(1.2.24)
CHAPTER 1: ELECTRODYNAMICS II 23
We now substitute (1.2.24) into (1.2.23b):
n
1
_
E
1

_
E
2
+E

2
e
ik2d
E
1
_
=n
2
_
E
2
E

2
e
ik2d

n
1
_
E
1
E
2
E

2
e
ik2d
+ E
2

=n
2
_
E
2
E

2
e
ik2d

2n
1
E
1
n
1
E
2
n
1
E

2
e
ik2d
=n
2
E
2
E

2
n
2
e
ik2d
n
2
E

2
e
ik2d
n
1
E

2
e
ik2d
=n
2
E
2
+ n
1
E
2
2n
1
E
1
E

2
= e
ik2d
_
E
2
(n
2
+n
1
) 2n
1
E
1
n
2
n
1
_
(1.2.25)
We now take (1.2.25) and substitute it into (1.2.23c):
E
2
e
ik2d
+ e
ik2d
_
E
2
(n
2
+n
1
) 2n
1
E
1
n
2
n
1
_
= E
3
E
3
(n
2
n
1
) =E
2
(n
2
n
1
)e
ik2d
+E
2
(n
2
+ n
1
)e
ik2d
2n
1
E
1
e
ik2d
2E
1
n
1
e
ik2d
+ E
3
(n
2
n
1
) =E
2
(n
2
n
1
)e
ik2d
+E
2
(n
2
+ n
1
)e
ik2d
=E
2
_
n
2
e
ik2d
n
1
e
ik2d
+n
2
e
ik2d
+ n
1
e
ik2d

=E
2
_
n
2
_
e
ik2d
+ e
ik2d
_
n
1
_
e
ik2d
e
ik2d
_
=E
2
[2n
2
cos(k
2
d) 2in
1
sin(k
2
d)]
=2E
2
[n
2
cos(k
2
d) in
1
sin(k
2
d)]
E
2
=
2E
1
n
1
e
ik2d
+E
3
(n
2
n
1
)
2 [n
2
cos(k
2
d) in
1
sin(k
2
d)]
(1.2.26)
We perform a similar exercise by substituting (1.2.25) into (1.2.23d):
n
3
E
3
=n
2
_
E
2
e
ik2d
e
ik2d
_
E
2
(n
2
+n
1
) 2n
1
E
1
n
2
n
1
__
n
3
E
3
=n
2
E
2
e
ik2d

E
2
(n
2
+ n
1
)n
2
e
ik2d
+ 2n
1
n
2
E
1
e
ik2d
n
2
n
1
n
3
(n
2
n
1
)E
3
=n
2
(n
2
n
1
)E
2
e
ik2d
n
2
E
2
(n
2
+n
1
)e
ik2d
+ 2n
1
n
2
E
1
e
ik2d
n
3
(n
2
n
1
)E
3
2n
1
n
2
E
1
e
ik2d
=n
2
E
2
_
n
2
e
ik2d
n
1
e
ik2d
n
2
e
ik2d
n
1
e
ik2d

=n
2
E
2
_
n
2
_
e
ik2d
e
ik2d
_
n
1
_
e
ik2d
e
ik2d
_
=n
2
E
2
[2in
2
sin(k
2
d) 2n
1
cos(k
2
)d)]
E
2
=
n
3
(n
2
n
1
)E
3
2n
1
n
2
E
1
e
ik2d
2n
2
[in
2
sin(k
2
d) n
1
cos(k
2
d)]
(1.2.27)
We can now set (1.2.26) and (1.2.27) equal to one another to eliminate E
2
:
2E
1
n
1
e
ik2d
+ E
3
(n
2
n
1
)
2 [n
2
cos(k
2
d) in
1
sin(k
2
d)]
=
n
3
(n
2
n
1
)E
3
2n
1
n
2
E
1
e
ik2d
2n
2
[in
2
sin(k
2
d) n
1
cos(k
2
d)]
(1.2.28)
If we let = sin k
2
d and = cos k
2
d, then (1.2.28) becomes
2E
1
n
1
e
ik2d
+E
3
(n
2
n
1
)
n
2
in
1

=
n
3
(n
2
n
1
)E
3
2n
1
n
2
E
1
e
ik2d
in
2
2
n
1
n
2

2E
1
n
1
e
ik2d
n
2
in
1

+
2n
1
n
2
E
1
e
ik2d
in
2
2
n
1
n
2

. .
I
=
n
3
(n
2
n
1
)E
3
in
2
2
n
1
n
2


E
3
(n
2
n
1
)
n
2
in
1

. .
II
W. Erbsen HOMEWORK #2
Where I have separated our expression for simplicity. The components are:
I =
2E
1
n
1
e
ik2d
n
2
in
1


n
2
+ in
1

n
2
+ in
1

+
2n
1
n
2
E
1
e
ik2d
in
2
2
n
1
n
2


in
2
2
+n
1
n
2

in
2
2
+n
1
n
2

=
2E
1
n
1
e
ik2d
(n
2
+ in
1
)
n
2
2

2
+ n
2
1

2

2n
1
n
2
E
1
e
ik2d
_
in
2
2
+ n
1
n
2

_
n
2
1
n
2
2

2
+ n
4
2

2
=
2E
1
n
1
(n
2
n
1
)E
1
(i + )
(n
2
+ in
1
)(n
1
+in
2
)
=
2E
1
n
1
(n
2
n
1
)E
1
(i sin(k
2
d) + cos(k
2
d))
(n
2
sin(k
2
d) +in
1
cos(k
2
d))(n
1
sin(k
2
d) +in
2
cos(k
2
d))
=
4in
1
(n
1
n
2
)E
1
2in
1
n
2
cos(2k
2
d) + (n
2
1
+ n
2
2
) sin(2k
d
d)
(1.2.29)
And now
II =
n
3
(n
2
n
1
)E
3
in
2
2
n
1
n
2


in
2
2
+ n
1
n
2

in
2
2
+ n
1
n
2


E
3
(n
2
n
1
)
n
2
in
1


n
2
+in
1

n
2
+in
1

=
n
3
(n
2
n
1
)E
3
_
in
2
2
+n
1
n
2

_
n
2
1
n
2
2

2
+ n
4
2

2

E
3
(n
2
n
1
) (n
2
+in
1
)
n
2
2

2
+n
2
1

2
= E
3
(n
2
n
1
)
_
n
3
n
2
(n
1
in
2
)
+
in
1
n
2

n
2
1

2
+ n
2
2

2
_
= E
3
(n
2
n
1
)
_
n
3
n
2
(n
1
cos(k
2
d) in
2
sin(k
2
d))
+
in
1
n
2
sin(k
2
d) cos(k
2
d)
n
2
1
sin(k
2
d)
2
+n
2
2
cos(k
2
d)
2
_
(1.2.30)
Setting (1.2.29) equal to (1.2.30) and shifting to one side, we see that
(n
1
n
2
)
_
E
3
(n
2
2
+n
1
n
3
) sin(k
2
d) +iE
3
n
2
(n
1
+ n
3
) cos(k
2
d) 2iE
1
n
1
n
2
_
n
2
(2in
1
n
2
cos(2k
2
d) + (n
2
1
+n
2
2
) sin(2k
2
d))
= 0 (1.2.31)
Solving (1.2.31) for E
3
yields
E
3
=
2n
1
n
2
n
2
(n
1
+n
3
) cos(k
2
d) i(n
2
2
+n
1
n
3
) sin(k
2
d)
E
1
Using the results from part (b), it is also easily seen that
E

1
=
_
n
2
(n
1
n
3
) cos(k
2
d) i(n
1
n
3
n
2
2
)
2
sin
2
(k
2
d)
n
2
(n
1
+n
3
) cos(k
2
d) i(n
1
n
3
+ n
2
2
)
2
sin
2
(k
2
d)
_
E
1
b) The transmission coecient can be found by
T =
n S
2
n S
1
=
n
3
n
1

E
3
E
1

2
=
n
3
n
1

2n
1
n
2
n
2
(n
1
+n
3
) cos(k
2
d) i(n
2
2
+ n
1
n
3
) sin(k
2
d)

2
(1.2.32)
From (1.2.32) it can be seen that the transmission coecient is nally given by
T =
4n
1
n
3
n
2
2
n
2
2
(n
1
+n
3
)
2
cos
2
(k
2
d) + (n
1
n
3
+ n
2
2
)
2
sin
2
(k
2
d)
(1.2.33)
Which matches (10.123). The reection coecient may be calculated via similar means, however
the easiest way is to recognize that R +T = 1, so that from (1.2.33), we calculate
CHAPTER 1: ELECTRODYNAMICS II 25
R =
n
2
2
(n
1
+n
3
)
2
cos
2
(k
2
d) + (n
1
n
3
+n
2
2
)
2
sin
2
(k
2
d)
n
2
2
(n
1
+n
3
)
2
cos
2
(k
2
d) + (n
1
n
3
+n
2
2
)
2
sin
2
(k
2
d)

4n
1
n
3
n
2
2
n
2
2
(n
1
+ n
3
)
2
cos
2
(k
2
d) + (n
1
n
3
+n
2
2
)
2
sin
2
(k
2
d)
=
n
2
2
(n
2
1
+n
2
3
+ 2n
1
n
3
) cos
2
(k
2
d) + (n
2
1
n
2
3
+n
4
2
+ 2n
1
n
3
n
2
2
) sin
2
(k
2
d) 4n
1
n
3
n
2
2
n
2
2
(n
1
+ n
3
)
2
cos
2
(k
2
d) + (n
1
n
3
+n
2
2
)
2
sin
2
(k
2
d)
=
n
2
2
(n
2
1
+n
2
3
+ 2n
1
n
3
) cos
2
(k
2
d) + (n
2
1
n
2
3
+n
4
2
+ 2n
1
n
3
n
2
2
) sin
2
(k
2
d) 4n
1
n
3
n
2
2
(sin
2
(k
2
d) + cos
2
(k
2
d))
n
2
2
(n
1
+n
3
)
2
cos
2
(k
2
d) + (n
1
n
3
+ n
2
2
)
2
sin
2
(k
2
d)
Combining terms yields
R =
n
2
2
(n
1
n
3
)
2
cos
2
(k
2
d) + (n
1
n
3
n
2
2
)
2
sin
2
(k
2
d)
n
2
2
(n
1
+n
3
)
2
cos
2
(k
2
d) + (n
1
n
3
+n
2
2
)
2
sin
2
(k
2
d)
c) To nd the thickness of the coating, we use (10.127), which reads
d =

1
4
_
n
1
n
3
Applying the provided values,
d =
5000 10
10
m
4
_
1
1.5
d 1.02 10
7
m
And by looking at our equation for the reection coecient, we can see that it will be zero if
n
2
=

n
1
n
3
, so the index of refraction of the coating is n
2
1.22 .
1.3 Homework #3
Problem 11.1
A plane electromagnetic wave is incident at an angle from vacuum onto the at surface of a perfect conductor.
a) Use conservation of momentum to nd the radiation pressure on the surface of the conductor.
b) For a wave that is polarized perpendicular to the plane of incidence, nd the radiation pressure by calculating
the magnetic force on the surface current.
c) For a wave that is polarized parallel to the plane of incidence, nd the radiation pressure by calculating the
magnetic and electric forces on the surface and surface charge.
Solution
a) The equation for radiation pressure was derived using conservation of momentum by Franklin as
(10.33):
p
rad
=

4
[E
0
[
2
(1.3.1)
W. Erbsen HOMEWORK #3
This is for the case that the incident light is completely reected (no absorption) and is normal to
the reecting surface. Since the radiation is at an angle , we must take the normal component of
(1.3.1) in order to discount the radiation not contributing momentum pressure. Furthermore, we
must also recognize that we must only recognize the component of the actual momentum normal
to the conductor (since we are recognizing both incoming and outgoing rays, the other component
cancels). Therefore,
p
rad
() =

4
[E
0
[
2
cos cos p
rad
=
1
4
[E
0
[
2
cos
2
(1.3.2)
Since we are coming from vacuum,
0
1 in CGS units.
b) &c) It is clear from (1.3.2) that the radiation pressure is independent of polarization, so the result
will once again be:
p
rad
=
1
4
[E
0
[
2
cos
2

Problem 11.2
Use the physical properties of copper to estimate the frequency and wave length for which its conductivity develops
an appreciable ( 10%) imaginary part. Assume two conduction electrons per atom.
Solution
We are primarily motivated by the availability of (11.62) in Franklin:
() =
z
c
Ne
2
m( i)
(1.3.3)
We can separate (1.3.3) into its real and imaginary parts,
Re () =
z
c
Ne
2
m

2
+
2
(1.3.4a)
Im() =
z
c
Ne
2
m

2
+
2
(1.3.4b)
We are told that z
c
is the number of conduction of electrons, which in the case of Copper is just 2.
Furthermore, the combined quantity z
c
N is the number of free electrons per unit volume, while is the
damping constant. According to Jackson on p. 312, the relevant quantities can be given numerically as:
?
N 8 10
28
atoms/m
3
5.9 10
7
( m)
1
CHAPTER 1: ELECTRODYNAMICS II 27
The damping constant can now be calculated using (1.3.3):

z
c
Ne
2
m

(2)(8 10
28
atoms/m
2
)(1.602 10
19
C)
2
(9.109 10
31
kg)(5.9 10
7
( m)
1
)
7.641 10
13
Hz (1.3.5)
We are interested at the frequencies at which the imaginary part of (1.3.3) develops an imaginary part
that is equal to 10% of the real part:
Im() = (0.10)Re () = (0.10) (1.3.6)
Where I used the real and imaginary components of (1.3.3) as shown in (1.3.4a) and (1.3.4b). According
to (1.3.6), the approximate frequency where this is possible is:
7.641 10
12
Hz (1.3.7)
If we recall that = 2c/, and using (1.3.7) we can nd the corresponding wavelength:
=
2(3 10
8
m/s)
(7.641 10
12
Hz)
246.7 m
Problem 11.3
a) Find the real and imaginary parts of , as given in (11.52).
b) Show that is analytic in the upper half of the complex plane. In nding any particular pole position, the
contributions from other modes can be ignored since they are analytic in that region.
Solution
a) Equation (11.52) reads
() = 1 + 4 = 1 +
4N
i
z
i

i
1
4
3

i
z
i

i
(1.3.8)
And we also recall that the molecular polarizability is from (11.52) in Franklin

i
=
e
2
m[
2
i

2
i
i
]
(1.3.9)
We also note that the frequency-dependent susceptibility takes the form
=
N
i
z
i

i
1
4
3

i
z
i

i
=
Ne
2
m
1

2
i

2
i
2
i
(1.3.10)
W. Erbsen HOMEWORK #3
Then the real and imaginary parts of from (1.3.8) are
Re () = 1 +
4Ne
2
m

2
i

(
2
i

2
)
2
+
2

2
Im() =
4Ne
2
m

(
2
i

2
)
2
+
2

2
b) To show that is analytic in the upper half of the complex plane, we rst substitute (1.3.10) into
(1.3.8):
() = 1 +
4Ne
2
m
1

2
i

2
i
2
i
(1.3.11)
Taking the complex integral of (1.3.11) leads to
_

() d =
_

_
1 +
4Ne
2
m
1

2
i

2
i
2
i
_
d
=
4Ne
2
m
_

2
i

2
i
2
i
d (1.3.12)
It is clear that (1.3.12) has poles in the lower half plane at

1,2
=
i
i
2

_

2
i


2
i
4
_
Since there are no poles at the boundary or in the upper half plane, and if we assume that the
kernel is nite, then we can say that () is analytic in the upper half plane .
Problem 11.4
Find the Fourier transform of the function in (11.74) to verify (11.75). (Hint : Complete the square in the
exponent in doing the integral.)
Solution
We rst recall that the functional forms of the integral Fourier transform is
f(x, 0) =
_

f(, 0)e
ikx
dk (1.3.13a)
A(k, 0) =
1
2
_

f(x, 0)e
ikx
dx (1.3.13b)
And the wave packet function (11.74) is given by
f(x, 0) = e
x
2
/2L
2
e
ik0x
(1.3.14)
To nd the Fourier transform of (1.3.14), we substitute it into (1.3.13b):
CHAPTER 1: ELECTRODYNAMICS II 29
A(k, 0) =
1
2
_

_
exp
_

x
2
2L
2
_
exp [ik
0
x]
_
exp [ikx] dx
=
1
2
_

exp
_

x
2
2L
2
_
exp [i (k
0
k) x] dx
=
1
2
_

exp
_

x
2
2L
2
ix
_
dx (let = k k
0
) (1.3.15)
At this point, per the suggestion in the prompt, we wish to complete the square in the exponent in the
integrand in (1.3.15):

x
2
2L
2
ix =
1
2L
2
_
x
2
+ 2iL
2
x
_

1
2L
2
_
_
x + iL
2

_
2
+
_
L
2

_
2
_
(1.3.16)
We now substitute (1.3.16) into (1.3.15),
A(k, 0) =
1
2
_

exp
_

1
2L
2
_
_
x +iL
2

_
2
+
_
L
2

_
2
_
_
dx
=
1
2
exp
_

L
2

2
2
_ _

exp
_

1
2L
2
_
_
x + iL
2

_
2
_
_
dx (1.3.17)
We wish to employ u

du

substitution (where the primes are present to avoid confusion later on), where
u

=
1

2L
2
_
x +iL
2

_
, and du

=
1

2L
2
dx
With this substitution, (1.3.17) becomes
A(k, 0) =
1
2
exp
_

L
2

2
2
_

2L
2
_

e
u
2
du

=
L

2
exp
_

L
2

2
2
_ _

e
u
2
du

(1.3.18)
The integral in (1.3.18) is just the standard Gaussian integral, whose solution is simply

, and in
substituting our value for , (1.3.18) becomes
A(k, 0) =
L

2
exp
_

L
2
(k k
0
)
2
2
_
(1.3.19)
We can see that (1.3.19) is identical to (1.3.13b), which is also the same as (11.75), which is what we
were asked to show.
Problem 11.5
A wave packet is given by
f(x, 0) = e
imx/L
, 0 < x < L, (1.3.20)
W. Erbsen HOMEWORK #3
and f(x, 0) is zero elsewhere.
a) Plot [f(x, 0)[
2
.
b) Find the Fourier transform, A(k), of f(x, 0). Plot [A[
2
for m = 1 and m = 2.
c) Dene a reasonable x and k for this wave packet. Calculate x, k, and xk.
Solution
We rst note that due to boundary conditions, we can rewrite (1.3.20) as
f(x, 0) = i sin
_
mx
L
_
(1.3.21)
a) The item to be plotted is (please see last page for printout of plots):
[f(x, 0)[
2
= sin
2
_
mx
L
_
(1.3.22)
b) The Fourier transform can be found the same way as we did in the last problem:
A(k, 0) =
i
2
_
L
0
sin
_
mx
L
_
e
ikx
dx
A(k, 0) =
iL
2
e
ikL
_
m cos(m) + ikLsin(m) me
ikL

k
2
L
2
m
2

2
(1.3.23)
c) A reasonable value for x is L, and likewise for k is 1/L, so that xk = 1:
x = L, k =
1
L
, xk = 1
Problem 11.6
a) Derive (11.81) and (11.82).
b) The spectral line in the decay of an excited state of sodium has a wave length ( = 5, 893

A), and a lifetime
of 2 10
8
seconds. Calculate its natural line width.
Solution
a) In order to derive (11.81), we must take the Fourier Transform of (11.80), which reads
E(x, t) =
_
E
0
exp [i (k
0
x t)] exp
_
xct
2c

, x < ct,
0 x > ct
Assuming we are in the rst region, at if we take t = 0, (11.80) becomes
CHAPTER 1: ELECTRODYNAMICS II 31
E(x, 0) = E
0
exp
_
i
_
k
0

i
2c
_
x
_
(1.3.24)
We take the Fourier Transform of (1.3.24) by substituting it into (1.3.13b),
E(k) =
E
0
2
_

exp
_
i
_
k
0

i
2c
_
x
_
exp [ikx] dx
=
E
0
2
_

exp
__
i (k
0
k) +
1
2c
_
x
_
dx
=
E
0
2
_
1
i (k
0
k) + 1/(2c)
exp
__
i (k
0
k) +
1
2c
_
x
_

E(k) =
E
0
2 [i(k
0
k) + 1/(2c)]
(1.3.25)
It is clear that (1.3.25) is identical to (11.81). In order to derive (11.82), we simply take the modulus
squared of (1.3.25)
[E(k)[
2
=
_
E
0
2 [i(k
0
k) + 1/(2c)]
__
E
0
2 [i(k
0
k) + 1/(2c)]
_
(1.3.26)
At this point we make the following assignments: = (k
0
k), and = 1/(2c), since otherwise
we will run out of paper. With these new denitions, (1.3.26) becomes
[E(k)[
2
=
_
E
0
2 [i + ]
__
E
0
2 [i + ]
_
=
E
2
0
4
2
1
(i +) (i +)
=
E
2
0
4
2
1

2
+
2
(1.3.27)
Backsubstituting our previous denitions for and , (1.3.27) becomes
[E(k)[
2
=
E
2
0
4
2
[(k k
0
)
2
+ 1/(2c)
2
]
(1.3.28)
And it is easy to see that (1.3.28) is identical to (11.82).
b) According to (11.84)

, the natural line width is given by


=

2
2c
(1.3.29)
With the data provided, this becomes
=
_
5893 10
10
m
_
2
2 (3.00 10
8
m/s) (2 10
8
s)
9.212 10
15
m

with correction
W. Erbsen HOMEWORK #3
Problem 11.7
A glass block has an index of refraction given by
n = 1.350
100

A

(1.3.30)
in the visible region.
a) What is the angular dierence between red light (Use = 6, 500

A) and blue light ( = 4, 500

A) in the glass
if the light enters with an angle of incidence of = 30
o
?
b) What are the phase and group velocities for each color light in the glass?
Solution
a) This problem requires a simple application of Snells Law; so solving for the exiting angle
2
,
n
1
sin(
1
) = n
2
sin(
2
)
2
= sin
1
_
n
1
n
2
sin(
1
)
_
(1.3.31)
So the goal here would be to calculate the exiting angle for both red and blue light, and take the
dierence. We rst address incident light in the case that it is red, and nd the index of refraction
from (1.3.30):

n
r
= 1.350
100

A
6, 500

A
1.3346 (1.3.32)
We now take (1.3.32) and substitute it into (1.3.31):

r
= sin
1
_
1
1.3346
sin(30
o
)
_
22.002
o
(1.3.33)
We now do the same for the shorter wavelength, starting with calculating the index of refraction
from (1.3.30):
n
b
= 1.350
100

A
4, 500

A
1.3278 (1.3.34)
And now we can calculate the angle from (1.3.31):

b
= sin
1
_
1
1.3278
sin(30
o
)
_
22.121
o
(1.3.35)
And therefore the angular dierence is given by
=
b

r
= 22.121
o
22.002
o
= 0.11927
o
b) The phase velocities for both wavelengths in the glass are given simply by (11.93):
v
p
=
c
n
(1.3.36)

Two assumption have been made. First, I have assumed that all the constant in (1.3.30), as well as all given wavelengths, are exact,
so that we can take the result out to arbitrary precision. Otherwise, we would be bound to the rules of signicant gures, which would
limit the transparency of the answer. Second, I have taken the incident index of refraction to be that of a vacuum, i.e. n
1
= 1.00.
CHAPTER 1: ELECTRODYNAMICS II 33
Completing this equation using rst the index of refraction of the glass for the longer wavelength
form (1.3.32), we have
v
p,r
=
2.9979 10
8
m/s
1.3346
v
p,r
2.2463 10
8
m/s
And now nding the phase velocity for the shorter wavelength we do the same calculation using
(1.3.34),
v
p,b
=
2.9979 10
8
m/s
1.3278
v
p,b
2.2578 10
8
m/s
To nd the group velocity, we rst recall that v
g
= d(k)/dk, and also that (k) = ck/n(k).
With this information, as well as the expression given (1.3.30), the generic expression for the group
velocity is
v
g
=
d
dk
_
ck
1.350 100

A k/(2)
_
=
d
dk
_
2ck
21.350 k100

A
_
=2c
_
1
21.350 k100

A
d
dk
(k) + k
d
dk
_
1
21.350 k100

A
__
=2c
_
1
21.350 k100

A
+
k100

A
_
21.350 k100

A

2
_
=
2c
21.350 k100

A
_
1 +
k100

A
21.350 k100

A
_
=
2c
21.350 100

A(2/)
_
1 +
100

A(2/)
21.350 100

A(2/)
_
=
c
1.350 100

A()
1
_
1 +
100

A()
1
1.350 100

A()
1
_
(1.3.37)
And at this point we can now calculate the group velocity given our two frequency components.
Starting with the longer wavelength,
v
r,g
=
2.9979 10
18

A/s
1.350 100

A
_
6500

A
_
1
_
1 +
100

A
_
6500

A
_
1
1.350 100

A
_
6500

A
_
1
_
v
r,g
1.7719 10 m/s
And now for the shorter wavelength,
v
b,g
=
2.9979 10
18

A/s
1.350 100

A
_
4500

A
_
1
_
1 +
100

A
_
4500

A
_
1
1.350 100

A
_
4500

A
_
1
_
v
b,g
1.7902 10 m/s
W. Erbsen HOMEWORK #4
1.4 Homework #4
Problem 12.1
A coaxial wave guide consists of two concentric copper cylinders (conductivity = 0.50 10
18
sec
1
), an in-
ner cylinder of radius A = 0.10 cm, and an outer clinder of radius B = 0.40 cm. The dielectric between the
clinders has a permittivity = 2.0, and permeability = 1.0. The outer cylinder is grounded, and a potential
V
0
= 120 volts (convert this to esu), oscillating at a frequency = 12 MHz, is applied to the inner cylinder.
a) Find the elds E(r, , z, t) and H(r, , z, t) for a wave traveling between the cylinders in the positive z-
direction.
b) Find the power (convert it to Watts) transmitted by this wave.
c) Find the attenuation length for this wave.
Solution
a) We start by solving Laplaces Equation in cylindrical coordinates:

2
T
=
1
r

r
_
r

r
_
+
1
r
2

2
= 0 (1.4.1)
Where we note that due to symmetry, the potential is independent of the angle , and so (1.4.1)
becomes
1
r

r
_
r

r
_
= 0 (r) = C
1
ln r +C
2
(1.4.2)
Our boundary conditions are that the outer conductor is grounded, (b) = 0, and the inner con-
ductor is at some potential (120 V in our case), (a) = V
0
. Applying the rst boundary condition,
(B) = C
1
lnB + C
2
= 0 C
2
= C
1
ln B (1.4.3)
Substituting (1.4.3) into (1.4.2) and simplifying, we have
(r) = C
1
ln(
r
/
B
) (1.4.4)
We now apply the second boundary condition to (1.4.4),
(A) = C
1
ln
_
A
/
B
_
= V
0
C
1
=
V
0
ln(
A
/
B
)
(1.4.5)
And now substitute (1.4.5) into (1.4.4),
(r) =
V
0
ln(
A
/
B
)
ln (
r
/
B
) (1.4.6)
If we take the negative derivative of (1.4.6), we can nd the eld:
E = E =
V
0
ln(
B
/
A
)
1
r
r (1.4.7)
To nd the magnetic eld, we use (8.28) from Jackson, which reads:
CHAPTER 1: ELECTRODYNAMICS II 35
H =

z E
Using (8.28), the magnetic eld becomes
H =

V
0
ln (
B
/
A
)
1
r
z r H =

V
0
ln (
B
/
A
)
1
r

(1.4.8)
We can include the additional space and time dependence of the electric and magnetic elds from
(1.4.7) and (1.4.8), respectively:
E(r, , z, t) =
V
0
ln(
B
/
A
)
1
r
e
i(kzt)
r, H(r, , z, t) =

V
0
ln (
B
/
A
)
1
r
e
i(kzt)

b) The transmitted power can be found by integrating the component of the Poynting vector pointing
in the z-direction over the cross-sectional area:
P =
c
8
_
(E

H) z dA (1.4.9)
Substituting the expressions calculated for the elds in the previous section into (1.4.10), we obtain
P =
c
8
_ __
V
0
ln (
B
/
A
)
1
r
e
i(kzt)
r
_

V
0
ln (
B
/
A
)
1
r
e
i(kzt)

__
z r drd
=
c
8
_
V
0
ln (
B
/
A
)
_
2

_
2
0
d
_
B
A
1
r
dr
=
c
4
_
V
0
ln(
B
/
A
)
_
2

ln
_
A
/
B
_
=
c

4
V
2
0
ln (
B
/
A
)
(1.4.10)
And substituting the appropriate constants into (1.4.10) yields

P =
3 10
8
m/s
_
(1)(2)
4
(0.4 esu)
2
ln(.004/.001)
P = 1.224 10
7
esu
2
m/s
P = 1.224 10
2
Watts m
Problem 12.3
A rectangular wave guide with copper walls has cross section dimensions 4.0 6.0 mm. It is lled with air.
a) Find the rst ve TM cuto frequencies (in Hz) for this wave guide.
b) Sketch nodal planes of E
z
for each of these TM modes.
c) Find the rst ve TE cuto frequencies for this wave guide.
d) Sketch nodal planes of H
z
for each of these TE modes.

Yes, I realize that these numbers are ridiculous.


W. Erbsen HOMEWORK #4
Solution
a) The relation for the TM cuto frequencies in Hz is given by (12.57) in Franklin:
f
m
=
c
2

_

2
A
2
+
m
2
B
2
_
1
/2
(1.4.11)
Where we note that neither nor m can equal zero. Furthermore,

= n = 1, and (1.4.11)
becomes
f
m
=
c
2
_

2
A
2
+
m
2
B
2
_
1
/2
(1.4.12)
Table 1.1: TM
,m
cuto frequencies
m f
m
(Ghz)
1 1 45.07
2 1 62.50
1 2 79.06
3 1 83.85
2 2 90.14
b) See attached plots
c) The expression for the TE cuto frequencies is identical to (1.4.11) except that either or m can
equal zero:
Table 1.2: TE
,m
cuto frequencies
m f
m
(Ghz)
1 0 25.00
0 1 37.50
1 1 45.07
2 0 50.00
2 1 62.50
d) See attached plots
Problem 12.4
a) The magnitude of E
z
at the center of the wave guide in the preceding problem is 6, 000 volts/meter (convert
to esu). Find the power transmitted by this wave guide for a TM wave with a frequency halfway between the
lowest TM cuto frequencies.
CHAPTER 1: ELECTRODYNAMICS II 37
b) Find the attenuation length for this wave.
Solution
a) We know that 1 esu = 300 volt, so 6, 000 volts/meter = 20 esu/meter. Furthermore, the frequency
halfway between the two lowest TM cuto frequencies is f = (62.46 45.04)/2 + 45.04 = 53.75
GHz. We know from (12.75) in Franklin that
P =
kE
2
0
AB
32
2
(1.4.13)
And we also know from class that
k

2

f
f
c
c

_
f
f
c
_
2
1 (1.4.14)
Substituting (1.4.14) into (1.4.13), we have
P =
E
2
0
AB
32
f
f
c
_

_
f
f
c
_
2
1 (1.4.15)
And now substituting the appropriate constants into (1.4.15) with =
0
= 1 and =
0
= 1,
P =
(20 esu/m)
2
(.004 m)(.006 m)
32

53.75 10
9
Hz
45.04 10
9
Hz
3 10
8
m/s

_
53.75 10
9
Hz
45.04 10
9
Hz
_
2
1
P = 2.227 10
4
esu m/s
b) The attenuation is given by (12.86) and (12.87) in Franklin as
L
atten
=
_

c
2
c
_1
/2
_
AB
_

2
B
2
+m
2
A
2
_

2
B
3
+ m
2
A
3
__
1
2
c
/
2

(1.4.16)
Jiggiling (1.4.16) around a little bit and noting that = m = 1,
L
atten
=
_

c
4
_1
/2
_
AB
_
B
2
+A
2
_
B
3
+A
3
_

1 f
2
c
/f
2
f
(1.4.17)
Substituting the appropriate values into (1.4.17) yields
L
atten
=
_
0.50 10
18
sec
1
4
_
1
/2
_
(.004 m)(.006 m)
_
(.006 m)
2
+ (.004 m)
2

(.006 m)
3
+ .(004 m)
3
_

_
1 (45.04)
2
/(53.75)
2
53.75 10
9
Hz
L
atten
= 3.709 m
W. Erbsen HOMEWORK #4
Problem 12.5
a) The magnitude of H
z
at the corner of the wave guide in the preceding problem is 0.20 gauss. Find the power
transmitted by this wave guide for a TE wave with a frequency halfway between the two lowest TE cuto
frequencies.
b) Find the attenuation length for this wave.
Solution
a) As in the previous problem, we nd the cuto frequency halfway between the two lowest TE
frequencies to be (37.47 23.98)/2 +23.98 = 30.73 GHz. The power is given by (12.76) in Franklin
as
P =
kH
2
0
AB
16
2
(1.4.18)
If we substitute (1.4.14) into (1.4.18) and simplify,
P =
H
2
0
AB
16
f
f
c
c

_
f
f
c
_
2
1 (1.4.19)
And substituting the appropriate values into (1.4.19) yields
P =
(.20 gauss)
2
(.004 m)(.006 m)
16

30.73 10
9
Hz
24.98 10
9
Hz
3 10
8
m/s

_
30.73 10
9
Hz
24.98 10
9
Hz
_
2
1
P = 5.047 gauss m/s
b) The attenuation length can be found from (12.86) and (12.88):
L
atten
=
_

c
2
c
_1
/2
_
AB
_

2
B
2
+m
2
A
2
_
AB(
2
B + m
2
A) + (
c
/)
2
(
2
B
3
+m
2
A
3
)
__
1
2
c
/
2

(1.4.20)
Getting rid of the appropriate constants and recalling that for the lowest cuto frequency = 1 and
m = 0, (1.4.21) becomes
L
atten
=
_

c
4
_1
/2
_
AB
A/2 + B(f
c
/f)
2
_

1 f
2
c
/f
2
f
(1.4.21)
And substituting the appropriate constants into (1.4.21),
L
atten
=
_
0.50 10
18
sec
1
4
_
1
/2
_
(.004 m)(.006 m)
(.004 m)/2 + (.006 m)(24.98/30.73)
2
_
_
1 24.98
2
/30.73
2
30.73 10
9
GHz
L
atten
= 7.085 m
Problem 12.7
CHAPTER 1: ELECTRODYNAMICS II 39
A circular wave guide with copper walls has a radius R = 4.0 mm. It is lled with air.
a) Find the rst ve TM cuto frequencies (in Hz) for this wave guide.
b) Sketch nodal surfaces of E
z
for each of these TM modes.
c) Find the rst ve TE cuto frequencies for this wave guide.
d) Sketch nodal surfaces of H
z
for each of these TE modes.
Solution
a) An expression for the TM cuto frequency can be found from (12.68) in Franklin:
f
m
=
c
2
j
m
R
(1.4.22)
Substituting the values from Table 12.2 from Franklin into (1.4.22) yields
Table 1.3: TM
,m
cuto frequencies
m f
m
(Ghz)
0 1 28.71
1 1 45.74
2 2 61.30
0 2 65.89
1 2 83.74
b) See attached plots
c) Using (12.69) we can nd the cuto frequency:
f
m
=
c
2
j

m
R
(1.4.23)
And using the values from Table 12.2 we can evaluate (1.4.23) for the desired modes:
Table 1.4: TE
,m
cuto frequencies
m f
m
(Ghz)
1 1 21.98
2 1 36.46
0 2 45.74
1 2 63.64
2 2 80.00
W. Erbsen HOMEWORK #5
d) See attached plots
Problem 12.8
The magnitude of E
z
at the center of the wave guide in the preceding problem is 6, 000 volts/meter (convert
to esu). Find the power transmitted by this wave guide for a TM wave with frequencies halfway between the two
lowest TM cuto frequencies.
Solution
As before, 6, 000 volts/meter = 20 esu/meter. Additionally, the frequency halfway between the two
lowest TM cuto frequencies is f = (45.74 28.71)/2 + 28.71 = 37.23 GHz. We know from (12.79) in
Franklin that
P =
kR
2
E
2
0
16
2
m
[J

m
(
m
R)]
2
(1.4.24)
To nd the derivative of the m
th
Bessel function we can use a recursion relation. Using Mathematica,
we nd that J

m
(
m
R) = 0.5192, and (1.4.24) becomes
P =

2
(37.23 GHz)
2
(.004 m)
2
(20 esu/m)
2
4(3 10
8
m/s)(2.4048/.004 m)
2
[0.5192]
2
P = 5.439 10
4
esu m/s
1.5 Homework #5
Problem 12.10
A rectangular cavity (with and = 1) has dimensions 2.0 3.0 4.0 cm.
a) Find the rst six resonant frequencies of this cavity, and identify any degenerate modes.
b) Write down the E and H elds for the lowest frequency resonant mode.
Solution
We rst recall that for a TM wave, E
z
must vanish at the walls, and according to (12.107) in Franklin,
E
z
= E
0
sin
_
lx
A
_
sin
_
my
B
_
cos
_
nz
C
_
[TM mode] (1.5.1)
CHAPTER 1: ELECTRODYNAMICS II 41
From which we can see that only n may be zero for a TM wave. Similarly, for a TE wave H
z
must vanish
at the walls, and from (12.108),
H
z
= H
0
cos
_
lx
A
_
cos
_
my
B
_
sin
_
nz
C
_
[TE mode] (1.5.2)
We can gather from this that both l and m can be zero for a TE wave. For both TM and TE waves, the
resonant frequency is given by (12.109),
f
lmn
=
c
2

_
_
l
A
_
2
+
_
m
B
_
2
+
_
n
C
_
2
_1
/2
(1.5.3)
a) We can nd the rst six resonant frequencies for the cavity by evaluating (1.5.3) for a variety of
dierent modes and nding the lowest six:
Resonant frequencies
l m n f
lmn
(Ghz) TM/TE d
0 1 1 6.250 TE 1
1 0 1 8.350 TE 1
1 1 0 9.014 TM 2
0 1 2 9.014 TE 2
1 1 1 9.768 TE 1
1 0 2 10.61 TE 1
0 2 1 10.68 TE 1
We note that both l and m cannot both be zero, otherwise all transverse components of E and H
will vanish, and we will not have a standing wave.
b) The eld components are just given by (12.63)-(12.65), if we allow A > B:
H
z
(x, y) =H
0
cos
_
x
A
_
H
z
(x, y) =
ikA

H
0
sin
_
x
A
_
E
y
(x, y) =
iA
c
H
0
sin
_
x
A
_
Problem 12.11
A cubical cavity of dimensions 2.0 2.0 2.0 cm has copper walls. Find the fundamental frequency and Q
value for that frequency.
Solution
W. Erbsen HOMEWORK #5
From (1.5.3),
f
lmn
=
c
2

_
_
l
.02 m
_
2
+
_
m
.02 m
_
2
+
_
n
0.02 m
_
2
_1
/2
f
110
= f
101
= f
011
= 10.61 GHz
The derivation for the expression for the Q value is found in the next problem, and is given by (1.5.21)
as
Q =
4L
c

f
c

c
(1.5.4)
Where L is the length of each side of the cavity. The conductivity of copper is given by
c
= 5.96
10
17

1
m, while we let
c
= 1. Accordingly, (1.5.4) leads to Q 0.2107 .
Problem 12.12
And they shall make an ark of acacia-wood: two cubits and a half shall be the length thereof, and a cubit
and a half the breadth thereof, and a cubit and a half the height thereof. And thou shalt overlay it with pure
gold. (Exodus XXV, 10:11). Find the fundamental resonant frequency and a half width for this biblical cavity.
Use the length from your ngertip to your elbow as a measure of a biblical cubit.
Solution
The length from my elbow to my beefy ngertip is 0.5 m, so the dimensions of our cavity are A = 1.25
m, B = 0.75 m, and C = 0.75 m. The fundamental frequency may be readily deduced from (1.5.3):
f
lmn
=
c
2

_
_
l
1.25 m
_
2
+
_
m
0.75 m
_
2
+
_
n
0.75 m
_
2
_1
/2
f
110
= f
101
= 0.2330 GHz
Which is clearly degenerate. We can see that f
110
corresponds to a TM mode, while f
101
corresponds to
a TE mode. The half width is found by dividing (12.131) by a factor of 2,
=

0
2Q
(1.5.5)
While the Q factor is given by (12.125) as
Q =
U
dU/dt
(1.5.6)
Where we know that
dU
dt
=
c
8
_

c
8
c
_
[H
surface
[
2
dA (1.5.7)
And if we recall from (1.5.1) and (1.5.2),
CHAPTER 1: ELECTRODYNAMICS II 43
E
z
=E
0
sin
_
lx
A
_
sin
_
my
B
_
cos
_
nz
C
_
(1.5.8)
H
z
=H
0
cos
_
lx
A
_
cos
_
my
B
_
sin
_
nz
C
_
(1.5.9)
For the TM mode, we know that l = 1, m = 1 and n = 0, and (1.5.8) becomes
E
z
= E
0
sin
_
x
A
_
sin
_
y
B
_
(1.5.10)
And similarly, for the TE mode, we recall that l = 1, m = 0 and l = 1, and (1.5.9) becomes
H
z
= H
0
cos
_
x
A
_
sin
_
z
C
_
(1.5.11)
We know that H
surface
will be a linear combination of both elds, so we can go ahead and evaluate the
modulus squared:
[H
surface
[
2
=
_
E
0
sin
_
x
A
_
sin
_
y
B
_
+ H
0
cos
_
x
A
_
sin
_
z
C
__
2
=E
2
0
sin
2
_
x
A
_
sin
2
_
y
B
_
. .

I
+H
2
0
cos
2
_
x
A
_
sin
2
_
z
C
_
. .

II
+ 2E
0
H
0
sin
_
x
A
_
sin
_
y
B
_
cos
_
x
A
_
sin
_
z
C
_
. .

III
=E
2
0

I
+H
2
0

II
+ 2E
0
H
0

III
(1.5.12)
Instead of directly substituting (1.5.12) into (1.5.7), we can go ahead and integrate
I
,
II
and
III
separately over space and then combine them later to avoid some of the mess. We rst recall the following
nifty trig identities:
sin
2
x =
1
2
(1 cos(2x)) cos
2
x =
1
2
(1 + cos(2x))
Using these, we rst evaluate
I
,
_

I
dA =
_
A
0
sin
2
_
x
A
_
dx
_
B
0
sin
2
_
y
B
_
dy
=
1
4
_
A
0
_
1 cos
_
2x
A
__
dx
_
B
0
_
1 cos
_
2y
B
__
dy
=
1
4
__
_
A
0
dx
_
A
0
cos
_
2x
A
_
dx
_

_
_
B
0
dy
_
B
0
cos
_
2y
B
_
dy
__
=
1
4
__
A
A
2
_
sin
_
2x
A
_

A
0
_

_
B
B
2
_
sin
_
2y
B
_

B
0
__
=
AB
4
(1.5.13)
And now doing the same for
II
,
W. Erbsen HOMEWORK #5
_

II
dA =
_
A
0
cos
2
_
x
A
_
dx
_
C
0
sin
2
_
z
C
_
dz
=
1
4
_
_
A
0
_
1 + cos
_
2x
A
__
dx
_
C
0
_
1 cos
_
2z
C
__
dz
_
=
1
4
__
A +
_
A
0
cos
_
2x
A
_
dx
_

_
C
_
C
0
cos
_
2z
C
_
dz
__
=
1
4
__
A +
A
2
_
sin
_
2x
A
_

A
0
_

_
C
C
2
_
sin
_
2z
C
_

C
0
__
=
AC
4
(1.5.14)
And now for
III
,
_

III
dA =
_
A
0
sin
_
x
A
_
cos
_
x
A
_
dx
. .
=0

_
B
0
sin
_
y
B
_
dy
_
C
0
sin
_
z
C
_
dz = 0 (1.5.15)
Using (1.5.13)-(1.5.15), we can now evaluate (1.5.7):
dU
dt
=
c
8
_

c
8
c
_
E
2
0
AB
4
+ H
2
0
AC
4
_
=
c
64
_
f
c

c
A
_
E
2
0
B + H
2
0
C

(1.5.16)
We now nd the energy within the cavity, U, using (12.121):
U =
_
E
2
0
+ H
2
0
_
ABC
32
_
(l/A)
2
+ (m/B)
2
+ (n/C)
2
(l/A)
2
+ (m/B)
2
_
(1.5.17)
If we take the frequency for the TM mode we let l = 1, m = 1, and n = 0, and (1.5.17) becomes
U =
_
E
2
0
+H
2
0
_
ABC
32
(1.5.18)
We now recall that = = 1 and B = C, so (1.5.18) becomes
U =
_
E
2
0
+ H
2
0
_
AB
2
32
(1.5.19)
Making the same assumptions for (1.5.16) yields
dU
dt
=
c
64
_
f
c

c
AB
_
E
2
0
+H
2
0

(1.5.20)
Now substituting (1.5.19) and (1.5.20) into (1.5.6) yields
Q = 2f
AB
2
32
_
E
2
0
+H
2
0
_

64
c
_

c
f
c
1
AB(E
2
0
+ H
2
0
)
CHAPTER 1: ELECTRODYNAMICS II 45
=
4B
c

f
c

c
(1.5.21)
The conductivity of gold can be found on p. 286 of Griths to be
c
= 4.525 10
7

1
m. Taking

c
= 1, (1.5.21) becomes
Q =
4(0.75 m)
(3 10
8
m/s)
_
(0.233 10
9
Hz)(4.525 10
7

1
m Q 3.226 (1.5.22)
Substituting (1.5.22) into (1.5.5),
=
f
0
Q
= 0.2269 GHz
It is clear that this answer does not make sense, if we compare it to the resonant frequency found in
the rst part of the problem. I repeated the latter part using Mathematica, which yielded a half width
of = .03612 GHz . Please see attached Mathematica printout for details.
Problem 12.13
A circular cavity (with and = 1) has a radius R = 2.0 cm, and a length L = 3.0 cm.
a) Find the rst six resonant frequencies of this cavity, and identify the type (TM or TE) of mode.
b) Sketch nodal surfaces for each of these six modes.
Solution
a) The relation for the resonant frequencies for both TM and TE modes is given by (12.112),
f
lmn
=
c
2

_
_
j
ml
R
_
2
+
_
n
L
_
2
_1
/2
[TM mode] (1.5.23)
f

lmn
=
c
2

_
_
j

ml
R
_
2
+
_
n
L
_
2
_1
/2
[TE mode] (1.5.24)
The calculated values are displayed in the following table (next page):
b) Please see attached plots.
W. Erbsen HOMEWORK #6
Resonant frequencies
l m n f
lmn
(Ghz) TM/TE
1 0 1 5.000 TE
1 1 1 6.650 TE
1 1 0 9.148 TM
1 0 2 10.00 TE
1 1 1 10.43 TM
1 2 0 12.26 TM
1.6 Homework #6
Problem 13.3
A thin linear antenna of length L carries an oscillating current
I(z, t) = I
0
sin
_
2[z[
L
_
e
it
, [z[ <
L
2
(1.6.1)
a) Sketch this current as a function of z.
b) Find the radiation pattern for this antenna. Sketch the angular distribution and plot it as a function of cos .
Plot the pattern for kL = and kL = 2.
Solution
a) Please see attached plot (at the end). Values chosen to emphasize oscillations.
b) Following Franklins lead in section (13.5), we recall that the eective radiation current is given by
(13.39) as
J(kr) = k
_
j (r

) e
ikr

r
d
3
r

(1.6.2)
For the current case, we have a linear antenna, which is oriented along the z-direction, and (1.6.2)
becomes
J (kz) = k
_
I(z, t)e
ikz cos
dz (1.6.3)
Substituting (1.6.1) into (1.6.3) yields
J(kz) = k
_
L/2
L/2
_
I
0
sin
_
2[z[
L
_
e
it
_
e
ikz cos
dz (1.6.4)
At this point, we must separate [z[ into the positive and negative parts, and (1.6.4) becomes
J(kz) =kI
0
e
it
_
_
L/2
0
sin
_
2[z[
L
_
e
ik|z| cos
dz +
_
0
L/2
sin
_

2[z[
L
_
e
ik|z| cos
dz
_
=kI
0
e
it
_
_
L/2
0
sin(az)e
ibz
dz
. .

_
0
L/2
sin(az)e
ibz
dz
. .

II
_
(1.6.5)
CHAPTER 1: ELECTRODYNAMICS II 47
Where I have set a = 2/L and b = k cos , and also let [z[ z for notational convenience. We
now recall the following trig integral:
_
sin(ax) cos(bx) dx =
cos [(a b)x]
2(a b)

cos [(a + b)x]
2(a + b)
(1.6.6a)
_
sin(ax) sin(bx) dx =
sin [(a b)x]
2(a b)

sin[(a + b)x]
2(a +b)
(1.6.6b)
We now begin by evaluating
I
in (1.6.5) using Eulers identity:

I
=
_
L/2
0
sin(az) [cos(bz) i sin(bz)] dz
=
_
L/2
0
sin(az) cos(bz) dz
. .

Ia
i
_
L/2
0
sin(az) sin(bz) dz
. .

Ib
(1.6.7)
We now apply (1.6.6a) to
Ia
in (1.6.7),

Ia
=
_

cos [(a b)z]


2(a b)

cos [(a +b)z]
2(a + b)

L/2
0
=
cos [(a b)L/2]
2(a b)

cos [(a + b)L/2]
2(a + b)
+
1
2(a b)
+
1
2(a +b)
=
a cos(aL/2) cos(bL/2) b sin(aL/2) sin(bL/2)
a
2
b
2
+
a
a
2
+ b
2
=
4Lcos
2
[(kL/4) cos ]
4
2
(kL)
2
cos
2

(1.6.8)
Now apply (1.6.6b) to
Ib
in (1.6.7),

Ib
=
_
sin [(a b)x]
2(a b)

sin [(a +b)x]
2(a + b)

L/2
0
=
sin [(a b)L/2]
2(a b)

sin [(a +b)L/2]
2(a + b)
=
b cos(bL/2) sin(aL/2) a cos(aL/2) sin(bL/2)
a
2
b
2
=
2Lsin [(kL/2) cos ]
4
2
(kL)
2
cos
2

(1.6.9)
Combining (1.6.8) and (1.6.9),

I
=
Ia
i
Ib
=
2L1 + cos [(kL/2) cos ] i sin [(kL/2) cos ]
4
2
(kL)
2
cos
2

=
2L
_
1 +e
i(kL/2) cos
_
4
2
(kL)
2
cos
2

(1.6.10)
Fantastic. If we repeat the same process for
II
in (1.6.5), which yields

II
=
2L1 cos [(kL/2) cos ] + i sin [(kL/2) cos ]
4
2
(kL)
2
cos
2

W. Erbsen HOMEWORK #6
=
2L
_
1 + e
i(kL/2) cos
_
4
2
(kL)
2
cos
2

(1.6.11)
At this point, we substitute
I
from (1.6.10) and
II
from (1.6.11) into (1.6.5),
J(kz) =kI
0
e
it

I
+
II

=kI
0
e
it
_
2Lsin [(kL/2) cos ]
4
2
(kL)
2
cos
2

+
2L
_
1 + e
i(kL/2) cos
_
4
2
(kL)
2
cos
2

_
=
4kL
_
1 +e
i(kL/2) cos
_
4
2
(kL)
2
cos
2

(1.6.12)
And we recall that the angular distribution of the spherical wave is given by (13.50) as
dP
d
=
1
8c
[J (kz)[
2
(1.6.13)
Substituting (1.6.12) into (1.6.13),
dP
d
=
1
8c
__
4kL
_
1 +e
i(kL/2) cos
_
4
2
(kL)
2
cos
2

__
4kL
_
1 +e
i(kL/2) cos
_
4
2
(kL)
2
cos
2

__
=
1
8c
_
64
2
(kL)
2
cos [(kL/4) cos ]
2
[4
2
+ (kL)
2
cos
2
]
2
_
=
8 (kL)
2
c
cos [(kL/4) cos ]
2
[4
2
+ (kL)
2
cos
2
]
2
(1.6.14)
Taking the transverse component squared of (1.6.14) leads us to
dP
d
=
8 (kL)
2
c
cos [(kL/4) cos ]
2
[4
2
+ (kL)
2
cos
2
]
2
sin
2

dP
d
=
8 (kL)
2
c
cos [(kL/4) cos ]
2
[4
2
+ (kL)
2
cos
2
]
2
_
1 cos
2

_
(1.6.15)
The function in (1.6.15) has been plotted both as a function of cos in cartesian coordinates, and
also with respect to in polar coordinates. Both of these have been done both for kL = and
kL = 2. Please see attached plots (at the very end). We can go ahead and evaluate (1.6.15) for
both of these values:
dP
d
=
8
c
cos [(/4) cos ]
2
[4 + cos
2
]
2
_
1 cos
2

_
[kL=]
dP
d
=
2
c
cos [(/2) cos ]
2
[1 + cos
2
]
2
_
1 cos
2

_
[kL=2]
CHAPTER 1: ELECTRODYNAMICS II 49
Problem 13.4
For the antenna in the previous problem, nd the maximum dP/d, and the angle for which this occurs.
Solution
The best way to nd the maximum dP/d is to nd where the rst derivative of the relevant function
is zero in the cos -domain. Specically, we start with (1.6.15), which reads
dP
d
=
8 (kL)
2
c
cos [(kL/4) cos ]
2
[4
2
+ (kL)
2
cos
2
]
2
_
1 cos
2

_
(1.6.16)
We now let cos x in (1.6.16), which leads to
dP
d
=
8 (kL)
2
c
cos [(kL/4) x]
2
[4
2
+ (kL)
2
x
2
]
2
(1 x) (1.6.17)
We must expand the trig term in the exponent of (1.6.17):
cos [(kL/4) x]
2
= 1
(kL)
2
x
2
32
+
(kL)
4
x
4
6144

(kL)
6
x
6
2949120
+ ... (1.6.18)
Clearly this converges very quickly, so we take the rst two terms only and substitute it into (1.6.17):
dP
d
=
8 (kL)
2
c
[(kL)
2
x
2
/32]
2
[4
2
+ (kL)
2
x
2
]
2
(1 x) (1.6.19)
The name of the game is then to take the rst derivative of (1.6.19), set it equal to zero, and solve for
x. The values should represent the maxima/minima:
cos =3.179 [kL=] (1.6.20a)
cos = [kL=2] (1.6.20b)
The maximum value is found by evaluating (1.6.19) at these points:
(dP/d)
max
=1.7980 [kL=]
(dP/d)
max
=2.1247 [kL=2]
Please see attached Mathematica printout for the details of this calculation.
Problem 13.5
Evaluate the lifetime of a Rutherford hydrogen atom from (13.84). Use as data: R = .53

A, mc
2
= 511 KeV, and
the binding energy of hydrogen is 13.6 eV.
Solution
W. Erbsen HOMEWORK #6
We begin with (13.84), which I will convert to SI units, in part by following the same logic as in the
following problem.
T =
R
4c
_
mc
2
e
2
/R
_
2
=
cR
4
_
mc
e
2
/R
_
2
=
cR
3
4
_
mc
e
2
_
2
=cR
3
_
2
0
mc
e
2
_
2
(1.6.21)
The appropriate constants are
c =2.998 10
10
m/s
e =1.6023 10
10
C
R =0.529 10
10
m
m =9.109 10
31
Kg

0
=8.854 10
12
F/m
Plugging all of these into (1.6.21), we have
T =
_
2.998 10
10
m/s
_ _
0.529 10
10
m
_
3

_
2
_
8.854 10
12
F/m
_ _
9.109 10
31
Kg
_ _
2.998 10
10
m/s
_
(1.6023 10
10
C)
2
_
2
T 1.322 10
11
sec
Problem 13.6
Consider a model of the hydrogen atom as a heavy positive proton, surrounded by a sphere of radius R, with
a uniform charge and mass density, having a total charge and mass of an electron.
a) Show that the angular frequency of oscillation for this atom (with amplitude A R) is
=
_
(e
2
/R)
mc
2
_
1
/2
c
R
(1.6.22)
b) Evaluate and the corresponding wavelength of radiation. (Use numerical values from the previous problem).
c) Find the oscillating electric dipole moment of this atom.
d) Find the total power radiated by this atom for A
0
= R.
CHAPTER 1: ELECTRODYNAMICS II 51
e) Find the lifetime for exponential decay of the oscillation. Evaluate the lifetime in seconds.
Solution
a) We begin by equating Newtons second law with Coulombs law,
F =
mv
2
r
= m
2
r =
1
4
0
e
2
r
2

2
r
3
=
1
4
0
e
2
m
(1.6.23)
The energy as a function of r is
E =T +V
=
mv
2
r

1
4
0
e
2
r
=
1
2
_
1
4
0

2
r
_

1
4
0
e
2
r
=
1
8
0
e
2
r
(1.6.24)
To nd the power loss we take the rst derivative of (1.6.24),
dE
dt
=
1
8
0
e
2
r
2
dr
dt
(1.6.25)
The Lamor formula for radiation caused by an accelerating charge is given by (13.94)
P =
2e
2
[a[
2
3c
3
(1.6.26)
If we recognize that the power radiated can be thought of as the power lost, or the rate of energy
leaving as a function of time, then we can say that:
dE
dt
=
2e
2
[a[
2
3c
3
(1.6.27)
We now substitute in the acceleration for a charge circulating the proton,
dE
dt
=
2e
2
_

2
r
_
2
3c
3
(1.6.28)
Substitute (1.6.23) into (1.6.28),
dE
dt
=
2e
2
3c
3
_
1
4
0
e
2
m
_
2
(1.6.29)
Equating (1.6.25) to (1.6.29),
1
8
0
e
2
r
2
dr
dt
=
2e
2
3c
3
_
1
4
0
e
2
m
_
2

dr
dt
=
1
4
0

4e
4
3m
2
c
3
T = 4
0

3m
2
c
3
4e
4
_
0
R
1
r
2
dr
T = 4
0

m
2
c
3
4e
4
1
R
3
W. Erbsen HOMEWORK #6
T = 4
0
R
4c
_
mc
2
e
2
/R
_
2
(1.6.30)
If we drop the factor of 4
0
, then (1.6.30) is the same as (13.84). Furthermore, we arrive at (13.161)
by evaluating (1.6.23) at r = R:

2
R
3
=
1
4
0
e
2
m
=
c
R
_
1
4
0
e
2
/R
mc
2
_
(1.6.31)
If we let 4
0
= 1 in (1.6.31), we are left with
=
c
R
_
e
2
/R
mc
2
_
(1.6.32)
Which is identical to (1.6.22).
b) To evaluate , we simply use (1.6.23) and use the data from the previous problem:
=
_
1
4
0
e
2
m
1
R
3
_
1
/2
=
_
1
4 (8.854 10
12
F/m)
_
1.6023 10
10
C
_
2
(9.109 10
31
Kg)
1
(0.529 10
10
m)
3
_
1
/2
(1.6.33)
4.496 10
16
rad
sec
(1.6.34)
The wavelength of radiation will be given by:
=
2c

=
2
_
2.998 10
10
m/s
_
4.496 Hz
4.193 10
8
m
c) The oscillating electric dipole moment is just the electric dipole moment, multiplied by a sinusoidal
time dependent frequency part:
p(t) =ed cos (t)
Where d in our case is equal to the separation of charge:
p(t) =eRcos (t) z (1.6.35)
Substituting in the appropriate values,
p(t) =
_
1.6023 10
10
C
_ _
0.529 10
10
m
_
cos (t) z p(t) =
_
8 10
30
C m
_
cos (t) z
Where is of course given by (1.6.33).
d) The total power radiated is just (1.6.28):
P =
2e
2
_

2
r
_
2
3c
3
Putting in the numbers,
P =
2
_
1.6023 10
10
C
_
2
_
4.496 10
16
rad/sec
_
4
_
0.529 10
10
m
_
2
3 (2.998 10
10
m/s)
3
P = 1.291 10
7
Watts
CHAPTER 1: ELECTRODYNAMICS II 53
Problem 13.7
Find and sketch the absolute square of the Fourier transform of the step function in (13.95), and estimate its
spread in . (Use the rst zero of the transform to estimate)
Solution
The step function in (13.95) reads
(x, t) =
_
q/L, [x vt[ < L/2
0, [x vt[ > L/2
(1.6.36)
And now we recall that the functional form of the integral Fourier transform is
f(x, t) =
_
f(, t)e
ikx
dk
A(k, t) =
1
2
_
f(x, t)e
ikx
dx (1.6.37)
Where the integrals are to be taken over the appropriate range. In our case, we substitute (1.6.36) into
(1.6.37), which yields
A(k, t) =
1
2
_
L/2
L/2
_
q
L
_
e
ikx
dx
=
1
2
_
q
L
_
_

e
ikx
ik

L/2
L/2
=
1
2
_
q
L
_
_
e
ikL/2
+ e
ikL/2
ik
_
=
q

sin(kL/2)
kL
(1.6.38)
Taking the absolute square of (1.6.38) leads to
A(k, t) =
q
2

2
sin
2
(kL/2)
(kL)
2
(1.6.39)
In order to estimate the spread of (1.6.39) in , we evaluate the function at the rst zero:
q
2

2
sin
2
(kL/2)
(kL)
2
= 0 kL = 2
L

= 1 = 2L
W. Erbsen HOMEWORK #7
1.7 Homework #7
Problem 13.8
A beam of 2 KeV electrons is stopped in a distance of 0.01 cm.
a) Calculate the total energy of the radiation emitted by each electron (assume constant acceleration).
b) Find the ratio of the emitted energy to the initial electron energy. Use this ratio to nd the radiated power
for an x-ray machine with 300 Watts of power input to accelerate the electrons.
c) Calculate the maximum intensity of the radiation at a distance of 20 cm from the stopping target.
Solution
a) To nd the total energy radiated by each electron, we rst convert the initial kinetic energy of the
electrons (T) from eV to Joules:
T = 2 10
3
eV
1.602 10
19
J
1 eV
T = 3.204 10
16
J
We now recall that the kinetic is given by
T =
1
/
2
mv
2
v =
_
2T
m
(1.7.1)
And, since we are told that the acceleration is constant, we can use one of the kinematic equations
of motion:
v
2
= v
2
o
+ 2ax a =
v
2
o
2x
(1.7.2)
If we take (1.7.1) and substitute it into (1.7.2) and put in the appropriate values,
a =
2T/m
2x

T
mx

_
3.204 10
16
J
_
(9.109 10
31
kg) (.01 10
2
m)
a = 3.518 10
18
m/s
2
We can now nd the power from the following expressions:
P =
2e
2
[a[
2
3c
3
[CGS] (1.7.3a)
P =
e
2
[a[
2
6
0
c
3
[SI] (1.7.3b)
Where (1.7.3a) comes from (13.94) in Franklin, and (1.7.3b) comes from Wikipedia. Doing the
calculation in SI units rst,
P =
(1.602 10
19
C)
2
(3.518 10
18
m/s
2
)
2
6(8.854 10
12
F/m)(2.998 10
8
m/s)
3
P = 7.063 10
17
Watts (1.7.4)
And doing the same in CGS units with (1.7.3a),
P =
24.803 10
10
StatC)
2
(3.518 10
20
cm/s
2
)
2
3(2.998 10
10
cm/s)
3
P = 7.063 10
10
erg/s (1.7.5)
CHAPTER 1: ELECTRODYNAMICS II 55
In order to nd the energy emitted, we must nd the time over which this happens. We again turn
to the kinematic equations of motion:
v = v
o
+at t =
v
o
a
Substituting in the expression for v
o
from (1.7.1) into this yields
t =
_
2T/m
a


_
(2) (3.204 10
16
J) / (9.109 10
31
kg)
3.518 10
18
m/s
2
t = 7.540 10
12
s
Now, if we recall that [power]=[energy]/[time], then we can nd the energy by multiplying our
expressions for power from (1.7.4) and (1.7.5), respectively by the time we just calculated:
E =
_
7.063 10
17
Watts
_ _
7.540 10
12
s
_

E = 5.326 10
28
J
= 3.324 10
9
eV
[SI]
E =
_
7.063 10
10
erg/s
_ _
7.540 10
12
s
_
E = 5.326 10
21
erg [CGS]
b) We nd the ratio of the energy emitted to the incident energy (T

/T) simply by taking the specied


ratio from the energy found in the last problem (T

here is the same as E in the last part):


T

T
=
3.324 10
9
eV
2 10
3
eV

T

T
= 1.662 10
12
(1.7.6)
We can use this ratio to nd the power emitted (P

) given the incident power as follows:


P

P
=
T

T
= 1.662 10
12
P

= P 1.662 10
12
Substituting in the appropriate values,
P

=(300 Watts)
_
1.662 10
12
_

P

= 4.986 10
10
Watts
= 3.112 10
9
eV/s
[SI]
P

=
_
4.986 10
10
Watts
_ _
10
7
erg/J
_
P

= 4.986 10
10
erg/s [CGS]
c) To nd the maximum intensity at a given distance, we start with the expressions for the radiation
pattern:
dP
d
=
e
2
[a[
2
8
2

o
c
3
sin
2
[SI] (1.7.7a)
dP
d
=
e
2
[a[
2
4c
3
sin
2
[CGS] (1.7.7b)
Where (1.7.7b) is (13.93) from Franklin, and (1.7.7a) was deduced from (11.69) from Griths. We
also recall from (13.50) that
dP
d
= r
2
r S S =
1
r
2
dP
d
(1.7.8)
Starting in SI units rst, we substitute (1.7.7a) into (1.7.8), which yields
S =
e
2
[a[
2
8
2

0
c
3
1
r
2
sin
2
(1.7.9)
W. Erbsen HOMEWORK #7
The maximum intensity will occur when = /2, so that sin
2
1. Also, we recall that the
intensity is really just the time-averaged Poynting vector, so we include a factor of
1
/
2
into our
expressions for the Poynting vector to get the intensity:
[S[ =
e
2
[a[
2
16
2

o
c
3
1
r
2

_
1.602 10
19
C
_
2
_
3.518 10
18
m/s
2
_
2
16
2
(8.854 10
12
F/m) (2.998 10
8
m/s)
3
1
(20 10
2
m)
2

[S[ = 2.108 10
16
Watts/m
2
= 1.316 10
3
eV
[SI]
Similarly, to nd the maximum intensity in CGS units we substitute (1.7.7b) into (1.7.8):
[S[ =
e
2
[a[
2
8c
3
1
r
2
=
_
4.803 10
10
StatC
_
2
(3.518 10
20
cm/s
2
)
2
8 (2.998 10
10
cm/s)
3
1
(20 cm)
2
[S[ = 1.054 10
13
erg/(cm
2
s) [CGS]
Problem 13.10
a) Find the quadrupole moment (Q
0
) of the antenna in problem 13.3.
b) Find the quadrupole radiation pattern. Sketch the angular distribution, and compare it to the exact distri-
bution.
c) Find the maximum intensity and the angle for which it occurs for the quadrupole, and compare these to the
exact results.
d) Find the total quadrupole power radiated and compare to the exact power (evaluate the integral numerically).
Solution
a) We rst recall the relation for the electric quadrupole dyadic from (13.114) of Franklin:
[Q] =
1
2
_
_
3r r r
2

n
_
d
3
r (1.7.10)
We can expand (1.7.10) in equation form as follows:
[Q] =
1
2
_
_
_
3x
2
r
2
3xy 3xz
3yx 3y
2
r
2
3yz
3zx 3zy 3z
2
r
2
_
_
d
3
r (1.7.11)
The current density, , is non-zero only for the z-component, the other components are zero ((x) =
(y) = 0). Therefore, when we expand r
2
= x
2
+y
2
+ z
2
, only the z-component remains. For this
same reasoning all of the odiagonal elements in (1.7.11) go to zero. Therefore, (1.7.11) becomes
[Q] =
1
2
_
_
_
z
2
0 0
0 z
2
0
0 0 2z
2
_
_
dz [Q] =
_
_
_
1
/
2
0 0
0
1
/
2
0
0 0 1
_
_
z
2
dz
CHAPTER 1: ELECTRODYNAMICS II 57
The matrix can be taken out of the integral, and we nd that
[Q] =
_
_
1
/
2
0 0
0
1
/
2
0
0 0 1
_
_
_
z
2
dz
. .
Q
0
(1.7.12)
Where I have bracketed the portion of (1.7.12) that represents Q
0
. To nd Q
0
, we must rst nd
the charge density, . To do this we begin with the continuity equation:
j =

t
=
_
t
0
j dt (1.7.13)
Where the charge density j is really a linear charge density, and can be found by expressed in
terms of the current as
j =
I
L
Substituting this in to (1.7.13),
=
1
L
_
t
0
I dt
=
1
L
_
t
0

z
I(z, t) dt
=
1
L
_
t
0

z
_
I
o
sin
_
2[z[
L
_
e
it
_
dt
=
2
L
2
I
o
cos
_
2[z[
L
__
t
0
e
it
dt
_ _
take only the real part, or
else will be complex!
_
=
2
L
2
I
o
cos
_
2[z[
L
__
t
0
cos(t) dt
=
2
L
2
I
o
cos
_
2[z[
L
__
sin(t)

t
0
=
2
L
2
I
o
cos
_
2[z[
L
_
sin(t) (1.7.14)
Hooray! We can now nd the quadrupole moment by substituting (1.7.14) into (1.7.12):
Q
0
=
_
z
2
dz
=
2
L
2
I
o
sin(t)
_
L/2
L/2
z
2
cos
_
2[z[
L
_
dz
=
2
L
2
I
o
sin(t)
_
_
L/2
0
z
2
cos
_
2[z[
L
_
dz +
_
0
L/2
z
2
cos
_
2[z[
L
_
dz
_
=
4
L
2
I
o
sin(t)
_
L/2
0
z
2
cos
_
2z
L
_
dz (1.7.15)
Where the last step is completed by realizing that the integral is symmetric. We can integrate
(1.7.15) by repeated use of integration by parts:
W. Erbsen HOMEWORK #7
I =
_
L/2
0
z
2
cos
_
2z
L
_
dz
_
u = z
2
du = 2z
dv = cos(2z/L) v = (L/2) sin(2z/L)
=
_
L
2
z
2
sin
_
2z
L
_

L/2
0

_
L/2
0
z sin
_
2z
L
_
dz
=
L
2

_
L
2
_
2
sin
_
2
L

L
2
_

_
L/2
0
z sin
_
2z
L
_
dz
=
L
3
8
sin()
L

_
L/2
0
z sin
_
2z
L
_
dz
=
L

_
L/2
0
z sin
_
2z
L
_
dz
_
u = z du = 1
dv = sin(2z/L) v =

(L/2) cos(2z/L)
=
L

_
_

L
2
z cos
_
2z
L
_

L/2
0
+
L
2
_
L/2
0
cos
_
2z
L
_
dz
_
=
L

L
2

L
2
cos
_
2
L

L
2
_
+
L
2
_
L
2
sin
_
2z
L
_

L/2
0
_
=
L

L
2
4
cos() +
L
2
4
2
sin
_
2
L

L
2
__
=
L

_
L
2
4
+
L
2
4
2
sin()
_
=
L
3
4
2
(1.7.16)
Whew! We can now substitute (1.7.16) into (1.7.15) to get the quadrupole moment:
Q
0
=
4
L
2
I
o
sin(t)
_

L
3
4
2
_
Q
0
=
L

I
o
sin(t) (1.7.17)
b) The quadrupole radiation pattern is given by (13.121) in Franklin as:
dP
d
=
ck
6
Q
2
0
32
cos
2
() sin
2
() (1.7.18)
Substituting (1.7.17) into (1.7.18) yields
dP
d
=
ck
6
32
_
L

I
o
sin(t)
_
2
cos
2
() sin
2
()
=
ck
6
32

L
2

2
I
2
o
sin
2
(t) cos
2
() sin
2
()
=
ck
6
L
2
32
3

2
I
2
o
sin
2
(t) cos
2
() sin
2
() (1.7.19)
If we recall that
2
= c
2
k
2
, then (1.7.19) becomes
dP
d
=
ck
6
L
2
32
3
(c
2
k
2
)
I
2
o
sin
2
(t) cos
2
() sin
2
()
=
k
4
L
2
32
3
c
I
2
o
sin
2
(t) cos
2
() sin
2
() (1.7.20)
CHAPTER 1: ELECTRODYNAMICS II 59
If we take the time-average of the time-dependent term in (1.7.20), then we nd that sin
2
(t)
1
/
2
,
and we are left with
dP
d
=
k
4
L
2
64
3
c
I
2
o
cos
2
() sin
2
() (1.7.21)
I have plotted (1.7.21), and is at the very end. The pattern is unnormalized, since we are not given
the values for any of the constants. We do know that within the mess of constant is a factor of
1/64, which tells us that the quadrupole radiation pattern contributes signicantly less to
the total radiated power of the antenna. This makes sense, since the antenna in question is
a dipole antenna, and so it emits mostly dipole radiation.
c) As done previously, we can nd the intensity from the time-averaged Poynting vector, given as
(13.50) in Franklin:
dP
d
= r
2
r S S =
1
r
2
dP
d
r (1.7.22)
So presumably, all we would need to do is substitute (1.7.21) into (1.7.22):
[S[ =
1
r
2
k
4
L
2
64
3
c
I
2
o
cos
2
() sin
2
() (1.7.23)
The intensity of (1.7.24) will reach a maximum when = /4, 3/4, so that cos
2
(/4) sin
2
(/4) =
1/4, and we are nally left with
[S[ =
1
r
2
k
4
L
2
256
3
c
I
2
o
(1.7.24)
d) The total power radiated from a symmetric quadrupole is given by (13.122) in Franklin as
P =
ck
6
Q
2
0
60
(1.7.25)
So, we substitute (1.7.17) into (1.7.25):
P =
ck
6
60
_
L

I
o
sin(t)
_
2
=
ck
6
L
2
60
2

2
I
2
o
sin
2
(t) (1.7.26)
We remember that
2
= c
2
k
2
, and that according to time averaging sin
2
(t) =
1
/
2
, and (1.7.26)
becomes
P =
K
4
L
2
120
2
c
I
2
o
(1.7.27)
Problem 13.11
A conducting sphere of radius R is an oscillating magnetic eld B = B
0
e
it
acquires an induced magnetic
moment .
a) Find the surface current K induced on the conducting sphere (Use the boundary condition on B, including
the magnetic eld due to the induced dipole).
W. Erbsen HOMEWORK #7
b) Integrate the current over the sphere to nd the induced magnetic moment.
Solution
a) This problem bears extreme resemblance to the very similar situation of a conducting sphere in an
external electric eld. One of the most immediately apparent discrepancies is that for a sphere in
an electric eld we are concerned with the electric dipole moment, p, whereas with the magnetic
eld we concern ourselves with the magnetic moment, .
We rst recall that Amperes circuit law states that
H =
3j
c
+
1
c

t
D (1.7.28)
We know that j = 0 inside and outside the sphere as well, so (1.7.28) reduces to
H =
1
c

t
D (1.7.29)
Since the sphere is a conductor, the electric eld inside will be zero, and outside as well, so (1.7.29)
becomes
H = 0 (1.7.30)
Which applies to both interior and exterior of the sphere. We also note that K must only exist at
the interface.
At this point, we introduce the concept of the magnetic scalar potential,
m
, which shares many
of the same characteristics as the electrical analogue,
e
. Most notably, from (7.88) in Franklin,
H =
m
B =
m
(1.7.31)
Now, we remember that from Maxwells equations that monopoles most certainly do not exist:
B = 0 (1.7.32)
And substituting (1.7.31) into (1.7.32) yields
(
m
) = 0
2

m
= 0 (1.7.33)
Which we immediately recognize to be Laplaces equation. We can nd solutions for this equation
by way of analyzing boundary conditions of a sphere imagined to be in an external electric eld,
and then replace the electric potential (
e
) by the magnetic potential (
m
).
So lets get started! For the case of an electric eld, our boundary conditions are

e
(r, ) =
_
0 at r = R
E
0
r cos at r
And the series solution we will be using is

e
(r, ) =

=0
_
A

+
B

r
+1
_
P

(cos ) (1.7.34)
Applying the rst boundary condition to (1.7.34) yields:
CHAPTER 1: ELECTRODYNAMICS II 61

e
(R, ) = A

+
B

R
+1
= 0 B

= A

R
2+1
(1.7.35)
We now substitute (1.7.35) back in to (1.7.34):

e
(r, ) =

=0
_
A

R
2+1
r
+1
_
P

(cos ) (1.7.36)
We now apply the second boundary condition to (1.7.36):

e
(r , ) =

=0
A

(cos ) = E
0
r cos (1.7.37)
By looking at the rst few Legendre Polynomials, ee can see that the only way that the second
boundary condition is satised is if = 1 and A

= A
1
= E
0
. Applying all this to (1.7.36) leads
us to

e
(r, ) = E
0
_
r
R
3
r
2
_
cos (1.7.38)
But not so fast! We must recognize that we must separate (1.7.34) to describe the interior and
exterior potentials of our sphere:

in
m
=

=0
A

(cos ) (1.7.39a)

out
m
=

=0
B

r
+1
P

(cos ) (1.7.39b)
Substituting in the values for the constants we found previously, (1.7.39a) and (1.7.39b) become,
respectively:

in
m
=0 (1.7.40a)

out
m
=B
0
R
3
r
2
cos (1.7.40b)
Under the same logic as the rst boundary condition, we choose the potential inside the sphere to
be zero, so that the eld is zero. We can now nd the interior and exterior elds by substituting
(1.7.40a) and (1.7.40b) into (1.7.33), in turn. We rst recall that the gradient is given (in spherical
coordinates) by
f =
f
r
r +
1
r
f

+
1
r sin
f

(1.7.41)
Since we have no -dependence, (1.7.41) becomes
f =
f
r
r +
1
r
f

(1.7.42)
Substituting (1.7.42), (1.7.40a)-(1.7.40b) into (1.7.33), we have:
B
in
=
in
m
= 0
B
out
=
out
m
=
_

r
r +
1
r

_ _
B
0
R
3
r
2
cos
_
W. Erbsen HOMEWORK #7
=
_
B
0
R
3
_

r
1
r
2
_
cos r + B
0
R
3
r
3
_

cos
_

_
=2B
0
R
3
r
3
cos r B
0
R
3
r
3
sin

(1.7.43)
We now recall from (7.74) that
4
c
K = n
_
B
out
B
in
_
K =
c
4
n
_
B
out
B
in
_
(1.7.44)
Substituting (1.7.43) into (1.7.44),
K =
c
4
n
_
2B
0
R
3
r
3
cos r B
0
R
3
r
3
sin

_
(1.7.45)
We recall that n is r, and r r = 0, but r

, so (1.7.45) becomes
K =
c
4
B
0
R
3
r
3
sin

(1.7.46)
The surface current is found by evaluating (1.7.46) at r = R:
K =
c
4
B
0
sin e
it

(1.7.47)
Where, you guessed it, I neglected the exponential term and put it back in at the last moment.
b) To nd the magnetic moment ,
=
1
2c
_
r K dS
=
1
2c
_
r
_
c
4
B
0
sin e
it

_
dS
=
B
0
8
2
e
it
_
_
r

_
sin dS
=
B
0
8
2
e
it
_
r sin dS
=
B
0
8
2
e
it
_ _
r sin r
2
sin dd
=
B
0
8
2
e
it
R
3
_
4
2
_
Where I have absorbed the direction into the magnitude B
0
to signify that the magnetic moment
is along the same axis (opposite direction) as the incident eld. We are then left with
=
B
0
2
R
3
e
it
CHAPTER 1: ELECTRODYNAMICS II 63
1.8 Homework #8
Problem 14.1
A space ship is moving with velocity 0.6c shoots a projectile with muzzle velocity 0.8c. Find the projectiles
velocity with respect to a xed target if it is red
a) Straight ahead.
b) Straight backward.
c) At right angles to the ships velocity.
d) Redo parts a,b,c if the muzzle velocity were c.
Solution
The relativistic addition of velocity equations are given by (14.24) and (14.25):
u

=
u

+ v
1 +u

v/c
2
(1.8.1a)
u

=
u

(1 +u

v/c
2
)
(1.8.1b)
And we also know that the Lorentz factor, , is given by
=
1
_
1 v
2
/c
2

1
_
1
2
(1.8.2)
Where = v/c. In our case, the moving frame (S

) is that of the spaceship, which is moving at a velocity


v. Accordingly, we can go ahead and calculate the Lorentz factor from (1.8.2):
=
1
_
1 (0.6c)
2
/c
2
=
1
_
1 (0.6)
2
=
1

1 0.36
=
1

0.64
=
1
0.8
=1.25 (1.8.3)
a) If the projectile is red straight ahead, then u

= 0.8c, and u

= 0. From (1.8.1a),
u

=
u

+v
1 +u

v/c
2
=
(0.8c) + (0.6c)
1 + (0.8c)(0.6c)/c
2
W. Erbsen HOMEWORK #8
=
1.4c
1 + 0.48
=
1.4
1.48
c u

0.9459c
And now from (1.8.1b),
u

=
u

(1 + u

v/c
2
)
u

= 0
b) If the projectile is red straight backwards, then u

= 0.8c, and u

= 0. Once again using


(1.8.1a),
u

=
u

+ v
1 +u

v/c
2
=
(0.8c) + (0.6c)
1 + (0.8c)(0.6c)/c
2
=
0.2c
1 0.48
=
0.2c
0.52
c u

0.3846c
And from (1.8.1b)
u

=
u

(1 + u

v/c
2
)
u

= 0
c) If it is red perpendicular to the ships motion, then we can say that u

= 0, and u

= 0.8c, and
(1.8.1a) becomes
u

=
u

+v
1 +u

v/c
2

0.6c
1
u

= 0.6c
And for the perpendicular component, (1.8.1b) leads us to
u

=
u

(1 +u

v/c
2
)

0.8c
1.25(1)
u

= 0.64c
d) Repeating parts a)-c) warrants no real surprises.
a) From (1.8.1a),
u

=
u

+v
1 +u

v/c
2
=
(c) + (0.6c)
1 + (c)(0.6c)/c
2
=
1.6c
1 + 0.6
=
1.6
1.6
c u

= c
And the perpendicular component is surprisingly uneventful,
u

=
u

(1 +u

v/c
2
)
u

= 0
CHAPTER 1: ELECTRODYNAMICS II 65
b) For the backwards case we have u

= c and u

= 0, we have from (1.8.1a) that


u

=
u

+ v
1 + u

v/c
2
=
(c) + (0.6c)
1 + (c)(0.6c)/c
2
=
0.4c
1 0.6
=
0.4c
0.4
c u

= c
And from (1.8.1b),
u

=
u

(1 +u

v/c
2
)
u

= 0
c) And lastly, but perhaps most interestingly, we let u

= 0 and u

= c, so (1.8.1a) becomes
u

=
u

+v
1 +u

v/c
2

0.6c
1
u

= 0.6c
And for the perpendicular component, (1.8.1b) says that
u

=
u

(1 + u

v/c
2
)

c
1.25(1)
u

= 0.8c
Problem 14.2
a) Use the relativistic velocity addition equations to prove that c added in any direction to any velocity results
in velocity c.
b) Prove that adding any two velocities cannot give a velocity greater than c.
Solution
a) To prove this we start with the relativistic velocity addition equation for parallel trajectories, given
by (14.24) in Franklin:
u

=
u

+ v
1 + u

v/c
2
(1.8.4)
Now, if we let u

= c, then (1.8.4) becomes


u

=
c +v
1 + (c) v/c
2
=
c +v
1 +v/c
=
c + v
1/c (c +v)
u

= c
W. Erbsen HOMEWORK #8
The same result applies if we let u

= c as well.
b) For this next proof, we again start with (1.8.4), but must apply some more algebraic manipulations:
u

=
u

+ v
1 + u

v/c
2
=
c(u

/c +v/c)
1 +u

v/c
2
=
c(1 1 +u

v/c
2
u

v/c
2
+ u

/c + v/c)
1 + u

v/c
2
=
c[1 +u

v/c
2
(1 +u

v/c
2
u

/c v/c)]
1 +u

v/c
2
=c
_
1 + u

v/c
2
1 + u

v/c
2

(1 +u

v/c
2
u

/c v/c)
1 +u

v/c
2
_
=c
_
1
(1 u

/c v/c + u

v/c
2
)
1 +u

v/c
2
_
=c
_
1
(1 u

/c) (1 v/c)
1 + u

v/c
2
_
(1.8.5)
We can see from the bracketed fraction in (1.8.5) that if u

= c, then u

= c, and also if v = c,
then u

= c. If Both are equal to c, then u

= c. If either (or both) are anything less than c, then


u

< c. Therefore, adding any two velocities cannot result in a velocity greater than c .
Problem 14.3
Derive (14.26) and (14.27) for the transformation of acceleration.
Solution
The equations that we are ultimately trying to derive are:
a

=
a

3
(1 +u

v/c
2
)
3
(1.8.6a)
a

=
a

+ (u

) v/c
2

2
(1 + u

v/c
2
)
3
(1.8.6b)
A good place to start is with the Lorentz transformation equations, given by (14.19)-(14.22) in Franklin:
t

=
_
t xv/c
2
_
x

= (x vt)
y

=y
CHAPTER 1: ELECTRODYNAMICS II 67
z

=z
We can nd the inverse transform equations of these by replacing t t

, x x

, y y

, z z

, and
v v:
t =
_
t

+x

v/c
2
_
(1.8.7a)
x = (x

+vt

) (1.8.7b)
y =y

z =z

Using (1.8.7a), we calculate the following derivative:


t
t

=

t

_
t

+ x

v/c
2
_
=
__
t

_
+
v
c
2
_
x

__
=
_
1 +u

v/c
2
_
=
_
1 +u

v/c
2
_
(1.8.8)
Taking the reciprocal of (1.8.8),
t

t
=
1
(1 +u

v/c
2
)
(1.8.9)
Fantastic. Now, we use the chain rule to nd the following derivative:
u

=
x
t
v
=
x
t

t
v
=

t
[ (x

+vt

)]
1
(1 +u

v/c
2
)
v
=
__
x

_
+ v
_
t

__

1
(1 + u

v/c
2
)
v
=
u

+ v
1 + u

v/c
2
v
=
u

+ v
1 + u

v/c
2
(1.8.10)
Where I used both (1.8.7b) and (1.8.9) in deriving (1.8.10). This is of course the relativistic velocity
addition equation for parallel trajectory, which is (14.24) from Franklin. Similarly, we repeat this process
for the transverse component (I will denote the unit vector perpendicular to the forward motion of v by
v

):
u

=
x
t
v

=
x
t

t
v

W. Erbsen HOMEWORK #8
=
x
t


1
(1 +u

v/c
2
)
v

=
u

(1 +u

v/c
2
)
v

=
u

(1 +u

v/c
2
)
(1.8.11)
Which is of course none other than (14.25) from Franklin.
But wait! We want the acceleration, not the velocity. But be not afraid, as the process of nding a

and a

is very much the same as what we did to nd u

and u

. We start by nding the derivative of


the parallel velocity component:
a

=
u

t
=
u

t
=

t

_
u

+v
1 +u

v/c
2
_

1
(1 + u

v/c
2
)
=
1
(1 +u

v/c
2
)
_
1
1 + u

v/c
2
_

t

_
u

+ v
_
_
+
_
u

+v
_
_

t

_
1
1 +u

v/c
2
___
=
1
(1 +u

v/c
2
)
_
1
1 +u

v/c
2
_
u

_
+
_
u

+v
_
_

v
c
2
u

_
1
(1 +u

v/c
2
)
2
___
=
1
(1 +u

v/c
2
)
_
a

1 +u

v/c
2
+
_
u

+v
_
_

v/c
2
(1 + u

v/c
2
)
2
__
=
1
(1 +u

v/c
2
)
_
a

1 +u

v/c
2

1 + u

v/c
2
1 + u

v/c
2
+
_
u

+ v
_
_

v/c
2
(1 +u

v/c
2
)
2
__
=
1
(1 +u

v/c
2
)
_
a

_
1 +u

v/c
2
_
(1 +u

v/c
2
)
2
+
_
u

+v
_
_

v/c
2
(1 + u

v/c
2
)
2
__
=
1
(1 +u

v/c
2
)
3
_
a

_
1 +u

v/c
2
_

_
u

+v
_
a

v/c
2
_
=
1
(1 +u

v/c
2
)
3
_
a

+ a

(u

v) /v
2
a

(u

v) /c
2
a

v
2
/c
2
_
=
1
(1 +u

v/c
2
)
3
_
a

v
2
/c
2
_
=
a

(1 +u

v/c
2
)
3
_
1 v
2
/c
2
_
(1.8.12)
If we recognize that the term in braces in (1.8.14) is just
2
, then we are nally left with
a

=
a

3
(1 + u

v/c
2
)
3
Which we can see is the same as (1.8.1a). For the perpendicular component,
CHAPTER 1: ELECTRODYNAMICS II 69
a

=
u

t
=
u

t
=

t

_
u

(1 +u

v/c
2
)
_

1
(1 + u

v/c
2
)
=
1

2
(1 +u

v/c
2
)
_
1
1 + u

v/c
2
_
u

_
+u

_

t

1
1 +u

v/c
2
__
=
1

2
(1 +u

v/c
2
)
_
a

1 +u

v/c
2
+u

v
c
2
_
u

_
1
(1 +u

v/c
2
)
2
__
=
1

2
(1 +u

v/c
2
)
_
a

1 +u

v/c
2
u

v/c
2
(1 +u

v/c
2
)
2
_
=
1

2
(1 +u

v/c
2
)
_
a

1 +u

v/c
2

1 + u

v/c
2
1 + u

v/c
2
u

v/c
2
(1 + u

v/c
2
)
2
_
=
1

2
(1 +u

v/c
2
)
_
a

_
1 +u

v/c
2
_
(1 +u

v/c
2
)
2
u

v/c
2
(1 +u

v/c
2
)
2
_
=
1

2
(1 +u

v/c
2
)
3
_
a

_
1 + u

v/c
2
_
u

(a

v) /c
2
_
(1.8.13)
At this point, it should be noted that we must be extremely careful with our vectors and scalar products.
In particular, we recall that a

v = a

v v v = a

v. Continuing (1.8.15),
a

=
1

2
(1 +u

v/c
2
)
3
_
a

+ a

_
u

v
_
/c
2
u

_
a

v
_
/c
2
_
=
1

2
(1 +u

v/c
2
)
3
_
a

+
_
a

_
u

v/c
2
u

v/c
2
_
=
1

2
(1 +u

v/c
2
)
3
_
a

+ a

v/c
2
a

v/c
2
u

v/c
2
_
=
1

2
(1 +u

v/c
2
)
3
_
a

+ a

v/c
2
a

v/c
2
_
u

+u

__
=
1

2
(1 +u

v/c
2
)
3
_
a

+ a

v/c
2
a

v/c
2
_
=
1

2
(1 +u

v/c
2
)
3
_
a

+
1
c
2
_
a

v a

v
_
_
(1.8.14)
We now recall the triple product A (BC) = B(A C) C(A B). We can apply this to the
bracketed term in (1.8.14):
_
a

v a

v
_
=
_
a

v a

v
_
=
_
u

_
a

v
_
a

_
u

v
__
= [u

(v a

) a

(v u

)]
= [v (u

)]
W. Erbsen HOMEWORK #8
=(u

) v (1.8.15)
Substituting (1.8.15) into (1.8.14),
a

=
1

2
(1 + u

v/c
2
)
3
_
a

+
1
c
2
[(u

) v]
_
Bringing in the factor from outside and bringing up the factor c
2
, we are left with
a

=
a

+ (u

) v/c
2

2
(1 +u

v/c
2
)
3
Which we can see is identical to (1.8.6b).
Problem 14.4
An object falls from constant acceleration (in its rest system) of g.
a) Find its velocity after a time t. (Answer: v = gt/
_
1 +g
2
t
2
/c
2
)
b) What is its velocity after one year?
c) Find p/m of the object at time t. Sketch graphics of v(t) and p(t)/m.
Solution
a) To nd the velocity, we start with Newtons 2
nd
Law:
F = ma
dp
dt
(1.8.16)
However, we must be careful here - the momentum is relativistic, and must treat it accordingly:
p = mv
mv
_
1 v
2
/c
2
(1.8.17)
Substituting (1.8.17) into (1.8.16), we nd that
a =
d
dt
_
v
_
1 v
2
/c
2
_

_
t
0
a(t) dt =
v
_
1 v
2
/c
2
(1.8.18)
Recalling that in our case a = g, so that (1.8.18) becomes
g
_
t
0
dt =
v
_
1 v
2
/c
2
gt +C =
v
_
1 v
2
/c
2
(1.8.19)
Our object starts at rest, so v(0) = 0, so C = 0, so (1.8.19) becomes
gt =
v
_
1 v
2
/c
2
CHAPTER 1: ELECTRODYNAMICS II 71
g
2
t
2
=
v
2
1 v
2
/c
2
v
2
=g
2
t
2
_
1 v
2
/c
2
_
v
2
=g
2
t
2
v
2
g
2
t
2
/c
2
g
2
t
2
=v
2
+v
2
g
2
t
2
/c
2
g
2
t
2
=v
2
_
1 + g
2
t
2
/c
2
_
v
2
=
g
2
t
2
1 + g
2
t
2
/c
2
(1.8.20)
From (1.8.20) it is easy to see that we are left with
v(t) =
gt
_
1 +g
2
t
2
/c
2
(1.8.21)
b) Does this question ask us to nd the velocity in the frame of an observer on Earth, or in the frame
of the ship? Lets nd out both! The velocity according to an observer on ship is easy, we can just
use (1.8.21), but rst lets nd t:
1 yr
365 day
1 yr

24 hr
1 day

60 min
1 hr

60 sec
1 min
= 31.539 10
6
sec
Substituting this value into (1.8.21), we nd that
v (1 yr) =
(9.810 m/s
2
)(31.539 10
6
s)
_
1 + (9.810 m/s
2
)(31.539 10
6
s)/(2.998 10
8
m/s)]
2
v (1 yr) = 0.7181c [Ship frame]
Now, to nd the velocity after one year in the Earths frame, we must instill a little mathematical
voodoo. We start with the denition of acceleration:
a

=
du

dt
=
du

dt


dt

dt
=
d
dt

_
u

+v
1 +u

v/c
2
_

1
(1 + u

v/c
2
)
(1.8.22)
Letting u

0, (1.8.22) becomes
a

=
1

dv
dt

(1.8.23)
Now, we take a look at (14.26) from Franklin, and also let u

0:
a

=
a

3
(1 + u

v/c
2
)
3
a

=
a

3
(1.8.24)
Equating (1.8.23) and (1.8.24),
1

dv
dt

=
a

3

dv
dt

=
a

2
(1.8.25)
W. Erbsen HOMEWORK #8
If we notice that a

= g, then (1.8.25) becomes

2
dv = g dt


_
v
0
1
1 v
2
/c
2
dv = g
_
t

0
dt

(1.8.26)
Things get a little easier if we let u = v/c, du = (1/c)dv. With this, (1.8.26) becomes
c
_
v
0
1
1 u
2
du = gt

(1.8.27)
This integral requires using hyperbolic trig substitutions, and is a well-known integral. Pressing on,
(1.8.27) becomes
c
_
tanh
1
(u)

= gt

c
_
tanh
1
_
v
c
__
= gt

v
c
= tanh
_
gt

c
_
v = c tanh
_
gt

c
_
(1.8.28)
Hooray! Substituting in the appropriate values into (1.8.28) yields
v (1 yr) = tanh
_
(9.810 m/s
2
)(31.539 10
6
s)
2.998 10
8
m/s
_
c
v (1 yr) = 0.7757c [Earth frame]
c) To nd p/m as a function of t, we must go back and use the result from the rst part of the problem.
The momentum is of course given by p(t) = mv(t), however which v(t) should we use? Lets use
both! I will go ahead and use the rst result, the one derived in part a). We begin by nding the
Lorentz factor, , with this velocity:
=
_
1
v
2
c
2
_

1
/2
=
_
_
1
1
c
2
_
gt
_
1 + g
2
t
2
/c
2
_
2
_
_

1
/2
=
_
1
1
c
2
_
g
2
t
2
1 +g
2
t
2
/c
2
__

1
/2
=
_
1
g
2
t
2
c
2
+g
2
t
2
_

1
/2
=
_
c
2
+ g
2
t
2
c
2
+ g
2
t
2

g
2
t
2
c
2
+g
2
t
2
_

1
/2
=
_
c
2
c
2
+ g
2
t
2
_

1
/2
=
_
c
2
+ g
2
t
2
c
2
_
1
/2
(1.8.29)
We can now use (1.8.29) to nd the p(t)/m:
CHAPTER 1: ELECTRODYNAMICS II 73
p(t)
m
=v(t)
=
_
c
2
+g
2
t
2
c
2
_
1
/2
gt
_
1 + g
2
t
2
/c
2
=
_
c
2
+g
2
t
2
c
2
_
1
/2
_
c
2
c
2
+ g
2
t
2
_
1
/2
gt (1.8.30)
From (1.8.30) it is clear that we are left with:
p(t)
m
= gt (1.8.31)
We now do the same for the velocity in the Earths frame from (1.8.28):
p(t)
m
=v(t)
=
1
_
1 v
2
/c
2
v(t)
=
1
_
1 (c tanh (gt/c))
2
/c
2
c tanh
_
gt
c
_
=
1
_
1 tanh
2
(gt/c)
c tanh
_
gt
c
_
=
1
_
sech
2
(gt/c)
c tanh
_
gt
c
_
=cosh (gt/c)
sinh
2
(gt/c)
cosh
2
(gt/c)
c (1.8.32)
It is easy to see that (1.8.32) leads us to
p(t)
m
=
sinh
2
(gt/c)
cosh (gt/c)
c (1.8.33)
A plot of v(t) from (1.8.21) and (1.8.28) is attached (they look the same). There is also a plot of
(1.8.33), however I have not included (1.8.31), as it is just a straight line, and straight lines are
never fun. Plots can be seen at the end of this homework assignment.
Problem 14.5
A space ship 100 meters long ies through an open hanger 80 meters long.
a) How fast must the space ship y in order to permit both doors of the hanger to be briey closed at the same
time while the space ship is inside?
b) How would the pilot of the space ship interpret what happens as the doors close and then reopen?
W. Erbsen HOMEWORK #8
Solution
a) We begin with the equation for length contraction, which reads
L

=
L

(1.8.34)
While the Lorentz factor, , is given by
=
1
_
1 v
2
/c
2
(1.8.35)
Substituting (1.8.35) into (1.8.34),
L

L
=
_
1 v
2
/c
2

_
L

L
_
2
= 1
v
2
c
2
(1.8.36)
Solving (1.8.36) for v,
v =

_
_
1
_
L

L
_
2
_
c
2
=

1
_
80 m
100 m
_
2
c
=

1
_
4
5
_
2
c
=
_
1
16
25
c
=
_
9
25
c
=
3
5
c v = 0.6c (1.8.37)
b) First of all, the doors will not be able to close simultaneously, which is dependent on the observer.
We note that although a person standing next to the hanger will observe the space ship appearing
shorter, the pilot will also see the hanger getting shorter.
At the end of the day, whether this event is observed to happens depends on the observer, and
also their denition of simultaneous. A person standing next to the hanger will say that the back
end of the space ship will make it through the entrance to the hanger before the front of the ship
hits the forward wall of the hanger. However, the pilot sees it just the other way around. To him,
his ship is too long to t in the hanger, and he sees the front of his ship hit the far wall of the
hanger before the tail end of the ship makes it all the way through the entrance of the hanger.
Problem 14.6
A jet plane circles the Earth at the equator with a speed of Mach 4. By how much time is the pilots watch
slow when he lands?
CHAPTER 1: ELECTRODYNAMICS II 75
Solution
We know that the kilometer was originally dened as the distance from the equator to the North pole
divided by ten million. Assuming that this value is correct, then this implies that the circumference of
the Earth is 40, 000, 000 m, or 4 10
7
m. I will assume that the pilot is ying at sea level - since no
other values were specied.
We also know that the Mach system denes velocity in terms of the speed of sound in the local
media. At sea level, Mach 4 turns out to be 1.361 10
3
m/s. We also know the speed of light, so the
constants needed in this problem are:
x =4.000 10
7
m
v =1.361 10
3
m/s
c =2.998 10
8
m/s
And if we dene = v/c, then we can say that
=
v
c
=
1.361 10
3
m/s
2.998 10
8
m/s
= 4.540 10
6
If we let t designate the time measured on Earth, and t

0
be the time measured in the local reference
frame of the ship, then the change in time is
t = t t

0
(1.8.38)
And the equation for time dilation is
t

0
=
t

(1.8.39)
Substituting (1.8.39) into (1.8.38),
t =t
t

=t
_
1
1

_
=t
_
1
_
1
2
_
(1.8.40)
Since 1, we can expand (1.8.40) in a series expansion:
t = t
_
1
_
1

2
2


4
8


6
16
...
__
Taking only the rst two terms,
t = t
_
1
_
1

2
2
__
t =
t
2
2
(1.8.41)
We can now say that from (1.8.41) the time lost per second is given by
t
t
=

2
2

_
4.540 10
6
_
2
2

t
t
= 1.030 10
11
(1.8.42)
W. Erbsen HOMEWORK #8
This means that the space ships clock lags behind by t/t = 10.30 ps for every second that it is at
speed.
The time that the ship is orbiting is given by
v =
x
t
t =
x
v

4.000 10
7
m
1.361 10
3
m/s
t = 2.945 10
4
s
Using (1.8.42), we nd that the total time dierence between the clocks after the ship returns is
_
t
t
_
(t) =
_
1.030 10
11
_ _
2.945 10
4
s
_
t = 3.035 10
7
s (30.35 ns)
Problem 14.8
a) Show that the matrix equations [

L][G][L] = [G] holds for the Lorentz matrix [L


(x)
()].
b) Find the Lorentz matrix for a Lorentz transformation of velocity v = c(

i +

j)/

2
Solution
a) We recall that from (14.66) in Franklin that x

= [L]x. In Matrix form, spanning all four-vectors,


_
_
_
_
ct

_
_
_
_
= [L]
_
_
_
_
ct
x
y
z
_
_
_
_
(1.8.43)
We also recall the value of [G] from (14.69):
[G] =
_
_
_
_
1 0 0 0
0 1 0 0
0 0 1 0
0 0 0 1
_
_
_
_
The question asks us to prove the orthogonality condition for only [L
(x)
()], but it can be shown to
be true in a more general case through linear algebra. Before we begin, we recall that ([A] [B])
T
=
[B]
T
[A]
T
. Using this identity, in conjunction with (1.8.43), we see that
_
ct x y z
_
[G]
_
_
_
_
ct
x
y
z
_
_
_
_
=
_
ct x y z
_
_
_
_
_
1 0 0 0
0 1 0 0
0 0 1 0
0 0 0 1
_
_
_
_
_
_
_
_
ct
x
y
z
_
_
_
_
=
_
ct x y z
_
_
_
_
_
ct
x
y
z
_
_
_
_
CHAPTER 1: ELECTRODYNAMICS II 77
=c
2
t
2
x
2
y
2
z
2
(1.8.44)
We can do the same calculation using the orthogonality condition in place of [G]:
_
ct x y z
_
[

L][G][L]
_
_
_
_
ct
x
y
z
_
_
_
_
=
_
_
ct x y z
_
[

L]
_
[G]
_

_
[L]
_
_
_
_
ct
x
y
z
_
_
_
_
_

_
=
_

_
[L]
_
_
_
_
ct
x
y
z
_
_
_
_
_

_
T
[G]
_

_
[L]
_
_
_
_
ct
x
y
z
_
_
_
_
_

_
=
_

_
_
_
_
_
ct

_
_
_
_
_

_
T
[G]
_

_
_
_
_
_
ct

_
_
_
_
_

_
=
_
ct

_
_
_
_
_
1 0 0 0
0 1 0 0
0 0 1 0
0 0 0 1
_
_
_
_
_
_
_
_
ct

_
_
_
_
=
_
ct

_
_
_
_
_
ct

_
_
_
_
=c
2
t
2
x
2
y
2
y
2
(1.8.45)
We note that the results from (1.8.44) and (1.8.45) are remarkably similar, and seem surprisingly
familiar. Of course! They are Lorentz invariants these values are independent of coordinate
transformations, and so they must be equal. Therefore, from (1.8.44) and (1.8.45), we have shown
that:
[

L][G][L] = [G]
b) To nd the Lorentz matrix for the velocity given, we rst break it down into its two components:
v =
c

2
_

i +

j
_

v
x
= c/

2

i = v
x
/

2

i
v
y
= c/

2

j = v
y
/

2

j
It is now in our best interest to dene:

x
=
v
x
c

2
, and
y
=
v
y
c

2
The equation that allows you to nd the Lorentz matrix for an arbitrary velocity is given in many
textbooks, and is given by:
W. Erbsen HOMEWORK #9
[L()] =
_
_
_
_
_
_
_
_
_
_

x

y

z

x
1 + ( 1)

2
x

2
( 1)

2
( 1)

y
( 1)

2
1 + ( 1)

2
y

2
( 1)

z
( 1)

2
( 1)

2
1 + ( 1)

2
z

2
_
_
_
_
_
_
_
_
_
_
We know the values for
x
and
y
, and also that
z
= 0, so this matrix becomes
[L()] =
_
_
_
_
_
_
_
_
_

x

y
0

x
1 + ( 1)

2
x

2
x
+
2
y
( 1)

x

2
x
+
2
y
0

y
( 1)

y

2
x
+
2
y
1 + ( 1)

2
y

2
x
+
2
y
0
0 0 0 0
_
_
_
_
_
_
_
_
_
If we substitute back in the values for
x
and
y
, this leaves us with
[L()] =
_
_
_
_
_
_
_
_
_

vx
c

2

vy
c

2
0

v
x
c

2
1 + ( 1)
v
2
x
v
2
x
+v
2
y
( 1)
v
x
v
y
v
2
x
+v
2
y
0

v
y
c

2
( 1)
v
y
v
x
v
2
x
+ v
2
y
1 + ( 1)
v
2
y
v
2
x
+ v
2
y
0
0 0 0 0
_
_
_
_
_
_
_
_
_
1.9 Homework #9
Problem 14.11
a) Derive the nonrelativistic result for stellar aberration by considering the motion (with v c) of a telescope
transverse to incoming light.
b) Show that, for v c, the relativistic formula for aberration agrees with the nonrelativistic result.
c) Calculate the maximum aberration angle due to the Earths orbital velocity of 30 km/sec.
Solution
a) Solution shown in part b).
CHAPTER 1: ELECTRODYNAMICS II 79
b) We rst recall the equations relating to the relativistic addition of velocities, from (14.24) and
(14.25) in Franklin, respectively:
u

=
u

+v
1 +u

v/c
2
(1.9.1a)
u

=
u

(1 + u

v/c
2
)
(1.9.1b)
Excellent. Lets examine each of these equations; lets do (1.9.1a) rst. We recall that u

= u

cos .
Applying this to (1.9.1a),
u

=
u

cos +v
1 + (u

cos ) v/c
2
(1.9.2)
And since space is a vacuum, then we can replace u

c u

. Then (1.9.2) becomes


u

=
c cos u

+v
1 +v cos /c
(1.9.3)
Now, we do the same thing for (1.9.1b). First, we note that u

= u

sin , and (1.9.1b) becomes


u

=
u

sin
(1 + (u

cos ) v/c
2
)
(1.9.4)
We now let u

c u

, and now (1.9.4) reads


u

=
c sin u

(1 +v cos /c)
(1.9.5)
We now, seemingly arbitrarily, divide both sides of (1.9.3) by c and let u

ucos

c cos

u:
u

c
=
1
c
_
c cos u

+v
1 +v cos /c
_
ucos

c
=
cos + v/c
1 + v cos /c
u

c cos

c
u =
cos +
1 + cos
u

cos

=
cos +
1 + cos
(1.9.6)
We now do the same mantra for (1.9.5):
u

c
=
1
c
_
c sin u

(1 +v cos /c)
_
usin

c
=
sin
(1 + v cos /c)
u

c sin

c
u =
sin
(1 + cos )
u

sin

=
sin
(1 + cos )
(1.9.7)
We now divide (1.9.7) by (1.9.6),
W. Erbsen HOMEWORK #9
sin

cos

=
_
sin
(1 + cos )
_ _
1 + cos
cos +
_
tan

=
sin
(cos + )
(1.9.8)
We recognize (1.9.8) to be none other than our good friend (14.109) out of Franklin. Now, in the
nonrelativistic limit, we know that 1, and =

1. Just something to keep in mind.


We now wish to solve (1.9.6) for cos :
cos

=
cos +
1 + cos
(1.9.9)
Perhaps the best way to do this is to expand the denominator of (1.9.9) in powers of :
1
1 + cos
1 cos +
2
cos
2

3
cos
3
+ ...
We now take the rst two terms and substitute in to (1.9.9):
cos

=(cos +) (1 cos )
=cos cos
2
+ +
2
cos (1.9.10)
We neglect
2
, and (1.9.10) becomes
cos

=cos cos
2
+
=cos
_
1 sin
2

_
+
=cos + sin
2
+
=cos + sin
2
(1.9.11)
We now innocently recall the following trig identity:
cos ( ) = cos cos sin sin
Now, playing with the LHS of (1.9.11), and applying this identity,
cos

=cos [(

) +]
=cos (

) cos sin (

) sin (1.9.12)
Now, we recall the series exansions:
cos (

) =1
(

)
2
2
+
(

)
4
24
...
sin (

) =(

)
(

)
3
6
+
(

)
5
120
...
Taking on the rst terms in both of these expansions and substituting in to (1.9.12) yields
cos

=cos (

) sin (1.9.13)
Equating (1.9.11) and (1.9.13),
cos + sin
2
=cos (

) sin
sin
2
= (

) sin
CHAPTER 1: ELECTRODYNAMICS II 81
(

) = sin (1.9.14)
Recalling that =

, and substituting in the appropriate value of , we are left with


=
v
c
sin (1.9.15)
Which is of course the nonrelativistic result. Hurrah!
c) Using the nonrelativistic result which we just derived ending with (1.9.15), we note that the maxi-
mum aberration angle will be obtained when sin = 1, and so we have:

max
=
v
c

30 10
3
m/s
2.998 10
8
m/s
1.000 10
4
rad (1.9.16)
Recalling that 1arcsecond = 1

= 4.848 10
6
rad, then (1.9.16) becomes

max
=
1.000 10
4
rad
4.848 10
6
rad/arcsecond

max
= 20.64

Problem 14.13
Find the velocity of 1 TeV protons, and of 20 GeV electrons (Express your answer as 1 v).
Solution
We rst recall the equations for total energy, which read:
E
tot
=T +m
0
c
2
(1.9.17a)
=( 1) m
0
c
2
(1.9.17b)
=m
0
c
2
(1.9.17c)
So, recalling that for a proton m
p
c
2
= 938 MeV, from (1.9.17c) we have
E
tot
=m
p
c
2
E
tot
m
p
c
2
=
1
_
1 v
2
/c
2
m
p
c
2
E
tot
=
_
1
v
2
c
2
(m
p
c
2
)
2
(E
tot
)
2
=1
v
2
c
2
v
2
c
2
=1
(m
p
c
2
)
2
(E
tot
)
2
v
2
=c
2
_
1
(m
p
c
2
)
2
(E
tot
)
2
_
W. Erbsen HOMEWORK #9
v =c

1
(m
p
c
2
)
2
(E
tot
)
2
(1.9.18)
Substituting the appropriate values into (1.9.18) yields
v (p) = c

1
(938 10
6
eV)
2
(1 10
12
eV)
2
v (p) = 0.9999996c
Or in natural units,
1 v (p) = 4.399 10
7
Now we must do the same for 20 GeV electrons, but rst recall that m
e
c
2
= 0.511 MeV. Still using
(1.9.18), we have
v
_
e

_
= c

1
(.511 10
6
eV)
2
(20 10
9
eV)
2
v
_
e

_
= 0.9999999996c
Or in natural unites,
1 v
_
e

_
= 3.264 10
10
Problem 14.14
Antiprotons were rst produced by colliding a beam of protons with protons at rest (xed target) in the reaction
p + p p + p + p + p.
a) What minimum beam energy is required to produce antiprotons in this process?
b) In the actual experiment, the protons were bound inside heavi nuclei, which resulted in some of them having
a kinetic energy of 40 MeV. What incident energy would be required to produce antiprotons o these protons
if their velocity was directed toward the beam?
Solution
The minimum beam energy is also known as the threshold energy, or when the incident energy
is just large enough to cover the rest mass energy of the resultant particles with none wasted on
kinetic energy.
Initially (before collision), we have
Momentum : p
p
Energy : E
2
p
= p
2
p
c
2
+ (m
p
c
2
)
2
Total energy : E
p
+ E
0
=
_
p
2
p
c
2
+ (m
p
c
2
)
2
+ m
p
c
2
And nally (after collision), we have
CHAPTER 1: ELECTRODYNAMICS II 83
Energy : E
2
= (4m
p
c
2
)
2
Now, lets make a table:
Lab Frame COM Frame
E
tot
E
b
+ m
p
c
2
4m
p
c
2
p
tot
p
b
0
Invariant (E
b
+m
p
c
2
)
2
p
2
b
c
2
(4m
p
c
2
)
2
Fantastico. Now, we set the invariant terms equal to one another, since their values do not change
under coordinate transformations:
(4m
p
c
2
)
2
=(E
b
+m
p
c
2
)
2
p
2
b
c
2
16m
2
p
c
4
=E
2
b
+m
2
p
c
4
+ 2E
b
m
p
c
2
p
2
b
c
2
16m
2
p
c
4
=
_
E
2
b
p
2
b
c
2

+ 2E
b
m
p
c
2
+ m
2
p
c
4
(1.9.19)
We now recall that E
2
b
p
2
b
c
2
= m
2
p
c
4
. Then (1.9.19) becomes
16m
2
p
c
4
=
_
m
2
p
c
4

+ 2E
b
m
p
c
2
+ m
2
p
c
4
16m
2
p
c
4
=2m
2
p
c
4
+ 2E
b
m
p
c
2
14m
2
p
c
4
=2E
b
m
p
c
2
(1.9.20)
From which it is easy to see that
E
b
= 7m
p
c
2
6.5660 GeV (1.9.21)
We know that (1.9.21) of course represents the total energy of the beam, the kinetic energy of the
beam is
E
b
= T
b
+ m
p
c
2
T
b
= E
b
m
p
c
2
T
b
= 6m
p
c
2
5.6280 GeV
Problem 14.15
a) A
+
(140) meson decays at rest into a
+
(106) lepton and a (0) neutrino (the mass in MeV is given in
parenthesis for each particle). Find the energy, momentum, and velocity of the produced muons. Muons
produced in this way are identied by the fact that they all have the same energy.
b) A neutron at rest decays in the beta decay process n(939.6) p(938.3) + e(0.51) +(0). Find the electrons
energy if there were no neutrino emitted in this decay. The fact that the decay electrons did not all have this
energy was the rst clue to the existence of the neutrino.
Solution
a) We recall that the total beam energy is given by
E
tot
=T + m
0
c
2
(1.9.22a)
W. Erbsen HOMEWORK #9
=
_
p
2
c
2
+ (m
0
c
2
)
2
(1.9.22b)
Initially, the total energy comes entirely from the rest mass energy of the pion, since it is at rest,
so:
E
i
tot
_

+
_
= m

c
2
(1.9.23)
Finally, the energy is attributed to the total energy of the muon, and the neutrino. The total energy
of the muon is given by
E
f
tot
_

+
_
=T + m

c
2
=
_
p
2

c
2
+ (m

c
2
)
2
(1.9.24)
And similarly, for the neutrino (recall that the neutrino has a zero rest mass):
E
f
tot
() =T +m

c
2
=
_
p
2

c
2
=p

c (1.9.25)
We now set the initial energy (from (1.9.23)) equal to the total nal energy (given by (1.9.24) and
(1.9.25)):
E
i
tot
_

+
_
=E
f
tot
_

+
_
+ E
f
tot
()
m

c
2
=
_
p
2

c
2
+ (m

c
2
)
2
+ p

c (1.9.26)
If we recognize that the magnitude of the momentum of the muon (p

) is the same as that for the


neutrino (p

), then (1.9.26) becomes


m

c
2
=
_
p
2

c
2
+ (m

c
2
)
2
+ p

c
m

c
2
p

c =
_
p
2

c
2
+ (m

c
2
)
2
_
m

c
2
p

c
_
2
=p
2

c
2
+ (m

c
2
)
2
(m

c
2
)
2
+ p
2

c
2
2m

c
3
p

=p
2

c
2
+ (m

c
2
)
2
(m

c
2
)
2
2m

c
3
p

=(m

c
2
)
2
2m

c
3
p

=(m

c
2
)
2
(m

c
2
)
2
p

c =
1
2
_
(m

c
2
)
2
(m

c
2
)
2
m

c
2
_
(1.9.27)
Plugging in the appropriate values in to (1.9.27), we have
p

c =
1
2
_
(140 10
6
eV)
2
(106 10
6
eV)
2
140 10
6
eV
_
p

c = 29.87 MeV (1.9.28)


To nd the nal energy of the muon, we go back to (1.9.24):
E
f
tot
_

+
_
=
_
p
2

c
2
+ (m

c
2
)
2
=
_
(29.87 10
6
eV)
2
+ (106 10
6
)
2
E
f
tot
_

+
_
= 110.1 MeV (1.9.29)
CHAPTER 1: ELECTRODYNAMICS II 85
And the velocity can be found by recalling that the total energy can be expressed as
E
tot
= m
0
c
2
(1.9.30)
And also that the Lorentz factor is given by
=
1
_
1 v
2
/c
2
(1.9.31)
Substituting (1.9.31) into (1.9.30),
E
tot
=
m
0
c
2
_
1 v
2
/c
2
(E
tot
)
2
=
(m
0
c
2
)
2
1 v
2
/c
2
1
v
2
c
2
=
(m
0
c
2
)
2
(E
tot
)
2
v
2
c
2
=1
(m
0
c
2
)
2
(E
tot
)
2
v =c

1
(m
0
c
2
)
2
(E
tot
)
2
(1.9.32)
Applying this to the case of the muon, and substituting in the previosu values calculated, we have
v =c

1
(106 10
6
eV)
2
(110.1 10
6
eV)
2
v = 0.2712c
b) At the beginning, all the energy comes from the rest mass energy of the neutron:
E
i
tot
(n) = m
n
c
2
(1.9.33)
Finally, the total energy comes from the total energy of the proton, electron, and neutrino. The
total energy of the proton is
E
f
tot
(p) =T + m
p
c
2
=
_
p
2
p
c
2
+ (m
p
c
2
)
2
(1.9.34)
The total energy of the electron is
E
f
tot
(e

) =T + m
e
c
2
=
_
p
2
e
c
2
+ (m
e
c
2
)
2
(1.9.35)
If we assume that no neutrino is ejected, as the prompt suggests, then from (1.9.33)-(1.9.35),
conservation of energy says that
E
i
tot
(n) =E
f
tot
(p) +E
f
tot
(e

)
m
n
c
2
=
_
p
2
p
c
2
+ (m
p
c
2
)
2
+
_
p
2
e
c
2
+ (m
e
c
2
)
2
(1.9.36)
If we assume that the magnetude of the momentum of the proton is equal to that of the electron
(p
p
= p
e
= p), then (1.9.36) becomes
W. Erbsen HOMEWORK #9
m
n
c
2
=
_
p
2
c
2
+ (m
p
c
2
)
2
+
_
p
2
c
2
+ (m
e
c
2
)
2
(m
n
c
2
)
2
=
_
_
p
2
c
2
+ (m
p
c
2
)
2
+
_
p
2
c
2
+ (m
e
c
2
)
2
_
2
=p
2
c
2
+ (m
p
c
2
)
2
+ p
2
c
2
+ (m
e
c
2
)
2
+ 2
_
(p
2
c
2
+ (m
p
c
2
)
2
) (p
2
c
2
+ (m
e
c
2
)
2
)
=2p
2
c
2
+ (m
p
c
2
)
2
+ (m
e
c
2
)
2
+ 2
_
(p
2
c
2
)
2
+ p
2
c
2
(m
e
c
2
)
2
+p
2
c
2
(m
p
c
2
)
2
+ (m
p
c
2
)
2
(m
e
c
2
)
2
Seeing as how the width of my paper has already been exceeded, the rest of this calculation will be
done in Mathematica. The result for the momentum is given by
pc =
_
(M
e
M
n
M
p
) (M
e
+M
n
M
p
) (M
e
M
n
+ M
p
) (M
e
+M
n
+ M
p
)
2M
n
(1.9.37)
Where M
e
= m
e
c
2
, M
p
= m
p
c
2
and M
n
= m
n
c
2
. Substituting in the appropriate values into
(1.9.37) (steps not shown). The result is
pc = 1.1945 MeV
The total energy of the ejected electron is then
E
tot
_
e

_
=
_
((1.1945 10
6
eV)
2
+ (0.511 10
6
eV)
2
E
tot
_
e

_
= 1.299 MeV
Problem 14.16
A (1116) hyperon with a kinetic energy of 2400 MeV decays in ight into a proton (938) and a pion (140).
The pion has its maximum laboratory energy when it is emitted in the same (forward) direction as the . What
is this maximum pion energy?
Solution
Be warned - I will skip muchos stepos in this problema. Our reaction is
p +
And before we get down to any funny business, we recall the following Lorentz invariant:
E
2
= p
2
c
2
+ (m
0
c
2
)
2
(1.9.38)
In terms of the four-vectors, and our element becomes:
E
2

p
2

c
2
=(E
p
+E

)
2
(p
p
c + p

c)
2
(m

c
2
)
2
=E
2
p
+ E
2

+ 2E
p
E

p
2
p
p
2

2(p
p
c)(p

c)
=
_
E
2
p
p
2
p

+
_
E
2

p
2

+ 2E
p
E

2(p
p
c)(p

c)
=(m
p
c
2
)
2
+ (m

c
2
)
2
+ 2 (E
p
E

(p
p
c)(p

c)) (1.9.39)
CHAPTER 1: ELECTRODYNAMICS II 87
Rearranging things a bit in (1.9.39) leads to
(m

c
2
)
2
(m
p
c
2
)
2
(m

c
2
)
2
=2 (E
p
E

(p
p
c)(p

c))
(m

c
2
)
2
(m
p
c
2
)
2
(m

c
2
)
2
2
=E
p
E

(p
p
c)(p

c) (1.9.40)
Now, we musnt forget that momentum is a vector, so that the scalar product on the RHS of (1.9.40)
must be appropriately altered:
(m

c
2
)
2
(m
p
c
2
)
2
(m

c
2
)
2
2
=E
p
E

(p
p
c)(p

c) cos (1.9.41)
If the pion is ejected forward, and the proton backwards, then cos(180
o
) = 1, so now the magnitudes
are equal (p
p
= p

= p), and (1.9.41) becomes


(m

c
2
)
2
(m
p
c
2
)
2
(m

c
2
)
2
2
=E
p
E

p
2
c
2
(m

c
2
)
2
(m
p
c
2
)
2
(m

c
2
)
2
2
=E
p
E

+E
2

(m

c
2
)
2
(m

c
2
)
2
(m
p
c
2
)
2
(m

c
2
)
2
2
=E

(E
p
+ E

) (m

c
2
)
2
(m

c
2
)
2
(m
p
c
2
)
2
(m

c
2
)
2
2
=E

(m

c
2
) (m

c
2
)
2
We in the home stretch now buddy!
E

(m

c
2
) =
(m

c
2
)
2
(m
p
c
2
)
2
+ (m

c
2
)
2
2
E

=
(m

c
2
)
2
(m
p
c
2
)
2
+ (m

c
2
)
2
2(m

c
2
)
(1.9.42)
Substituting in the appropriate values leads us to
E

= 172.6 MeV
Problem 14.17
The muon has a lifetime of = 2.2 10
6
sec. Muons are produced by cosmic rays hitting the upper atmo-
sphere at 80 km above the Earth. What energy must a muon have to be able to reach the Earth within its
lifetime? The fact that the large numbers of such muons do reach the Earth is a verication of the time dilation
prediction of special relativity.
Solution
Since the muons are traveling relativistically, then the particles will last longer by a vactor of , allowing
them to reach Earth before decaying.
W. Erbsen HOMEWORK #9
The rst step, then, is to nd the speed that our thrifty muons must travel at in order to achieve
this feat. We recall that v = x/t x = vt, where t = . We now have:
x =v
x =
v
_
1 v
2
/c
2
v =x
_
1 v
2
/c
2
v
2

2
=x
2
_
1
v
2
c
2
_
v
2

2
=x
2

v
2
x
2
c
2
x
2
=v
2

2
+
v
2
x
2
c
2
x
2
=v
2
_

2
+
x
2
c
2
_
v
2
=
x
2

2
+ x
2
/c
2
v =
x
_

2
+ x
2
/c
2
(1.9.43)
Hooray! Plugging in the appropriate values to (1.9.43) leads us to
v =
80 10
3
m
_
(2.2 10
6
s) + (80 10
3
m)
2
/ (2.998 10
8
m/s)
=2.998 m/s
=0.99997c (1.9.44)
The total energy is given by
E
tot
= m
o
c
2
(1.9.45)
If we recall that the rest mass energy of muon is 106 MeV, and substituting in the value obtained from
(1.9.44), we have
E
tot
() =
106 10
6
eV
_
1 (0.99997)
2
E
tot
() = 12.96 GeV
Chapter 2
Quantum Mechanics II
2.1 Homework #1
Exercise 9.3
In a two-slit interference experiment with particles of denite wave number (energy) k, the slits A and B are
located at positions r
A
and r
B
. At large distances from the slits, the amplitudes r[A) and r[B) are to reasonable
accuracy represented by r[A) e
ik|rAr|
and r[B) e
ik|rBr|
. Show how, to within a constant of proportional-
ity, the probability of nding the particle at position r depends on the dierence of the distances from A to B to
the point of observation, and on the relative magnitude and phase of the amplitudes A[) and B[), which are
determined by the experimental arrangement.
Solution
We rst recall that we can expand the coecients r[) as
r[) =r[A)A[) + r[B)B[) (2.1.1)
Finding the magnitude of (2.1.1),
[r[)[
2
=[r[A)A[) +r[B)B[)[
2
=[r[A)A[)[
2
+[r[B)B[)[
2
+ 2Re r[A)A[)r[B)B[) (2.1.2)
Where we note that we can write the following:
[r[A)A[)[
2
=(r[A)A[))

r[A)A[)
=A[r)[A)r[A)A[)
=[A)A[r)r
..
1
[A)A[)
=[A) A[A)
. .
1
A[)
=[A)A[)
=[A[)[
2
(2.1.3)
W. Erbsen HOMEWORK #1
Using the same logic as what lead to (2.1.3), we can also say that
[r[B)B[)[
2
=[B[)[
2
(2.1.4)
Using (2.1.3) and (2.1.4), we can now rewrite (2.1.2):
[r[)[
2
=[A[)[
2
+ [B[)[
2
+ 2Re r[A)A[)r[B)B[) (2.1.5)
We recall from the prompt that
r[A) e
ik|rAr|
(2.1.6a)
r[B) e
ik|rBr|
(2.1.6b)
Substituting (2.1.6a)-(2.1.6b) into (2.1.5), we have:
[r[)[
2
=[A[)[
2
+[B[)[
2
+ 2Re
_
e
ik|rAr|
A[)e
ik|rBr|
B[)
_
(2.1.7)
Lets assume that r
A
> r
B
, r
A
> r, r
B
< r. Therefore, we can rewrite (2.1.6a)-(2.1.6b) as
r[A) e
ik|rAr|
e
ik(rAr)
(2.1.8a)
r[B) e
ik|rBr|
e
ik(rrB)
(2.1.8b)
Using (2.1.8a)-(2.1.8b), (2.1.7) becomes
[r[)[
2
=[A[)[
2
+[B[)[
2
+ 2Re
_
e
ik(rAr)
A[)e
ik(rrB)
B[)
_
=[A[)[
2
+[B[)[
2
+ 2Re
_
e
ik(rAr)
e
ik(rrB)
A[)B[)
_
=[A[)[
2
+[B[)[
2
+ 2Re
_
e
ik(rArB)
A[)B[)
_
(2.1.9)
We now expand the cocients in the bracket in (2.1.9) as:
A[) =[A[e
iA
(2.1.10a)
B[) =[B[e
iB
(2.1.10b)
Substituting (2.1.10a)-(2.1.10b) back into (2.1.9), we have
[r[)[
2
=[A[)[
2
+[B[)[
2
+ 2Re
_
e
ik(rArB)
[A[e
iA
[B[e
iB
_
(2.1.11)
From (2.1.11) it is easy to see that
[r[)[
2
= [A[)[
2
+[B[)[
2
+ 2[A[[B[Re
_
e
ik(rArB)
e
i(A+B)
_
(2.1.12)
Recalling (2.1.10a)-(2.1.10b), we can dissect (2.1.12) a little more to increase its transparency:
[r[)[
2
=[A[
2
+ [B[
2
+ 2[A[[B[Re
_
e
ik(rArB)
e
i(A+B)
_
=[B[
2
_
[A[
2
[B[
2
+ 1 + 2
[A[
[B[
Re
_
e
ik(rArB)
e
i(A+B)
_
_
(2.1.13)
CHAPTER 2: QUANTUM MECHANICS II 91
Rewriting (2.1.13) once more, we see that
[r[)[
2
= [B[
2
_
[A[
2
[B[
2
. .
ii
+1 + 2
[A[
[B[
..
ii
Re
_
i
..
e
ik(rArB)
iii
..
e
i(A+B)
_
_
(2.1.14)
Where we let
i) The dierence of the distance from A to B to the point of observation
ii) The relative amplitude of A[) to B[)
iii) The relative phase of A[) to B[)
Exercise 9.7
Show that for a linear operator
(
b
, A
a
) =

j
b

i
A
ij
a
j
(2.1.15)
and write this equation in matrix form. If A were antilinear, how would the corresponding equation look?
Solution
We rst recall that Merzbacher writes his inner products in a funny way, and that we can rewrite (2.1.15)
as

b
[A[
a
) =

j
b

i
A
ij
a
j
(2.1.16)
Alrighty, through straightforward substitution, (2.1.16) becomes

b
[A[
a
) =
_

i
b
i

j
a
j

a
_
=

j
b
i

b
[ A[a
j

a
)
=

j
b

i
a
j

b
[A[
a
)
. .
A
ij
(2.1.17)

I will not allow myself to be apart of this notational charade no longer. Merzbachers notation in his book is inconsistent with
all works of any selected literatire, not to mention lectures. Its barbaric, and it scares me. Ergo, from hereonin I will adopt the
universally accepted form:
(a, c) a|c
In the future, all inner-products will have this form, as I choose to follow the convention that most others do, and how its taught in
other classes
W. Erbsen HOMEWORK #1
Noticing that the underbraced term in (2.1.17) is none other than the coecient A
ij
, and (2.1.17) becomes

b
[A[
a
) =

j
b

i
A
ij
a
j
(2.1.18)
If A is anti-linear, we rst must recall that such operators are governed by
A( ) =

A
And so, applying this to (2.1.16), we have

b
[A[
a
) =

j
b

i
A
ij
a

j
Exercise 9.8
If F
A
(
a
,
b
) is a complex-valued (scalar) functional of the two variables
a
and
b
with the linearity prop-
erties
F
A
(
a
+
c
) [
b
) =

F
A

a
[
b
) +

F
A

c
[
b
) (2.1.19a)
F
A

a
[(
b
+
c
) ) =F
A

a
[
b
) +F
A

a
[
c
) (2.1.19b)
Show that F
A
can be represented as an inner product.
F
A

a
[
b
) =
a
[A[
b
) (2.1.20)
For every
a
and
b
, and that (2.1.20) denes a linear operator A uniquely (compare with Exercise 9.4).
Solution
Essentially, it all boils down to this: we are given both (2.1.19a) and (2.1.19b), and we wish to show that
F
A
can be represented as some inner product, as given in (2.1.19a). Capiche?
Lets purposely turn this around a little bit. Assuming that (2.1.19a) and (2.1.19b) represent ab-
solute truth and salvation, we may hypothesize that (2.1.20) is true, and possibly verify this by way of
substituting into said equation from (2.1.19a)-(2.1.19b).
For ease of the picken hands we reckon that itll be easiest if we apply our linear operator to only two
vectors (any two vectors). Irregardless, it aint gonna be so bad, so hold on to yer britches and saddle
on up cowboy!
Lets apply these basic principles to (2.1.20):
F
A

a
[
b
) =
_

i
a
i

b
_
[Using (2.1.19a)]
=

i
a

i
F
A

i
[
b
) (2.1.21)
CHAPTER 2: QUANTUM MECHANICS II 93
Continuing on to the ket in (2.1.21),
F
A

a
[
b
) =

i
a

i
F
A
_

j
b
j

j
_
[Using (2.1.19b)]
=

j
a

i
F
A
b
j

i
[
j
) (2.1.22)
We now innocently recall (2.1.18) from a previous problem, which says that

b
[A[
a
) =

j
b

i
A
ij
a
j

a
[A[
b
) =

j
a

i
A
ij
b
j
(2.1.23)
Using (2.1.23), we can rewrite (2.1.22), and nd that:
F
A

a
[
b
) =
b
[A[
a
) (2.1.24)
Where in proclaiming (2.1.24), we have dened:
A
ij
=
i
[A[
j
) (2.1.25)
Applied to our particular case, (2.1.25) becomes
A
ij
=
i
[F
A
[
j
) A
ij
= F
A

i
[
j
) (2.1.26)
The million dollar question is then, how exactly can our linear operator F
A
be uniquely represented
by A? From (2.1.24), we can see that F
A
does indeed create a matrix, whose elements are compactly
expressed in bra-ket notation as (2.1.25). Furthermore, from our statement (2.1.26), we can see that the
entire set A
ij
does indeed uniquely represent F
A
.
If we wanted to be a little more thorough, we could employ a proof by contradiction. We wish to
prove that (2.1.26) is unique, which we can do by proving that it is not unique by way of, for lack of a
better term, contradiction. If we have two linear operators, which we assume are unique (eg, A ,= B).
Then (2.1.26) becomes
F
A
i[j) =i[A[j)
=i[B[j)
=i[A B[j)
=0 (2.1.27)
We can see that (2.1.27) implies that A = B, which is a contradiction of our earlier statement.
It would appear as though (9.41) also denes uniquely, although via Rieszs representation theorem:
f
a
() =
a
[)
W. Erbsen HOMEWORK #1
Exercise 9.13
Prove that products of unitary operators are also unitary.
Solution
We rst recall that a unitary operator is dened by
U

= U
1
(2.1.28)
And also that for unitary operators,
I = U

U = UU

= 1 (2.1.29)
And also, we mustnt forget that
(AB)
1
= B
1
A
1
(2.1.30)
So, given two unitary operators A and B, and using (2.1.28)-(2.1.30), we have
(AB)

(AB) =(AB)
1
(AB)
=B
1
A
1
AB
=B
1
_
A
1
A
_
B
=B
1
(1)B
=B
1
B
From this it is easy to see that
(AB)

(AB) = 1
Exercise 9.17
Show that under an active unitary transformation, the following conditions hold:
a) A Hermitian operator remains Hermitian
b) A unitary operator remains unitary
c) A normal operator remains normal
d) A symmetric matrix does not in general remain symmetric
Solution
CHAPTER 2: QUANTUM MECHANICS II 95
a) We recall that the dening property of Hermitian operators is that A

= A. With this in mind,


A = S

AS
_
A
_

=
_
S

AS
_

= S

S = S

AS (2.1.31)
So, we can summarize (2.1.31) with
A =
_
A
_

= S

AS
b) Unitary operators are dened by AA

= 1, and given that A = S

AS, we can say that


AA

=
_
S

AS
_

_
S

AS
_
=
_
S

S
_

_
S

AS
_
=
_
S
1
A
1
S
_

_
S
1
AS
_
=S
1
A
1
SS
1
. .
1
AS
=S
1
A
1
A
. .
1
S
=S
1
S (2.1.32)
From (2.1.32) we can straightfowardly proclaim that
AA

= 1
c) Normal operators have the unique property that AA

= A

A, and using A = S

AS as before, we
can say that
A

A =
_
S

AS
_

_
S

AS
_
=
_
S

S
_ _
S

AS
_
=
_
S

AS
_ _
S

S
_
(2.1.33)
From (2.1.33) it is easy to see that
A

A = AA

d) We are trying to show that a symmetric matrix does not remain symmetric under an active unitary
transformation, eg that A ,=

A. Once again using the fact that A = S

AS, we have:

A =
_

AS
_
=
_

S
__

A
_
=

S

A

(2.1.34)
Noting that

S

= S

= S

, (2.1.34) becomes

A =

S

AS

(2.1.35)
Noting that (2.1.35) does not directly correspond with our previous denition of A, we can say that

A ,= A
W. Erbsen HOMEWORK #1
Exercise 9.18
Show by the use of bra-ket notation that
Tr A =

i
K
i
[A[K
i
) (2.1.36)
is independent of the choice of the basis [K
i
), and that
Tr AB = Tr BA (2.1.37)
Solution
It is possible (and convenient) to show that (2.1.36) and (2.1.37) are true in the same process. We can
do this by rst assuming that (2.1.36) is true, and working towards (2.1.37):
Tr AB =

i
K
i
[AB[K
i
)
=

j
K
i
[A[K
j
)K
j
[B[K
i
)
=

j
K
j
[B[K
i
)K
i
[A[K
j
)
=

j
K
j
[B([ K
i
)K
i
. .
1
[A[K
j
) (2.1.38)
From (2.1.38) it is easy to see that
Tr AB = Tr BA
Therefore, both (2.1.36) and (2.1.37) are true.
Problem 9.1
If
n
(r) is the normalized eigenfunction of the time-independent Schr odinger equation, corresponding to energy
eigenvalue E
n
, show that [
n
(r)[
2
is not only the probability density for the coordinate vector r, if the system
has energy E
n
, but also conversely the probability of nding the energy to be E
n
, if the system is known to be at
position r.
Solution
The rst part of this question is rather straightforward; we rst recall that the time-independent
Schr odinger equation is given, of course, by
H
n
(r) = E
n

n
(r) (2.1.39)
CHAPTER 2: QUANTUM MECHANICS II 97
Where in (2.1.39) we are in the position representation. We can expand into the position representation
from the arbitrary representation as follows:
[
n
) =

i
[r
i
)r
i
[
n
) (2.1.40)
Where the inner product in (2.1.40) is the wave function projected into the position basis, which can
then be projected into energy representation:
r
i
[
n
) =

j
r
i
[E
j
)E
j
[
n
) (2.1.41)
Using (2.1.41), we can rewrite (2.1.40) as:
[
n
) =

j
[r
i
) [r
i
[E
j
)E
j
[
n
)]
=

j
([r
i
)r
i
[) [E
j
)E
j
[
n
)
=

j
(1) [E
j
)E
j
[
n
)
=

j
[E
j
)E
j
[
n
) (2.1.42)
Using (2.1.42), we can now nd [
n
(r)[
2
:
[
n
(r)[
2
=

i
[r
i
)r
i
[
n
)

2
, [
n
(r)[
2
=

j
[E
j
)E
j
[
n
)

2
(2.1.43)
The results shown in (2.1.43) are actually quite profound: given an arbitrary state [
n
) we can equivalently
express it in terms of energy eigenstates and also in terms of position eigenstates.
Problem 9.2
Using the momentum representation, calculate the bound-state energy eigenvalue and the corresponding eigen-
function for the potential V (x) = g(x) (for g > 0). Compare with the results in Section 6.4.
Solution
We rst note that by their very nature, bound-state energies are negative (E < 0). Furthermore, the
TISE becomes
(T +V ) (x) = E(x) (2.1.44)
Where the kinetic and potential energies are given by
W. Erbsen HOMEWORK #1
T(p) =
p
2
2m
, V (x) = g(x)
We also know from Merzbacher that
(x) =
1

2
_

exp
_

px
_
(p) dp (2.1.45a)
(p) =
1

2
_

exp
_
i

px
_
(x) dx (2.1.45b)
Using our denition for the potential energy V (x), we can now say that V (p) becomes
V (p) =
1

2
_

exp
_
i

px
_
(g(x)) dx
=
g

2
(2.1.46)
Which comes from the dening property of the delta function. We can now rewrite the TISE from
(2.1.44) as
(T(p) +V (x)) (x) = E(x) (T(p) E(x)) (x) =V (x)(x)
_
p
2
2m
E
_
(p) =
1

2
_

V (p)(p) dp
=
_
1

2
__

2
__

(p) dp
=
g
2
_

(p) dp (2.1.47)
Innocently multiplying both sides of (2.1.47) by 2m, we are left with
_
p
2
2mE
_
(p) =
gm

(p) dp (2.1.48)
Recalling that the energy is negative, so letting E E, (2.1.48) becomes
_
p
2
+ 2mE
_
(p) =
gm

(p) dp (2.1.49)
Now, lets let the RHS of (2.1.49) be equal to some number, call it a. Then, we have
_
p
2
+ 2mE
_
(p) = a (p) =
1
p
2
+ 2mE
a (2.1.50)
Where we have set
a =
gm

(p) dp (2.1.51)
Substituting (2.1.50) into (2.1.51),
a =
gm

a
p
2
+ 2mE
dp (2.1.52)
CHAPTER 2: QUANTUM MECHANICS II 99
Lets let b
2
= 2mE. Then (2.1.52) becomes
a =
gm

a
p
2
+b
2
dp (2.1.53)
The proper way to complete the integral in (2.1.54) (analytically) is to cast it into the complex plane,
and undergo a contour integration:
a =
gma

_
C
1
z
2
+ b
2
dz (2.1.54)
The integrand in (2.1.54) can be rewritten as
f(z) =
1
(z + ib) (z ib)
(2.1.55)
From which it is easy to see that in the complex plane, we have singularities at z
0
= ib and z
0
= ib.
Recalling from The Residue Theorem, for a pole of order one,
Res (z
0
) = (z z
0
) f(z)

z=z0
(2.1.56)
Since in contour integration, the name of the game is to integrate over either the upper or the lower
hemisphere. Since there is only one singularity in both, it makes no dierence which we choose. I choose
to integrate over the upper hemisphere, since up is my favorite color. So, applying (2.1.56) at z
0
= ib,
we have
Res (ib) = (z ib)
1
(z +ib) (z ib)

z=ib
=
1
z + ib

z=ib
=
1
2ib
(2.1.57)
So, the integral of our integrand over the complex plane becomes
_
C
1
z
2
+b
2
dz =2i
_
1
2ib
_
=

b
(2.1.58)
Since from The Residue Theorem,
_
f(z) dz = 2i

Res. From (2.1.58), we now see that (2.1.54)


becomes
a =
gma

b
_
(2.1.59)
Recalling that b
2
= 2mE, (2.1.59) becomes
a =
gm

2mE
a (2.1.60)
In order for (2.1.60) to work, we must solve for E, and noticing that a is washed neatly from our hands,
we are left with:
W. Erbsen HOMEWORK #1
1 =
gm

2mE
E =
g
2
m
2
2
(2.1.61)
Now, substituting this value of E into (p) from (2.1.50), we have
(p) =
1
p
2
g
2
m
2
/
2
a (2.1.62)
In order to nd a in (2.1.62), we must normalize our wave function:
[(p)[
2
= a
2
_

_
1
p
2
g
2
m
2
/
2
_
2
dp = 1 (2.1.63)
Letting b = gm/, (2.1.63) becomes
1 = a
2
_

_
1
p
2
+b
2
_
2
dp (2.1.64)
We must once again employ contour integration, this time applied to (2.1.64). With this, the integrand
becomes
f(z) =
1
(z
2
+ b
2
)
2
=
1
(z +ib)
2
(z ib)
2
(2.1.65)
Now, since n = 2 in this case, the residues are given by
Res (z
0
) =
d
dz
(z z
0
)
2
f(z)

z=z0
(2.1.66)
Again integrating over the upper half plane, we can apply (2.1.66) to our integrand at z
0
= ib like:
Res (ib) =
d
dz
(z ib)
2
1
(z +ib)
2
(z ib)
2

z=ib
=
d
dz
1
(z + ib)
2

z=ib
=
2
(z + ib)
3

z=ib
=
2
(2ib)
3
(2.1.67)
Now, in accordance to the residue theorem,
_
C
1
(z + ib)
2
(z ib)
2
dz =2i
_

2
(2ib)
3
_
=

2b
3
(2.1.68)
CHAPTER 2: QUANTUM MECHANICS II 101
Recalling from our previous denition that b =

2mE , (2.1.68) becomes

2b
3
=

2 (2mE)
3


16m
3
E
3
(2.1.69)
And continuing from (2.1.64), we have
1 = a
2
_

_
1
p
2
+ b
2
_
2
dp a =
_
16m
3
E
3

(2.1.70)
And nally, from (2.1.70), we can now rewrite (2.1.62) as
(p) =
_
16m
3
E
3

1
p
2
2mE
(E < 0) (2.1.71)
To compare my results with 6.4: we see that my result for the eigenenergies, as seen in (2.1.61), is
identical to Merzbachers (6.54).
2.2 Homework #2
Exercise 10.9
What are the eigenvalues of the kinetic and potential energy operators of the harmonic oscillator? Explain why
these dont add up to the (discrete) eigenvalues of the total energy.
Solution
The Hamiltonian for the Harmonic Oscillator is given by
H =
1
2m
p
2
. .
T
+
m
2
2
x
2
. .
V
(2.2.1)
Where both T and V can be expressed in terms of the ladder operators a

and a:
T[
n
) =
1
2m
p
2
[
n
) (2.2.2)
=
1
2m
_
i
_
m
2
(a

a)
_
2
[
n
)
=
1
2m
m
2
(a

a)
2
[
n
)
=

4
(a
2
a

a aa

+a
2
)[
n
)
W. Erbsen HOMEWORK #2
=

4
(a
2
+ a
2
(a

a + aa

))[
n
) (2.2.3)
Now recall that [a, a

] = 1 aa

= a

a + 1, so (2.2.3) becomes
T[
n
) =

4
(a
2
+ a
2
(a

a +a

a + 1))[
n
)
=

4
_
(a
2
+ a
2
)[
n
) (2a

a + 1)[
n
)

=

4
_
(a
2
+ a
2
)[
n
) (2(a

a)[
n
) +[
n
))

=

4
_
(a
2
+ a
2
)[
n
) (2n)[
n
) + [
n
))

=

4
_
(a
2
+ a
2
) (2n + 1)

[
n
)
=

4
_
(a
2
+a
2
) + (2n + 1)

[
n
)
The eigenvalue for the kinetic energy operator is then dened by
T[
n
) =

4
_
(a
2
+a
2
) + (2n + 1)

[
n
) (2.2.4)
And now lets do the same thing for the potential energy operator V :
V [
n
) =
m
2
2
x
2
[
n
) (2.2.5)
=
m
2
2
__

2m
(a

+ a)
_
2
[
n
)
=
m
2
2

2m
(a

+ a)
2
[
n
)
=

4
_
a
2
+ a

a + aa

+ a
2
_
[
n
)
=

4
_
(a
2
+a
2
)[
n
) + (2a

a + 1)[
n
)

4
_
(a
2
+a
2
)[
n
) + (2n)[
n
) +[
n
))

4
_
(a
2
+a
2
) + (2n + 1)

[
n
)
And nally, the dening equation for the eigenvalue for the potential energy operator is given by
V [
n
) =

4
_
(a
2
+ a
2
) + (2n + 1)

[
n
) (2.2.6)
At this point it should be mentioned that I have thoughtlessly used the same eigenstate for both
(2.2.2) and (2.2.5). This would imply that both T and V are simultaneously observable, which is clearly
nonsense. T depends explicitly on p and V on x, and one of the most fundamental results of quantum
theory is that [x, p] ,= 0.
CHAPTER 2: QUANTUM MECHANICS II 103
But in nding the eigenvalues for T and V I made no mention that the eigenvalues were being obtained
simultaneously. And because the operators are not acting simultaneously, it follows that the eigenvalues
can not be collected at the same time. Therefore we cannot add the eigenvalues of T and V to get H.
Exercise 10.12
Transcribe (10.77) and (10.88) in the coordinate (q) representation and calculate q

[n) from these dierential


equations. Using the mathematical tools of 5.3, verify (10.96).
Solution
We rst note that from (10.88) in Merzbacher, that

n
= [n)
1

n!
_
a

_
n

0
(2.2.7)
Where we note that the vacuum state is dened as
0
= [0). Furthermore, we note that (10.77) from
Merzbacher reads:
a[
0
) = 0 (2.2.8)
In words, (2.2.8) means that we cannot lower the lowest state any lower. We recall from (10.69) and
(10.71) that
a =
_
m
2
_
q + i
p
m
_
(2.2.9a)
a

=
_
m
2
_
q i
p
m
_
(2.2.9b)
Applying (2.2.9a) to (2.2.8), we have:
_
m
2
_
q +i
p
m
_
[
0
) =0
_
q +i
p
m
_
[
0
) =0 (2.2.10)
At this point, we let [
0
)
0
(q), and also recalling that p = /i /q, (2.2.10) becomes:
_
q +i
1
m


i

x
_

0
(q) =0
_
q +

m

x
_

0
(q) =0 (2.2.11)
Assuming that
0
(q) is only dependent on q, then d, and (2.2.11) becomes:
_
q +

m
d
dx
_

0
(q) =0
W. Erbsen HOMEWORK #2
q
0
(q) +

m
d
0
(q)
dq
=0
d
0
(q)
dq
+
m

q
0
(q) =0 (2.2.12)
Using the method of integrating factors, we can see that:
(q) =exp
__
m

q dq
_
=exp
_
m
2
q
2
_
(2.2.13)
Multiplying (2.2.12) by our integrating factor (q) from (2.2.13),
exp
_
m
2
q
2
_
d
0
(q)
dq
+ exp
_
m
2
q
2
_
m

q
0
(q) = 0 (2.2.14)
It is easy to see that (2.2.14) will only work if we require that:

0
(q) = C exp
_

m
2
q
2
_
(2.2.15)
To nd the constant C, we must normalize (2.2.15):
_

_
C exp
_

m
2
q
2
__

_
C exp
_

m
2
q
2
__
dq =1
C
2
_

exp
_

q
2
_
dq =1
C
2
_


m
=1 (2.2.16)
From (2.2.16) it is easy to see that the normalization constant, C, is given by:
C =
_
m

_1
/4
(2.2.17)
Substituting (2.2.17) back into (2.2.15), we have:

0
(q) =
_
m

_1
/4
exp
_

m
2
q
2
_
(2.2.18)
In order to utilize the tools of 5.3, we rst recall that the harmonic oscillator wave function is given
from (5.27) as

n
(q) = C
n
H
n
__
m

q
_
exp
_

m
2
q
2
_
(2.2.19)
Recalling that Merzbacher makes the following denition,
=
_
m

q (2.2.20)
According to (2.2.20), we can rewrite (2.2.19) as
CHAPTER 2: QUANTUM MECHANICS II 105

n
(q) = C
n
H
n
() exp
_

2
2
_
(2.2.21)
From (2.2.21) we apply ladder operators and arrive at a dierential equation, which may be integrated
and normalized to yield (10.96), with the aid of the recursion relations derived in Appendix B.
Exercise 10.14
Show that for any coherent state [),
R

[) = C

[e
i
) (2.2.22)
Where [e
i
) is again a coherent state and C

is a phase factor. Interpret the meaning of this result in the complex


eigenvalue plane (See Fig. (10.1)).
Figure 2.1: This is Fig. 10.1 from Merzbacher
Solution
We rst recall that (10.105) and (10.106) from Merzbacher says that, respectively:
R

=exp
_
ia

(2.2.23a)
R

aR

=exp[i]a (2.2.23b)
W. Erbsen HOMEWORK #2
Since R

is unitary, we can manipulate (2.2.23b) as:


R

_
R

aR

= exp [i] a
_
aR

=R

exp [i] a (2.2.24)


Multiplying both sides of (2.2.24) by [) from the right,
aR

[) = exp [i] R

a[) (2.2.25)
Recalling from (10.97) that a[) = [), (2.2.25) becomes
aR

[) = exp [i] R

[) aR

[) = exp [i] R

[) (2.2.26)
From (2.2.26), we can denitively say that R

[) is an eigenvector of a with eigenvalue exp [i].


Therefore, R

[) creates another coherent state [exp [i] ) that is oset by some phase factor, which
Merzbacher denotes as C

. Therefore, we can say that:


R

[) = C

[e
i
)
It appears as though the unitary operator R

is appropriately denoted - it seems to rotate some initial


coherent state [) by some angle within the complex plane (See Figure 10.1 above).
Exercise 10.20
For a coherent state [), evaluate the expectation value of the number operator a

a, its square and its vari-


ance, using the commutation relation from (10.72). Check the results by computing the expectation values of n,
n
2
, and (n)
2
directly from the Poisson distribution, (10.110).
Solution
The commutation relation is [a, a

] = aa

a = 1. We also know that


a[) = [) (2.2.27)
Where [) is the coherent state. First lets nd the expectation value of N:
N) =[N[)
=[a

a[)
=
_
[a

_
(a[))
=

=[[
2
(2.2.28)
And similarly, we now nd the expectation value of N
2
:
N
2
) =[(a

a)
2
[)
CHAPTER 2: QUANTUM MECHANICS II 107
=[a

aa

a[)
=[a

(aa

)a[)
=[a

(a

a + 1)a[)
=[a

aa[) + [a

a[)
=[a
2
a
2
[) + [[
2
=
2

2
+[[
2
=[[
4
+[[
2
(2.2.29)
Where I used the commutation relation and also the result from (2.2.28). Now to nd the variance we
insert (2.2.28) and (2.2.29) into the typical equation:
N
2
=N
2
) N)
2
=[[
4
+[[
2
([[
2
)
2
=[[
4
+[[
2
[[
4
=[[
2
(2.2.30)
The results (2.2.28), (2.2.29) and (2.2.30) constitute the answer to the rst part of the question. Now
we are asked to nd the same quantities using the Poisson distribution:
P
n
() = e
||
2 [[
2n
n!
(2.2.31)
And recall that if we expand the coherent state [) in terms of the number operator eigenstates [n) we
get:
[) =

n=0
[n)e
||
2
/2

n

n!
(2.2.32)
Which is (10.109) on pg. 227 of Merzbacher. We can then say that
[ =

n=0
n[e
||
2
/2

n

n!
(2.2.33)
Then to nd the expectation value of the number operator, we sandwich n between (2.2.33) and (2.2.32),
respectively:
n) =[n[)
=([) (n[))
=
_

n=0
n[e
||
2
/2

n

n!
__

n=0
n[n)e
||
2
/2

n

n!
_
=e
||
2

n=0
n

n
n!
n[n)
=e
||
2

n=0
n
[[
2n
n!
(2.2.34)
W. Erbsen HOMEWORK #2
Now we must jiggle this equation around a little to arrive at the solution. First it is important to notice
that
n
n!
=
n
1 2 3 ... n 1 n
=
1
1 2 3 ... n 1
=
1
(n 1)!
(2.2.35)
Also recall the power series expansion of e
x
2
e
x
2
=

k=0
x
2k
k!
(2.2.36)
Applying (2.2.35) and (2.2.36) to n) in (2.2.34),
n) =e
||
2

n=0
n
[[
2n
n!
=[[
2
e
||
2

n=0
[[
2(n1)
(n 1)!
(2.2.37)
Letting k = n 1,
n) =[[
2
e
||
2

k=0
[[
2k
k!
=[[
2
e
||
2
e
||
2
=[[
2
(2.2.38)
Which agrees with (2.2.28). Now lets do the same thing for n
2
). Using the results from (2.2.34):
n
2
) =

n=0
n
2
[[
2n
n!
=

n=0
n
[[
2n
(n 1)!
(2.2.39)
Now go ahead and shift the index, making this easier to solve. Let n = k + 1 k = n 1:
n
2
) =e
||
2

k=0
(k + 1)
[[
2(k+1)
k!
=e
||
2

k=0
k
[[
2(k+1)
k!
+e
||
2

k=0
[[
2(k+1)
k!
=e
||
2
[[
4

k=0
[[
2(k1)
(k 1)!
+ e
||
2
[[
2

k=0
[[
2k
k!
CHAPTER 2: QUANTUM MECHANICS II 109
=e
||
2
[[
4
e
||
2
+ e
||
2
[[
2
e
||
2
=[[
4
+ [[
2
(2.2.40)
Which agrees with (2.2.29).
Problem 10.2
Assuming a particle to be in one of the stationary states of an innitely high one-dimensional box, calculate
the uncertainties in position and momentum, and show that they agree with the Heisenberg uncertainty relation.
Also show that in the limit of very large quantum numbers, the uncertainty in x equals the root-mean-square
deviation of the position of a particle moving in the enclosure classically with the same energy.
Solution
Assuming that our innite potential well has a length denoted by a, the (normalized) eigenfunctions are
given by

n
(x) =
_
2
a
sin
_
nx
a
_
(2.2.41)
To nd the uncertainties in both position and momentum in the quantum regime, we must calculate the
standard deviation of each quantity
x =
_
x
2
) x)
2
, p =
_
p
2
) p)
2
(10.2.2a,b)
So, our task is to calculate x), x
2
), p), and p
2
). In these calculations I will be using two integration
techniques: integration by parts and trig substitutions. In particular I will use sin
2
(ax) = 1/2(1
cos(2ax)) and also
_
sin(ax) cos(ax) dx = 1/2a sin
2
(ax). Lets go ahead and calculate x) rst:
x) =
_
a
0

n
(x)x
n
(x) dx
=
2
a
_
a
0
x sin
2
_
nx
a
_
dx
=
2
a
_
a
0
x
_
1
2

1
2
cos
_
2nx
a
__
dx
=
1
a
__
a
0
x dx
_
a
0
x cos
_
2nx
a
_
dx
_
=
1
a
_
a
2
2

_
a
0
x cos
_
2nx
a
_
dx
_
=
1
a
_
a
2
2

__
xa
2n
sin
_
2nx
a
_

a
0

a
2n
_
a
0
sin
_
2nx
a
_
dx
__
=
1
a
_
a
2
2

_
a
2
2n
sin(2n) +
_
a
2n
_
2
_
cos
_
2nx
a
_

a
0
__
W. Erbsen HOMEWORK #2
=
1
a
_
a
2
2

_
_
a
2n
_
2
(cos(2n) cos(0))
__
=
1
a
_
a
2
2
_
=
a
2
(2.2.3)
This is not surprising, and in fact this fact coincides with the classical result, which I discuss at the end
of this problem. Now lets do the same for x
2
):
x
2
) =
_
a
0

n
(x)x
2

n
(x) dx
=
2
a
_
a
0
x
2
sin
2
_
nx
a
_
dx
=
2
a
_
a
0
x
2
_
1
2

1
2
cos
_
2nx
a
__
dx
=
2
a
__
a
0
x
2
2
dx
_
a
0
x
2
2
cos
_
2nx
a
_
dx
_
=
2
a
_
a
3
6

1
2
_
a
0
x
2
cos
_
2nx
a
_
dx
_
=
2
a
_
a
3
6

1
2
__
ax
2
2n
sin
_
2nx
a
_

a
0

a
n
_
a
0
x sin
_
2nx
a
_
dx
__
=
2
a
_
a
3
6
+
a
2n
__
a
0
x sin
_
2nx
a
_
dx
__
=
2
a
_
a
3
6
+
a
2n
__
ax
2n
cos
_
2nx
a
_

a
0
+
a
2n
_
a
0
cos
_
2nx
a
_
dx
__
=
2
a
_
a
3
6
+
a
2n
_
a
2
2n
__
=
2
a
_
a
3
6

a
3
4n
2

2
_
=
a
2
3

a
2
2n
2

2
(2.2.4)
This is not immediately intuitive, but it is possible to draw understanding from it when comparing it to
the classical analogue. Now lets head o to calculate the momentum expectation value p):
p) =
_
a
0

n
_

i
d
dx
_

n
(x) dx
=
2
a

i
_
a
0
sin
_
nx
a
_
d
dx
sin
_
nx
a
_
dx
=
2
a

i
n
a
_
a
0
sin
_
nx
a
_
cos
_
nx
a
_
dx
=
2
a

i
n
a
_
1
2a
sin
2
_
nx
a
_

a
0
=0 (2.2.5)
CHAPTER 2: QUANTUM MECHANICS II 111
We could have guessed this result since
n
(x) is real. On the other hand, p
2
) is not zero:
p
2
) =
_
a
0

n
_

i
d
dx
_
2

n
(x) dx
=
2
a
_

i
_
2
_
a
0
sin
_
nx
a
_
_
d
dx
_
2
sin
_
nx
a
_
dx
=
2
a
_

i
_
2
n
a
_
a
0
sin
_
nx
a
_
_
d
dx
_
cos
_
nx
a
_
dx
=
2
a
()
2
_
n
a
_
2
_
a
0
sin
2
_
nx
a
_
dx
=
2
a
()
2
_
n
a
_
2
_
a
0
_
1
2

1
2
cos
_
2nx
a
__
dx
=
2
a
()
2
_
n
a
_
2
_
1
2
_
a
0
dx
1
2
_
a
0
cos
_
2nx
a
_
dx
_
=
2
a
()
2
_
n
a
_
2
_
a
2
_
=

2
n
2

2
a
2
(2.2.6)
So now we can go ahead and compute the uncertainties in position and momentum by substituting (2.2.3)
and (2.2.4) into (10.2.2a) and also (2.2.5) and (2.2.6) into (10.2.2b), respectively. Starting rst with the
position,
x =
_
x
2
) x)
2
=

_
a
2
3

a
2
2n
2

2
_

_
a
2
_
2
=
_
a
2
3

a
2
2n
2

2

a
2
4
2
x =
_
a
2
12

a
2
2n
2

2
(2.2.7)
And now the momentum,
p =
_
p
2
) p)
2

2
n
2

2
a
2
_
0 p =
n
a
(2.2.8)
To check to see if these results are consistent with the Heisenberg uncertainty relation, multiply (2.2.7)
and (2.2.8):
xp =
_
a
2
12

a
2
2n
2

2
_
1
/2
_

2
n
2

2
a
2
_
1
/2
=
_

2
n
2

2
12


2
2
_
1
/2
=

2
_
n
2

2
3
2
_
1
/2
(2.2.9)
W. Erbsen HOMEWORK #2
The portion of (2.2.9) in the parenthesis is guaranteed to be greater than 1, thus Heisenbergs uncertainty
relation is satised since xp /2. To see this lets plug in some conservative numbers. Let n = 1
(ground state), and = 3. Then (2.2.9) says that
xp =

2
_
n
2

2
3
2
_
1
/2
=

2
_
(1)
2
(3)
2
3
2
_
1
/2
=

2
(1)
1
/2
=

2
Which is the minimum uncertainty. Since we underestimated , (2.2.9) is certain to be greater than /2,
which satises the Heisenberg uncertainty principle. In the classical analogue, the particle can be found
anywhere between 0 and a. This is an example of a continuous uniform distribution, whose mean and
variance are dened by ?
=
(a +b)
2

2
=
(b a)
2
12
And in our case
x =
_
a
2
12
x =
a
2

3
(2.2.10)
Which is the root-mean-square deviation. Now lets compare this with the quantum result (2.2.7) as
n
x =
_
a
2
12

a
2
2n
2

2

_
a
2
12

1

x =
a
2

3
(2.2.11)
Which agrees with (2.2.10).
Problem 10.5
Rederive the one-dimensional minimum uncertainty wave packet by using the variational calculus to minimize
the expression I = (x)
2
(p)
2
subject to the condition
_
[[
2
dx = 1 (2.2.12)
Show that the solution of this problem satises a dierential equation which is equivalent to the Schr odinger
equation for the harmonic oscillator, and calculate the minimum value of xp.
CHAPTER 2: QUANTUM MECHANICS II 113
Solution
Using variational calculus, we wish to comploy constraint optimization by way of Lagrange Multipliers
to minimize the action of:
I = (x)
2
(p)
2
(2.2.13)
Subject to the constraint:
[[)[
2
= 1 [) = 1 (2.2.14)
The rst step is to recall that we may express the variance of the position, (x)
2
, as:
(x)
2
=
_
(x x))
2
_
= = x
2
) x)
2
=[x
2
[) [[x[)[
2
(2.2.15)
In accordance with variational calculus, in order to minimize (2.2.13) subject to the constraint from
(2.2.14), we must require that:
I F
1
= 0 (2.2.16)
Where is the so-called Lagrange Multiplier, while F
1
is our rst constraint (we only have one in this
problem), given by (2.2.14). Substituting in the appropriate values into (2.2.16), we have:

_
(x)
2
(p)
2
[)
_
= 0 (2.2.17)
Where denotes a small variation. We can vary our dummy variable as:
+ (2.2.18)
Where in (2.2.18) is completely arbitrary (this property will become useful in due course). Applying
this methodology, we can modify the (arbitrary) state vectors in precisely the same way:
[) [) +[) (2.2.19a)
[ [ +[ (2.2.19b)
Where we note that (2.2.19a) and (2.2.19b) are independent. We are now in the optimal position to
rewrite (2.2.17):

_
(x)
2
(p)
2
_
[) =0
(x)
2
(p)
2
+ (p)
2
(x)
2
[)
. .
=0 (2.2.20)
Before we go any further, we must nd the variation of our condition, denoted by the underbraced term
in (2.2.20). Accordingly, from (2.2.19a), this becomes:
[) =[ +) [)
=[) +[) [)
=[) (2.2.21)
W. Erbsen HOMEWORK #2
Substituting (2.2.21) back into (2.2.20), we have:
(x)
2
(p)
2
+ (p)
2
(x)
2
[) =0 (2.2.22)
At this point, we must nd out what the variation of the variances in (2.2.22) are in turn. Starting with
the position, using (2.2.15), we have:
(x)
2
=
_
[x
2
[) [[x[)[
2
_
=
_
[x
2
[)
_

_
[[x[)[
2
_
(2.2.23)
Where, we note that it is possible to write:
[x
2
[) =[x
2
[ +) [x
2
[)
=[x
2
[) +[x
2
[) [x
2
[)
=[x
2
[) (2.2.24)
Similarly, we can say that:
[[x[)[
2
= [x[) [x[)
=[x[)[x[) +[x[)[x[)
=2[x[)[x[)
=2[x[) [x[ + ) [x[)
=2[x[) [x[) + [x[) [x[)
=2[x[)[x[) (2.2.25)
We can now substitute (2.2.24) and (2.2.25) back into (2.2.23), which yields:
(x)
2
=[x
2
[) 2[x[)[x[)
=[x
2
[) 2x)[x[)
=

x
2
2x)x

_
(2.2.26)
Following the same steps which led is to (2.2.26), we can make an analagous statement for the variation
of the variance in momentum:
(p)
2
=

p
2
2p)p

_
(2.2.27)
Substituting our results from (2.2.26) and (2.2.27) back into (2.2.22), we have:
(x)
2
_

p
2
2p)p

_ _
+ (p)
2
_

x
2
2x)x

_ _
[) =0
_

(x)
2
_
p
2
2p)p
_
+ (p)
2
_
x
2
2x)x
_


_
=0 (2.2.28)
Multiplying (2.2.28) on the left by x[) and on the right by [), we nd that:
_
x

(x)
2
_
p
2
2p)p
_
+ (p)
2
_
x
2
2x)x
_


_
=0 (2.2.29)
We recall that x[) the arbitrary vector [) in the basis of [x). Therefore, from (2.2.29), we can
wright:
CHAPTER 2: QUANTUM MECHANICS II 115
_
(x)
2
_
p
2
2p)p
_
+ (p)
2
_
x
2
2x)x
_

_
(x) =0
(x)
2
_
p
2
2p)p
_
(x) +
_
(p)
2
_
x
2
2x)x
_

_
(x) =0 (2.2.30)
At this point, we innocently recall that the momentum operator is dened by:
p =

i

x
(2.2.31)
Applying (2.2.31) to (2.2.30), we have:
(x)
2
_
_

x
_
2
2p)
_

x
_
_
(x) +
_
(p)
2
_
x
2
2x)x
_

_
(x) =0
(x)
2
_

2

2
x
2
(x)
2
i
p)

x
(x)
_
+
_
(p)
2
_
x
2
2x)x
_

_
(x) =0
(x)
2
_

(x)
2
i
p)

(x)
_
+
_
(p)
2
_
x
2
2x)x
_

_
(x) =0

(x)
2
i
p)

(x) +
1

2
(x)
2
_
(p)
2
_
x
2
2x)x
_

_
(x) =0 (2.2.32)
At this point, it is very important that we evaluate , and substitute it into (2.2.32). We do this by rst
going back to (2.2.28), and multiplying on the right by [) which reads:
_

(x)
2
_
p
2
2p)p
_
+ (p)
2
_
x
2
2x)x
. .
_

_
=0 (2.2.33)
At this point, we must evaluate the underbraced term in (2.2.33). We can manipulate the position inner
product and nd out:
[2x)x[) =2[x)x[)
=2[x[)[x[)
=2x)x)
=2x)
2
(2.2.34)
Using the same logic that led to (2.2.34) for the position inner product, we may declare that for the
momentum inner product, we have:
[2p)p[) =2p)
2
(2.2.35)
Using (2.2.34) and (2.2.35), we can rewrite (2.2.33) as:
_

(x)
2
_
p
2
2p)
2
_
+ (p)
2
_
(x
2
2x)
2
_

_
=0
_

(x)
2
_
p
2
2p)
2
_
+ (p)
2
_
(x
2
2x)
2
_

_
= (2.2.36)
Now, using the results from (2.2.15) and its analogue for momentum, we can rewrite the bracketed terms
in (2.2.36) as:
x
2
) 2x)
2
=x
2
) x)
2
x)
2
W. Erbsen HOMEWORK #2
=(x)
2
x)
2
(2.2.37)
And similarly, for the momentum:
p
2
) 2p)
2
=(p)
2
p)
2
(2.2.38)
Applying (2.2.37) and (2.2.38) to (2.2.36), we have:
(x)
2
_
(p)
2
p)
2
_
+ (p)
2
_
(x)
2
x)
2
_
=
(x)
2
(p)
2
(x)
2
p)
2
+ (p)
2
(x)
2
(p)
2
x)
2
=
2 (x)
2
(p)
2
(x)
2
p)
2
(p)
2
x)
2
= (2.2.39)
Now, substituting (2.2.39) back into (2.2.32)

(x)
2
i
p)

(x)
+
1

2
(x)
2
_
(p)
2
_
x
2
2x)x
_

_
2 (x)
2
(p)
2
(x)
2
p)
2
(p)
2
x)
2
__
(x) = 0

(x)
2
i
p)

(x)
+
1

2
(x)
2
_
(p)
2
_
x
2
2x)x
_
2 (x)
2
(p)
2
+ (x)
2
p)
2
+ (p)
2
x)
2
_
(x) = 0

(x)
2
i
p)

(x)
+
1

2
(x)
2
_
(p)
2
_
x
2
2x)x +x)
2
_
2 (x)
2
(p)
2
+ (x)
2
p)
2
_
(x) = 0

(x)
2
i
p)

(x) +
1

2
(x)
2
_
(p)
2
(x x))
2
2 (x)
2
(p)
2
+ (x)
2
p)
2
_
(x) = 0

(x)
2
i
p)

(x) +
1

2
_
(p)
2
(x)
2
(x x))
2
2 (p)
2
+p)
2
_
(x) = 0 (2.2.40)
At this point, we suppose that the wave function (x) can be expressed as:
(x) = exp
_
ip)x

_
(x) (2.2.41)
Whose rst and second derivative are

(x) =
ip)

exp
_
ip)x

_
(x) + exp
_
ip)x

(x) (2.2.42a)

(x) =
p)
2

2
exp
_
ip)x

_
(x) +
ip)

exp
_
ip)x

(x) +
ip)

exp
_
ip)x

(x) + exp
_
ip)x

(x)
(2.2.42b)
CHAPTER 2: QUANTUM MECHANICS II 117
We now wish to substitute (2.2.41), (2.2.42a), and (2.2.42b) back into (2.2.40). We rst recognize that
the exponential terms cancel, and so we are left with:
_

p)
2

2
+
ip)

+
ip)

2
i
p)
_
ip)

_
+
1

2
_
(p)
2
(x)
2
(x x))
2
2 (p)
2
+p)
2
_
= 0

p)
2

2
+
2ip)

2p)
2

2

2p)
i

+
1

2
_
(p)
2
(x)
2
(x x))
2
2 (p)
2
+p)
2
_
= 0

+
1

2
_
(p)
2
(x)
2
(x x))
2
2 (p)
2
+p)
2
p)
2
2p)
2
_
= 0

+
1

2
_
(p)
2
(x)
2
(x x))
2
2
_
(p)
2
+p)
2
_
_
= 0

+
1

2
_
(p)
2
(x)
2
(x x))
2
2p
2
)
_
= 0
(2.2.43)
At this point, we let (p) / (x) m (the reasons why we chose our constants as m and will become
clear momentarily. Accordingly, (2.2.43) becomes

+
_
m
2

2
(x x))
2
2p
2
)
_
=0 (2.2.44)
We can divide both sides of (2.2.44) by 2m, which yields:

2
2m

(x) +
m
2
2
(x x))
2
(x) =
p
2
)
m
(x) (2.2.45)
We can see that (2.2.45) looks an awful lot like the time-independent Schr odinger Equation with the 1-D
Harmonic Oscillator Hamiltonian. We can manipulate it slightly more,
_
p
2
2m
+
m
2
2
(x x))
2
_
(x) =
p
2
)
m
(x) (2.2.46)
We now recall that for the 1-D Harmonic Oscillator, from (A.9) and (A.12), we recall that:
x) = 0, p
2
) =
m
2
(2n + 1)
Applying this to (2.2.46), we have:
_
p
2
2m
+
m
2
2
x
2
_
(x) =
_
n +
1
/
2
_
(x) (2.2.47)
Letting E
n
=
_
n +
1
/
2
_
for obvious reasons, we should now be suciently convinced that the solution
of this problem is fully equivalent to the Schr odinger equation for the 1-D Harmonic Oscillator. Letting
(x)
n
(x), (2.2.47) becomes:
_
p
2
2m
+
m
2
2
x
2
_

n
(x) = E
n

n
(x)
W. Erbsen HOMEWORK #2
The solution for this dierential equation is given by (5.49) in Merzbacher as:

n
(x) = 2
n/2
(n!)

1
/2
_
m

_1
/4
exp
_

m
2
x
2
_
H
n
__
m

x
_
(2.2.48)
We can nd the 1-D minimum uncertainty wave packet by other means, though. Following Merzbachers
work on pgs. 218-219, starting with (10.55), we have:
A
2
B
2
[(, (A A))(B B)))[
2
(2.2.49)
Where in our case A x and B p. Remember that we are looking for the minimum uncertainty
state, so to nd the ground state wave function with minimum uncertainty we would take the in
(2.2.49) to be a =. This holds if and only if
(p p)) = (x x)) (2.2.50)
Which is analogous to (10.56) in Merzbacher. With a little manipulation we can see that (2.2.50) is in
fact a dierential equation
(p p)) (x x)) =0
_

x
p)
_
(x x)) =0 (2.2.51)
Where Merzbacher has provided us with the derivation and value of = iC)/2A
2
, where in our case
C) = . Substituting this into (2.2.51):
_

x
p)
_

i
2x
2
(x x)) =0
d
dx
+
_
ip)

+
(x x))
2x
2
_
=0 (2.2.52)
Which is a separable dierential equation, and the solution is found by
1

d =
_
ip)


(x x))
2x
2
_
dx
log () =
_

(x x))
2x
2
dx +
_
ip)

dx
=
(x x))
2
4x
2
+
ip)x

+ C
= C exp
_

(x x))
2
4x
2
+
ip)x

_
(2.2.53)
And normalizing,
_

[[
2
dx =1
_

C exp
_

(x x))
2
4x
2
+
ip)x

2
dx =1
CHAPTER 2: QUANTUM MECHANICS II 119
C
2
_

exp
_

(x x))
2
2x
2
_
=1
C
2
_
2x
2
_1
2
=1
C =
_
2x
2
_

1
/4
(2.2.54)
So our ground state wave function, (2.2.53), combined with the appropriate normalization constant,
(2.2.54), is given by
=
_
2x
2
_
1/4
exp
_

(x x))
2
4x
2
+
ip)x

_
(2.2.55)
Which is identical to (10.66). An important note would be that (2.2.55) minimizes the uncertainty, which
will come in handy later on when we reach problems addressing with coherent and squeezed states.
Problem 10.6
The Hamiltonian representing an oscillating LC circuit can be expressed as
H =
Q
2
2C
+

2
2L
(2.2.56)
Establish that Hamiltons equations are the correct dynamical equations for this system, and show that the charge
Q and the magnetic ux can be regarded as canonically conjugate variables q, p (or the dual pair p, q).
Work out the Heisenberg relation for the product of the uncertainties in the current I and the voltage V . If a
mesoscopic LC circuit has an eective inductance of L = 1 H and an eective capacitance C = 1 pF, how low
must the temperature of the device be before quantum uctuations become comparable to thermal energies? Are
the corresponding current-voltage tolerances in the realm of observability?
Solution
Hamiltons equations are given by:
q
i
=
H
p
i
(2.2.57a)
p
i
=
H
q
i
(2.2.57b)
Where (2.2.57a) and (2.2.57b) are from (8.18) from Goldstein on pg. 337 ?. We note that in our case the
position is the magnetic ux, and the momentum is the charge. Accordingly, (2.2.57a) and (2.2.57b)
become

=
H
Q
(2.2.58a)


Q =
H

(2.2.58b)
W. Erbsen HOMEWORK #2
Substituting (2.2.56) into (2.2.58a) and (2.2.59b),

=

Q
_
Q
2
2C
+

2
2L
_


=
Q
C
(2.2.59a)


Q =

_
Q
2
2C
+

2
2L
_


Q =

L
(2.2.59b)
The best way to proceed with this problem is to mimic the same procedure for the Quantum Harmonic
Oscillator. We begin by drawing a direct analogue between the position and momentum operators
with the magnetic ux and charge operators, respectively. Starting with the position magnetic ux
relationship, we recall from (A.2a) that
q =
_

2m
_
a

+a
_
(2.2.60)
In the case where we let q , we must also let (LC)
1
/2
, and m C. Accordingly, (2.2.60)
becomes:
=
_

2
_
L
C
_1
/4
_
c

+ c
_
(2.2.61)
Where c and c

are to be dened momentarily. We now draw correlation between the momentum operator
from (A.2b) to the charge operator:
Q = i
_

2
_
C
L
_1
/4
_
c

c
_
(2.2.62)
It is easy to see from (A.9) and (A.11) that ) = Q) = 0. From (A.10), we have:

2
) =
_
L
C
_
n +
1
/
2
_
(2.2.63)
While from (A.12) we can see that:
Q
2
) =
_
C
L
_
n +
1
/
2
_
(2.2.64)
From (2.2.63) and (2.2.64), we can now calculate the variance of the magnetic ux and charge:
()
2
=
2
) )
2

_
L
C
_
n +
1
/
2
_
(2.2.65a)
(Q)
2
=Q
2
) Q)
2

_
C
L
_
n +
1
/
2
_
(2.2.65b)
We are asked to nd the product of the uncertainties in the current I and the voltage V , however (2.2.65a)
and (2.2.65b) are in terms of and Q, respectively. We can use (2.2.60) and (2.2.59b) to connect the
two respective quantities:

V =
Q
C
Q = CV (2.2.66a)


Q I =

L
= LI (2.2.66b)
CHAPTER 2: QUANTUM MECHANICS II 121
So, using (2.2.66a) and (2.2.66b), then (2.2.65a) and (2.2.65b) become
(I)
2
=
_
1
LC
_
n +
1
/
2
_
(2.2.67a)
(V )
2
=
_
1
LC
_
n +
1
/
2
_
(2.2.67b)
We now multiply (2.2.67a) and (2.2.67b), which yields
IV =

LC
_
n +
1
/
2
_
Given the provided parameters, we are now in a position to nd the temperature at which quantum
uctuations become apparent. We achieve this by setting the zero-point energy equal to the average
thermal energy, and solving for T:

2
1

LC
T =

2k
B
1

LC
=
1.055 10
34
J s
2 (1.381 10
23
J s
1
)
1
_
(1 10
6
H) (1 10
12
F)
T 3.82 10
3
K (2.2.68)
Problem 10.7
If a coherent state [) (eigenstate of a) of an oscillator is transformed into a squeezed state by the unitary
operator
U = exp
_

2
(a
2
a
2
)
_
(2.2.69)
calculate the value of that will reduce the width of the Hermitian observable (a + a

)/

2 to 1 percent of its
original coherent-state value. What happens to the width of the conjugate observable (a a

)/

2 i in this trans-
formation?
Solution
We know that [) is an eigenstate of a and satises a[) = [). What we want to do is go from our
coherent state [) to a squeezed state [, ). We arrive at the squeezed state by applying a unitary
operator, (2.2.69), to our coherent state: U[) = [, ). To do this, we rst nd the squeezed state
operator, b, which we dene as:
b =UaU

b =aU

BU =a (2.2.70)
W. Erbsen HOMEWORK #2
So we can say:
[a[) =[U

bU[)
=, [b[, ) (2.2.71)
The easiest way to nd b would be to utilize some identities made available by Merzbacher on pgs. 39-40.
He says that if
f() = e
A
Be
A
Then we can write
f() = Bcosh
_
+
[A, B]

sinh
_
(2.2.72)
Which we can use only if is constant, and the operators A and B satisfy [A, [A, B]] = B. In our case,
f() b(), and A = (a
2
a
2
) and B = a. To nd out if we can use (2.2.72):
[A, B] =[(a
2
a
2
), a]
=[a
2
, a] [a
2
, a]
= [a
2
, a]
= a

[a

, a] [a

, a]a

= a

(1) (1)a

=2a

(2.2.73)
[A, [A, B]] =[(a
2
a
2
), 2a

]
=2
_
[a
2
, a

] [a
2
a

]
_
=2
_
a[a, a

] + [a, a

]a
_
=4a (2.2.74)
It seems that we can in fact use (2.2.72), since both of our conditions are satised with = /2 and
= 4. We now have,
b() =a cosh
_
+
[A, B]

sinh
_

=a cosh /2

4 +
2a

4
sinh /2

4
=a cosh +a

sinh
=a cosh a

sinh (2.2.75)
Following Merzbachers lead on pgs. 230-231, we dene = cosh and = sinh , where here is
dierent than the one used before. We can now rewrite (2.2.75):
b = a +a

(2.2.76)
CHAPTER 2: QUANTUM MECHANICS II 123
And taking the Hermitian conjugate,
b

= a

+a (2.2.77)
Since both and are real. Inverting these two equations
a = b b

(2.2.78)
a

= b

b (2.2.79)
And we can now write
1

2
[a[) =
1

2
, [b[, )
=
1

2
, [a + a

[, ) (2.2.80)
1

2
[a

[) =
1

2
, [b

[, )
=
1

2
, [a

+ a[, ) (2.2.81)
Now that we have the framework laid down, we can now attack our problem head on and directly
investigate the Hermitian conjugate we are to minimize, (a + a

)/

2 :
1

2
[a + a

[) =
1

2
_
[a[) + [a

[)

=
1

2
_
, [a +a

[, ) + , [a

+ a[, )

=
1

2
, [a +a

+a

+a[, )
=
1

2
, [a( +) +a

( + )[, )
=
1

2
( +), [a +a

[, )
=
1

2
(cosh sinh ), [a + a

[, )
=
1

2
e

, [a +a

[, ) (2.2.82)
And we are asked to calculate the value of that squeezes (a +a

)/

2 to 1% of its original coherent


state value. So take the appropriate fraction, and solve for the squeezing parameter :
1/

2 e

, [a +a

[, )
1/

2 [a + a

[)(100)
=
e

100
(2.2.83)
And nally
e

= 100 = log 100 4.60517 . . .


W. Erbsen HOMEWORK #3
Figure 2.2: Graph of p = /2x.
Do note that the last part of the question where we are asked what happens to the width of the conjugate
observable (a a

)/

2 i yields the same result.


As a side comment, I found it most informative to investigate squeezed states graphically. Most notably,
by solving the uncertainty relation for p in terms of x and graphing it (See Figure 2 on next page).
The product of the coordinates at any point along the curve is equal to exactly /2; as an example, in
the gure x
1
p
1
= x
2
p
2
= /2. States where this is satised are referred to as squeezed states,
since the uncertainty relation is saturated. If there exists a point above the curve (shaded region), then
this implies that xp /2 and that it is not minimized. Points below the curve cannot exist, for
obvious reasons.
2.3 Homework #3
Exercise 14.3
Show that in the Heisenberg picture the density operator for the state [),
= [(0))(0)[ = [

[ (2.3.1)
Satises the equations
i

t
= iT

(t, 0)

t
T(t, 0) =
_

H,

and
d
dt
= 0 (2.3.2)
and that the expectation value of is constant as in (14.21).
CHAPTER 2: QUANTUM MECHANICS II 125
Solution
First recall that the Schr odinger equation in the Heisenberg representation takes the form
i
d
dt
[
H
) = H
H
[
H
) (2.3.3)
And similarly,
i
d
dt

H
[ =
H
[H
H
(2.3.4)
We begin by rewriting the rst part of (2.3.2):
i

H
t
=i
__
d
dt
[
H
)
_

H
[ + [
H
)
_
d
dt

H
[
__
=i
__
1
i
H
H
[
H
)
_

H
[ [
H
)
_
1
i

H
[H
H
__
=[H
H
([
H
)
H
[) ([
H
)
H
[) H
H
]
=[H
H
(
H
) (
H
) H
H
]
=[H
H
,
H
] (2.3.5)
Now lets do the same for the middle part. We know that

t
= T(t, 0)

H
t
T

(t, 0) (2.3.6)
We can manipulate this further by multiplying (2.3.6) on the left by T

(t, 0) and on the right by T(t, 0).


This gives
T

(t, 0)

t
T(t, 0) =

H
t
(2.3.7)
Which is (14.31) in Merzbacher and also is the same as the left side of (2.3.2), which we have already
shown to be equal to the right side. We can quantify this by inserting the result from (2.3.5) into (2.3.7),
which yields

H
t
= T

(t, 0)

t
T(t, 0) =
1
i
[H
H
,
H
] i

H
t
= iT

(t, 0)

t
T(t, 0) = [H
H
,
H
] (2.3.8)
Where (2.3.8) is identical to (2.3.2).
Exercise 14.4
Show that the expression

, t
2
[

T (t
2
, t
1
)[

, t
1
) (2.3.9)
W. Erbsen HOMEWORK #3
for the transition amplitude is quite general and gives the correct answer if the Schr odinger (H
0
= 0) or Heisenberg
(H
0
= H) pictures are employed.
Solution
First, we recall that

T(t
2
, t
1
) represents the time evolution operator in the interaction picture. Its eect
is the same as we would expect, from (14.48):
[

(t)) =

T(t
2
, t
1
)[

(0)) (2.3.10)
And it can be expressed in terms of unitary transformations from (14.49) as

T(t
2
, t
1
) = U(t
2
)T(t
2
, t
1
)U

(t
1
) (2.3.11)
Where we recall from (14.46) that
U(t) = exp
_
i

H
0
t
_
(2.3.12)
Merzbacher states in (14.50) that
[

, t) = U(t)[A

) (2.3.13a)

, t[ = B

[U

(t) (2.3.13b)
Multiplying (2.3.13a) on the left by U

, and (2.3.13b) on the right by U,


U

(t)[

, t) = [A

) (2.3.14a)

, t[U(t) = B

[ (2.3.14b)
Substituting (2.3.14a) and (2.3.14b) into (2.3.9),

, t
2
[

T (t
2
, t
1
)[

, t
1
) = B

[T(t
2
, t
1
)[A

) (2.3.15)
The equivalence has thus been shown. We can further quantify this by recalling (14.17), which reads
T(t, t
0
) = exp
_

(t t
0
)H
0
_
(2.3.16)
Substituting (2.3.16) into (2.3.15),
B

[T(t
2
, t
1
)[A

) =B

[e

(tt0)H0
[A

) (2.3.17)
If we let H
0
0, as in the Schr oginer picture, then (2.3.17) becomes
B

[T(t
2
, t
1
)[A

) = B

[A

)
Which is what we would expect in the Schr odinger picture. Similarly, if we let H
0
H, as in the
Heisenberg picture, then (2.3.17) becomes
CHAPTER 2: QUANTUM MECHANICS II 127
B

[e

(tt0)H0
[A

) = B

[e

(tt0)H
[A

)
Which is, according to Merzbacher, also what we would expect for the Heisenberg picture.
Exercise 14.6
Illustrate the validity of (14.60) by letting G = x
2
and F = p
2
x
, and evaluating both the operator expression
on the left, in the limit 0, and the corresponding Poisson bracket on the right.
Solution
Equation (14.60) reads,
lim
0
GF FG)
i
=
G
x
F
p
x

F
x
G
p
x
+
G
y
F
p
y

F
y
G
p
y
+
G
z
F
p
z

F
z
G
p
z
(2.3.18)
Lets start by evaluating the numerator of the limit on the LHS of (2.3.18):
GF FG) =x
2
p
2
x
p
2
x
x
2
)
=[x
2
, p
2
x
])
=x[x, p
2
x
] + [x, p
2
x
]x)
=x([x, p
x
]
. .
i
p
x
+ p
x
[x, p
x
]
. .
i
) + ([x, p
x
]
. .
i
p
x
+ p
x
[x, p
x
]
. .
i
)x)
=2ixp
x
+p
x
x)
=2i[xp
x
) + p
x
x)]
=2i
__

(x)xp
x
(x) dx +
_

(x)p
x
x(x) dx
_
=2i
_

i
_

(x)x
_

x
_
(x) dx +

i
_

(x)
_
d
dx
_
x(x) dx
_
=2i
_

i
_

(x)x
_

x
_
(x) dx +

i
__

(x)x
_

x
_
(x) dx +
_

(x)(x) dx
__
=2i
_
2
i
_

(x)x
_

x
_
(x) dx +

i
_

(x)(x) dx
_
=2i
_
2xp
x
) +

i
(x) [ (x))
_
=2i
_
2xp
x
) +

i
_
(2.3.19)
Where I assumed that (x) is normalized, and I also liberally used the fact that [x, p
x
] = i and also
the following identities from (3.50): [AB, C] = A[B, C] + [A, C]B and [A, BC] = [A, B]C +B[A, C]. We
now plug (2.3.19) into the left side of (2.3.18):
W. Erbsen HOMEWORK #3
lim
0
GF FG)
i
= lim
0
2i
_
2xp
x
) +

i
_
i
= lim
0
2
_
2xp
x
) +

i
_
=4xp
x
) (2.3.20)
To solve the Poisson bracket on the right, you should rst notice that all partial derivatives on the right
are zero except the rst:
G
x
F
p
x
=
(x
2
)
x
(p
2
x
)
p
x
4xp
x
(2.3.21)
So, the left side of (2.3.18) is given by (2.3.20) as 4xp
x
) and the right side is given by (2.3.21) as 4xp
x
:
4xp
x
) = 4xp
x
xp
x
) = xp
x
This makes sense; the Poisson Bracket is dened as the classical analogue of the quantum-mechanical
commutator and the expectation value is an average. This therefore demonstrates how classical me-
chanics is a limiting case of quantum mechanics.
Exercise 14.7
Show that the transformation
UaxU

=ax cos +bp


x
sin (2.3.22a)
Ubp
x
U

= ax sin +bp
x
cos (2.3.22b)
is canonical, if a and b are real-valued constants, and is a real-valued angle parameter. Construct the unitary
operator U that eects this transformation. For the special case of = /2, calculate the matrix elements of U in
the coordinate representation. Noting that this transformation leaves the operator a
2
x
2
+b
2
p
2
x
invariant, rederive
the result of Exercises 3.8 and 3.21.
Solution
In the spirit of 14.4, we recall that in order to verify canonically conjugate transformations,
qp pq = i1 (2.3.23)
Which is of course (14.63). We also note that q represents the canonically conjugate position, while p is
the canonically conjugate momentum. Claiming that q corresponds to (2.3.22a), while p corresponds to
(2.3.22b), we can compute the commutator in (2.3.23) straightforwardly,
qp pq = (ax cos +bp
x
sin) (ax sin + bp
x
cos ) (ax sin + bp
x
cos ) (ax cos +bp
x
sin )
(2.3.24)
CHAPTER 2: QUANTUM MECHANICS II 129
To reduce the cumbersomeness of evaluating this, we evaluate qp and pq separately, and then combine
them. Computing the product qp rst,
qp =(ax cos + bp
x
sin) (ax sin + bp
x
cos )
= a
2
x
2
cos sin + axbp
x
cos
2
bp
x
ax sin
2
+b
2
p
2
x
sin cos (2.3.25)
And now doing the same thing for the product pq,
pq =(ax sin + bp
x
cos ) (ax cos +bp
x
sin )
= a
2
x
2
sin cos axbp
x
sin
2
+ bp
x
ax cos
2
+b
2
p
2
x
cos sin (2.3.26)
Simply by visual inspection, we can see that in subtracting (2.3.26) from (2.3.25), the rst and last terms
in both equations cancel. We are left with,
qp pq =axbp
x
cos
2
bp
x
ax sin
2
+axbp
x
sin
2
bp
x
ax cos
2

=(axbp
x
bp
x
ax) cos
2
+ (axbp
x
bp
x
ax) sin
2

=[ax, bp
x
]
_
cos
2
+ sin
2

_
=[ax, bp
x
] (2.3.27)
Recalling that a and b are constants, (2.3.27) becomes
qp pq =ab [x, p
x
] qp pq = abi (2.3.28)
According to (2.3.23), the transformation in (2.3.22a) and (2.3.22b) are canonically conjugate . Now,
following closely Merzbachers work on pgs. 327-328, we dene the innitesimal unitary operator U

from (14.72) as
U

= 1 +
i

G (2.3.29)
Where G is the generator of this innitesimal transformation. We now recall that according to (14.68),
F(q, p) = F(x, p
x
) +

i
[F, G] (2.3.30)
While we also note that from (14.71),
F(q, p) = U

F(x, p
x
)U

(2.3.31)
Comparing (2.3.31) to (2.3.22a) and (2.3.22b), and applying the results to (2.3.30), we have
UaxU

=ax +
a
i
[x, G] (2.3.32a)
Ubp
x
U

=bp
x
+
b
i
[p
x
, G] (2.3.32b)
Applying the RHS of (2.3.22a) and (2.3.22b) to (2.3.32a) and (2.3.32b), respectively,
ax cos +bp
x
sin =ax +
a
i
[x, G] (2.3.33a)
ax sin + bp
x
cos =bp
x
+
b
i
[p
x
, G] (2.3.33b)
W. Erbsen HOMEWORK #3
Now recalling (14.70) from Merzbacher,
F(q, p) =
_
1 +
i

G
_
F(x, p
x
)
_
1
i

G
_
(2.3.34)
Applying (2.3.34) to (2.3.33a) rst,
ax cos +bp
x
sin =
_
1 +
i

G
_
ax
_
1
i

G
_
=
_
1 +
i

G
__
ax
iax

G
_
=ax
i

Gax + ax
i

G+
ax
2

2
G
2
(2.3.35)
First order approximation in allows us to rewrite (2.3.35) as
ax +bp
x
= ax
i

[G, ax] bp
x
=
ia

[x, G] (2.3.36)
And we can analogously apply (2.3.34) to (2.3.33b),
ax =
ib

[p
x
, G] (2.3.37)
Now recalling (14.64) from Merzbacher,
q = x +
G
p
x
, p = p
x

G
x
(2.3.38)
Applying (2.3.38) to (2.3.22a) and (2.3.22b),
UaxU

=ax + a
G
p
x
(2.3.39a)
Ubp
x
U

=bp
x
b
G
x
(2.3.39b)
Using (2.3.22a), (2.3.32a) and (2.3.39a), we now have
ax
b
=
G
x
(2.3.40)
And similarly, using (2.3.22b), (2.3.32b), and (2.3.39b),
bp
x
a
=
G
p
x
(2.3.41)
We can solve the dierential equations in (2.3.40) and (2.3.41) as
ax
b
=
G
x
G =
a
b
x
2
2
(2.3.42a)
bp
x
a
=
G
p
x
G =
b
a
p
2
x
2
(2.3.42b)
Having established that , and combining (2.3.42a) and (2.3.42b), we have
CHAPTER 2: QUANTUM MECHANICS II 131
G =
a
2b
x
2
+
b
2b
p
2
x
(2.3.43)
We now recall (14.75) from Merzbacher, which reads
U = exp
_
i

G
_
(2.3.44)
Substituting (2.3.43), and recognizing the fact that ,
U = exp
_
i

_
a
2b
x
2
+
b
ab
p
2
x
__
(2.3.45)
For the special case of = /2, (2.3.45) becomes
U = exp
_
i
2
_
a
2b
x
2
+
b
2a
p
2
x
__
(2.3.46)
And in order to calculate the matrix elements of U in the coordinate representation (letting q x),
x

[U[x) =
_
x

exp
_
i
2
_
a
2b
x
2
+
b
2b
p
2
x
__

x
_
(2.3.47)
Substituting the series expansion of the exponential function into (2.3.47),
x

[U[x) =
_
x

N=0
1
N!
_
i
2
_
a
2b
x
2
+
b
2b
p
2
x
__
N

x
_
(2.3.48)
Expanding to the rst few terms of (2.3.48),
x

[U[x) =x

[x) +
_
x

i
2
_
a
2b
x
2
+
b
2b
p
2
x
_

x
_
+
_
x

1
2
_
i
2
_
a
2b
x
2
+
b
2b
p
2
x
__
2

x
_
+ ... (2.3.49)
Rewriting the last element in (2.3.48),
_
x

1
2
_
i
2
_
a
2b
x
2
+
b
2b
p
2
x
__
2

x
_
=
i
32
_
x

_
a
b
x
2
+
b
a
p
2
x
_
2

x
_
(2.3.50)
To make our lives easier, lets examine the main part of (2.3.50) without the constants:
x

[(x
2
+p
2
x
)
2
[x) =x

[(x
2
+p
2
x
+ 2xp
x
)[x) x

[x
2
[x) + x

[p
2
x
[x) + 2x

[p
x
[x) (2.3.51)
Taking inspiration from the harmonic oscillator, we know that for x ,= x

, then (2.3.51) goes to zero.


The same can be seen for the rst two matrix elements as well, and therefore we only allow for diagonal
entries.
We now recall that for a linear harmonic oscillator, the Schr odinger Equation can be expressed in
momentum representation with (3.84) as
W. Erbsen HOMEWORK #3
1
2m
p
2
x

E
(x)
1
2
m
2
d
2

E
(x)
dp
2
x
= E
E
(x) (2.3.52)
We now recall that the Hamiltonian for the Quantum Harmonic Oscillator reads
H =
p
2
x
2m
+
1
2
m
2
x
2
(2.3.53)
We may also rewrite the unitary transformation in (2.3.45) as
U = exp
_
i
2
_
a
2
x
2
+ b
2
p
2
x

_
(2.3.54)
To show that the operator inside the brackets of (2.3.54) remains invariant,
U
_
a
2
x
2
+b
2
p
2
x
_
U

=exp
_
i
2
_
a
2
x
2
+ b
2
p
2
x

_
_
a
2
x
2
+ b
2
p
2
x
_
exp
_

i
2
_
a
2
x
2
+ b
2
p
2
x

_
=
_
1 +
_
i
2
_
a
2
x
2
+b
2
p
2
x

__
_
a
2
x
2
+ b
2
p
2
x
_
_
1
_
i
2
_
a
2
x
2
+b
2
p
2
x

__
=a
2
x
2
+ b
2
p
2
x
Therefore, the transformation leaves a
2
x
2
+ b
2
p
2
x
invariant . We also note ,

, given in (2.3.45) rep-


resents the time evolution operator for the quantum harmonic oscillator, then it lends itself that the
position-space wave function can be found by taking the position-space function projected onto momentum-
space from (2.3.48) and solving the position-space Schr odinger equation from (2.3.52). The result is:
(p
x
, t) =
e
i

m

_
p
x
m
, t
_
Exercise 14.11
For a free particle in one dimension and an arbitrary initial wave packet, calculate the time development of
(q)
2
t
and show (as in Problem 2 in Chapter 3) that
(q)
2
t
= (q)
2
0
+
t
m
[qp + pq)
0
2q)
0
p)] +
(p)
2
t
2
m
2
(2.3.55)
Verify that for the minimum uncertainty wave packet this result agrees with (14.97). Also compare with the value
of the variance (q)
2
as a function of time for a beam of freely moving classical particles whose initial positions
and momenta have distributions like variances (q)
2
0
and (p)
2
.
Solution
To nd the time development of (q)
2
t
, it is rst helpful to take care of some denitions. First, recall
CHAPTER 2: QUANTUM MECHANICS II 133
that the time derivative of the expectation value of an operator is given by
i
d
dt
A) = [A, H]) +
_
A
t
_
(2.3.56)
The operators we will be dealing with in this problem are time-independent, so the last part of (2.3.56)
will go to zero. Furthermore, we can go ahead and nd p)
t
and p
2
)
t
since they will be needed later on
in the problem:
i

t
p) =[p, T])
=
1
2m
[p, p
2
]) = 0 (2.3.57)
Integrating (2.3.57) we nd that we are only left with a constant: p)
t
= p)
0
. With the same logic we
can arrive at p
2
)
t
= p
2
)
0
, and the variance of p) is therefore p
2
) = p
2
)
0
p)
2
0
. It will also be
convenient to nd the following commutators,
[q, p
2
] = [q, p]p +p[q, p] = 2ip (2.3.58)
Where I liberally used the fact that [q, p] = i and also the following identities from (3.50): [AB, C] =
A[B, C] +[A, C]B and [A, BC] = [A, B]C+B[A, C]. Armed with these tools, we can go ahead and start
nding (q)
2
t
= q
2
)
t
q)
2
t
. Start by nding q):
i

t
q) =[q, T])
=
1
2m
[q, p
2
])
=
1
m
ip)
t
q)
t
=
p)
0
t
m
+ q)
0
(2.3.59)
Doing the same for q
2
),
i

t
q
2
) =
1
2m
[q
2
, p
2
])
=
1
2m
q [q, p
2
]
. .
2ip
+[q, p
2
]
. .
2ip
p)
=
i
m
qp +pq)
q
2
)
t
=
qp + pq)
t
t
m
+ q
2
)
0
(2.3.60)
And we must now nd qp + pq)
t
:
i

t
qp +pq) =
1
2m
[(qp + pq), p
2
])
=
1
2m

[qp, p
2
] + [pq, p
2
]
_
W. Erbsen HOMEWORK #3
=
1
2m
q [p, p
2
]
. .
0
+[q, p
2
]
. .
2ip
p + p [q, p
2
]
. .
2ip
+[p, p
2
]
. .
0
q)
=
i
m
p
2
)
qp + pq)
t
=
p
2
)
0
t
m
+qp + pq)
0
(2.3.61)
Substituting (2.3.61) into (2.3.60)
q
2
)
t
=
1
m
_
p
2
)
0
t
m
+qp + pq)
0
_
t + q
2
)
0
=
p
2
)
0
t
2
m
2
+
qp + pq)
0
t
m
+q
2
)
0
(2.3.62)
And now we nd (q)
2
t
. Using (2.3.59) and (2.3.62),
(q)
2
t
=q
2
)
t
q)
2
t
=
_
p
2
)
0
t
2
m
2
+
qp +pq)
0
t
m
+q
2
)
0
_

_
p)
0
t
m
+q)
0
_
2
=
p
2
)
0
t
2
m
2
+
qp + pq)
0
t
m
+ q
2
)
0

p)
2
0
t
2
m
2
2
p)
0
q)
0
t
m
q)
2
0
=q
2
)
0
q)
2
0
+
qp + pq)
0
t
m

2q)
0
p)
0
t
m
+
p
2
)
0
t
2
m
2

p)
2
0
t
2
m
2
=(q)
2
0
+
t
m
[qp + pq)
0
2q)
0
p)
0
] +
(p)
2
0
t
2
m
2
(2.3.63)
Which answers the rst part of the question

. For a beam of freely moving classical particles, nding


(q)
2
t
is simply a matter of realizing that there is no dierence between qp and pq, and (2.3.63) becomes:
(q)
2
t
=(q)
2
0
+
t
m
[2qp 2qp] +
(p)
2
0
t
2
m
2
(q)
2
t
= (q)
2
0
+
(p)
2
0
t
2
m
2
(2.3.64)
Which is identical to (14.97).
Exercise 14.13
In either the Heisenberg or Schr odinger picture, show that if at t = 0 a linear harmonic oscillator is in a co-
herent state, with eigenvalue , it will remain in a coherent state, with eigenvalue e
it
, at time t.
Solution

You will notice that my answer (2.3.63) diers ever so slightly from Merzbachers (2.3.107). I do believe that Merzbacher made a
mistake; the last term in the square brackets should be p
0
, as opposed to p.
CHAPTER 2: QUANTUM MECHANICS II 135
Recall from (14.7) that the time development operator T(t, t
0
) can relate the initial state [(t
0
)) to the
nal state [(t)). Applying this principle to our system where t
0
= 0,
[(t)) = T(t, t
0
)[(t
0
))
t0=0
[(t)) = T(t, 0)[(0)) (2.3.65)
Where the state in our case is a coherent state, [). Using (10.110), we can say that
[) =

n=0
[n)e
||
2
/2

n

n!
(2.3.66)
And according to (14.17) that the time development operator T(t, t
0
) at t
0
= 0 is given by
T(t, t
0
) = e

(tt0)H
t0=0
T(t, 0) = e

Ht
(2.3.67)
And the series expansion of the exponential function e
x
is given by:
e
x
=

n=0
x
n
n!
(2.3.68)
So, using (2.3.66), (2.3.67) and (2.3.68), we can now rewrite (2.3.65):
[(t)) =e

Ht
[(0))
=e

Ht

n=0
[n)e
||
2
/2

n

n!
=

n=0
(iHt/)
n
n!
[n)e
||
2
/2

n

n!
=

n=0
(it(n + 1/2))
n
n!
[n)e
||
2
/2

n

n!
=

n=0
(itn it/2)
n
n!
[n)e
||
2
/2

n

n!
=

n=0
e
itn
e
it/2
[n)e
||
2
/2

n

n!
=e
it/2

n=0
_
e
it
_
n
[n)e
||
2
/2

n

n!
=e
it/2

n=0
[n)e
||
2
/2
_
e
it
_
n

n!
=e
it/2

e
it
_
(2.3.69)
Where I used (2.3.66), by using the fact that [[
2
is independent of phase, since it refers to a magnitude.
Since

e
it
_
is an eigenstate of the energy operator, it satises a[) = [), and
a[(t)) =ae
it/2

e
it
_
=e
it/2
a

e
it
_
W. Erbsen HOMEWORK #3
=e
it/2
_
e
it
_

e
it
_
=e
it
_
e
it/2

e
it
_
_
=e
it
[(t)) (2.3.70)
So, according to , if a linear harmonic is in a coherent state at t
0
= 0, it will remain in a coherent state
with eigenvalue e
it
:
a[(t)) = e
it
[(t))
Problem 10.1
A particle of charge q moves in a uniform magnetic eld B which is directed along the z axis. Using a gauge
in which A
z
= 0, show that q = (cp
x
qA
x
)/qB and p = (cp
y
qA
y
)/c may be used as suitable canonically conju-
gate coordinate and momentum together with the pair z, p
z
. Derive the energy spectrum and the eigenfunctions
in the q-representation. Discuss the remaining degeneracy. Propose alternative methods for solving this eigenvalue
problem.
Solution
To show that q and p are canonically conjugate, we must test to see if they satisfy the fundamental
Poisson bracket relations: q
i
, q
j
= 0, p
i
, p
j
= 0 and q
i
, p
j
=
ij
. The Poisson bracket is dened by
the right hand side of (14.60):
lim
0
[G, F])
i
=
G
x
F
p
x

F
x
G
p
x
+
G
y
F
p
y

F
y
G
p
y
+
G
z
F
p
z

F
z
G
p
z
(2.3.71)
The easiest way to nd out of Poissons conditions are satised would be to evaluate the commutator on
the left of (2.3.71). First we must choose our gauge; and I convienently choose to work in the Landau
Gauge since it has only one term (the symmetric gauge should also work): A(x, y, z) = (yB
0
, 0, 0).
Rewriting our given values of q and p,
q =
cp
x
qA
x
qB
=
cp
x
qB
0
+ y (2.3.72)
And,
p =
cp
y
qA
y
c
= p
y
(2.3.73)
Just by visual inspection it is possible to tell that x, y = x, z = y, z = 0, and also that p
x
, p
y
=
p
x
, p
x
= p
y
, p
z
= 0. For the same reasons, we can see that q
x
, p
x
= q
z
, p
z
= 0. To nd q
y
, p
y
,
q
y
, p
y
= lim
0
[q
y
, p
y
])
i
= lim
0
[y, p
y
])
i
CHAPTER 2: QUANTUM MECHANICS II 137
= lim
0
i)
i
=1 (2.3.74)
Which indicates that q and p may be used as canonically conjugate coordinates. To nd the energy spec-
trum and eigenfunctions, start o by remembering the denition of a Hamiltonian in an electromagnetic
eld (4.93)
H =
1
2m
_
p
q
c
A
_
2
+ q (2.3.75)
Where in our case = 0. Still working in the Landau gauge, (2.3.75) becomes
H =
1
2m
_
_
p
x

q
c
A
x
_
2
+
_
p
y

q
c
A
y
_
2
+
_
p
z

q
c
A
z
_
2
_
=
1
2m
_
p
x
+
q
c
yB
0
_
2
+
1
2m
_
p
2
y
+ p
2
z
_
=
1
2m
_
p
2
x
+
2q
c
B
0
yp
x
+
q
2
c
2
y
2
B
2
0
_
+
1
2m
_
p
2
y
+ p
2
z
_
=
1
2m
_
p
2
x
+ p
2
y
+p
2
z
+
2q
c
B
0
yp
x
+
q
2
c
2
y
2
B
2
0
_
=
1
2m
_
p
2
x
+ p
2
y
+p
2
z
+
2q
c
yp
x
B
0
+
q
2
c
2
y
2
B
2
0
_
=
x
2
+p
2
y
+ p
2
z
2m
+
_
qB
0
mc
_
yp
x
+
m
2
_
qB
0
mc
_
2
y
2
=
p
2
x
+ p
2
y
+p
2
z
2m
+
c
yp
x
+
m
2

2
c
y
2
(2.3.76)
Where I used the fact that the cyclotron resonance frequency is dened by
c
= qB
0
/mc. At this point we
should notice that our Hamiltonian commutes with both p
x
and p
z
but not with p
y
: [H, p
x
] = [H, p
z
] = 0.
This implies that it is possible to nd a common eigenstate between both p
x
, p
z
and H. Lets nd an
eigenfunction for p
x
:
p
x
(x) =p
x
(x)

i
d
dx
(x) =p
x
(x)
1
(x)
d(x) =
i

p
x
dx
(x) =e
ipxx

(2.3.77)
And WLOG we can assume this holds for p
z
as well. At this point we still need to nd an eigenstate for
H; one way to nd it is to deploy separation of variables:
(x, y, z) =(x)(y)(z)
=e
ipxx/
(y)e
ipz z/
=e
i(pxx+pzz)/
(y) (2.3.78)
W. Erbsen HOMEWORK #3
Using our Hamiltonian (2.3.76) and our eigenstate (2.3.78), we can construct the Schr odinger equation,
keeping the eigenfunction arbitrary (for now):
_
1
2m
_
p
2
x
+p
2
y
+p
2
z
_
+
c
yp
x
+
m
2

2
c
y
2
_
(x, y, z) =E(x, y, z) (2.3.79)
Where the x, z, p
x
and p
z
are now numbers instead of operators, so:
_
p
2
y
2m
+
c
yp
x
+
m
2

2
c
y
2
_
(x, y, z) =
_
E
(p
2
x
+ p
2
z
)
2m
_
(x, y, z) (2.3.80)
At this point we want to undergo a change of variables with the denitions of q and p given in the
prompt:
y

y +
p
z
m
And
p

y
p
y
While
E

E
p
2
z
2m
Solving and substituting into (2.3.80):
_
p
2
y
2m
+
c
p
x
_
y

p
x
m
_
+
m
2

2
c
_
y

p
x
m
_
2
_
(x, y, z) =E

(x, y, z)
_
p
2
y
2m
+
c
p
x
y

p
2
x
m
+
m
2
c
2
_
y
2

2y

p
x
m
c
+
p
2
x
m
c
_
_
(x, y, z) =E

(x, y, z)
_
p
2
y
2m
+
c
p
x
y

p
2
x
m
+
m
2
c
y
2
2

c
p
x
y

+
p
2
x
2m
_
(x, y, z) =E

(x, y, z)
_
p
2
y
2m

p
2
x
2m
+
m
2
c
y
2
2
_
(x, y, z) =E

(x, y, z)
_
p
2
y
2m
+
m
2
c
y
2
2
_
(x, y, z) =E

(x, y, z) (2.3.81)
The operator acting on (x, y, z) on the left of (2.3.81) is none other than the Hamiltonian for the
harmonic oscillator. This implies that it is a straightforward matter to nd both the eigenenergies and
the eigenfunctions. From (2.3.81) it is clear that E =
c
(n +
1
/
2
), as it would be for the harmonic
oscillator. Therefore, the eigenvalues for the energy operator in our case is given by:
E =
c
_
n +
1
2
_
+
p
2
z
2m
(2.3.82)
To nd the eigenfunctions make our separation of variables substitution from (2.3.78),
CHAPTER 2: QUANTUM MECHANICS II 139
_
p
2
y
2m
+
m
2
c
y
2
2
_
exp
_
i(p
x
x + p
z
z)

_
(y) =E

exp
_
i(p
x
x + p
z
z)

_
(y) (2.3.83)
So that the eigenfunctions are just those of the harmonic oscillator from (5.39) multiplied by (2.3.78):
(x, y, z) = 2
n/2
(n!)
1/2
_
m
c

_
1/4
exp
_

m
c
2
+
i(p
x
x +p
z
z)

_
x
2
H
n
__
m
c

x
_
(2.3.84)
These results agree with the literature; the energy spectrum is referred to as Landau Levels. In
fact, the whole of the problem alludes to Landau Quantization, or the quantization cyclotron orbits
of charged particles subjected to an external magnetic eld. The fact that the system boils down to the
harmonic oscillator problem is no surprise; when we think of a classical particle in a static magnetic eld,
the particle follows a helical path, moving in the direction of the eld and oscillating in a plane normal
to the direction of propagation. The quantum case, however is more interesting.
The momentum components in the x and y plane do not commute: [p
x
, p
y
] ,= 0. Therefore it is impossible
for us to precisely dene the momentum in these two directions, which causes degeneracy. This is made
more obvious when we notice that neither of these terms are present in (2.3.82). We also note that the
degeneracy is directly proportional to the strength of the applied magnetic eld B
0
. ?
Problem 10.2
A linear harmonic oscillator is subjected to a spatially uniform external force F(t) = C(t)e
t
where is a
positive constant and (t) the Heaviside step function (A.23). If the oscillator is in the ground state at t < 0,
calculate the probability of nding it at time t in an oscillator eigenstate with quantum number n. Assuming
C = (m
3
)
1
/2
, examine the variation of the transition probabilities with n and with the ratio /, being the
natural frequency of the harmonic oscillator.
Solution
This is an instance of a forced linear harmonic oscillator, whose Hamiltonian is given by (14.105):
H =
p
2
2m
+
1
2
m
2
x
2
qQ(t) pP(t) (2.3.85)
Where Q(t) is a real-valued function of t and corresponds to an external time-dependent force (F(t) in our
case). P(t) is a velocity dependent term and neednt be considered for this problem. The Hamiltonian
can also be written in terms of the raising and lowering operators:
H =
_
a

a +
1
2
_
+f(t)a +f

(t)a

(2.3.86)
Which is (14.106). The function f(t) is given by
f(t) =
_

2m
Q(t) +i
_
m
2
P(t)
W. Erbsen HOMEWORK #3
=
_

2m
C(t)e
t
=
_

2m

m
3
(t)e
t
=
_

3
2
(t)e
t
(2.3.87)
Since the oscillator is initially in the ground state at t < 0 and we are asked to nd the probability at
some time t, we should use the time evolution operator, [(t)) = T(t, 0)[(0)), where T(t, 0) can be
found from (14.148), letting t
2
= t, t
1
= 0 and H
0
= (a

a +
1
/
2
) from (14.121):
T(t, 0) =e
i(t,0)
exp
_

(t, 0)ae
it
+(t, 0)a

e
it
_
e
i(a

a+1/2)t
(2.3.88)
And to nd the probability at some later time t of nding the oscillator in an eigenstate with quantum
number n would be given by
P
n
(t) =[n[(t)) [
2
=[n[T(t, 0) [ (0))[
2
=

n[ e
i(t,0)
exp
_

(t, 0)ae
it
+(t, 0)a

e
it
_
e
i(a

a+1/2)t
[(0))

2
(2.3.89)
At this point we should note that
e
i(a

a+1/2)t
[(0)) =

k=0
_
i(a

a + 1/2)t
_
k
k!
[(0))
=

k=0
(ia

a it/2)
k
k!
[(0))
=e
it/2

k=0
(ia

a)
k
k!
[(0))
=e
it/2

k=0
(i)
k
k!
(a

a)
k
[(0))
=e
it/2
_
[(0)) +

k=1
(i)
k
k!
(a

a)
k
[(0))
_
=e
it/2
[(0)) (2.3.90)
Because the number operator acting on the ground state (any number of times) has an eigenvalue of zero
since you cant go any lower than the ground state: a

a[(0)) a

(a[(0))) = 0. We also know that


(t, 0) is given by (14.140):
(t, 0) =
i
2
2
_
t
0
dt

_
t
0
dt

_
f(t

)f

(t

)e
i(t

)
f

(t

)f(t

)e
i(t

)
_
(2.3.91)
Luckily, we dont have to evaluate (2.3.91) since is real, such that e
i(t,0)
is itself a phase and whose
magnitude is 1, and it therefore doesnt matter. The same argument can be applied to (2.3.90). Therefore,
(2.3.89) becomes
CHAPTER 2: QUANTUM MECHANICS II 141
P
n
(t) =

exp
_

(t, 0)ae
it
+(t, 0)a

e
it
_

(0)
_

2
(2.3.92)
Where (t, 0) can be found from (14.138):
(t, 0) =
i

_
t
0
e
it

(t

) dt

=i
_

3
2
_
t
0
e
it

(t

)e
t

dt

(2.3.93)
And recall that from (A.23), the Heaviside step function is
(x) =
_
x

(u) du =
_
1 x > 0
0 x < 0
(2.3.94)
Since t > 0, then (t

) = 1 in (2.3.93), according to (2.3.94). So, (t, 0) becomes:


(t, 0) =i
_

3
2
_
t
0
e
i(+i)t

dt

=i
_

3
2
1
i( + i)
_
e
i(+i)t

=t
t

=0
=
_

3
2
1
+ i
e
i(+i)t
(2.3.95)
Using this, we can now rewrite (2.3.92):
P
n
(t) =

_
n

exp
___

3
2
1
+i
e
i(+i)t
_
a

e
it

__

3
2
1
i
e
i(+i)t
_
ae
it
_

(0)
_

2
=

_
n

exp
___

3
2
1
+i
e
t
_
a

__

3
2
1
i
e
i
_
a
_

(0)
_

2
(2.3.96)
Dene a new complex number (t):
(t) =
_

3
2
1
+i
e
t
(2.3.97)
For reasons to become clear momentarily. Then (2.3.96) becomes
P
n
(t) =

exp
_
(t)a

(t)a
_

(0)
_

2
(2.3.98)
Which reminds us of the displacement operator (10.98):
D

=e
a

a
(2.3.99)
Which has the property
[) =D

[0) (2.3.100)
Which is (10.101). Using (2.3.99) and (2.3.100) we can now rewrite (2.3.98):
P
n
(t) =[n[D

[ (0))[
2
W. Erbsen HOMEWORK #3
=[n[)[
2
(2.3.101)
Recall that we can dene [) in terms of eigenstates of the number operator with the Poisson distribution,
(10.109), such that (2.3.101) becomes (10.110)
P
n
(t) =e
||
2 [[
2n
n!
(2.3.102)
Where we can nd from (2.3.97):
[[
2
=

3
2
1
+ i
e
t

2
=
__

3
2
1
i
e
t
___

3
2
1
+ i
e
t
_
=

3
2
e
2t
1
( i)( +i)
=

3
2(
2
+
2
)
e
2t
(2.3.103)
So that (2.3.102) becomes
P
n
(t) =exp
_

3
2(
2
+
2
)
e
2t
_
1
n!

3
2(
2
+
2
)
e
2t

n
=exp
_

3
2(
2
+
2
)
e
2t
_
1
n!
__

3
2(
2
+
2
)
_
n
e
2nt
_
(2.3.104)
We see that (2.3.104) can be easily rewritten as
P
n
(t) = exp
_

3
2(
2
+
2
)
_ _

3
2(
2
+
2
)
_
n
1
n!
e
2t(n+1)
(2.3.105)
Which answers the rst part of the question. To see how the transition probability P
n
(t) varies with
n and /, it will be most instructive to create a graph. For the rst graph we seek to see how P
n
(t)
varies with n, so we set = = 1 in (2.3.105) and construct a 3-D graph (Fig. (1)).
Similarly, to see how P
n
(t) varies with (which I have dened as = /), simply plot (2.3.105)
with for various values of n and see how it looks (Figs. (2)-(4)).
From Fig. (1) it is clear that as n increases the probability that the system will remain in the
lowest state subjected to the time-dependent force F(t) decreases with higher n. Similarly, from Figs.
(2)-(4) we can see that as increases the probability that the system will remain in the ground state
decreases with respect to lower .
Problem 10.3
CHAPTER 2: QUANTUM MECHANICS II 143
Figure 2.3: P
n
(t) vs Figure 2.4: P
n
(t) vs n for n = 0
Figure 2.5: P
n
(t) vs n for n = 1 Figure 2.6: P
n
(t) vs n for n = 2
If the term V (t) in the Hamiltonian changes suddenly (impulsively) between time t and t + t, in a time
t short compared with all relevant periods, and assuming only that [V (t

), V (t

)] = 0 during the impulse, show


that the time development operator is given by
T(t + t, t) = exp
_

_
t+t
t
V (t

) dt

_
(2.3.106)
Note especially that the state vector remains unchanged during a sudden change of V by a nite amount.
Solution
Recall that (14.11) reads
i
dT(t, t
0
)
dt
= H(t)T(t, t
0
) (2.3.107)
Where in our case we recognize that t
0
t and t t + t. We also note that H(t) = H
0
+ V (t). So,
lets solve (2.3.107) for T(t + t, t):
1
T(t + t, t)
dt(t + t, t) =
1
i
H(t

) dt

W. Erbsen HOMEWORK #4
T(t + t, t) =exp
_

_
t+t
t
H
0
+V (t

) dt

_
=exp
_

_
H
0
_
t+t
t
dt

+
_
t0+t
t
V (t

) dt

__
(2.3.108)
Where we recognize that since the Hamiltonian changes suddenly, the rst integral is approximately zero:
_
t+t
t
dt

0, and (2.3.108) becomes


T(t + t, t) = exp
_

_
t+t
t
V (t

) dt

_
(2.3.109)
Which agrees with (2.3.106). To see that the state vector remains unchanged during a sudden change of
V by a nite amount, recall that
[(x, t)) =T(t, 0)[(x, 0)) [(x, t + t)) = T(t + t, t)[(x, t)) (2.3.110)
And with a sudden change t 0, such that the right side of (2.3.110) becomes
T(t, t)[(x, t)) = [(x, t))
2.4 Homework #4
Problem 14.4
A linear harmonic oscillator in its ground state is exposed to a spatially constant force which at t = 0 is suddenly
removed. Compute the transition probabilities to the excited states of the oscillator. Use the generating function
for Hermite polynomials to obtain a general formula. How much energy is transferred?
Solution
The Hamiltonian for a forced harmonic oscillator is given by (15.65) in Merzbacher as
H = H
0
+H


p
2
2m
+
1
2
m
2
x
2
xF(t) (2.4.1)
Where the perturbing term, H

= xF(t) in this case. We now wish to employ perturbation theory to


see how the energy changes when this force is added. Due to parity, the rst order shift is zero:
E
(1)
n
=
(0)
n
[H

[
(0)
n
) 0
The reason for this is because the perturbation is odd and we are integrating over the entire universe.
The second order shift is not zero, though. The easiest way to solve this problem is to follow the work
done by Merzbacher on Pgs. 84-88. From (5.42),
CHAPTER 2: QUANTUM MECHANICS II 145
n[x[k) =
_

n
(x) x
k
(x) dx
=
_

m
_
_
n
2

k,n1
+
_
n + 1
2

k,n+1
_
(2.4.2)
And all the other terms vanish. So, we have
E
(2)
n
=

n=k
[n[H

[k)[
2
E
(0)
k
E
(0)
n
=

n=k
[n[Fx[k)[
2
E
(0)
n
E
(0)
k
=
F
2

2m
[

n
n,k1
+

n + 1
n,k+1
[
2
E
(0)
k
E
(0)
n
=
F
2

2m
_
[

n
n,k1
[
2
E
(0)
k
E
(0)
n
+
[

n + 1
n,k+1
[
2
E
(0)
k
E
(0)
n
_
=
F
2

2m
_
[

n
n,k1
[
2
(n +
3
/
2
) (n +
1
/
2
)
+
[

n + 1
n,k+1
[
2
(n
1
/
2
) (n +
1
/
2
)
_
=
F
2

2m
_
n


n + 1

_
=
F
2

2m
_

_
=
F
2
2m
2
(2.4.3)
We know that (2.4.3) is, of course, approximate. We can nd the exact value in a few dierent ways.
The method I am choosing to employ entails substituting the perturbing Hamiltonian into the TISE. We
start by dening the following quantity:
x

= x
F
m
2
(2.4.4)
And we recall that the TISE is
_


2
2m

2
x
2
+V (x)
_
(x) = E(x) (2.4.5)
We now recall from (2.4.1), that our potential in this problem is given by
V (x) =
1
2
m
2
x
2
Fx (2.4.6)
Substituting (2.4.4) into (2.4.48),
V (x) =
1
2
m
2
_
x

+
F
m
2
_
2
F
_
x

+
F
m
2
_
=
1
2
m
2
_
x
2
+
F
2
m
2

4
+
2Fx

m
2
_
F
_
x

+
F
m
2
_
W. Erbsen HOMEWORK #4
=
1
2
m
2
x
2
+
m
2
2

F
2
m
2

2
+
m
2
2

2Fx

m
2
Fx

F
2
m
2
=
1
2
m
2
x
2
+
F
2
2m
2
+ Fx

Fx

F
2
m
2
=
1
2
m
2
x
2

F
2
2m
2
(2.4.7)
Substituting (2.4.48) back into the TISE from (2.4.5),
_


2
2m

2
x
2
+
1
2
m
2
x
2

F
2
2m
2
_
(x

) = E(x

)
_

2
2m

2
x
2
+
1
2
m
2
x
2
_
(x

) =
_
E +
F
2
2m
2
_
(x

) (2.4.8)
If we allow for the following substitution, we see that our old harmonic oscillator dierential equation
and can jump to the solution:
E

= E +
F
2
2m
2
(2.4.9)
Then we can rewrite (2.4.8) as
_


2
2m

2
x
2
+
1
2
m
2
x
2
_
(x

) = E

(x

) (2.4.10)
The solution of (2.4.10) is as we would expect,
E

n
=
_
n +
1
/
2
_
(2.4.11)
Substituting (2.4.11) back into (2.4.9),
_
n +
1
/
2
_
= E
n
+
F
2
2m
2
E
n
=
_
n +
1
/
2
_

F
2
2m
2
(2.4.12)
The solution for the eigenfunctios of (2.4.10) have been solved exhaustively, and the solution is given by
(5.39) in Merzbacher (with a few modications) as

n
(x

) =
1

2
n
n!
_

_1
/4
exp
_

2
2
_
H
n
(

) (2.4.13)
Where

may be deduced from the original denition and our denition of x

from (2.4.4),

=
_
m

=
_
m

_
x
F
m
2
_
=
_
m

x
_
m

F
m
2
=
F

m
3
CHAPTER 2: QUANTUM MECHANICS II 147
= a (2.4.14)
Where a is constant. From (2.4.13), we may deduce that our ground state wave function is

0
(x

) =
_

_1
/4
exp
_

2
2
_
(2.4.15)
Our new goal is to nd the transition probability, which is dened as P
n
= [
n
(x)[
0
(x))[
2
. Using our
previous denitions for
n
(x) and
0
(x

), we see that
P
n
=[
n
(x)[
0
(x))[
2
=
_

n
(x)
0
(x) dx
=
_

_
1

2
n
n!
_

_1
/4
exp
_

2
2
_
H
n
()
__
_

_1
/4
exp
_

2
2
_
_
d (2.4.16)
Substituting in our value for

from (2.4.14) into (2.4.16),


P
n
=

_
1

2
n
n!
_

_1
/4
exp
_

2
2
_
H
n
()
__
_
( a)

_1
/4
exp
_

( a)
2
2
__
d

2
(2.4.17)
We now note that (2.4.17) strikes an uncanny resemblance to integral which led to the result obtained
in Problem 6 in Chapter 5, where we are asked to nd the transitional probability of a forced harmonic
oscillator. In this problem, we found that
P
n
=

2n
2
n
n!
exp
_

1
2

2
_
(2.4.18)
Where we dened to be
=
_
m

a
Where, uncoincidentally, I have chosen a in my problem to coincide with a from Problem 5.6. Therefore,
using our previous denition of a, we can use (2.4.18) to say
P
n
=
1
2
n
n!
_
F
2
m
3
_
n
exp
_

1
2
F
2
m
3
_
Problem 14.5
In the nuclear beta decay of a tritium atom (
3
H) in its ground state, an electron is emitted and the nucleus
changes into an
3
He nucleus. Assume that the change is sudden, and compute the probability that the atom is
found in the ground state of the helium atom after the emission. Compute the probability of atomic excitation to
the 2S and 2P states of the helium ion. How probable is excitation to higher levels, including the continuum?
W. Erbsen HOMEWORK #4
Solution
In decay, we know that a neutron decays into a proton, electron, and electron anti-neutrino. For
Tritium,
3
H
3
He + e

+ , where is an anti-neutrino. To nd the probability that the atom is in


the ground state of helium after the emission, we must nd P = [
H
1,0,0
(r, , )[
He
1,0,0
(r, , ))[
2
, which
is the overlap integral between the two states particular to the transition in question. The superscript
H signies the state of Tritium where Z = 1, and similarly He is the Helium ion with Z = 2.
Since both
3
H and
3
He
+
have only one electron, they are Hydrogenic, and we can therefore use
(12.98) from Merzbacher:

1,0,0
(r, , ) =
_
Z
3
a
3
_
1
/2
exp
_

Zr
a
_
(2.4.19)
Additionally, we must know the functional dependence of the radius (a) on Z, which is given by (4.72)
in Griths: ?
a =
4
0

2
me
2
(2.4.20)
And m in our case will translate to the reduced mass , which will most certainly be dierent. The
reduced mass of Tritium is
(
3
H) =
m
e
3m
p
m
e
+ 3m
p
m
e
(2.4.21)
And similarly for (
3
He
+
), which just says that a(
3
H) = a(
3
He
+
) = a
0
. Now we must determine (2.4.19)
for both Tritium and the
3
He nucleus. For Tritium Z = 1, so that (2.4.19) is just

H
1,0,0
(r, , ) =
_
1
3
a
3
_
1
/2
exp
_

r
a
_
(2.4.22)
And for
3
He
+
Z = 2, so:

He
1,0,0
(r, , ) =
_
8
a
3
_1
/2
exp
_

2r
a
_
(2.4.23)
Taking the square of the overlap integral between (2.4.22) and (2.4.23),
P =[
H
1,0,0
(r, , )[
He
1,0,0
(r, , ))[
2
=

__
_
1
a
3
_1
/2
exp
_

r
a
_
_

_
_
8
a
3
_1
/2
exp
_

2r
a
_
__

2
=
_

8
a
3
_
2
0
d
_

0
sin d
_

0
r
2
exp
_

3r
a
_
dr
_
2
=
8

2
a
6
_
4
_

0
r
2
exp
_

3r
a
_
dr
_
2
=
8

2
a
6
16
2
__

0
r
2
exp [r] dr
_
2
_
= 3/a
CHAPTER 2: QUANTUM MECHANICS II 149
=
128
a
6
_
2

3
_
2
=
128
a
6
_
2a
3
27
_
2
=
128
a
6

4a
6
729
=
512
729
P 0.702332 (2.4.24)
To nd the probability of excitation into the 2S and 2P states, the process is very much the same.
However instead of blindly implementing (12.98) we must manually nd the excited eigenfunctions. This
is not so bad, since we know that
n,l,m
(r, , ) = R
n,l
(r)Y
m
l
(, ). The respective radial wave functions
and spherical harmonics can be looked up in any QM textbook, such as chapter 4 in Griths. The wave
function for 2S ends up being,

He
2,0,0
(r, , ) =R
2,0
(r)Y
0
0
(, )
=
1
2

2
_
8
a
3
_1
/2
_
2
2r
a
_
exp
_

r
a
_

1
2
_
1

=
_
1
a
3
_1
/2 _
1
r
a
_
exp
_

r
a
_
(2.4.25)
Taking the inner product with (2.4.19) and squaring the result,
P =[
H
1,0,0
(r, , )[
He
2,0,0
(r, , ))[
2
=

__
_
1
a
3
_1
/2
exp
_

r
a
_
_

_
1
a
3
_1
/2 _
1
r
a
_
exp
_

r
a
_
_

2
=
1

2
a
6

__
exp
_

r
a
__

_
1
r
a
_
exp
_

r
a
__

2
=
1

2
a
6
__
2
0
d
_

0
sin d
_

0
r
2
_
1
r
a
_
exp
_

2r
a
_
dr
_2
=
16
a
6
__

0
r
2
exp
_

2r
a
_
dr
1
a
_

0
r
3
exp
_

2r
a
_
dr
_
2
=
16
a
6
__

0
r
2
exp [r] dr
1
a
_

0
r
3
exp [r] dr
_
2
_
= 2/a
=
16
a
6
_
2

3

1
a
6

4
_
2
=
16
a
6
_
2
8
a
3

3
8
a
3
_
2
=
16
64
P = 0.25 (2.4.26)
To nd the wave function corresponding to the 2P state we repeat the same process that we did in
(2.4.25):
W. Erbsen HOMEWORK #4

He
2,1,0
(r, , ) =R
2,1
(r)Y
0
1
(, )
=
_
4
3a
3
_1
/2
r
a
exp
_

r
a
_

1
2
_
3

cos
=
_
1
a
3
_1
/2
1
a
r exp
_

r
a
_
cos (2.4.27)
And now,
P =[
H
1,0,0
(r, , )[
He
2,1,0
(r, , ))[
2
=

__
_
1
a
3
_1
/2
exp
_

r
a
_
_

_
1
a
3
_1
/2
1
a
r exp
_

r
a
_
cos
_

2
=

1
a
4
__
exp
_

r
a
__

r exp
_

r
a
_
cos
_

2
=
_
1
a
4
_
2
0
d
_

0
sin cos d
_

0
r
3
exp
_

2r
a
_
dr
_
P = 0 (2.4.28)
Because of the integral. The probability of excitation to higher states is just the probability that is left
over from the previous states, eg 1.00 0.70 0.25 = 0.05. Sadly Tritium is not available to the public,
and this is unfortunate for restless high-schoolers who wish to make a low-power inertial connement
fusion device, as Tritium is the ideal source ?, ?.
Problem 14.7
At t < 0 a system is in a coherent state [) (eigenstate of a) of an oscillator and subjected to an impulsive
interaction
V (t) =
i
2

_
a
2
a
2
_
(t) (2.4.29)
Where is a real-valued parameter. Show that the sudden change generates a squeezed state. If the oscillator
frequency is , derive the time dependence of the variances
_

_
a +a

2
__
2
, and
_

_
a a

2
__
2
(2.4.30)
Solution
We know that [) is an eigenstate of a and satises a[) = [). What we want to do is go from our
coherent state [) to a squeezed state [, ). We see that we arrive at a squeezed state by rst nding
the squeezed state operator, b, where we can say
[a[) =[U

bU[)
CHAPTER 2: QUANTUM MECHANICS II 151
=, [b[, ) (2.4.31)
The easiest way to nd b would be to utilize some identities made available by Merzbacher on pgs. 39-40.
He says that if
f() = e
A
Be
A
Then we can write
f() = Bcosh
_
+
[A, B]

sinh
_
(2.4.32)
Which we can use only if is constant, and the operators A and B satisfy [A, [A, B]] = B. In our case,
f() b(), and A = (a
2
a
2
) and B = a. To nd out if we can use (2.4.32):
[A, B] =[(a
2
a
2
), a]
=[a
2
, a] [a
2
, a]
= [a
2
, a]
= a

[a

, a] [a

, a]a

= a

(1) (1)a

=2a

(2.4.33)
[A, [A, B]] =[(a
2
a
2
), 2a

]
=2
_
[a
2
, a

] [a
2
a

]
_
=2
_
a[a, a

] + [a, a

]a
_
=4a (2.4.34)
It seems that we can in fact use (2.4.32), since both of our conditions are satised with = /2 and
= 4. We now have,
b() =a cosh
_
+
[A, B]

sinh
_

=a cosh /2

4 +
2a

4
sinh /2

4
=a cosh +a

sinh
=a cosh a

sinh (2.4.35)
Following Merzbachers lead on pgs. 230-231, we dene = cosh and = sinh , where here is
dierent than the one used before. We can now rewrite (2.4.35):
b = a +a

(2.4.36)
And taking the Hermitian conjugate,
b

= a

+a (2.4.37)
W. Erbsen HOMEWORK #4
Since both and are real. Inverting these two equations
a = b b

(2.4.38)
a

= b

b (2.4.39)
And we can now write
1

2
[a[) =
1

2
, [b[, )
=
1

2
, [a + a

[, ) (2.4.40)
1

2
[a

[) =
1

2
, [b

[, )
=
1

2
, [a

+ a[, ) (2.4.41)
So, we can say that the impulsive action of (2.4.29) generates a squeezed state . We can nd the vari-
ances from (2.4.30) by recalling that we can write
_

_
a + a

2
__
2
=
1
2
_
, [
_
a +a

_
2
[, ) , [
_
a + a

_
[, )
2
_
(2.4.42a)
_

_
a a

2
__
2
=
1
2
_
, [
_
a a

_
2
[, ) , [
_
a a

_
[, )
2
_
(2.4.42b)
We can evaluate (2.4.42a) and (2.4.42b) according to the previous formulas laid out in this problem.
Unfortunately, I did not have time as I have been preparing a presentation for WildCorn. The framework
has been laid out, the rest of this problem is very straightforward.
Exercise 15.5
From the denition (15.31) of the propagator and the Hermitian character of the Hamiltonian, show that
K(r

, r; t

, t) = K

(r, r

; t, t

) (2.4.43)
linking the transition amplitude that reverses the dynamical development between spacetime points (r, t) and
(r

, t

) to the original propagator.


Solution
We rst recall (15.31), which states
K(r, r

; t, t

) = r[T(t, t

)[r

) (2.4.44)
CHAPTER 2: QUANTUM MECHANICS II 153
Where the time evolution operator is given by (14.41) as
T(t, t

) = exp
_

H(t t

)
_
(2.4.45)
Substituting (2.4.45) into (2.4.44) (letting 1 for convenience),
K(r, r

; t, t

) =r[e
iH(tt

)
[r

)
_
r[e
iHt
__
e
iHt

[r

)
_
(2.4.46)
We now take the complex conjugate of both sides of (2.4.46),
K

(r, r

; t, t

) =
__
r[e
iHt
__
e
iHt

[r

)
__

_
e
iHt

[r

)
_

_
r[e
iHt
_

(2.4.47)
Recalling the Hermitian property of the Hamiltonian H

= H

, we can rewrite (2.4.47) as


K

(r, r

; t, t

) =
_
r

[e
iHt

__
e
iHt
[r)
_
r

[e
iH(t

t)
[r) (2.4.48)
Going back to (2.4.46), if we interchange r and r

and also t and t

, then (2.4.46) becomes


K(r

, r; t

, t) =
_
r

[e
iHt

__
e
iHt
[r)
_
r

[e
iH(t

t)
[r) (2.4.49)
We now see that (2.4.49) is equal to (2.4.48), and we can now say
K(r

, r; t

, t) = K

(r, r

; t, t

)
Exercise 15.8
Verify that (15.48) solves the time-dependent Schr odinger equation and agrees with the initial contition (15.32). If
the initial (t

= 0) state of a free particle is represented by (x, 0) = e


ik0x
, verify that (15.35) produces the usual
plane wave.
Solution
Equation (15.48) reads
K(x; x

; t t

) =
_
m
2i(t t

)
exp
_
m(x x

)
2
2i(t t

)
_
(2.4.50)
We want to use this in the time-dependent Schr odinger equation (15.33):
i

t
K(r, r

; t, t

) =
_


2
2m
_

iq
c
A
_
2
+ V
_
K(r, r

; t, t

) (2.4.51)
The LHS of (2.4.51) is
W. Erbsen HOMEWORK #4
i

t
K(r, r

; t, t

) =i

t
__
m
2i(t t

)
exp
_
m(x x

)
2
2i(t t

)
__
=
m(x x

)
2
+i(t t

)
2(t t

)
2

_

im
2t 2t

_1
/2
exp
_

im(x x

)
2
2(t t

)
_
(2.4.52)
While the RHS of (2.4.51) is
_


2
2m
_

iq
c
A
_
2
+V
_
K(r, r

; t, t

) =
_


2
2m
_

iq
c
A
_
2
+ V
_
_
m
2i(t t

)
exp
_
m(x x

)
2
2i(t t

)
_
=

2
2m

2
_
m
2i(t t

)
exp
_
m(x x

)
2
2i(t t

)
_
=
m(x x

)
2
+i(t t

)
2(t t

)
2

_

im
2t 2t

_1
/2
exp
_

im(x x

)
2
2(t t

)
_
(2.4.53)
Since we have a free particle and A = 0 and V = 0. All derivatives were computed with Mathematica.
We do notice, however that the left side of (2.4.51), (2.4.52) is equal to the right side, (2.4.53), therefore
(2.4.50) solves the time-dependent Schr odinger equation.
We are now asked to verify that this agrees with the initial condition given by (15.32), which says:
K(r, r

; t, t) = (r r

) (2.4.54)
Which clearly satises the TDSE for tt

tt = 0 because of the completeness of the eigenfunctions:


K(r, r

; t, t

) =

n=0

n
(r

)
n
(r)e
iEnt/
(2.4.55)
And in our case,
K(r, r

; t, t) =

n=0

n
(r

)
n
(r) = (r r

) (2.4.56)
For the last part of the question, (15.35) reads
(r, t) =
_
K(r, r

; t, t

)(r

, t

)d
3
r

(2.4.57)
Where in our case t

= 0, (x, 0) = e
ik0x
and K(r, r

; t, t

) is given by (2.4.50). So, (2.4.57) becomes:


(r, t) =
_

K(r, r

; t, 0)(r

, 0)d
3
r

=
_

_
m
2it
exp
_
m(x x

)
2
2it
_
e
ik0x

dx

=
_

_
m
2it
exp
_
m(x x

)
2
2it
+ ik
0
x

_
dx

CHAPTER 2: QUANTUM MECHANICS II 155


=
_
m
2it
_

exp
_
m(x
2
2xx

x
2
)
2it
+ ik
0
x

_
dx

=
_
m
2it
_

exp
_
mx
2
2it

mxx

it

mx
2
2it
+ ik
0
x

_
dx

=
_
m
2it
_

exp
_

mx
2
2it

mxx

it
+
mx
2
2it
+ ik
0
x

_
dx

=
_
m
2it
_

exp
_
_

m
2it
_
x
2
+
_
ik
0

mx
it
_
x

+
_
mx
2
2it
__
dx

=
_
m
2it
_

exp
_
ax
2
+ bx

+c
_
dx

=
_
m
2it
_

a
exp
_
b
2
4a
+c
_
(2.4.58)
Where I have set
a =
m
2it
(2.4.59a)
b =ik
0

mx
it
(2.4.59b)
c =
mx
2
2it
(2.4.59c)
We now wish to evaluate (2.4.58) one bit at a time. The rst factor is
_

a
=
_

2it
m

_
2it
m
(2.4.60)
The exponential term becomes
exp
_
b
2
4a
+ c
_
=exp
_

_
ik
0

mx
it
_
2
4
_

m
2it
_ +
mx
2
2it
_
=exp
_
1
4

2it
m
_
k
2
0

2mxk
0
t
+
m
2
x
2

2
t
2
_
+
mx
2
2it
_
=exp
_

ik
2
0
t
2m
ixk
0
+
imx
2
2t

imx
2
2t
_
=exp
_

ik
0
2m
(k
0
t + 2mx)
_
(2.4.61)
Substituting (2.4.60) and (2.4.61) back into (2.4.58),
(r, t) =
_
m
2it

_
2it
m
exp
_

ik
0
2m
(k
0
t + 2mx)
_
(r, t) = exp
_

ik
0
2m
(k
0
t + 2mx)
_
(2.4.62)
Which is indeed a plane wave.
W. Erbsen HOMEWORK #4
Exercise 15.9
Calculate [(x, t)[
2
from (15.50) and show that the wave packet moves uniformly and at the same time spreads so
that
(x)
2
t
= (x)
2
0
_
1 +

2
t
2
4m
2
(x)
4
0
_
(2.4.63)
Solution
Equation (15.50) reads:
(x, t) = [2(x)
2
0
]

1
/4

_
1 +
it
2m(x)
2
0
_

1
/2
exp
_
_

x
2
4(x)
2
0
+ ik
0
x ik
2
0
t
2m
1 +
it
2m(x)
2
0
_
_
(2.4.64)
To make calculating [(x, t)[
2
from (2.4.64) easier recall that if we have (a +ib)/(c +id), then
a +ib
c +id

c id
c id

(a +ib)(c id)
c
2
+d
2
(2.4.65)
Furthermore we remember that if f(x) = exp (e + if) then [f(x)[ = exp (e if) exp (e +if) = exp (2e),
so in further calculations I will recognize that the imaginary phase cancels out ahead of time to simplify
the calculations. So, we have:
[(x, t)[
2
=

[2(x)
2
0
]

1
/4

_
1 +
it
2m(x)
2
0
_

1
/2
exp
_
_

x
2
4(x)
2
0
+ ik
0
x ik
2
0
t
2m
1 +
it
2m(x)
2
0
_
_

2
=

[2(x)
2
0
]

1
/4

_
1 +
it
2m(x)
2
0
_

1
/2

2
. .
[
I
[
2

exp
_
_

x
2
4(x)
2
0
+ik
0
x ik
2
0
t
2m
1 +
it
2m(x)
2
0
_
_

2
. .
[
II
[
2
(2.4.66)
Such that
[
I
[
2
=

[2(x)
2
0
]

1
/4

_
1 +
it
2m(x)
2
0
_

1
/2

2
[2(x)
2
0
]

1
/2

_
1 +

2
t
2
4m
2
(x)
4
0
_

1
/2
(2.4.67)
And also
[
II
[
2
=

exp
_
_

x
2
4(x)
2
0
+ ik
0
x ik
2
0
t
2m
1 +
it
2m(x)
2
0
_
_

2
=exp
_
_

x
2
4(x)
2
0
ik
0
x + ik
2
0
t
2m
1
it
2m(x)
2
0
+

x
2
4(x)
2
0
+ ik
0
x ik
2
0
t
2m
1 +
it
2m(x)
2
0
_
_
CHAPTER 2: QUANTUM MECHANICS II 157
=exp
_
_

x
2
4(x)
2
0
ik
0
x + ik
2
0
t
2m
1
it
2m(x)
2
0

1 +
it
2m(x)
2
0
1 +
it
2m(x)
2
0
+

x
2
4(x)
2
0
+ ik
0
x ik
2
0
t
2m
1 +
it
2m(x)
2
0

1
it
2m(x)
2
0
1
it
2m(x)
2
0
_
_
=exp
_
_

x
2
4(x)
2
0
+
k0xt
2m(x)
2
0

k
2
0

2
t
2
4m
2
(x)
2
0

x
2
4(x)
2
0
+
k0xt
2m(x)
2
0

k
2
0

2
t
2
4m
2
(x)
2
0
1 +

2
t
2
4m(x)
4
0
_
_
=exp
_
_

x
2
2(x)
2
0
+
k0xt
m(x)
2
0

k
2
0

2
t
2
2m
2
(x)
2
0
1 +

2
t
2
4m(x)
4
0
_
_
=exp
_
_

x
2
2
+
k0xt
m

k
2
0

2
t
2
2m
2
(x)
2
0
_
1 +

2
t
2
4m(x)
4
0
_
_
_
(2.4.68)
Combining (2.4.67) and (2.4.68),
[(x, t)[
2
=[
I
[
2
[
II
[
2
=[2(x)
2
0
]

1
/2

_
1 +

2
t
2
4m
2
(x)
4
0
_

1
/2
exp
_
_
_
k0t
m
x
_ _

k0t
2m
+
x
2
_
(x)
2
0
_
1 +

2
t
2
4m(x)
4
0
_
_
_
(2.4.69)
Which is the answer. By graphing this we can see that this does indeed move uniformly and spreads in
time. If we substitute (2.4.63) into (2.4.69),
[(x, t)[
2
= [2(x)
2
t
]

1
/2
exp
_
(k
0
t/mx) (k
0
t/(2m) + x/2)
(x)
2
t
_
(2.4.70)
With the denition of (x)
t
provided by (2.4.63), (2.4.70) is very clearly Gaussian (not that it didnt
look that way before). Please see attached Mathematica printout for plots.
Exercise 15.13
For the harmonic oscillator, derive (x, t) directly from (x, 0) by expanding the initial wave function, which
represents a displaced ground state as in (15.60), in terms of stationary states. Use the generating function (5.33)
to obtain the expansion coecients and again to sum the expansion. Rederive (15.61).
Solution
Using the denition of the propagator from (15.59),
K(x, x

; t) =

..
_
m
2isin(t)
_1
/2
exp
_

m
2isin(t)
(x
2
cos(t) 2xx

+ x
2
cos(t))
_
W. Erbsen HOMEWORK #4
=exp
_

m
2isin(t)
(x
2
cos(t) 2xx

+x
2
cos(t))
_
(2.4.71)
We can nd (x, t) using
(x, t) =
_

K(x, x

; t)(x

, 0) dx

=N
_

exp
_

m
2isin(t)
(x
2
cos(t) 2xx

+ x
2
cos(t))
_
exp
_

m
2
(x

x
0
)
2
_
dx

=N
_

exp
_

m
2isin(t)
(x
2
cos(t) 2xx

+ x
2
cos(t))
m
2
(x

x
0
)
2
_
dx

=N
_

exp
_
m
2
_

x
2
cos(t)
i sin(t)
+
2xx

i sin(t)

x
2
cos(t)
i sin(t)
(x

x
0
)
2
__
dx

=N
_

exp
_
m
2
_

x
2
cos(t)
i sin(t)
+
2xx

i sin(t)

x
2
cos(t)
i sin(t)
x
2
+ 2x

x
0
x
2
0
__
dx

=N
_

exp
_

mx
2
cos(t)
2isin(t)
+
2mxx

2isin(t)

mx
2
cos(t)
2isin(t)

mx
2
2
+
2mx

x
0
2

mx
2
0
2
_
dx

=N
_

exp
_

mx
2
2

mx
2
cos(t)
2isin(t)
+
mxx

isin(t)
+
mx

x
0

mx
2
cos(t)
2isin(t)

mx
2
0
2
_
dx

=N
_

exp
__

m
2

m cos(t)
2isin(t)
_
x
2
+
_
mx
isin(t)
+
mx
0

_
x

+
_

mx
2
cos(t)
2isin(t)

mx
2
0
2
__
dx

(2.4.72)
Where we set
a =
m
2

m cos(t)
2isin(t)
=
m
2
_
1 +
cos(t)
i sin(t)
_
(2.4.73)
b =
mx
isin(t)
+
mx
0

=
m

_
x
i sin(t)
+ x
0
_
(2.4.74)
c =
mx
2
cos(t)
2isin(t)

mx
2
0
2
=
m
2
_
x
2
cos(t)
i sin(t)
+ x
2
0
_
(2.4.75)
Such that (2.4.72) becomes
(x, t) =N
_

exp
_
ax
2
+ bx

+ c
_
dx

=N
__

a
exp
_

b
2
4a
+c
__
(2.4.76)
Where we have
CHAPTER 2: QUANTUM MECHANICS II 159
_

a
=
_

_
m
2
_
1 +
cos(t)
i sin(t)
__
1
_1
/2
=
_

_
m
2
_
i sin(t) + cos(t)
i sin(t)
__
1
_1
/2
=
_
2
m
_
i sin(t)
i sin(t) + cos(t)
__1
/2
(2.4.77)
And also, we can say that
exp
_

b
2
4a
+c
_
=exp
_
_
m

_
x
i sin(t)
+x
0
__
2

1
4
_
m
2
_
1 +
cos(t)
i sin(t)
__
1

m
2
_
x
2
cos(t)
i sin(t)
+ x
2
0
_
_
We can simplify this further,
exp % =exp
_
m
2

2
_
x
i sin(t)
+x
0
_
2

1
4
_
m
2
_
i sin(t) + cos(t)
i sin(t)
__
1

m
2
_
x
2
cos(t)
i sin(t)
+x
2
0
_
_
=exp
_
m
2
_

x
2
sin(
2
t)
+
2xx
0
i sin(t)
+ x
2
0
__
i sin(t)
i sin(t) + cos(t)
_

m
2
_
x
2
cos(t)
i sin(t)
+x
2
0
__
=exp
_
m
2
__

x
2
sin(
2
t)
+
2xx
0
i sin(t)
+ x
2
0
__
i sin(t)
i sin(t) + cos(t)
_

_
x
2
cos(t)
i sin(t)
+ x
2
0
___
=exp
_
m
2
__

ix
2
sin(t)
+ 2xx
0
+ix
2
0
sin(t)
__
1
i sin(t) + cos(t)
_

_
x
2
cos(t)
i sin(t)
+ x
2
0
___
=exp
_
_
_
m
2
_
_

ix
2
sin(t)
+ 2xx
0
+ ix
2
0
sin(t)
cos(t) +i sin(t)

(cos(t) i sin(t))
(cos(t) i sin(t))

_
x
2
cos(t)
i sin(t)
+x
2
0
_
_
_
_
_
_
=exp
_
_
_
m
2
_
_

ix
2
cos(t)
sin(t)
+ 2xx
0
cos(t) + ix
2
0
sin(t) cos(t) x
2
2ixx
0
sin(t) +x
2
0
sin(
2
t)
cos(
2
t) + sin(
2
t)

x
2
cos(t)
i sin(t)
x
2
0
__
=exp
_
m
2
_

ix
2
cos(t)
sin(t)
+ 2xx
0
cos(t) + ix
2
0
sin(t) cos(t) x
2
2ixx
0
sin(t) +x
2
0
sin(
2
t)

x
2
cos(t)
i sin(t)
x
2
0
__
=exp
_
m
2
_
2xx
0
cos(t) + ix
2
0
sin(t) cos(t) x
2
2ixx
0
sin(t) +x
2
0
sin(
2
t) x
2
0

_
=exp
_
m
2
_
2xx
0
cos(t) +
ix
2
0
sin(2t)
2
x
2
2ixx
0
sin(t) +x
2
0
_
1 cos(
2
t)
_
x
2
0
__
=exp
_
m
2
_
2xx
0
cos(t) +
ix
2
0
sin(2t)
2
x
2
2ixx
0
sin(t) x
2
0
cos(
2
t)
__
W. Erbsen HOMEWORK #4
=exp
_
m
2
_
x
2
+ 2xx
0
cos(t) x
2
0
cos(
2
t) 2ixx
0
sin(t) +
ix
2
0
sin(2t)
2
__
=exp
_

m
2
_
x
2
2xx
0
cos(t) + x
2
0
cos(
2
t)

mixx
0
sin(t)

+
imx
2
0
sin(2t)
4
_
=exp
_

m
2
(x x
0
cos(t))
2

mixx
0
sin(t)

+
imx
2
0
sin(2t)
4
_
=exp
_

m
2
(x x
0
cos(t))
2
i
_
mxx
0
sin(t)


mx
2
0
sin(2t)
4
__
(2.4.78)
Going back to (2.4.76), implementing (2.4.77) (keeping (2.4.78) arbitrary for the time being), and sub-
stituting back in our value for ,
(x, t) =N
_
m
2isin(t)
_1
/2
_

a
exp
_

b
2
4a
+ c
_
=N
_
m
2isin(t)
_1
/2

_
2
m
_
i sin(t)
cos(t) +i sin(t)
__1
/2
exp
_

b
2
4a
+c
_
=N
_
m
2isin(t)

2
m
_
i sin(t)
cos(t) +i sin(t)
__1
/2
exp
_

b
2
4a
+c
_
=N
_
1
i sin(t)
_
i sin(t)
cos(t) + i sin(t)
__1
/2
exp
_

b
2
4a
+ c
_
=N
_
1
cos(t) + i sin(t)
_1
/2
exp
_

b
2
4a
+c
_
=N
_
1
(e
it)
+ e
it)
)/2
_
+i
_
e
it)
e
it)
)/(2i)
_1
/2
exp
_

b
2
4a
+ c
_
=N
_
1
e
it)
/2 + e
it)
/2 + e
it)
/2 e
it)
/2
_1
/2
exp
_

b
2
4a
+c
_
=N
_
1
e
it)
_1
/2
exp
_

b
2
4a
+c
_
=Ne
it)/2
exp
_

b
2
4a
+ c
_
(2.4.79)
Now substituting (2.4.78) into (2.4.79),
(x, t) =Ne

it)
2
exp
_

m
2
(x x
0
cos(t))
2
i
_
m

xx
0
sin(t)
m
4
x
2
0
sin(2t)
__
=N exp
_

m
2
(x x
0
cos(t))
2
i
_
m

xx
0
sin(t)
m
4
x
2
0
sin(2t)
_

i
2
t
_
=N exp
_

m
2
(x x
0
cos(t))
2
i
_

2
t +
m

xx
0
sin(t)
m
4
x
2
0
sin(2t)
__
(2.4.80)
Such that we nally arrive at
(x, t) = N exp
_

m
2
(x x
0
cos(t))
2
i
_

2
t +
m

xx
0
sin(t)
m
4
x
2
0
sin(2t)
__
CHAPTER 2: QUANTUM MECHANICS II 161
Which is identical to (15.61).
20 10 10 20
x
0.2
0.4
0.6
0.8
1.0
x,t
Figure 2.7: Graph of [(x, t)[ vs. x for various values of t.
We can also show that [(x, t)[ oscillates without any change in shape, we proceed with the usual mantra
of complex analysis, recognizing that for a complex number z = [z[e
i
, and in our case:
[(x, t)[ =N exp
_

m
2
(x x
0
cos(t))
2
_
(2.4.81)
As can be seen in Fig. (2.4), plotting [(x, t)[ for various times (30 in this case) clearly shows that
[(x, t)[ does not change shape, eg it does not broaden, or otherwise distort. We also note that [(x, t)[
is normalized.
2.5 Homework #5
Part I
Problem 1
Consider the canonical ensemble, dened by the statistical operator
=
_
Tr
_
e
H/kBT
__
1
e
H/kBT
(2.5.1)
for the 1-D harmonic oscillator (with n-degenerate eigenvalues E
n
=
_
n +
1
/
2
_
with n = 0, 1, 2, ...)
a) Show that Tr
_

2
_
< 1. What happens for T 0?
b) Calculate the occupation probability of level n).
W. Erbsen HOMEWORK #5
c) Calculate the averaged energy E) and specic heat /TE).
d) Evaluate and discuss the limits of very large and very small temperatures in part c).
Solution
As is the case most of the time in world of statistical mechanics, we must rst nd the partition function
(the single-particle variety, in our case)
Q
1
=

n=0
exp
_

_
n +
1
/
2
_
=exp
_

1
/
2

n=0
exp [n]
=exp
_

1
/
2

n=0
exp []
n
=exp
_

1
/
2

n=0
1
1 exp []
=
1
exp [
1
/
2
] exp [
1
/
2
]
=
1
2 sinh [
1
/
2
]
(2.5.2)
a) We rst note that the density operator, , is given in the coordinate representation by
= q[ [q) =
_
m

tanh
_

2k
B
T
__1
/2
exp
_

mq
2

tanh
_

2k
B
T
__
(2.5.3)
Which is (5.4.25) from Pathria. To show that Tr
_

2

< 1, we evaluate
2
by squaring (2.5.3):

2
=
m

tanh
_

2k
B
T
_
exp
_

2mq
2

tanh
_

2k
B
T
__
(2.5.4)
Taking the trace of (2.5.4), we nd that
Tr
_

2

=
_


2
dq
=
_

_
m

tanh
_

2k
B
T
_
exp
_

2mq
2

tanh
_

2k
B
T
___
dq
=
m

tanh
_

2k
B
T
_ _

exp
_

2mq
2

tanh
_

2k
B
T
__
dq (2.5.5)
Lets dene:
=
m

tanh
_

2k
B
T
_
Then (2.5.5) becomes
CHAPTER 2: QUANTUM MECHANICS II 163
Tr
_

2

exp
_
2q
2

dq
=

_

2
=
_

2
=

1
2

m

tanh
_

2k
B
T
_
(2.5.6)
We can qualitatively show that Tr
_

2

< 1 by plotting (2.5.6) as a function of temperature (see


Fig. (1)). We see that Tr
_

2

approaches 0.4, which by most accounts is less than 1.


0 5 10 15 20 25
T 0.0
0.1
0.2
0.3
0.4
0.5
Tr^2
Figure 2.8: Graph of Tr
_

2

vs T for Problem 1.
b) In general, the way to nd the average number of things in statistical mechanics is
n) =
1
Q
1

n=0
n exp [nE
n
] (2.5.7)
In our case, (2.5.7) can be written
n) =
1
Q
1

n=0
n exp
_

_
n +
1
/
2
_
=
1
Q
1

n=0
n exp [n] exp
_

1
/
2

=
exp
_

1
/
2

Q
1

n=0
n exp [n] (2.5.8)
We now, seemingly arbitrarily, take the following derivative:

exp [n] = n exp [n] (2.5.9)


Manipulating this a bit more, (2.5.9) becomes
W. Erbsen HOMEWORK #5
nexp [n] =
1

exp [n] (2.5.10)


Substituting (2.5.10) back into (2.5.8),
n) =
exp
_

1
/
2

Q
1

n=0
_

exp [n]
_
=
1

exp
_

1
/
2

Q
1

n=0
exp [n]
_
=
1

exp
_

1
/
2

Q
1

_
1
1 exp []
_
=
1

exp
_

1
/
2

Q
1
_

exp []
(1 exp [])
2
_
=
1
Q
1
_
exp
_

3
/
2

(1 exp [])
2
_
(2.5.11)
Substituting in to (2.5.11) our partition function from (2.5.2), we are left with
n) = 2 sinh
_
1
/
2

_
exp
_

3
/
2

(1 exp [])
2
_
c) The average energy, E), can be found from the partition function from (2.5.2) as
E) =

log [Q
1
]
=

log
_
1
2 sinh[
1
/
2
]
_
=

log
_
2 sinh
_
1
/
2

(2.5.12)
Taking the simple derivative in (2.5.12), we arrive at for the average energy
E) =
1
/
2
coth
_
1
/
2

(2.5.13)
The specic heat is dened as
C
V
=
E)
T
(2.5.14)
Substituting (2.5.13) into (2.5.14),
C
V
=

T
_

2
coth
_

2k
B
T
__


2


2k
B
T
2
csch
2
_

2k
B
T
_
(2.5.15)
We see that (2.5.15) may be readily reduced to
C
V
=

2

2
4k
B
T
2
csch
2
_

2k
B
T
_
(2.5.16)
CHAPTER 2: QUANTUM MECHANICS II 165
d) By plotting (2.5.16) as a function of T (where we let = = k
B
= 1, we can clearly see the trend
at high and low temperatures (see Fig. (2)). As T 0, C
V
0 , and as T , C
V
1 .
For the low-temperature limit, we approach the quantum zero-point energy of E =
1
/
2
, which
is independent of temperature. Therefore, in the low-temperature limit, the specic heat at constant
volume approaches zero.
For the high-temperature limit, C
V
approaches 1, which is what we would expect classically.
0.0 0.5 1.0 1.5 2.0 2.5
T 0.0
0.2
0.4
0.6
0.8
1.0
CV
Figure 2.9: Graph of C
V
vs T for Problem 1.
Problem 2
An electron is in the spin state [)
z
with respect to the z-axis and you measure the electrons spin with re-
spect to a new quantization axis, given by the vector e = (0.6, 0, 0.8). Find the probability for measuring the
electrons spin in the +e direction.
Solution
We rst recall that S = /2 , where the pauli spin matrices are given by

x
=
_
0 1
1 0
_
,
y
=
_
0 i
i 0
_
,
z
=
_
1 0
0 1
_
Our electron is originally spin up in the z-direction, and our goal is to nd the probability that it is spin
up in the +e direction, where e = (0.6, 0, 0.8). To nd this probability, we must calculate:
P =

(e)
+
_

(z)
+

2
(2.5.17)
Where it is known that
(z)
+
is given simply by

(z)
+
=
_
1
0
_
(2.5.18)
W. Erbsen HOMEWORK #5
In order to nd
(e)
+
, we must diagonalize the spin matrix S
e
. We do that by taking the normal component
with S:
S
e
=e S
=0.6S
x
+ 0.8S
z
=

2
_
0.6
_
0 1
1 0
_
+ 0.8
_
1 0
0 1
__
=

2
__
0 0.6
0.6 0
_
+
_
0.8 0
0 0.8
__
=

2
_
0.8 0.6
0.6 0.8
_
(2.5.19)
To nd the eigenvalues and eigenvectors of (2.5.19), we take the determinate of the matrix and set it to
zero:

0.8 0.6
0.6 0.8

= 0 (0.8 ) (0.8 ) (0.6) (0.6) = 0


0.64 0.8 + 0.8 +
2
0.36 = 0
= 1
To nd the eigenvector corresponding to = +1,

2
_
0.8 1 0.6
0.6 0.8 1
_


2
_
0.2 0.6
0.6 1.8
_
(2.5.20)
To nd the eigenvectors,

2
_
0.2 0.6
0.6 1.8
__

_
=
_
0
0
_

0.2 + 0.6 = 0
0.6 1.8 = 0
For the rst line, we can let = 0.6, and = 0.2. Accordingly, our eigenspinor becomes

(e)
+
=
_
0.3
0.1
_
(2.5.21)
We can now substitute (2.5.21) and (2.5.18) back into (2.5.17) to nd the probability,
P =

_
0.3
0.1
_

_
1
0
_

_
0.3 0.1
_
_
1
0
_

2
[0.3[
2
P = 0.09 (2.5.22)
Part II
CHAPTER 2: QUANTUM MECHANICS II 167
Exercise 15.25
If an ensemble E consists of an equal-probability mixture of two nonorthogonal (but normalized) states [
1
)
and [
2
) with overlap C =
1
[
2
), evaluate the Shannon mixing entropy H(E ) and the von Neumann entropy,
S(). Compare the latter with the former as [C[ varies between 0 and 1. What happens as C 0?
Solution
The Shannon mixing entropy is given by (15.126):
H(E ) =
N

i=1
p
i
log [p
i
] (2.5.23)
And the density operator is given by
=
N

i=1
p
i

i

N

i=1
p
i
[
i
)
i
[
And since the states have equal probabilities,
p
1
= p
2
=
1
2
Substituting this into (2.5.23) we have
H(E ) =
2

i=1
p
i
log [p
i
] =
2

i=1
log
_
1
/
2

2
= log [2] = 0.693147... (2.5.24)
The Von Neumann entropy is dened by (15.128):
S() =
n

i=1
p
i
log [p
i
]
n

i=1
p
i
[ log [] [p
i
) Tr( log []) (2.5.25)
Where p
1,2
is given in Exercise 15.24. So,
p
1,2
=
1
_
[
1
[
2
)[
2
2

1 C
2
p
1
=
1 +C
2
, p
2
=
1 C
2
Putting this back into (2.5.25), we have
S() =
_
1 + C
2
_
log
_
1 + C
2
_

_
1 C
2
_
log
_
1 C
2
_
(2.5.26)
For the functional dependence of S() on C please see Fig. (3). As we can see, when C = 0 then
S() = log [2] = H(E ), however as C is increased S() decreases. Explicitly, as C 0, we have
S() = log [2]
W. Erbsen HOMEWORK #5
1.0 0.5 0.5 1.0
C
0.1
0.2
0.3
0.4
0.5
0.6
0.7
S
Figure 2.10: Graph of S() for Problem 15.25.
Problem 15.1
For a system that is characterized by the coordinate r and the conjugate momentum p, show that the expec-
tation value of an operator F can be expressed in terms of the Wigner distribution W(r

, p

) as
F) = [F[) =
__
F
W
(r

, p

)W(r

, p

) d
3
r

d
3
p

(2.5.27)
where
F
W
(r

, p

) =
_
e
ip

/
r

/2[F[r

+ r

/2) d
3
r

(2.5.28)
and where the function W(r

, p

) is dened in Problem 5 in Chapter 3. Show that for the special cases F = f(r)
and F = g(p) these formulas reduce to those obtained in Problem 5 and 6 in Chapter 3, that is, F
W
(r

) = f(r

)
and F
W
(p

) = g(p

).
Solution
The Wigner distribution function dened in Problem 5 in Chapter 3 is:
W(r

, p

) =
1
(2)
3
_
e
ip

_
r

2
_

_
r

+
r

2
_
d
3
r

(2.5.29)
So, putting (2.5.28) and (2.5.29) into (2.5.27),
F) =
__
F
W
(r

, p

)W(r

, p

) d
3
r

d
3
p

=
_ _ __
e
ip

/
r

/2[F[r

+r

/2) d
3
r

_
1
(2)
3
_
e
ip

(r

/2) (r

+ r

/2) d
3
r

_
d
3
r

d
3
p

CHAPTER 2: QUANTUM MECHANICS II 169


=
1
(2)
3
__ ___
e
ip

(r

)/
r

/2[F[r

+ r

/2)

(r

/2) (r

+ r

/2) d
3
r

d
3
r

_
d
3
r

d
3
p

(2.5.30)
We recognize that r

integral collapses when looking at the exponent, which reduces to the delta function
(r

). So, (2.5.30) becomes


F) =
1
(2)
3
__
r

/2[F[r

+r

/2)

(r

/2) (r

+ r

2) d
3
r

d
3
r

(2.5.31)
We also recall from Chapter 17 that the Wigner Exckart Theorem holds if the determinate of J = 1,
where J is
J =
_
_
_
_
_

_
r

2
_

r

_
r

2
_

_
r

+
r

2
_

r

_
r

+
r

2
_
_
_
_
_
_

_
1
1
/
2
1
1
/
2
_
Taking the determinate of J,
Det J =

1
1
/
2
1
1
/
2

(1)(
1
/
2
) (
1
/
2
)(1) 1 (2.5.32)
We can dene two quantities r
1
and r
2
as
r
1
=r

2
r
2
=r

+
r

2
Using these, we can then rewrite (2.5.31) as
F) =
__
r
1
[F[r
2
)

(r
1
) (r
2
) d
3
r
1
d
3
r
2
=
__
r
1
[F[r
2
)r
1
[r
2
) d
3
r
1
d
3
r
2
=
__
r
1
[F[r
2
) (r
1
r
2
) d
3
r
1
d
3
r
2
=F) (2.5.33)
Using the results from (2.5.32) and also the results from (2.5.33), we can therefore safely make the
conclusion that F) can be expressed in terms of W(r

, p

) as in (2.5.27).
The next parts of this problem ask us to show that F
W
(r

) = f(r

) and F
W
(p

) = g(p

). Starting
with F
W
(r

), we rewrite (2.5.28) as
F
W
(r

, p

) =
_
e
ip

/
r

/2[f(r)[r

+r

/2) d
3
r

=
_
e
ip

/
f(r)r

/2[r

+ r

/2) d
3
r

W. Erbsen HOMEWORK #5
=
_
e
ip

/
f(r) (r

/2 (r

+ r

/2)) d
3
r

=
_
e
ip

/
f(r) (r

) d
3
r

(2.5.34)
From (2.5.34) we can safely conclude that F
W
(r

, p

) = f(r

). To do the same for F


W
(p

), we proceed
in very much the same way, except we expand g(p

) in p eigenstates:
F
W
(r

, p

) =
_
e
ip

/
r

/2[g(p

)
__
[p)p[ d
3
p
_
[r

+ r

/2) d
3
r

=
__
e
ip

/
g(p

)r

/2[p)p[r

+r

/2) d
3
r

d
3
p (2.5.35)
Recalling from a previous homework set that
r[p) =
1
(2)
3
/2
e
irp/
(2.5.36)
Rewriting (2.5.35), and applying (2.5.36),
F
W
(r

, p

) =
__
e
ip

/
g(p

)r

/2[p) (r

+r

/2[p))

d
3
r

d
3
p
=
__
e
ip

/
g(p

)
_
1
(2)
3
/2
e
i(r

/2)p/
__
1
(2)
3
/2
e
i(r

+r

/2)p/
_
d
3
r

d
3
p
=
__
e
ip

/
g(p

)
_
1
(2)
3
/2
e
i(r

/2)p/
__
1
(2)
3
/2
e
i(r

+r

/2)p/
_
d
3
r

d
3
p
=
1
(2)
3
__
e
ip

/
g(p

)e
i(r

/2)p/
e
i(r

+r

/2)p/
d
3
r

d
3
p
=
1
(2)
3
__
g(p

)e
i(p

p)r

/
d
3
r

d
3
p (2.5.37)
It is easy to see that the exponential term in (2.5.37) collapses to a -function, and we are left with
nothing other than F
W
(r

, p

) = g(p

).
Exercise 16.8
Prove that the only matrix which commutes with all three Pauli matrices is a multiple of the identity. Also
show that no matrix exists which anti-commutes with all three Pauli matrices.
Solution
Imagining some matrix A:
CHAPTER 2: QUANTUM MECHANICS II 171
A =
_
a b
c d
_
To answer the rst part of the question, we look to evaluate:
[A, ] = [A,
x
] + [A,
y
] + [A,
z
] = 0 (2.5.38)
The rst commutator is given by
[A,
x
] =A
x

x
A
=
_
a b
c d
__
0 1
1 0
_

_
a b
c d
__
0 1
1 0
_
=
_
b a
d c
_

_
c d
a b
_
(2.5.39)
While the second is
[A,
y
] =i
_
b a
d c
_
i
_
c d
a b
_
(2.5.40)
And nally, the third commutator is
[A,
z
] =
_
a b
c d
_

_
a b
c d
_
(2.5.41)
Plugging (2.5.39)-(2.5.41) back into (2.5.38),
[A, ] =
_
b a
d c
_

_
c d
a b
_
+ i
_
b a
d c
_
i
_
c d
a b
_
+
_
a b
c d
_

_
a b
c d
_
=
_
b a
d c
_
+ i
_
b a
d c
_

_
c d
a b
_
+i
_
c d
a b
_
+
_
a b
c d
_

_
a b
c d
_
=
_
b + ib +c ic a d 2b
d + id (a +ia) + 2c c ic (b + ib)
_
(2.5.42)
This leaves us with four equations and four unknowns:
b(1 +i) +c(1 i) = 0 (2.5.43a)
a 2b d = 0 (2.5.43b)
d(1 + i) a(1 + i) + 2c = 0 (2.5.43c)
c(1 i) b(1 +i) = 0 (2.5.43d)
Solving (2.5.43a) for b(1 +i), our four equations become
b(1 +i) = c(1 i) (2.5.44a)
a + 2c
(1 i)
(1 + i)
d = 0 (2.5.44b)
d(1 +i) a(1 + i) + 2c = 0 (2.5.44c)
W. Erbsen HOMEWORK #5
c(1 i) + c(1 i) = 0 c = 0 (2.5.44d)
Now that we know that we require that c = 0, (2.5.44a)-(2.5.44c) become
b(1 +i) = 0 b = 0 (2.5.45a)
a d = 0 a = d (2.5.45b)
d(1 + i) a(1 + i) = 0 (2.5.45c)
From (2.5.44d), (2.5.45a) and (2.5.45b), it is clear that b = c = 0 and a = d. Therefore, we can now say
that the only matrix which commutes with all three Pauli matrices is a multiple of the identity. We note
that multiplying the commutator by a constant will not change this.
The next part of the question asks us to show that no matrix exists which anti-commutes with all
three Pauli matrices. To show this, we proceed in the same way we did in the last problem:
[A,
x
] =
_
b a
d c
_

_
c d
a b
_
,= 0 (2.5.46)
[A,
y
] =i
_
b a
d c
_
i
_
c d
a b
_
,= 0 (2.5.47)
[A,
z
] =
_
a b
c d
_

_
a b
c d
_
,= 0 (2.5.48)
Following the same logic as before, we can see that the anti-commutation condition will only exist if
a = b = c = d = 0, so therefore no matrix exists which anti-commutes with all three Pauli matrices.
Exercise 16.15
In many applications, conservation laws and selection rules cause a decaying two-level system to be prepared
in an eigenstate of
z
, say
_
1
0
_
, and governed by the simple normal Hamiltonian matrix
H = a1 +b
x
(2.5.49)
where a and b are generally complex constants. In terms of the energy dierence E = E
02
E
01
and the decay
rates
1
and
2
, calculate the probabilities of nding the system at time t in state or state , respectively.
Solution
We rst recall that from the prompt, we have =
_
1
0
_
, and according to (2.5.49),
H =a
_
1 0
0 1
_
+ b
_
0 1
1 0
_
=
_
a 0
0 a
_
+
_
0 b
b 0
_
=
_
a b
b a
_
(2.5.50)
CHAPTER 2: QUANTUM MECHANICS II 173
We also note that from the prompt that E = E
02
E
01
, and we recall from (16.81) from Merzbacher,
E
1
=E
01
i

1
2
(2.5.51a)
E
2
=E
02
i

2
2
(2.5.51b)
Taking the dierence of (2.5.51a) and (2.5.51b),
E
2
E
1
=E
02
i

2
2
E
01
i

1
2
=E i

2
(2.5.52)
Where =
2

1
. We also recall from (16.83),
[T(t, 0)[) =
[H[)
E
2
E
1
_
exp
_

iE
2
t

_
exp
_

iE
1
t

__
(2.5.53)
We already know what [H[) is:
[H[) =

_
a b
b a
_

=
_
0 1
_
_
a b
b a
__
1
0
_
=b (2.5.54)
Substituting (2.5.52) and (2.5.54) into (2.5.53),
[T(t, 0)[) =
b
E
i
/
2

_
exp
_

i
_
E
02

i
/
2

2
_
t

_
exp
_

i
_
E
01

i
/
2

1
_
t

__
(2.5.55)
Taking the modulus squared of (2.5.56),
P

=[[T(t, 0)[)[
2
=
[b[
2
[E
i
/
2
[
2
_
_
_

exp
_

i
_
E
02

i
/
2

2
_
t

_
exp
_

i
_
E
01

i
/
2

1
_
t

2
_
_
_
=
[b[
2
(E)
2

1
/
4
()
2
_
_
_

exp
_

i
_
E
02

i
/
2

2
_
t

_
exp
_

i
_
E
01

i
/
2

1
_
t

2
_
_
_
(2.5.56)
The bracketed term in (2.5.56) can be reduced to
... =
_
exp
_
i
_
E
02
+
i
/
2

2
_
t

_
exp
_
i
_
E
01
+
i
/
2

1
_
t

___
exp
_

i
_
E
02

i
/
2

2
_
t

_
exp
_

i
_
E
01

i
/
2

1
_
t

__
W. Erbsen HOMEWORK #6
=exp
_

_
exp
_

(2iE (
1
+
2
)) t
2
_
exp
_
(2iE (
1
+
2
)) t
2
_
+ exp
_

_
(2.5.57)
Substituting (2.5.57) into (2.5.56),
P

=
[b[
2
(E)
2

1
/
4
()
2
_
exp
_

_
exp
_

(2iE (
1
+
2
)) t
2
_
exp
_
(2iE (
1
+
2
)) t
2
_
+ exp
_

__
To nd P

, the only dierence is that [H[) = a, and so we have


P

=
[a[
2
(E)
2

1
/
4
()
2
_
exp
_

_
exp
_

(2iE (
1
+
2
)) t
2
_
exp
_
(2iE (
1
+
2
)) t
2
_
+ exp
_

__
2.6 Homework #6
Exercise 16.23
If a constant magnetic eld B
0
, pointing along the z-axis, and a eld B
1
, rotating with angular velocity in
the xy-plane, act in concert on a spin system (gyromagnetic ratio ), calculate the polarization vector P as a
function of time. Assume P to point in the z-direction at t = 0. Calculate the Rabi oscillations in the rotating
frame, and plot the average probability that the particle has spin down as a function of /
0
for a value of
B
1
/B
0
= 0.1. Show that a resonance occurs when = B
0
. (This arrangement is a model for all magnetic
resonance experiments.)
Solution
The processing magnetic eld, B
1
, can be expressed in terms of its angular frequency and the time t
as
B
1
= B
1
cos (t)

i B
1
sin(t)

j (2.6.1)
The sign in (2.6.1) determines the direction of rotation. Lets let it be rotating clockwise, so that .
Furthermore, we are told that the static eld points purely in the z-direction, so that B
0
= (0, 0, B
0
).
The total magnetic eld is of course
B = B
1
+B
0
B = B
1
cos (t)

i B
1
sin (t)

j + B
0

k (2.6.2)
CHAPTER 2: QUANTUM MECHANICS II 175
We now set our minds to nding the Hamiltonian matrix, H, which describes our system. We now recall
that in general, the Hamiltonian for a particle with magnetic moment in some magnetic eld B is
given by H = B, where H is a matrix. For the case of a spin in a magnetic eld, the magnetic
moment becomes = /2, where denotes the respective Pauli spin matrix.
Accordingly, the Hamiltonian matrix becomes
H =

2
_
B
1
cos (t)
x

i B
1
sin (t)
y

j +B
0

z

k
_
(2.6.3)
We can rewrite (2.6.3) by replacing our trig functions with exponentials:
H =

2
_
B
1
2
_
e
it
+ e
it
_

i
B
1
2i
_
e
it
e
it
_

j +B
0

z

k
_
=

2
_
B
1
2
_
_
e
it
+e
it
_

i
1
i
_
e
it
e
it
_

y

j
_
+B
0

z

k
_
(2.6.4)
We also recall that the Pauli spin matrices are

x
=
_
0 1
1 0
_
,
y
= i
_
0 1
1 0
_
,
z
=
_
1 0
0 1
_
Substituting in
x
,
y
and
z
into (2.6.4),
H =

2
_
B
1
2
_
_
e
it
+ e
it
_
_
0 1
1 0
_

1
i
_
e
it
e
it
_
i
_
0 1
1 0
__
+B
0
_
1 0
0 1
__
=

2
_
B
1
2
_
_
e
it
+ e
it
_
_
0 1
1 0
_

_
e
it
e
it
_
_
0 1
1 0
__
+B
0
_
1 0
0 1
__
(2.6.5)
We can combine the rst two terms in (2.6.5) as
_
0 e
it
+ e
it
e
it
+ e
it
0
_

_
0
_
e
it
e
it
_
e
it
e
it
0
_
= 2
_
0 e
it
e
it
0
_
Using this, (2.6.5) becomes
H =

2
_
B
1
_
0 e
it
e
it
0
_
+B
0
_
1 0
0 1
__
(2.6.6)
We now dene the following quantities

0
=
B
0
2
,
1
=
B
1
2
Accordingly, (2.6.6) becomes
H =
1
_
0 e
it
e
it
0
_

0
_
1 0
0 1
_
(2.6.7)
At this point, we dene the following time-dependent wave function
(t) =
_
c
1
(t)
c
2
(t)
_
(2.6.8)
W. Erbsen HOMEWORK #6
Whose rst derivative with respect to time is of course
d
dt
(t) =
_
c
1
(t)
c
2
(t)
_
(2.6.9)
We now require that our Hamiltonian and associated wave functions satisfy the TDSE as
i
d
dt
(t) = H(t) (2.6.10)
Substituting (2.6.7)-(2.6.9) into (2.6.10) yields
i
_
c
1
(t)
c
2
(t)
_
=
_

1
_
0 e
it
e
it
0
_

0
_
1 0
0 1
__

_
c
1
(t)
c
2
(t)
_
i
_
c
1
(t)
c
2
(t)
_
=
1
_
c
2
(t)e
it
c
1
(t)e
it
_

0
_
c
1
(t)
c
2
(t)
_
_
c
1
(t)
c
2
(t)
_
=i
1
_
c
2
(t)e
it
c
1
(t)e
it
_
+i
0
_
c
1
(t)
c
2
(t)
_
(2.6.11)
It is easy to see that (2.6.11) yields two coupled dierential equations,
c
1
(t) =i
1
c
2
(t)e
it
+ i
0
c
1
(t) (2.6.12a)
c
2
(t) =i
1
c
1
(t)e
it
i
0
c
2
(t) (2.6.12b)
The two equations (2.6.12a) and (2.6.12b) constitute a set of coupled dierential equations. The most
straightforward way to solve these coupled equations is to evaluate the derivative of one of the equations
and then plug in the other one. This was done via Mathematica, including the initial conditions. The
result is
c
1
(t) =
_
cos (

t/2) +
i
/

1
/
2

_
sin (

t/2)

e
it/2
(2.6.13a)
c
2
(t) =
_
i
/

_
(

)
2
(
1
/
2

0
)
2
sin (

t/2)
_
e
it/2
(2.6.13b)
Where I have dened

=
_
(
1
/
2

0
)
2
+
2
1
. We can further simplify (2.6.13a) and (2.6.13b) by
letting
=
_
(

)
2
(
1
/
2

0
)
2

, =

t
2
, =

0

, =
t
2
(2.6.14)
Using , and , we can now rewrite (2.6.13a) and (2.6.13b) as
c
1
(t) =[cos () + i sin ()] e
i
(2.6.15a)
c
2
(t) =isin () e
i
(2.6.15b)
We recall that the Pauli spin matrices from before, and to nd the polarization vector P as a function
of time, we must nd each component by taking the inner product of the respective spin matrix. For
instance, we can nd P
x
as follows:
P
x
=(t)[
x
[(t))
CHAPTER 2: QUANTUM MECHANICS II 177
=

(t)
x
(t)
=
_
c

1
(t) c

2
(t)
_
_
0 1
1 0
__
c
1
(t)
c
2
(t)
_
=
_
c

2
(t) c

1
(t)
_
_
c
1
(t)
c
2
(t)
_
=c

2
(t)c
1
(t) +c

1
(t)c
2
(t) (2.6.16)
Substituting in to (2.6.16) the results from (2.6.15a) and (2.6.15b),
P
x
=
_
isin () e
i
_

_
(cos () + i sin ()) e
i

+
_
(cos () + i sin ()) e
i

_
isin () e
i
_
= isin () e
i
(cos () +i sin ()) e
i
+i(cos () i sin()) e
i
sin() e
i
=i
_
sin () cos () e
2i
i sin
2
() e
2i
+ sin () cos () e
2i
i sin
2
() e
2i
_
=i
_
sin () cos ()
_
e
2i
e
2i

i sin
2
()
_
e
2i
+ e
2i
_
=i
_
sin () cos () [2i sin (2)] i sin
2
() [2 cos (2)]
_
=2sin () cos () sin (2) + 2 sin
2
() cos (2)
=2sin () [cos () sin (2) + sin () cos (2)] (2.6.17)
Similarly, we can nd P
y
in the following way:
P
y
=(t)[
y
[(t))
=

(t)
y
(t)
=
_
c

1
(t) c

2
(t)
_
_
0 i
i 0
__
c
1
(t)
c
2
(t)
_
=
_
ic

2
(t) ic

1
(t)
_
_
c
1
(t)
c
2
(t)
_
=ic

2
(t)c
1
(t) ic

1
(t)c
2
(t) (2.6.18)
Substituting in the appropriate values,
P
y
=i
_
isin () e
i
_

__
cos () + i sin () e
i
_
i
_
(cos () + i sin ()) e
i

_
isin () e
i
_
= sin() e
i
(cos () + i sin ()) e
i
+ (cos () i sin ()) e
i
sin () e
i
=sin ()
_
(cos () +i sin ()) e
2i
+ (cos () i sin ()) e
2i
_
=sin ()
_
cos () e
2i
+ i sin () e
2i
+ cos () e
2i
i sin () e
2i
_
=sin ()
_
cos ()
_
e
2i
+e
2i

+ i sin ()
_
e
2i
e
2i
_
=sin () cos () [2 cos (2)] + i sin () [2i sin (2)]
=2sin() cos () cos (2) sin () sin (2) (2.6.19)
And nally, for P
z
we have
P
z
=(t)[
z
[(t))
=

(t)
z
(t)
=
_
c

1
(t) c

2
(t)
_
_
1 0
0 1
__
c
1
(t)
c
2
(t)
_
W. Erbsen HOMEWORK #6
=
_
c

1
(t) c

2
(t)
_
_
c
1
(t)
c
2
(t)
_
=c

1
(t)c
1
(t) c

2
(t)c
2
(t) (2.6.20)
With the coecients, (2.6.20) turns in to
P
z
=
_
(cos () + i sin ()) e
i

_
(cos () +i sin ()) e
i

_
isin () e
i
_

_
isin () e
i
_
=(cos () i sin ()) (cos () + i sin ()) +
2
sin
2
()
=cos
2
() +
2
sin
2
() +
2
sin
2
()
=cos
2
() + sin
2
()
_

2
+
2
_
(2.6.21)
So, the polarization vector, P, is given by (2.6.17), (2.6.19) and (2.6.21) as
P
x
= 2sin () [cos () sin (2) + sin () cos (2)]
P
y
= 2sin () cos () cos (2) sin () sin (2)
P
z
= cos
2
() + sin
2
()
_

2
+
2
_
Where , , and are dened by (2.6.14). To nd the probability that the spin polarization is up as
a function of time, we evaluate
P

(t) =

(z)
+
[(t))

_
1 0
_
_
c
1
(t)
c
2
(t)
_

2
[c
1
(t)[
2
c

1
(t)c
1
(t) (2.6.22)
And now we have
P

(t) =[cos () + i sin ()]

[cos () + i sin ()] P

(t) = cos
2
() +
2
sin
2
() (2.6.23)
And similarly,
P

(t) =

(z)

[(t))

_
0 1
_
_
c
1
(t)
c
2
(t)
_

2
[c
2
(t)[
2
c

2
(t)c
2
(t) (2.6.24)
And now
P

(t) =(isin ())

(isin ()) P

(t) =
2
sin
2
() (2.6.25)
Resonance occurs when the probability from (2.6.25) equals 1. To evaluate the condition, we rewrite
(2.6.25) by substituting back in the values of and from (2.6.14):
P

(t) =
(
1
)
2
(
1
/
2

0
)
2
+ (
1
)
2
sin
2
_
_
_
(
1
/
2

0
)
2
+ (
1
)
2
2
t
_
_
(2.6.26)
The maximum value of the sin
2
(...) term is 1, so we are left with:
(
1
)
2
(
1
/
2

0
)
2
+ (
1
)
2
= 1
_
1
/
2

0
_
2
= 0 = 2
0
(2.6.27)
Using (2.6.27) and our previous denition of
0
, we nd that
CHAPTER 2: QUANTUM MECHANICS II 179
= 2
_
B
0
2
_
= B
0
We can rewrite (2.6.26) once again by expressing it in terms of /
0
as suggested in the prompt,
P

(t) =
(
1
/
0
)
2
(
1
/
2
/
0
1)
2
+ (
1
/
0
)
2
sin
2
_
1
/
2
_

2
0
_
(
1
/
2
/
0
)
2
+ (
1
/
0
)
2
_
t
_
(2.6.28)
The time-averaged version of (2.6.28) is (recalling that sin
2
(...)) =
1
/
2
)
P

) =
(
1
/
0
)
2
2
_
(
1
/
2
/
0
1)
2
+ (
1
/
0
)
2
_ (2.6.29)
Please see Fig. (2.11) for a plot of (2.6.29). We can clearly see that resonance occurs at /
0
= 2.
1 0 1 2 3 4

0
0.1
0.2
0.3
0.4
0.5
P
Figure 2.11: Resonance plot of P

) for Exercise 16.23.


Exercise 16.26
Show that if the incident spin state is a pure transverse polarization state, the scattering amplitudes for the
initial polarizations P
0
= n are g = ih and the scattering leaves the polarization unchanged, P = P
0
.
Solution
W. Erbsen HOMEWORK #6
We rst recall from (16.130) from Merzbacher that
P =
_
[g[
2
[h[
2
_
P
0
+ i (g

h gh

) n + 2[h[
2
P
0
n n + (g

h +gh

) P
0
n
[g[
2
+ [h[
2
+i (g

h gh

) P
0
n
(2.6.30)
If the initial beam has transverse polarization with the scattering plane perpendicular to P
0
where
P
0
= P
0
n, (2.6.30) becomes
P =
_
[g[
2
+ [h[
2
_
P
0
+i (g

h gh

)
[g[
2
+ [h[
2
+i (g

h gh

) P
0
n (2.6.31)
Which is of course (16.131) from Merzbacher. We now recall (16.129), which states
P =
Tr
_
S
inc
S

d/d
(2.6.32)
We now recall (16.127) and (16.125) from the book, which are (respectively):

inc
=
1
/
2
(1 +P
0
) ,
d
d
= Tr
_

inc
S

(2.6.33)
Substituting these equations from (2.6.33) into (2.6.32), we have
P =
Tr
_
S
1
/
2
(1 +P
0
) S

Tr [
1
/
2
(1 + P
0
) S

S]
(2.6.34)
With the scattering matrix S dened by (16.120) as
S =
_
S
11
S
12
S
21
S
22
_
(2.6.35)
While the Pauli spin matrices are
=
x

i +
y

j +
z

k (2.6.36)
Where the individual spin matrices that constitute (2.6.36) are

x
=
_
0 1
1 0
_
,
y
=
_
0 i
i 0
_
,
z
=
_
1 0
0 1
_
(2.6.37)
We now have the tools to nish the problem, unfortunately I have run out of time. Using (2.6.31)-(2.6.37),
the scattering amplitudes turn out to be g = i and the polarization vector unchanged, P = P
0
.
Exercise 16.28
Assuming that P
0
is perpendicular to the scattering plane, evaluate the asymmetry parameter A, dened as
a measure of the right-left asymmetry by
A =
(d/d)
+
(d/d)

(d/d)
+
+ (d/d)

(2.6.38)
CHAPTER 2: QUANTUM MECHANICS II 181
where the subscripts + and refer to the sign of the product P
0
n. Show that if P
0
= n, the asymmetry
A equals the degree of polarization P dened in (16.132). In particle polarization experiments, this quantity is
referred to as the analyzing power.
Solution
We rst note that (16.132) reads
P = P n = i
g

h gh

[g[
2
+ [h[
2
n (2.6.39)
We now recall that (16.128) from Merzbacher is
d
d
= [g[
2
+[h[
2
+i(g

h gh

)P
0
n (2.6.40)
Using (2.6.40) we can rewrite (2.6.38) as
A =
_
[g[
2
+[h[
2
+ i(g

h gh

)
_

_
[g[
2
+[h[
2
i(g

h gh

)
_
([g[
2
+[h[
2
+ i(g

h gh

)) + ([g[
2
+[h[
2
i(g

h gh

))
(2.6.41)
We note that (2.6.41) may be trivially reduced to
A = i
g

h gh

[g[
2
+ [h[
2
n (2.6.42)
Where we see that (2.6.42) is identical to (2.6.39).
Exercise 16.33
For a mixed state given by the density matrix
=
1
75
_
41 7 + 7i
7 7i 34
_
(2.6.43)
Check the inequalities (15.120), and calculate the eigenvalues and eigenstates. Evaluate the von Neumann entropy,
and compare this with the outcome entropy for a measurement of
z
.
Solution
We rst wish to evaluate the eigenvalues and eigenvectors of . We nd the eigenvectors in the usual
way,

41/75 (7 + 7i) /75


(7 7i) /75 34/75

= 0
_
41
75

__
34
75

_

1
75
2
(7 + 7i) (7 7i) = 0

(41)(34)
75
2

41
75

34
75
+
2

(49 49i + 49i + 49)


75
2
= 0
W. Erbsen HOMEWORK #6

2

_
36
75
_
2
= 0 (2.6.44)
Evaluating the quadratic equation from (2.6.44),

1,2
=
1
_
1 4 (36/75)
2
1

1
=
16
25
,
2
=
9
25
(2.6.45)
Lets nd the eigenvectors. Starting with
1
rst,
_
41/75 16/25 (7 + 7i) /75
(7 7i) /75 34/75 16/25
__

_
=
_
0
0
_
_
7/75 (7 + 7i) /75
(7 7i) /75 14/75
__

_
=
_
0
0
_
_
1 1 + i
1 i 2
__

_
=
_
0
0
_
(2.6.46)
The corresponding eigenvector for (2.6.46) is:

1
=
_
1 +i
1
_
(2.6.47)
Normalizing according to the usual prescription N = [
1
[
1
)]
1
/2
, (2.6.47) becomes

1
=
1

3
_
1 +i
1
_
(2.6.48)
Using
2
from (2.6.45) and following the same prescription, we nd that

2
=
_
1 +i
1
_
(2.6.49)
The normalization factor is, of course, the same. Therefore, our eigenvectors are
=
1
=
2
=
1

3
_
1 + i
1
_
(2.6.50)
The inequalities from (15.120) read
0 Tr
_

2
_
[Tr ]
2
= 1 (2.6.51a)

ii

jj
[
ij
[
2
(2.6.51b)
Lets tackle showing (2.6.51a) rst. We rst note that from (2.6.43), we can easily nd the trace:
Tr =
1
75
(41 + 34) 1 (2.6.52)
Now, we need to nd
2
:

2
=
1
(75)
2
_
41 7 + 7i
7 7i 34
_

_
41 7 + 7i
7 7i 34
_
CHAPTER 2: QUANTUM MECHANICS II 183
=
1
(75)
2
_
41 7 + 7i
7 7i 34
__
41 7 + 7i
7 7i 34
_
=
1
(75)
2
_
(41)
2
+ (7 7i)
2
41 (7 + 7i) + 34 (7 7i)
41 (7 + 7i) + 34 (7 7i) (7 + 7i)
2
+ (34)
2
_
(2.6.53)
The trace of (2.6.53) is
Tr
_

2
_
=(41)
2
+ (7 7i)
2
+ (7 + 7i)
2
+ (34)
2
0.50 (2.6.54)
Therefore, from (2.6.52) and (2.6.54), we can now say that
0 Tr
_

2
_
[Tr ]
2
= 1 (2.6.55)
Furthermore, from (2.6.43) we can now say that

ii

jj
=(41) (34) 1394 (2.6.56)
While we also see that
[
ij
[
2
= (7 + 7i) (7 7i) 98 (2.6.57)
Therefore, from (2.6.56) and (2.6.57) we can say that

ii

jj
[
ij
[
2
The von Neumann entropy is dened by (15.129) as
S() =
N

i=1
p
i
log [p
i
] (2.6.58)
Substituting in our eigenvalues to (2.6.58), we nd that
S() =
16
25
log
_
16
25
_

9
25
log
_
9
25
_
S() = 0.653
The outcome entropy is dened by (15.131) in Merzbacher as
H(K) =
n

j=1
p(K
j
) log [p(K
j
)] (2.6.59)
But we mustnt forget (15.132), which reads
p(K
j
) = Tr [K
j
)K
j
[
Which can be applied to our previous work as
Tr [K
1
)K
1
[ =
41
75
, Tr [K
2
)K
2
[ =
34
75
(2.6.60)
Therefore, applying (2.6.60) to (2.6.59), we nd that
W. Erbsen HOMEWORK #6
H(K) =
41
75
log
_
41
75
_

34
75
log
_
34
75
_
H(K) = 0.688 (2.6.61)
To nd the probability of measuring
(z)
+
, we rst must evaluate

(z)
+
[
z
[
(z)
+
) =
1
3
_
1 i 1
_
_
1 0
0 1
__
1 +i
1
_
=
1
3
_
1 i
1
__
1 + i
1
_
=
1
3
(1 i) (1 +i) 1
=
1
3
We see here that the outcome entropy is larger than the result from a measurement of
z
. Realizing
now that I may have misinterpreted this last part of the problem, we can also say that
H(K) > S()
Problem 16.1a
The spin-zero neutral kaon is a system with two basis states, the eigenstates of
z
, representing a particle K
0
and its antiparticle

K
0
: The operator
x
= CP represents the combined parity (P) and charge conjugation (C),
or particle-anti-particle, transformation and takes = [K
0
) into = [

K
0
). The dynamics is governed by the
Hamiltonian matrix
H = M i

2
(2.6.62)
Where M and are Hermitian 2 2 matrices, representing the mass-energy and decay properties of the system,
respectively. The matrix is positive denite. A fundamental symmetry (under combined CP and time reversal
transformations) requires that
x
M

= M
x
and
x

=
x
.
Show that in the expansion of H in terms of the Pauli matrices, the matrix
z
is absent. Derive the eigenvalues
and eigenstates of H in terms of the matrix elements of M and . Are the eigenstates orthogonal?

Solution
Charge-parity reversal requires that
x
[K
0
) = 1[

K
0
). Furthermore, we recall that the four matrices 1,

x
,
y
and
z
are linearally dependent, and any 2 2 matrix can be represented as
A =
0
1 +
1

x
+
2

y
+
3

z
Which is (16.57) from Merzbacher. Applying this principle to our matrix M,

Whats your favorite color?


CHAPTER 2: QUANTUM MECHANICS II 185
M =a
1
1 + b
1

x
+c
1

y
+d
1

z
=a
1
_
1 0
0 1
_
+b
1
_
0 1
1 0
_
+ c
1
_
0 i
i 0
_
+ d
1
_
1 0
0 1
_
=
_
a
1
+d
1
b
1
ic
1
b
1
+ ic
1
a
1
d
1
_
(2.6.63)
We recall from the prompt that
x
M

= M
x
, which is a requirement. Applying the LHS of this
statement to our matrix M from (2.6.63),

x
M

=
_
0 1
1 0
__
a
1
+ d
1
b
1
ic
1
b
1
+ ic
1
a
1
d
1
_

_
0 1
1 0
__
a
1
+d
1
b
1
+ ic
1
b
1
ic
1
a
1
d
1
_
(2.6.64)
We now compute the RHS of our statement:
M
x
=
_
a
1
+d
1
b
1
ic
1
b
1
+ic
1
a
1
d
1
__
0 1
1 0
_

_
b
1
ic
1
a
1
+d
1
a
1
d
1
b
1
+ic
1
_
(2.6.65)
So, equating (2.6.64) and (2.6.65),

x
M

= M
x

_
b
1
ic
1
a
1
d
1
a
1
+ d
1
b
1
+ ic
1
_
=
_
b
1
ic
1
a
1
+ d
1
a
1
d
1
b
1
+ic
1
_
(2.6.66)
This yields two equations,
b
1
ic
1
+a
1
d
1
= b
1
ic
1
+a
1
+d
1
d
1
= d
1
(2.6.67a)
a
1
+ d
1
+ b
1
+ ic
1
= a
1
d
1
+ b
1
+ ic
1
d
1
= d
1
(2.6.67b)
We see that in order for M to meet the requirement that
x
M

= M
x
, we must have d
1
= 0. We can
rewrite M from (2.6.63) with this change,
M =
_
a
1
b
1
ic
1
b
1
+ ic
1
a
1
_
(2.6.68)
We can equally well apply this principle to the matrix :
=
_
a
2
+d
2
b
2
ic
2
b
2
+ ic
2
a
2
d
2
_
(2.6.69)
Following the logic which led us to (2.6.68), we may use (2.6.69) to deduce that d
2
= 0 according to the
requirement that
x

=
x
, and we can now write
=
_
a
2
b
2
ic
2
b
2
+ ic
2
a
2
_
(2.6.70)
We now substitute (2.6.68) and (2.6.70) into (2.6.62),
H =
_
a
1
b
1
ic
1
b
1
+ ic
1
a
1
_

i
2
_
a
2
b
2
ic
2
b
2
+ ic
2
a
2
_
=
_
a
1
b
1
ic
1
b
1
+ ic
1
a
1
_

_
i
/
2
a
2
i
/
2
(b
2
ic
2
)
i
/
2
(b
2
+ ic
2
)
i
/
2
a
2
_
W. Erbsen HOMEWORK #6
=
_
a
1
b
1
ic
1
b
1
+ ic
1
a
1
_

_
ia
2
/2 (ib
2
+ c
2
) /2
(ib
2
c
2
) /2 ia
2
/2
_
=
_
a
1
ia
2
/2 b
1
ic
1
(ib
2
+ c
2
) /2
b
1
+ ic
1
(ib
2
c
2
) /2 a
1
ia
2
/2
_
(2.6.71)
We must now nd the eigenvalues of H, in the usual way

a
1
ia
2
/2 b
1
ic
1
(ib
2
+ c
2
) /2
b
1
+ ic
1
(ib
2
c
2
) /2 a
1
ia
2
/2

=0
_
a
1

i
/
2
a
2

_
2

_
b
1
ic
1

1
/
2
(ib
2
+ c
2
)
_ _
b
1
+ic
1

1
/
2
(ib
2
c
2
)
_
=0 (2.6.72)
We can just go ahead and solve for ,
= a
1

i
/
2
a
2

_
(b
1
ic
1

1
/
2
(ib
2
+c
2
)) (b
1
+ic
1

1
/
2
(ib
2
c
2
)) (2.6.73)
We can simplify (2.6.73) ever so slightly, giving eigenvalues

1,2
= a
1

i
/
2
a
2

_
(2b
1
ib
2
)
2
+ (2c
1
ic
2
)
2
(2.6.74)
To nd the eigenvectors, we substitute
1
and
2
from (2.6.74) into our Hamiltonian from (2.6.72) as
_
a
1
ia
2
/2
1,2
b
1
ic
1
(ib
2
+c
2
) /2
b
1
+ ic
1
(ib
2
c
2
) /2 a
1
ia
2
/2
1,2
__
c
1
c
2
_
=
_
0
0
_
(2.6.75)
The eigenvectors were found using Mathematica. The result is

1,2
= N
_

(2b1ib2)
2
+(2c1ic2)
2
2b1ib2+2ic1+c2
1
_
(2.6.76)
The normalization in (2.6.76) is obtained as we usually do:

1
[
1
) = 1 N
2

1

1
= 1 N
2
_
c
1
c
2
_

_
c
1
c
2
_
= 1 N =
1
_
c

1
c
1
+ c

2
c
2
(2.6.77)
This normalization constant was determined from (2.6.77) using Mathematica, the result is
N =
_
_
_
(2b
1
+ib
2
)
2
+ (2c
1
+ ic
2
)
2
2b
1
+ ib
2
2ic
1
+ c
2
+
_
(2b
1
ib
2
)
2
+ (2c
1
ic
2
)
2
2b
1
ib
2
+ 2ic
1
+ c
2
_
_
1
/2
(2.6.78)
Combining (2.6.76) and (2.6.78), the normalized eigenvectors are

1,2
=
_
_
_
(2b
1
+ ib
2
)
2
+ (2c
1
+ic
2
)
2
2b
1
+ ib
2
2ic
1
+ c
2
+
_
(2b
1
ib
2
)
2
+ (2c
1
ic
2
)
2
2b
1
ib
2
+ 2ic
1
+ c
2
_
_
1
/2
_

(2b1ib2)
2
+(2c1ic2)
2
2b1ib2+2ic1+c2
1
_
(2.6.79)
To see if the eigenstates from (2.6.79) are orthogonal, we apply the following test:
CHAPTER 2: QUANTUM MECHANICS II 187

1
[
2
) = 0 (if orthogonal) (2.6.80)
Using Mathematica, it is easy to show that (2.6.80) is not satised, and therefore
1
and
2
are not orthogonal .
2.7 Homework #7
Exercise 17.12
Derive the recursion relation (17.54), as indicated.
Solution
We rst recall three three relations dening various quantities specifying the behavior of the total angular
momentum:
J =J
1
+J
2
J
z
=J
z1
+J
z2
J
+
=J
1+
+J
2+
(2.7.1a)
We now recall the three equations from (17.27) of Merzbacher:
J
z
[j m) =m[j m) (2.7.2a)
J
+
[j m) =
_
(j m)(j +m + 1) [j
1
j
2
j m+ 1) (2.7.2b)
J

[j m) =
_
(j +m)(j m + 1) [j
1
j
2
j m 1) (2.7.2c)
We also recall (17.52), which reads
[j
1
j
2
j m) =

m1,m2
[j
1
j
2
m
1
m
2
)j
1
j
2
m
1
m
2
[j
1
j
2
j m) (2.7.3)
Which is the link between the direct product basis and the total angular momentum basis. According
to Merzbacher, in order to obtain the recursion relations from (17.54), all we have to do is apply J
+
and J

to (2.7.3). The behavior of J


+
and J

is dened by (2.7.2b) and (2.7.2c), which may be more


compactly redened as
J

[j m) =
_
(j m)(j m + 1) [j
1
j
2
j m 1) (2.7.4)
Similarly, we may generalize (2.7.1a) as
J

= J
1
+J
2
(2.7.5)
We can apply (2.7.5) to (2.7.3), which yields
W. Erbsen HOMEWORK #7
J

[j
1
j
2
j m) = J

_

m1,m2
[j
1
j
2
m
1
m
2
)j
1
j
2
m
1
m
2
[j
1
j
2
j m)
_
(2.7.6)
We must now evaluate the LHS and the RHS of (2.7.6) in turn. Starting with the LHS, we have
J

[j
1
j
2
j m) =
_
(j m)(j m+ 1) [j
1
j
2
j m 1) (2.7.7)
Which comes as no surprise. Evaluating the RHS of (2.7.6) poses a more arduous problem, which we
shall see momentarily:
J

m1,m2
[j
1
j
2
m
1
m
2
)j
1
j
2
m
1
m
2
[j
1
j
2
j m) =(J
1
+ J
2
)

m1,m2
[j
1
j
2
m
1
m
2
)j
1
j
2
m
1
m
2
[j
1
j
2
j m)
=J
1

m1,m2
[j
1
j
2
m
1
m
2
)j
1
j
2
m
1
m
2
[j
1
j
2
j m)
+J
2

m1,m2
[j
1
j
2
m
1
m
2
)j
1
j
2
m
1
m
2
[j
1
j
2
j m) (2.7.8)
We express the rst term on the RHS of (2.7.8) as I and the second as II, and we can rewrite it as
J

m1,m2
[j
1
j
2
m
1
m
2
)j
1
j
2
m
1
m
2
[j
1
j
2
j m) = I +II (2.7.9)
So, now we must evaluate I and II in turn. In doing this, we must recognize that J
1
and J
2
only work
on particle 1 and 2, respectively. Accordingly,
I =J
1

m1,m2
[j
1
j
2
m
1
m
2
)j
1
j
2
m
1
m
2
[j
1
j
2
j m)
=

m1,m2
_
(j
1
m
1
)(j
1
m
1
+ 1) [j
1
j
2
m
1
1 m
2
)j
1
j
2
m
1
m
2
[j
1
j
2
j m) (2.7.10)
And similarly, we can also evaluate II as
II =J
2

m1,m2
[j
1
j
2
m
1
m
2
)j
1
j
2
m
1
m
2
[j
1
j
2
j m)
=

m1,m2
_
(j
1
m
1
)(j
1
m
1
+ 1) [j
1
j
2
m
1
m
2
1)j
1
j
2
m
1
m
2
[j
1
j
2
j m) (2.7.11)
Substituting (2.7.10) and (2.7.11) into (2.7.9) and combining the result with (2.7.7),
_
(j m)(j m + 1) j
1
j
2
m
1
m
2
[j m 1) =
_
(j
1
m
1
)(j
1
m
1
+ 1) [j
1
j
2
m
1
1 m
2
)j
1
j
2
m
1
m
1
[j m)
+
_
(j
2
m
2
)(j
1
m
2
+ 1) [j
1
j
2
m
1
m
2
1)j
1
j
2
m
1
m
2
[j m)
Where I have suppressed the presence of j
1
j
2
in the ket to save room. We note that this is identical to
(12.54), and is what we were trying to show.
Exercise 17.13
CHAPTER 2: QUANTUM MECHANICS II 189
Show that a symmetry exists along the three quantum numbers j
1
, j
2
and j, and that in addition to (17.57)
they satisfy the equivalent relations
[j j
2
[ j
1
j +j
2
and [j j
1
[ j
2
j + j
1
(2.7.12)
Solution
We rst recall (17.57) from Merzbacher, which reads
[j
1
j
2
[ j j
1
+ j
2
(2.7.13)
We note that the magnitude on the LHS of (2.7.18) can in fact be two dierent things:
[j
1
j
2
[ = j
1
j
2
or [j
1
j
2
[ = j
2
j
1
With these possibilities, in addition to the RHS of (2.7.18), we have three inequalities to play with:
j j
1
+ j
2
(2.7.14a)
j
1
j +j
2
(2.7.14b)
j
2
j +j
1
(2.7.14c)
Using (2.7.19) and (2.7.21), we can write
j
1
j j
2
j
1
j
2
j
_
j
1
[j j
2
[ (2.7.15)
Using (2.7.22) and (2.7.20), we can now say
[j j
2
[ j
1
j + j
2
Similarly, using (2.7.19) and (2.7.20), we can declare that
j
2
j j
1
j
2
j
1
j
_
j
2
[j j
1
[ (2.7.16)
And now combining (2.7.16) with (2.7.21),
[j j
1
[ j
2
j + j
1
Exercise 17.15
Verify the values of two special C-G coecients:
j 1 j 0[j 1 j j) =

j
j + 1
(2.7.17a)
W. Erbsen HOMEWORK #7
j 2 j 0[j 2 j j) =

j(2j 1)
(j + 1)(2j + 3)
(2.7.17b)
Determine the value of the trivial C-G coecient j 0 m 0[j 0 j m). How does it depend on the value of m?
Solution
As was done in class, we start our journey by calculating the C-G coecients by starting with the largest
magnetic quantum number m = j
1
+j
2
and repeatedly applying the lowering operator J

= J
1+
+J
2
.
So,
[j
1
, j
2
, j = j
1
+j
2
, m = j
1
+j
2
) =

_
2(j
1
+ j
2
)
[j
1
, j
2
, m
1
= j
1
, m
2
= j
2
) (2.7.18)
Applying J

to the LHS of (2.7.18) rst,


J

[j
1
, j
2
, j = j
1
+j
2
, m = j
1
+j
2
) =[j
1
, j
2
, j = j
1
+j
2
, m = j
1
+j
2
1) (2.7.19)
And now applying J

to the RHS of (2.7.18),


J

N[j
1
, j
2
, m
1
= j
1
, m
2
= j
2
) =(J
1+
+ J
2
) N[j
1
, j
2
, m
1
= j
1
, m
2
= j
2
)
=N [j
1
, j
2
, m
1
= j
1
1, m
2
= j
2
) + [j
1
, j
2
, m
1
= j
1
, m
2
= j
2
1)
(2.7.20)
Using these results, and substituting back in the normalization factor N, we nd that
[j = j
1
+ j
2
, m = j
1
+ j
2
1) =

j
1
j
1
+j
2
[m
1
= j
1
1, m
2
= j
2
) +

j
2
j
1
+ j
2
[m
1
= j
1
, m
2
= j
2
1)
(2.7.21)
In our special case, (2.7.21) becomes
m
1
= j, m
2
= 0[j, m = j) =

j
j + 1
We continue along the same lines, and using Mathematica to take care of the matrix algebra we nd that
m
1
= j
1
2, m
2
= j
2
[j = j
1
+j
2
2, m = j
1
+ j
2
2) =

j
2
(2j
2
1)
(j
1
+ j
2
1) (2j
1
+ 2j
2
1)
(2.7.22)
Applying the appropriate constants, (2.7.22) becomes
m
1
= j, m
2
= 2[j = j, m = j) =

j (2j 1)
(j + 1) (2j + 3)
We evaluate the last part of the question by simply using (2.7.18). The result is
CHAPTER 2: QUANTUM MECHANICS II 191
m
1
= m, m
2
= 0[j = j, m = m) =
1

_
2 (j
1
+j
2
)
m
1
= m, m
2
= 0[j = j, m = m) =
1

2j
Exercise 17.16
Work out the results (17.63) from the recursion relations for C-G coecients and the normalization and stan-
dard phase conditions.
Solution
The results from (17.63) read

j
1
,
1
/
2
, m
1
/
2
,
1
/
2
[ j
1
,
1
/
2
, j
1

1
/
2
, m
_
=

j
1
m+
1
/
2
2j
1
+ 1
(2.7.23a)

j
1
,
1
/
2
, m +
1
/
2
,
1
/
2
[ j
1
,
1
/
2
, j
1

1
/
2
, m
_
=

j
1
m +
1
/
2
2j
1
+ 1
(2.7.23b)
While the recursion relation from (17.54) reads
_
(j m)(j m + 1) j
1
j
2
m
1
m
2
[j m 1) =
_
(j
1
m
1
)(j
1
m
1
+ 1) [j
1
j
2
m
1
1 m
2
)j
1
j
2
m
1
m
1
[j m)
+
_
(j
2
m
2
)(j
1
m
2
+ 1) [j
1
j
2
m
1
m
2
1)j
1
j
2
m
1
m
2
[j m)
(2.7.24)
Where it is important to recognize that the j
1
j
2
terms in the ket are still suppressed. We also notice that
(2.7.23a) and (2.7.23b) are C-G coecients, and must take the form of j
1
, j
2
, m
1
, m
2
[j
1
, j
2
, j, m).
Therefore, comparing this to the LHS of (2.7.23a), we may deduce that
j
2
=
1
/
2
, m
1
= m
1
/
2
, m
2
=
1
/
2
, j = j
1

1
/
2
(2.7.25)
We substitute these values into the recursion relation from (2.7.24). We denote the LHS of (2.7.24) as
LHS, while the rst and second terms of the RHS are RHS
1
and RHS
2
, respectively. Accordingly,
have:
LHS =
_
(j m)(j m + 1) j
1
, j
2
, m
1
, m
2
[j
1
, j
2
, j, m 1)
=
_
(j
1

1
/
2
m)(j
1

1
/
2
m + 1) j
1
,
1
/
2
, m
1
/
2
,
1
/
2
[j
1
,
1
/
2
, j
1

1
/
2
, m1)
RHS
1
=
_
(j
1
(m
1
/
2
))(j
1
(m
1
/
2
) + 1) [j
1
,
1
/
2
, m
1
/
2
1,
1
/
2
)j
1
,
1
/
2
, m
1
/
2
,
1
/
2
[j
1
,
1
/
2
, j
1

1
/
2
, m)
=
_
(j
1
m
1
/
2
)(j
1
m
1
/
2
+ 1) [j
1
,
1
/
2
, m
1
/
2
1,
1
/
2
)j
1
,
1
/
2
, m
1
/
2
,
1
/
2
[j
1
,
1
/
2
, j
1

1
/
2
, m)
RHS
2
=
_
(j
2
m
2
)(j
1
m
2
+ 1) [j
1
, j
2
, m
1
, m
2
1)j
1
, j
2
, m
1
, m
2
[j
1
, j
2
, j, m)
=
_
(
1
/
2

1
/
2
)(j
1

1
/
2
+ 1) [j
1
,
1
/
2
, m
1
/
2
,
1
/
2
1)j
1
,
1
/
2
, m
1
/
2
,
1
/
2
[j
1
,
1
/
2
, j
1

1
/
2
, m)
W. Erbsen HOMEWORK #7
Seemingly arbitrarily, we choose the top sign in the above equations:
LHS =
_
(j
1
+m +
1
/
2
) (j
1
m+
3
/
2
) j
1
,
1
/
2
, m
1
/
2
,
1
/
2
[j
1
+
1
/
2
, m
1
)
RHS
1
=
_
(j
1
+m
1
/
2
) (j
1
+ m+
1
/
2
) [j
1
,
1
/
2
, m +
1
/
2
,
1
/
2
)j
1
,
1
/
2
, m
1
/
2
,
1
/
2
[j
1
+
1
/
2
, m)
RHS
2
=0
Combining these equations and rearranging,
m
1
/
2
,
1
/
2
[j
1
+
1
/
2
, m) =

j
1
+ m +
1
/
2
j
1
+ m +
3
/
2
m +
1
/
2
,
1
/
2
[j
1
+
1
/
2
, m + 1) (2.7.26)
Shifting the index according to m m + 1, we now have
m +
1
/
2
,
1
/
2
[j
1
+
1
/
2
, m + 1) =

j
1
+m +
3
/
2
j
1
+m +
5
/
2
m +
3
/
2
,
1
/
2
[j
1
+
1
/
2
, m+ 2) (2.7.27)
Going back to (2.7.23a) and applying m = j
1
+
1
/
2
to (2.7.26) and (2.7.27), we nd that
j
1
,
1
/
2
, m
1
/
2
,
1
/
2
[j
1
,
1
/
2
, j
1

1
/
2
, m) =

j
1
+m +
1
/
2
2j
1
+ 1
(2.7.28)
Substituting back in the appropriate signs into (2.7.28),

j
1
,
1
/
2
, m
1
/
2
,
1
/
2
[ j
1
,
1
/
2
, j
1

1
/
2
, m
_
=

j
1
m +
1
/
2
2j
1
+ 1
(2.7.29)
Which we note is identical to (2.7.23a). To derive (2.7.23b), we repeat the same process. Evaluating the
LHS and RHS of our recursion relation yields
LHS =
_
(j
1

1
/
2
m)(j
1

1
/
2
m + 1) j
1
,
1
/
2
, m +
1
/
2
,
1
/
2
[j
1

1
/
2
, m1)
RHS
1
=
_
(j
1
(m+
1
/
2
))(j
1
(m +
1
/
2
) + 1) [j
1
,
1
/
2
, m +
1
/
2
1,
1
/
2
)j
1
,
1
/
2
, m+
1
/
2
,
1
/
2
[j
1

1
/
2
, m)
RHS
2
=
_
(
1
/
2
(
1
/
2
))(j
1
(
1
/
2
) + 1) [j
1
,
1
/
2
, m +
1
/
2
,
1
/
2
1)j
1
,
1
/
2
, m+
1
/
2
,
1
/
2
[j
1

1
/
2
, m)
The steps taken to arrive at (2.7.23b) are from henceforth identical to that which was done to derive
(2.7.23a), and accordingly they will not be shown. From the above equations, we choose an arbitrary
sign (which will be stuck back in at the end) in order to simplify the equation. We then shift the index
of m, nagle the equations and evaluating the coecient in terms of j
1
and m, we get

j
1
,
1
/
2
, m+
1
/
2
,
1
/
2
[ j
1
,
1
/
2
, j
1

1
/
2
, m
_
=

j
1
m +
1
/
2
2j
1
+ 1
Which is of course (2.7.23b).
Exercise 17.18
CHAPTER 2: QUANTUM MECHANICS II 193
Starting with the uncoupled basis (17.67), work out the 4 4 matrices S
z
and S
2
, and show by explicit diag-
onalzation that the singlet and triplet states are the eigenvectors with the appropriate eigenvalues.
Solution
We recall that (17.67) from Merzbacher reads

2
=
1

2
,
1

2
=
1

2
,
1

2
=
1

2
,
1

2
=
1

2
(2.7.30)
We introduce the following shorthand notation, as indicated in lecture, which constitutes the product
basis:
[++), [+), [+), [)
Where the rst term in each ket acts on particle 1, while the second acts on particle 2. We also recall
that we can write the total spin operator S in terms of the single particle spin operators S
1
and S
2
as
S = S
1
1
(2)
+ 1
(1)
S
2
(2.7.31)
We also note that when referring to the individual components of the spin operators, it will be most
convenient to work in terms of the raising and lowering spin operators (S
+
and S

, respectively). The
raising and lowering operators are described in terms of the cartesian components by
S

= S
z
iS
y
So that S(S
x
, S
y
, S
z
) S(S

, S
z
). We now recognize that the S acts on the individual spin kets
according to
S
z
[+) =

/
2
[+), S
z
[) =

/
2
[)
S
+
[+) = 0, S
+
[) = [+)
S

[+) = [), S

[) = 0
We also musnt forget how the total components act on each particle, following the convention of (2.7.31),
S
z
=S
1z
1
(2)
+ 1
(1)
S
2z
S
+
=S
1+
1
(2)
+1
(1)
S
2+
S

=S
1
1
(2)
+ 1
(1)
S
2
Accordingly, we can now write
S
z
[++) =

/
2
[++) +

/
2
[++) [++) (2.7.32a)
S
z
[+) =0 (2.7.32b)
S
z
[+) =0 (2.7.32c)
S
z
[) =

/
2
[)

/
2
[) [) (2.7.32d)
We can express (2.7.32a)-(2.7.32d) in matrix form as
W. Erbsen HOMEWORK #7
(S
z
)
m1,m2
=
_
_
_
_
0 0 0
0 0 0 0
0 0 0 0
0 0 0
_
_
_
_
(2.7.33)
The value of the eigenvalues from (2.7.34) are transparent,

1
= ,
2
=
We now wish to nd S
2
, rst representing it in the product basis:
S
2
=S
2
1
+S
2
2
+ 2S
1z
S
2z
+ S
1+
S
2
+S
1
S
2+
(2.7.34)
In order to evaluate all the matrix elements for S
2
, lets make a table:
S
2
1
S
2
2
S
1z
S
2z
S
1+
S
2
S
1
S
2+
[++)
3
/
4

2
[++)
3
/
4

2
[++)
1
/
2
[++)
1
/
2
[++) 0 [+) [+) 0
[+)
3
/
4

2
[+)
3
/
4

2
[+)
1
/
2
[+)
1
/
2
[+) 0 0 [) [+)
[+)
3
/
4

2
[+)
3
/
4

2
[+)
1
/
2
[+)
1
/
2
[+) [+) [) 0 0
[)
3
/
4

2
[)
3
/
4

2
[)
1
/
2
[)
1
/
2
[) [+) 0 0 [)
Substituting the values of the tabulated matrix elements into (2.7.34), we nd that:
S
2
[++) =
3
/
4

2
[++) +
3
/
4

2
[++) + 2
_
1
/
2
[++)
_ _
1
/
2
[++)
_
2
2
[++) (2.7.35a)
S
2
[+) =
3
/
4

2
[+) +
3
/
4

2
[+) + 2
_
1
/
2
[+)
_ _

1
/
2
[+)
_
+
2
[+)
2
([+) +[+) (2.7.35b)
S
2
[+) =
3
/
4

2
[+) +
3
/
4

2
[+) + 2
_

1
/
2
[+)
_ _
1
/
2
[+)
_
+
2
[+)
2
([+) +[+)) (2.7.35c)
S
2
[) =
3
/
4

2
[) +
3
/
4

2
[) + 2
_

1
/
2
[)
_ _

1
/
2
[)
_
2
2
[) (2.7.35d)
We can nd the expectation values of S
2
(i.e. the matrix elements) easily, and the non-zero elements are
+ + [S
2
[++) =2
2
+ [S
2
[+) =
2
+ [S
2
[+) =
2
+ [S
2
[+) =
2
+ [S
2
[+) =
2
+ + [S
2
[++) =2
2
While all the other elements are of course zero. We are now in a place to write down the matrix S
2
:
_
S
2
_
m1,m2
=
2
_
_
_
_
2 0 0 0
0 1 1 0
0 1 1 0
0 0 0 2
_
_
_
_
(2.7.36)
Which is not diagonal. We can remedy this by diagonalizing the central matrix, allowing us to nd the
eigenvalues and eigenvectors of S
2
.
CHAPTER 2: QUANTUM MECHANICS II 195
(S

)
2
=
_

2
_
Finding the eigenvalues,

2

2

= 0
_

_ _

4
= 0

_
2
2
_
= 0

1
= 0,
2
= 2
2
Whose corresponding eigenvectors are
V
1
=
1

2
([+) [+)) , V
2
=
1

2
([+) + [+))
The eigenvectors which we did not include here are found from the rst and fourth column of the matrix
S
2
. Combining the results, we arrive at a series of states whose total angular momentum is either 0 or
1, i.e. either singlet or triplet. The states are
[0 0) =
1

2
([+) [+))
[1 m) =[++)
[1 m) =
1

2
([+) + [+))
[1 m) =[)
Problem 17.1
In the notation of (17.64) the state of a spin one-half particle with sharp total angular momentum j, m is
aY
jm
j
1
/2
+ bY
jm
j+
1
/2
(2.7.37)
Assume this state to be an eigenstate of the Hamiltonian with no degeneracy other than that demanded by
rotational invariance.
a) If H conserves parity, how are the coecients a and b restricted?
b) If H is invariant under time reversal, show that a/b must be imaginary.
c) Verify explicitly that the expectation value of the electric dipole moment e r vanishes if either parity is
conserved or time reversal invariance holds (or both).
Solution
W. Erbsen HOMEWORK #7
We rst note that (17.64) reads
Y
jm

= Y

1
/2, m

=
1

2 + 1
_

_
m +
1
/
2
Y
m
1
/2

_
m +
1
/
2
Y
m+
1
/2

_
(2.7.38)
Where Y
jm

denotes the common eigenstates of J


z
and J
2
. Furthermore, if parity is conserved, then we
require that

P
_
aY
jm
j
1
/2
+ bY
jm
1+
1
/2
_
=(1)
j
1
/2
aY
jm
j
1
/2
+ (1)
j+
1
/2
bY
jm
j+
1
/2
(2.7.39)
Problem 17.4
The Hamiltonain of the positronium atom in the 1S state in a magnetic eld B along the z-axis is to good
approximation
H = AS
1
S
2
+
eB
0
mc
(S
1z
S
2z
) (2.7.40)
if all higher energy states are neglected. The electron is labeled as particle 1 and the positron as particle 2. Using
the coupled representation in which S
2
= (S
1
+S
2
)
2
and S
z
= S
1z
+S
2z
are diagonal, obtain the energy eigenvalues
and eigenvectors and classify them according to the quantum numbers associated with constants of the motion.
Empirically, it is known that for B = 0 the frequency of the 1
3
S 1
1
S transition is 2.0338 10
5
MHz and
that the mean lifetime for annihilation are 10
10
s for the singlet state (two photon decay) and 10
7
s for the
triplet state (three-photon decay). Estimate the magnetic eld strength B
0
which will cause the lifetime of the
longer lived m = 0 state to be reduced (quenched) to 10
8
s.
Solution
We start by nding the constants of motion of the problem, which is found by evaluating the commutator
of the Hamiltonian from (2.7.40) with various quantities and noting which equals zero. To do this, we
recall the following commutation relations for S
2
:
_
S
2
, S
z

=0
_
S
2
, S
z1

,=0,
_
S
2
, S
z2

,= 0
_
S
2
, S
1

=
_
S
2
, S
2

= 0
_
S
2
, S
2
1

=
_
S
2
, S
2
2

= 0
And similarly, some relations for S
z
are
[S
z
, S
z1
] =[S
z
, S
z2
] = 0
_
S
z
, S
2
1

=
_
S
z
, S
2
2

= 0
From the prompt, we are told to use the coupled representation, where S
2
= (S
1
+S
2
)
2
and S
z
= S
z1
+S
z2
are diagonal. We can evaluate the dot product in (2.7.40) by expanding the given denition of S
2
:
CHAPTER 2: QUANTUM MECHANICS II 197
S
2
=(S
1
+ S
2
)
2
S
2
1
+S
2
2
+ 2S
1
S
2
S
1
S
2
=
1
/
2
_
S
2
S
2
1
S
2
2
_
(2.7.41)
Substituting (2.7.41) into (2.7.40), the Hamiltonian becomes
H =
1
/
2
A
_
S
2
S
2
1
S
2
2
_
+
eB
0
mc
(S
1z
S
2z
) (2.7.42)
Lets not evaluate some commutators to nd the constants of motion:
_
H, S
2

=
__
1
/
2
A
_
S
2
S
2
1
S
2
2
_
+ (S
z1
S
z2
)
_
, S
2

=
1
/
2
A
__
S
2
, S
2

. .
=0

_
S
2
1
, S
2

. .
=0

_
S
2
2
, S
2

. .
=0
_
+
_ _
S
z1
, S
2

. .
=0

_
S
z2
, S
2

. .
=0
_
,= 0 (2.7.43)
[H, S
z
] =
__
1
/
2
A
_
S
2
S
2
1
S
2
2
_
+ (S
z1
S
z2
)
_
, S
z

=
1
/
2
A
__
S
2
, S
z

. .
=0

_
S
2
1
, S
z

. .
=0

_
S
2
2
, S
z

. .
=0
_
+
_
[S
z1
, S
z
]
. .
=0
[S
z2
, S
z
]
. .
=0
_
= 0 (2.7.44)
[H, S
1
] =
__
1
/
2
A
_
S
2
S
2
1
S
2
2
_
+ (S
z1
S
z2
)
_
, S
1

=
1
/
2
A
__
S
2
, S
1

. .
=0

_
S
2
1
, S
1

. .
=0

_
S
2
2
, S
1

. .
=0
_
+
_
[S
z1
, S
1
]
. .
=0
[S
z2
, S
1
]
. .
=0
_
= 0 (2.7.45)
[H, S
2
] =
__
1
/
2
A
_
S
2
S
2
1
S
2
2
_
+ (S
z1
S
z2
)
_
, S
2

=
1
/
2
A
__
S
2
, S
2

. .
=0

_
S
2
1
, S
2

. .
=0

_
S
2
2
, S
2

. .
=0
_
+
_
[S
z1
, S
2
]
. .
=0
[S
z2
, S
2
]
. .
=0
_
= 0 (2.7.46)
So, S
2
is not a constant of motion according to (2.7.43), while S
z
, S
1
, and S
2
are constants of motion
from (2.7.44)-(2.7.46). We now wish to switch to the following basis notation,
[++), [+), [+), [)
We now wish to evaluate the diagonal matrix elements for H, rst recalling that
H =
1
/
2
A
_
S
2
S
2
1
S
2
2
_
+ (S
1z
S
2z
)
S
2
S
2
1
S
2
1
S
1z
S
2z
[++) 2
2
[++)
3
/
4

2
[++)
3
/
4

2
[++)
1
/
2
[++)
1
/
2
[++)
[+) 0
3
/
4

2
[+)
3
/
4

2
[+)
1
/
2
[+)
1
/
2
[+)
[+) 0
3
/
4

2
[+)
3
/
4

2
[+)
1
/
2
[+)
1
/
2
[+)
[) 2
2
[)
3
/
4

2
[)
3
/
4

2
[)
1
/
2
[)
1
/
2
[)
Substituting the tabulated elements into our Hamiltonian,
H[++) =
1
/
2
A
_
2
2

3
/
4

3
/
4

2
_
[++)
1
/
4
A
2
[++)
H[+) =
1
/
2
A
_

3
/
4

3
/
4

2
_
[+) +
_
1
/
2
+
1
/
2

_
[+)
_

3
/
4
A
2
+
_
[+)
H[+) =
1
/
2
A
_

3
/
4

3
/
4

2
_
[+) +
_

1
/
2

1
/
2

_
[+)
_

3
/
4
A
2

_
[+)
H[) =
1
/
2
A
_
2
2

3
/
4

3
/
4

2
_
[)
1
/
4
A
2
[)
These calculations were clearly done in the direct product basis, whereas it could just as easily be done
in the total angular momentum basis.
W. Erbsen HOMEWORK #9
2.8 Homework #9
Problem 17.6
The magnetic moment operator for a nucleon of mass m
n
is = e (g

L +g
S
S) /2m
n
c, where g

= 1 and g
S
= 5.587
for a proton, g

= 0 and g
S
= 3.826 for a neutron. In a central eld with an additional spin-orbit interaction, the
nucleons move in shells characterized by the quantum numbers and j =
1
/
2
. Calculate the magnetic moment
of a single nucleon as a function of j for the two kinds of nucleons, distinguishing the two cases j = +
1
/
2
and
j =
1
/
2
. Plot j times the eective gyromagnetic ratio versus j, connected in each case the points by straight-line
segments (Schmidt lines).
Solution
We start by rewriting the magnetic moment operator :
=
e
2m
n
c
(g

L + g
s
S) (2.8.1)
Where m
n
is the mass of the nucleon. If we choose the quantization axis of the interaction to be along
the z-direction, the expectation value of the magnetic moment operator is given by

z
) = sjm
j
[
z
[sjm
j
) (2.8.2)
We can rewrite (2.8.1) according to our chosen quantization axis as

z
=
e
2m
n
c
(g

L
z
+ g
s
S
z
) (2.8.3)
Substituting (2.8.3) into (2.8.2), we have

z
) =
e
2m
n
c
sjm
j
[(g

L
z
+g
s
S
z
)[sjm
j
) (2.8.4)
At this point, we recall that J
z
= L
z
+S
z
L
z
= J
z
S
z
. Substituting this into (2.8.4),

z
) =
e
2m
n
c
sjm
j
[(g

(J
z
S
z
) + g
s
S
z
)[sjm
j
)
=
e
2m
n
c
sjm
j
[(g

J
z
+ (g
s
g

) S
z
)[sjm
j
)
=
e
2m
n
c
g

sjm
j
[J
z
[sjm
j
) + (g
s
g

) sjm
j
[S
z
[sjm
j
) (2.8.5)
Our task is now to evaluate the expectation values J
z
) and S
z
) on the RHS of (2.8.5). The tool that
allows us to do this is the Wigner Eckart Theorem (W.E.T.). Applying this to S
z
) rst,
sjm
j
[S
z
[sjm
j
) = j1m
j
0[j1jm
j
)j|S|j) (2.8.6)
We can apply the W.E.T. along the same lines to the expectation value of J
z
):
sjm
j
[J
z
[sjm
j
) = j1m
j
0[j1jm
j
)j|J|j) (2.8.7)
We now notice that the conveniently uncalculated C-G coecients in (2.8.6) and (2.8.7) are in fact equal.
We can therefore take the ratio of these two equations, thus ridding them from our lives:
CHAPTER 2: QUANTUM MECHANICS II 199
sjm
j
[S
z
[sjm
j
)
sjm
j
[J
z
[sjm
j
)
=
j1m
j
0[j1jm
j
)j|S|j)
j1m
j
0[j1jm
j
)j|J|j)
sjm
j
[S
z
[sjm
j
)
sjm
j
[J
z
[sjm
j
)
=
j|S|j)
j|J|j)
sjm
j
[S
z
[sjm
j
) =
j|S|j)
j|J|j)
sjm
j
[J
z
[sjm
j
) (2.8.8)
At this point, we make a small deviation to evaluate the reduced matrix elements. In order to do this,
we must evaluate J S) and J J):
sjm
j
[J S[sjm
j
) =j1m
j
0[j1jm
j
)j|J|j)j|S|j) (2.8.9a)
sjm
j
[J J[sjm
j
) =j1m
j
0[j1jm
j
)j|J|j)j|J|j) (2.8.9b)
Not surprisingly, we notice once again that the C-G coecients are again the same in (2.8.9a) and
(2.8.9b). Taking the ratio of these two equations,
sjm
j
[J S[sjm
j
)
sjm
j
[J J[sjm
j
)
=
j1m
j
0[j1jm
j
)j|J|j)j|S|j)
j1m
j
0[j1jm
j
)j|J|j)j|J|j)
sjm
j
[J S[sjm
j
)
sjm
j
[J J[sjm
j
)
=
j|S|j)
j|J|j)
(2.8.10)
Switching the LHS and RHS in (2.8.10), and substituting in the appropriate eigenvalues, (2.8.10) becomes
j|S|j)
j|J|j)
=
1
/
2
_
j(j + 1)
2
+s(s + 1)
2
( + 1)
2

j(j + 1)
2
=
j(j + 1) + s(s + 1) ( + 1)
2j(j + 1)
(2.8.11)
Substituting (2.8.11) back into (2.8.4),
sjm
j
[S
z
[sjm
j
) =
j(j + 1) + s(s + 1) ( + 1)
2j(j + 1)
sjm
j
[J
z
[sjm
j
) (2.8.12)
And nally, substituting (2.8.12) into (2.8.5):

z
) =
e
2m
n
c
_
g

sjm
j
[J
z
[sjm
j
) + (g
s
g

)
_
j(j + 1) +s(s + 1) ( + 1)
2j(j + 1)
_
sjm
j
[J
z
[sjm
j
)
_
(2.8.13)
Substituting in the appropriate eigenvalues for J
z
), (2.8.13) simplies to

z
) =
ej
2m
n
c
_
g

+ (g
s
g

)
_
j(j + 1) + s(s + 1) ( + 1)
2j(j + 1)
__
(2.8.14)
Now, in our particular case, each of the nucleons has s =
1
/
2
, and so (2.8.14) becomes

z
) =
ej
2m
n
c
_
g

+ (g
s
g

)
_
j(j + 1) ( + 1) +
3
/
4
2j(j + 1)
__
(2.8.15)
Now, we mustnt forget that we are interested when we do not have an empty sub-shell, since a lled
sub-shell must have a zero (total) angular momentum. Because j is always equal to an integer plus
1
/
2
,
W. Erbsen HOMEWORK #9
the occupancy of the sub-shell is always even.
Since the nuclear spin is non-zero in our case, we have a contribution from the nuclei towards a
magnetic dipole moment. This is justied since both the neutrons and the protons have an intrinsic
magnetic moment. Additionally, we must consider that the proton is charged, meaning that it can
produce an additional magnetic moment due to its orbital motion.
We therefore look at two unique cases of the total angular momentum: when j = +
1
/
2
, and also
when j =
1
/
2
. Lets look at j = +
1
/
2
rst. To do this, we substitute the value back into (2.8.15):

z
) =
e( +
1
/
2
)
2m
n
c
_
g

+ (g
s
g

)
_
( +
1
/
2
)(( +
1
/
2
) + 1) ( + 1) +
3
/
4
2( +
1
/
2
)(( +
1
/
2
) + 1)
__
=
e( +
1
/
2
)
2m
n
c
_
g

+ (g
s
g

)
_
( +
1
/
2
)( +
3
/
2
) ( + 1) +
3
/
4
2( +
1
/
2
)( +
3
/
2
))
__
=
e( +
1
/
2
)
2m
n
c
_
g

+ (g
s
g

)
_

2
+
3
/
2
+
1
/
2
+
3
/
4

2
+
3
/
4
2( +
1
/
2
)( +
3
/
2
))
__
=
e( +
1
/
2
)
2m
n
c
_
g

+ (g
s
g

)
_
+
3
/
2
2( +
1
/
2
)( +
3
/
2
))
__
=
e( +
1
/
2
)
2m
n
c
_
g

+ (g
s
g

)
1
2( +
1
/
2
)
_
=
e
2m
n
c
_
g

( +
1
/
2
) +
g
s
2

g

2
_
=
e
2m
n
c
_
g

+
g
s
2
_
(2.8.16)
We now wish to do the same thing for j =
1
/
2
:

z
) =
e(
1
/
2
)
2m
n
c
_
g

+ (g
s
g

)
_
(
1
/
2
)((
1
/
2
) + 1) ( + 1) +
3
/
4
2(
1
/
2
)((
1
/
2
) + 1)
__
=
e(
1
/
2
)
2m
n
c
_
g

+ (g
s
g

)
_
(
1
/
2
)( +
1
/
2
) ( + 1) +
3
/
4
2(
1
/
2
)( +
1
/
2
)
__
=
e(
1
/
2
)
2m
n
c
_
g

+ (g
s
g

)
_
1
2

( + 1) +
3
/
4
2(
1
/
2
)( +
1
/
2
)
__
=
e
2m
n
c
_
g

+
g

2g
s
( + 1)
2(2 + 1)
_
=
e
2m
n
c
_
g

_
+
1
2(2 + 1)
_
g
s
_
+ 1
2 + 1
__
=
e
2m
n
c
_

1
/
2
+
1
/
2
_
( + 1)g

g
s
2
_
_
(2.8.17)
So, in summary, we have from (2.8.16) and (2.8.17) that

z
) =
_

_
e
2m
n
c
_
g

+
g
s
2
_
, j = +
1
/
2
e
2m
n
c
_

1
/
2
+
1
/
2
_
( + 1)g

g
s
2
_
_
, j =
1
/
2
(2.8.18)
We can rewrite (2.8.18) slightly as
CHAPTER 2: QUANTUM MECHANICS II 201

z
) =
_

_
e
2m
n
c
_
g

+
g
s
2
_
, j = +
1
/
2
e
2m
n
c
_
j
j + 1
_
( + 1)g

g
s
2
_
_
, j =
1
/
2
A plot of jg

is plotted vs. j on attached printout.


Problem 17.9
A system that is invariant under rotation is perturbed by a quadrupole interaction
V =
2

q=2
C
q
T
q
2
(2.8.19)
where the C
q
are constant coecients and T
q
2
are the components of an irreducible spherical tensor operator,
dened by one of its components:
T
2
2
= (J
x
+iJ
y
)
2
(2.8.20)
a) Deduce the conditions for the coecients C
q
if V is to be Hermitian.
b) Consider the eect of the quandrupole perturbation on the manifold of a degenerate energy eigenstate of the
unperturbed system with angular momentum quantum j, neglecting all other unperturbed energy eigenstates.
What is the eect of the perturbation on the manifold of an unperturbed j =
1
/
2
state?
c) If C
2
= C
0
and C
1
= 0, calculate the perturbed energies for a j = 1 state, and plot the energy splittings
as a function of the interaction strength C
0
. Derive the corresponding unperturbed energy eigenstates.
Solution
a) In order for V to be Hermitian, we require that it be self-adjoint. From (4.5) in Merzbacher,
V

= V
Where self-adjoint implies that the matrix is its own conjugate transpose, e.g.
V

= V (2.8.21)
We must also recall that applying the self-adjoint condition to an operator with two elements, we
have:
(A B)

=
_
B A
_

B

A

In order to evaluate the specic conditions we must require for each of the components, we must
expand (2.8.19) and apply the Hermitian requirement from (2.8.21):
V =C
2
T
2
2
+ C
1
T
1
2
+C
0
T
0
2
+ C
1
T
1
2
+ C
2
T
2
2
(2.8.22)
W. Erbsen HOMEWORK #9
Applying the self-adjoint condition from (2.8.21) to (2.8.22),
V

=
_
C
2
T
2
2
_
+
_
C
1
T
1
2
_
+ (C
0
T
0
2
)

+(C
1
T
1
2
)

+ (C
2
T
2
2
)

=T
2
2
C

2
+ T
1
2
C

1
+ T
0
2
C

0
+ T
1
2
C

1
+ T
2
2
C

2
(2.8.23)
The coecients C
q
in (2.8.23) are non-conjugatable, as coecients tend not to be. Therefore,
(2.8.23) can be rewritten as
V

=T
2
2
C

2
+ T
1
2
C

1
+T
0
2
C

0
+T
1
2
C

1
+T
2
2
C

2
=T
2
2
C

2
+T
1
2
C

1
+T
0
2
C

0
+ T
1
2
C

1
+ T
2
2
C

2
(2.8.24)
Where I have rewritten the tensor operators as T
q
k
= T
q
k
, for reasons that will become clear
momentarily. In the last homework, we showed that T
q
k
= (1)
q
T
q
k
, which applied to (2.8.24)
yields
V

=
_
T
2
2
_

C

2
+
_
T
1
2
_

C

1
+
_
T
0
2
_

C

0
+
_
T
1
2
_

C

1
+
_
T
2
2
_

C

2
=T
2
2
C

2
T
1
2
C

1
+T
0
2
C

0
T
1
2
C

1
+T
2
2
C

2
(2.8.25)
Equating (2.8.22) and (2.8.25),
C
2
T
2
2
+C
1
T
1
2
+ C
0
T
0
2
+C
1
T
1
2
+ C
2
T
2
2
=T
2
2
C

2
T
1
2
C

1
+ T
0
2
C

0
T
1
2
C

1
+ T
2
2
C

2
(2.8.26)
We note that (2.8.26) can only be satised if:
C
0
R
C
1
= C

1
C
2
= C

2
b) In order to study the eect of the quadrupole perturbation on the manifold of a degenerate energy
eigenstate of the unperturbed system with generic total angular momentum quantum number j, we
must investigate the selection rules for an arbitrary irreducible 2
nd
rank tensor operator.
We rst recall the two main stipulations on q and k, dened by (17.88) and (17.89), respectively
as:
q = m

m (2.8.27a)
[j j

[ k j + j

(2.8.27b)
We also recollect the Wigner Eckart Theorem from (17.87):

[T
q
k
[jm) = jkmq[jkj

|T
k
|j) (2.8.28)
Where we require that the bounds on T
q
k
are limited by q. For a second rank irreducible tensor
operator, we have q = 2, and therefore:
q = 2, 1, 0, 1, 2
Now, applying the possible values of q to (2.8.27a) rst, we nd that
m = 2, 1, 0, 1, 2 m = 0, 1, 2 (2.8.29)
And similarly for (2.8.27b),
CHAPTER 2: QUANTUM MECHANICS II 203
[j[ 2, 1, 0, 1, 2 j j = 0, 1, 2 (2.8.30)
It is easy to see that (2.8.29) and (2.8.30) express the relevant selection rules for an irreducible
second-rank tensor operator T
2
q
. Another important result worth noting is that when both of these
conditions are satised, we require that

[T
q
k
[jm) , = 0 (2.8.31)
We now must nd a connection between j and k. Starting with arbitrary j, we recognize that the
condition where we are most likely to nd a limiting relationship between j and k is if we let j = j

:
0 k 2j k 2j j
k
2
(2.8.32)
This gives us a condition in which the expectation value from (2.8.31) is non-zero. We can extend
(2.8.32) to give us a condition in which T
q
k
is zero:
j <
k
2
(2.8.33)
In order to apply this result to our specic case where j =
1
/
2
we note that:
1
/
2
< 1 T
q
2
) = 0 for j =
1
/
2
(2.8.34)
We can verify this more quantitatively by considering the worst case, where we are most likely to
fail (my reasoning for believing that this is the case will be apparent in due course), is when j = 0
and m = 0. Therefore, the Wigner Eckart Theorem from (2.8.28),

[T
q
2
[jm) = 0200[0200)

|T
2
|j) (2.8.35)
The Clebsch-Gordon coecient in (2.8.35) can be found in Merzbacher from (17.62):
j2j0[j2jj) =

j(2j 1)
(j + 1)(2j + 3)
0200[0200) = 0 (2.8.36)
Substituting (2.8.36) into (2.8.35), we nd that
T
q
2
) = 0 (2.8.37)
Where (2.8.37) is the same as (2.8.34). In summary, it is clear from these result indicate that
the eect of the quadrupole perturbation on our system is null .
c) Using the fact that C
2
= C
0
and C
1
= 0, we can rewrite (2.8.22) as
V =C
0
T
2
2
+C
0
T
0
2
+C
0
T
2
2
V = C
0
_
T
2
2
+ T
0
2
+ T
2
2

(2.8.38)
We are also given one of the components of the irreducible spherical tensor operator from (2.8.20).
Even though it was not mentioned earlier, it is quite apparent that this component can be rewritten
in terms of J
+
:
T
2
2
= (J
x
+iJ
y
)
2
J
2
+
(2.8.39)
Using (2.8.39), we can now write

T
2
2
[jm) =

[J
2
+
[jm)
W. Erbsen HOMEWORK #9
=
_
(j m)(j + m+ 1)
_
(j m1)(j + m+ 2)
2

,m+2

jj

(2.8.40)
Now, from the W.E.T. , we have

[T
2
2
[jm) =j, 2, m, 2[j, 2, j

, m

|T|j) (2.8.41)
Rewriting the reduced matrix element on the RHS of (2.8.41),
j

|T|j) =
_
(j m)(j +m + 1)(j m 1)(j + m + 2)
2
j2m2[j2jm + 2)

jj
(2.8.42)
Since the reduced matrix element is independent of the choice of m, we arbitrary choose m = 1,
and (2.8.42) becomes
j

|T|j) =
_
j
2
(j + 1)
2

2
j, 2, 1, 2[j, 2, j, 1)

jj
(2.8.43)
Using (2.8.41) and (2.8.43), we can now express the matrix elements for all tensor components:
j

[T
q
2
[jm) =
j(j + 1)
2
j, 2, 2, 2[j, 2, j, 1)

jj
j, 2, m, q[j, 2, j

, m

) (2.8.44)
Where we recall that selection rules dictate that q = m

m. Dropping the in (2.8.44) for


convenience (since =

) and applying j = j

= 1,
1, m

[T
q
2
[1, m) =
2
2
1, 2, 2, 2[1, 2, 1, 1)
1, 2, m, q[1, 2, 1, m

) (2.8.45)
From (2.8.38), we can see that the only components we require are for T
2
2
, T
0
2
and T
2
2
, which we
will evaluate separately. Starting with T
2
2
, we evaluate (2.8.45) with q = 2, which yields
1, m

[T
2
2
[1, m) =
2
2
1, 2, 2, 2[1, 2, 1, 1)
1, 2, m, 2[1, 2, 1, m

) (2.8.46)
In order for (2.8.46) to be nonzero, the selection rules (derived previously) require that q = m

m,
or in our case m

= m 2. The only nonzero element comes from m = 1 m

= 1, and so
(2.8.46) becomes
1, 1[T
2
2
[1, 1) =
2
2
1, 2, 2, 2[1, 2, 1, 1)
1, 2, 1, 2[1, 2, 1, 1) 2
2
(2.8.47)
We now wish to evaluate T
0
2
in very much the same way. Substituting q = 0 into (2.8.45),
1, m

[T
0
2
[1, m) =
2
2
1, 2, 2, 2[1, 2, 1, 1)
1, 2, m, 0[1, 2, 1, m

) (2.8.48)
Selection rules require that m = m

, so we have three non-zero components, corresponding to


m = 1, m = 0, and m = 1 (respectively). Substituting each of these m-values into (2.8.48),
1, 1[T
0
2
[1, 1) =
2
2
1, 2, 2, 2[1, 2, 1, 1)
1, 2, 1, 0[1, 2, 1, 1)
2
2

6
(2.8.49a)
1, 0[T
0
2
[1, 0) =
2
2
1, 2, 2, 2[1, 2, 1, 1)
1, 2, 0, 0[1, 2, 1, 0)
4
2

6
(2.8.49b)
1, 1[T
0
2
[1, 1) =
2
2
1, 2, 2, 2[1, 2, 1, 1)
1, 2, 1, 0[1, 2, 1, 1)
2
2

6
(2.8.49c)
CHAPTER 2: QUANTUM MECHANICS II 205
And nally, we must evaluate the matrix elements for T
2
2
. We do this in very much the same way
as before, setting q = 2 in (2.8.45)
1, m

[T
2
2
[1, m) =
2
2
1, 2, 2, 2[1, 2, 1, 1)
1, 2, m, 2[1, 2, 1, m

) (2.8.50)
From the selection rules, we have m = 1 m

= 1, and so the only nonzero matrix element is


from (2.8.48)
1, 1[T
0
2
[1, 1) =
2
2
1, 2, 2, 2[1, 2, 1, 1)
1, 2, 1, 0[1, 2, 1, 1) 2
2
(2.8.51)
We can now begin to construct our matrices corresponding to the calculated matrix elements for
T
2
2
, T
0
2
and T
2
2
, which were calculated in (2.8.47), (2.8.49a)(2.8.49c), and (2.8.51), respectively.
Most generically, the matrix T
q
2
) is given by
T
q
2
) =
_
_
[1,

1) [1, 0) [1, 1)
1,

1[ 1,

1[T
q
2
[1,

1) 1,

1[T
q
2
[1, 0) 1,

1[T
q
2
[1, 1)
1, 0[ 1, 0[T
q
2
[1,

1) 1, 0[T
q
2
[1, 0) 1, 0[T
q
2
[1, 1)
1, 1[ 1, 1[T
q
2
[1,

1) 1, 1[T
q
2
[1, 0) 1, 1[T
q
2
[1, 1)
_
_
(2.8.52)
So, each of our three elements are given by
T
2
2
) =
_
_
[1,

1) [1, 0) [1, 1)
1,

1[ 0 0 2
2
1, 0[ 0 0 0
1, 1[ 0 0 0
_
_
(2.8.53a)
T
0
2
) =
_
_
[1,

1) [1, 0) [1, 1)
1,

1[ 2
2
/

6 0 0
1, 0[ 0

4
2
/

6 0
1, 1[ 0 0 2
2
/

6
_
_
(2.8.53b)
T
2
2
) =
_
_
[1,

1) [1, 0) [1, 1)
1,

1[ 0 0 0
1, 0[ 0 0 0
1, 1[ 2
2
0 0
_
_
(2.8.53c)
Combining (2.8.53a)-(2.8.53c) and substituting into (2.8.38),
V = 2C
0

2
_
_
[1,

1) [1, 0) [1, 1)
1,

1[ 1/

6 0 1
1, 0[ 0

2/

6 0
1, 1[ 1 0 1/

6
_
_
(2.8.54)
To make the solution for (2.8.54) more transparent, we interchange the 2
nd
and 3
rd
columns
V = 2C
0

2
_
_
[1,

1) [1, 1) [1, 0)
1,

1[ 1/

6 1 0
1, 0[ 0 0

2/

6
1, 1[ 1 1/

6 0
_
_
W. Erbsen HOMEWORK #9
And now do the same thing for the 1
st
and 3
rd
rows,
V = 2C
0

2
_
_
[1,

1) [1, 1) [1, 0)
1,

1[ 1/

6 1 0
1, 1[ 1 1/

6 0
1, 0[ 0 0

2/

6
_
_
(2.8.55)
It is easy to see that we neednt diagonalize the entire matrix in (2.8.55); rather, only the 2 2
submatrix:
V

= 2C
0

2
_
[1,

1) [1, 1)
1,

1[ 1/

6 1
1, 1[ 1 1/

6
_
(2.8.56)
Diagonalizing (2.8.56), we nd that

2C
0

2
/

6 2C
0

2
2C
0

2
2C
0

2
/

= 0
_
2C
0

__
2C
0

_
4C
0

2
= 0

4C
0

6

10C
2
0

4
3
= 0
= 2C
0

2
_
1

6
1
_
(2.8.57)
Using (2.8.57), we can now nd the corresponding eivenvectors. For = 2C
0

2
_
1

6
+ 1
_
, we have
2C
0

2
_

1 1
1

1
__
[1, 1)
[1,

1)
_
=
_
0
0
_
N
_
[1, 1) + [1,

1)
_
= 0 (2.8.58)
And similarly, for = 2C
0

2
_
1

6
1
_
,
2C
0

2
_
1 1
1 1
__
[1, 1)
[1,

1)
_
=
_
0
0
_
N
_
[1, 1) [1,

1)
_
= 0 (2.8.59)
Where the normalization constant N in (2.8.58) and (2.8.59) is simply 1/

2 . The eigenvalues and


eigenvectors are then

1
=
4C
0

6
, [1, 0) = 0 (2.8.60)

2
=2C
0

2
_
1

6
+ 1
_
,
1

2
_
[1, 1) +[1,

1)
_
= 0 (2.8.61)

3
=2C
0

2
_
1

6
1
_
,
1

2
_
[1, 1) [1,

1)
_
= 0 (2.8.62)
The eigenvalues in (2.8.60) are the energy shifts (E), while the eigenvectors correspond to the
unperturbed energy Eigenstates. The energy shifts are plotted in Fig. (3) (see attached Mathamatica
printout).
Exercise 21.4
CHAPTER 2: QUANTUM MECHANICS II 207
From the commutation relations for a
i
and a

i
, deduce that the operator N
i
= a

i
a
i
has as eigenvalues all non-
negative integers in the case of Bose-Einstein statistics, but only 0 and 1 in the case of Fermi-Dirac statistics.
Solution
The commutation relations for a
i
and a

i
are stated for the Bose-Einstein case in (21.34) as
a

k
a

k
= 0
a
k
a

a
k
= 0
a
k
a

a
k
=
k
1
_
_
_
B.E. (2.8.63)
And similarly, under Fermi-Dirac statistics, we have from (21.35)
a

k
a

+ a

k
= 0
a
k
a

+ a

a
k
= 0
a
k
a

+ a

a
k
=
k
1
_
_
_
F.D. (2.8.64)
We also recall that the occupation number operators, stated in the prompt and also given by (21.30), as
N
i
= a

i
a
i
(2.8.65)
We can describe the eect of the annihilation and creation operators in general by studying (21.36):
a
i
[n
1
, n
2
, ..., n
i
, ...) = e
i

n
i
[n
1
, n
2
, ..., n
i
1, ...) (2.8.66)
From (2.8.66), we can extrapolate the eect of a
i
and a

i
under Bose-Einstein and Fermi-Dirac statistics,
respectively:
a[n) =

n [n), a

[n) =

n + 1 [n)
_
B.E. (2.8.67a)
a[0) = 0, a

[0) = e
i
[1)
a[1) = e
i
[0), a

[1) = 0
_
F.D. (2.8.67b)
Where we note that (2.8.67a) is the same as (12.38) in Merzbacher, while (2.8.67b) is the same as
(12.39). The connection between (2.8.66) and (2.8.67a) for Bose-Einstein statistics stems from the fact
that we require that = 0. Similarly, we go from (2.8.66) to (2.8.67b) by recognizing that for the
anti-commutation relations from (2.8.64), we let e
i
= 1 if the number of occupied single-particle states
is even, while e
i
= 1 if they are odd.
We are now tasked with nding the eigenvalues of the occupation number operators from (2.8.65) un-
der both Bose-Einstein statistics as well as Fermi-Dirac statistics. Starting with Bose-Einstein statistics
rst, we apply the action of (2.8.67a) to [n) as follows:
N[n) aa

[n) =

n + 1 (a[n + 1))
=

n + 1

n + 1 [n)
=(n + 1) [n) (2.8.68)
But we arent nished yet. We recall from (2.8.63) that aa

a = 1 (1 1 in this special case).


Tinkering with this a bit yields
W. Erbsen HOMEWORK #9
aa

a = 1 aa

= N + 1 (2.8.69)
Substituting (2.8.69) back into (2.8.68),
(N + 1) [n) = (n + 1) [n) N[n) = n[n) (2.8.70)
Recognizing that the Bosonic occupation number operator may be any positive number, therefore we
can say that the eigenvalues of N may take any non-negative integer .
In order to repeat this same task within the framework of Fermi-Dirac statistics, we must go about the
problem in a slightly dierent way. We again start with the occupation number operator from (2.8.65),
but this time we wish to study the square, N
2
:
N
2
=
_
a

a
_ _
a

a
_
(2.8.71)
At this point, we remember the last equation in (2.8.64), from which we can say
aa

+a

a = 1 aa

= 1 a

a (2.8.72)
Substituting (2.8.72) back into (2.8.71),
N
2
=a

_
1 a

a
_
a a

a a

aa
. .
0
N
2
= N
From which we can easily say that the only condition in which the eigenvalues of N
2
and N are equal
for Fermi-Dirac statistics is when the eigenvalues are equal either 0 or 1.
Exercise 21.8
Prove from the commutation relations that
0[a
i
a
j
a

k
a

[0) =
jk

i

ik

j
(2.8.73)
the sign depending on the statistics, B.E. (+) or F.D. (). Also calculate the vacuum expectation value 0[a
h
a
i
a
j
a

k
a

m
[0).
Solution
We begin our journey by rearranging the algebraic equations describing the relationship between two
unique ladder operators a
k
and a

, described by (21.34) in Merzbacher for Bose-Einstein statistics and


(21.35) for Fermi-Dirac statistics (the last equation in (2.8.63) and (2.8.64) in the previous problem).
a
k
a

a
k
=
k
1 a
k
a

=
k
1 +a

a
k
_
B.E. (2.8.74a)
a
k
a

+ a

a
k
=
k
1 a
k
a

=
k
1 a

a
k
_
F.D. (2.8.74b)
We can combine (2.8.74a) and (2.8.74b) in a much more compact form,
CHAPTER 2: QUANTUM MECHANICS II 209
a
k
a

=
k
1 a

a
k
[B.E. +, F.D. ] (2.8.75)
We now need to determine how our operators act on the vacuum state [0). We recall from (21.6) and
(21.7) that
a

i
[0) =
(1)
i
= [0, 0, ..., n
i
= 1, 0, ...) (2.8.76a)
a
i

(1)
j
=a
i
[0, 0, ..., n
i
= 1, 0, ...) =
ij

(0)
(2.8.76b)
Where we note that for the vacuum state,
(0)
i
=
(0)
j
=
(0)
= [0). Using this logic, we can evaluate
(2.8.73) by substituting into it (2.8.75) and using (2.8.77) and (2.8.78):
0[aa
i
a
j
a

k
a

[0) =
(0)
a
i
a
j
a

k
a

(0)
=

(1)
i
..
(
(0)
a
i
) a
j
a

(1)

..
(a

(0)
)
=
(1)
i
a
j
a

(1)

(2.8.77)
Substituting into (2.8.77) our derived result from (2.8.75),

(1)
i
a
j
a

(1)

=
(1)
i
_

jk
1 a

k
a
j
_

(1)

=
(1)
i
(
jk
1)
(1)


(1)
i
_
a

k
a
j
_

(1)

=
jk

(1)
i

(1)

. .
i

(1)
i
a

k
. .
ik
(1)
k
a
j

(1)

. .
j
(0)
(2.8.78)
From (2.8.78) we can say that

(1)
i
a
j
a

(1)

=
jk

ik

j
(2.8.79)
Substituting (2.8.79) back into (2.8.77), we are left with

(0)
a
i
a
j
a

k
a

(0)
=
jk

i

ik

j
Which is the same as (2.8.73).
Problem 21.1
a) Show that if V (r) is a two-particle interaction that depends only on the distance r between the particles, the
matrix element of the interaction in the k-representation may be reduced to
k
3
k
4
[V [k
1
k
2
) = (k
1
+ k
2
k
3
k
4
)
1
(2)
3
_
V (r)e
iqr
d
3
r (2.8.80)
where q is the momentum transfer (k
3
k
1
).
W. Erbsen HOMEWORK #9
b) For this interaction, show that the mutual potential energy operator is
V =
1
2
___

(k
1
+ q)

(k
2
q) (k
2
) (k
1
) F (q) d
3
k
1
d
3
k
2
d
3
q (2.8.81)
where F(q) is the Fourier transform of the displacement-invariant interaction.
Solution
Before we get down to business, we must lay down some formalism. We recall that within the framework
of identical particles, a
i
and a

i
destroy and create particles with some quantum number K
i
. We also recall
that L represents a complete set of one-particle observables, whose eigenvalues are L
q
. The relationship
between K
i
and L
q
is given by (21.9) as
[K
i
) =

q
[L
q
)L
q
[K
i
) (2.8.82)
We can also evaluate the expectation value of K by expanding it in terms of K
i
:
L
q
[K[L
r
) =

i
L
q
[K
i
)K
i
K
i
[L
r
) (2.8.83)
Which is of course (21.43). We also wish to dene an additive two-particle operator as
V =
1
2

ij
a

i
a

j
a
j
a
i
V
ij

1
2

qrst
b

q
b

r
b
s
b
t
qr[V [ts) (2.8.84)
Where, from (21.48),
qr[V [st) =

ij
L
q
[K
i
)K
i
[L
t
)L
r
[K
j
)K
j
[L
s
) V
ij
(2.8.85)
We note that (2.8.85) is the generalized two-particle matrix element, and according to Merzbachers
chosen convention we require that r and s belong to one particle, while q and t to the other. We also
recall the following symmetry property:
qr[V [ts) = rq[V [st)
Which should come as no surprise.
a) We are now tasked with nding the two-particle matrix element in the k-representation. The most
general form is given by (2.8.85), and we note that since our interaction stems from a mutual
potential energy, (2.8.84) and (2.8.85) can be readily used.
The rst thing we wish to do is rewrite our k-representation two-particle matrix element from
the LHS of (2.8.80) in terms of the more generic form of (2.8.85):
k
3
k
4
[V [k
1
k
2
) =

ij
k
3
[r
i
)r
i
[k
1
)k
4
[r
j
)r
j
[k
2
) V
ij
(2.8.86)
Our particle coordinates are r
1
and r
2
, while the interaction potential between the two particles is
only dependent on the distance between them (call this distance r), so that V
ij
= V (r
1
r
2
) = V (r).
Accordingly, (2.8.86) becomes
CHAPTER 2: QUANTUM MECHANICS II 211
k
3
k
4
[V [k
1
k
2
) =

12
k
3
[r
1
)r
1
[k
1
)k
4
[r
2
)r
2
[k
2
) V (r) (2.8.87)
It is very important that we recognize that the summation indicies in (2.8.87) are referring to the
coordinates r
1
and r
2
, and mustnt be applied to k. Since r
1
and r
2
are continuous variables, we
may take the sums in (2.8.87) to be integrals. We now have
k
3
k
4
[V [k
1
k
2
) =
__
k
3
[r
1
)r
1
[k
1
)k
4
[r
2
)r
2
[k
2
) V (r) d
3
r
1
d
3
r
2
(2.8.88)
At this point, we wish to reexpress (2.8.88) in terms of momentum eigenstates, which we will do in
the usual way. Most generally,
(r) =r
1 =
_
[k)k[ dk

[)
=
_
r[k)k[) dk (2.8.89)
We also recall that (k[r))

= r[k) Ne
ikr/
, while normalizing this yields
[N[
2
_

e
ik(rr

)/
N =
1

2
, < (r r

) (2.8.90)
Compensating for the fact that dr d
3
r, we can now say that
r[k) =
1
(2)
3
/2
e
ikr/
(2.8.91)
We now rewrite (2.8.88) in a slightly more transparent way,
k
3
k
4
[V [k
1
k
2
) =
__
r
1
[k
3
)

r
1
[k
1
) r
2
[k
4
)

r
2
[k
2
) V (r) d
3
r
1
d
3
r
2
(2.8.92)
We can now evaluate each of the coecients in (2.8.93) by applying (2.8.91):
r
1
[k
3
)

=
1
(2)
3
/2
e
ik3r1/
r
1
[k
1
) =
1
(2)
3
/2
e
ik1r1/
r
2
[k
4
)

=
1
(2)
3
/2
e
ik4r2/
r
2
[k
2
) =
1
(2)
3
/2
e
ik2r2/
And substituting these into (2.8.93),
k
3
k
4
[V [k
1
k
2
) =
1
(2)
6
__
exp
_
i

(k
3
r
1
+k
1
r
1
k
4
r
2
+k
2
r
2
)
_
V (r) d
3
r
1
d
3
r
2
(2.8.93)
By substituting r = r
1
r
2
r
1
= r
2
+r into (2.8.93) and completing the square, we arrive at
W. Erbsen HOMEWORK #9
k
3
k
4
[V [k
1
k
2
) =
_
e
iqr2
e
i(k4k2)r2
d
3
r
2
_
e
iqr2
_
V (r)e
iqr1
d
3
r
1
e
iqr1
_
V (r)e
iqr2
d
3
r
2
_
=
1
(2)
3
_
e
i(k1+k2k3k4)r2/
d
3
r
2
_
V (r)e
iqr/
d
3
r (2.8.94)
Noticing that the rst integral in (2.8.94) is a -function, we are left with
k
3
k
4
[V [k
1
k
2
) = (k
1
+ k
2
k
3
k
4
)
1
(2)
3
_
V (r)e
iqr
d
3
r
Problem 21.2
Show that the diagonal part of the interaction operator V , found in Problem 1 in the k-representation, arises
from momentum transfers q = 0 and q = k
2
k
1
, respectively. Write down the two interaction terms and identify
them as direct (q = 0) and exchange (q = k
2
k
1
) interactions. Draw the corresponding diagrams (Figure 21.1).
Solution
We rst recall from (21.51) that

(r

(r

(r

(r

) = 0

(r

(r

(r

(r

) = 0

(r

(r

(r

(r

) = (r

_
_
_
B.E. (2.8.95)
And similarly, under Fermi-Dirac statistics, we have from (21.52)

(r

(r

) +

(r

(r

) = 0

(r

(r

) +

(r

(r

) = 0

(r

(r

) +

(r

(r

) = (r

_
_
_
F.D. (2.8.96)
We also recall from (21.53) that
N =

(r

(r

) d
3
r

(2.8.97)
From the previous problem, we have
V =
1
2
___

(k
1
+ q)

(k
2
q) (k
2
) (k
1
) F (q) d
3
k
1
d
3
k
2
d
3
q (2.8.98)
Substituting in to (2.8.98) the appropriate value for q and simplifying, we have
V =
1
2
____

(k
3
)

(k
4
)(k
2
)(k
1
)k
3
k
4
[k
2
k
1
) d
3
k
1
d
3
k
2
d
3
k
3
d
3
k
4
(2.8.99)
We now wish to sandwich (2.8.99) between the state
n
as follows:
CHAPTER 2: QUANTUM MECHANICS II 213

n
[V [
n
) =
1
2
____

n
[

(k
3
)

(k
4
)(k
2
)(k
1
)[
n
)k
3
k
4
[k
2
k
1
) d
3
k
1
d
3
k
2
d
3
k
3
d
3
k
4
(2.8.100)
The generic interaction term we are interested in is given from (2.8.100) as

n
[

(k
3
)

(k
4
)(k
2
)(k
1
)[
n
) (2.8.101)
Now, if q = 0, then we require from the prompt that k
1
= k
3
and k
2
= k
4
, and so (2.8.101) becomes

n
[

(k
1
)

(k
2
)(k
2
)(k
1
)[
n
) =
n
[

(k
1
)(k
1
)

(k
2
)(k
2
)[
n
) (2.8.102)
Where we recall that the number operator acts like N
i
= a

i
a
i
. Accordingly, if we recall that N
i
= n
i
,
from (2.8.102) we have

n
[

(k
1
)(k
1
)

(k
2
)(k
2
)[
n
) =n
1
n
2
(2.8.103)
If, on the other hand, q = 0 because k
1
= k
2
= k
3
= k
4
k, then

n
[

(k
1
)

(k
2
)(k
2
)(k
1
)[
n
) =
n
[

(k)(k)

(k)(k)[
n
)
=
n
[

(k)

(k)(k)(k)[
n
)
=
n
[

(k)

(k)(k)(k)[
n
) (2.8.104)
The result of (2.8.104) is not the same for each avor of particles; for symmetric particles, the RHS of
(2.8.104) is n(n 1), and for anti-symmetric particles, it is equal to 0, which comes as no surprise.
Following the same logic, we have for q ,= 0 that k
3
= k
2
and k
1
= k
4
. The number operators act
as before, and we end up with

n
[

(k
1
)(k
1
)

(k
2
)(k
2
)[
n
) =
n
[

(k
1
)(k
1
)

(k
2
)(k
2
)[
n
)
A n
1
n
2
(2.8.105)
Where for symmetric particles, A = 1, and for anti-symmetric particles A = 1. We nally can say that
the direct terms come from q = 0 , while the exchange terms come from q = k
1
k
2
. Please see
diagrams in Fig. (4) and (5) (attached at the end of this assignment).
2.9 Homework #10
Exercise 22.3
Construct explicitly in terms of the states of the form a

jm2
a

jm1
[0) the total angular momentum eigenstates for
two neutrons in the congurations (p1
/2
)
2
and (p3
/2
)
2
. How would the angular momentum eigenstates look if the
two particles were a neutron and a proton but otherwise had the same quantum numbers as before?
Solution
W. Erbsen HOMEWORK #10
Before we get down to business, we recall (22.4), which reads

(2)
JM
= [JM) =
1

m1m2
a

jm2
a

jm1
jjm
1
m
2
[jjJM) [0) (2.9.1)
And that all the individual particles at play here, i.e. proton, neutrons, and electrons, are all Fermions,
whereby we must obey the Pauli exclusion principle.
For our rst case, where both neutrons are in the (p1
/2
)
2
conguration, we have the separate quantum
numbers as:
j
1
=
1
/
2
, m
1
=
1
/
2
j
2
=
1
/
2
, m
2
=
1
/
2
_
J = 0, 1
Since we are closely following the mantra of the Pauli exclusion principle, our total angular momentum
quantum number J cannot obtain an odd value, and since in this rst case we only have J = 0 and
J = 1, we only must calculate the former. Applying the relevant quantum numbers to (2.9.1), we have

[0, 0) =
1

m1,m2
a

1
/2,m2
a

1
/2,m1

1
/
2
,
1
/
2
, m
1
, m
2
[
1
/
2
,
1
/
2
, 0, 0) [0) (2.9.2)
We must expand the sum in (2.9.2) for all allowed combinations of m
1
and m
2
. In our case, there are
only two possible permutations: m
1
=
1
/
2
, m
2
=
1
/
2
and m
1
=
1
/
2
, m
2
=
1
/
2
. We therefore only have
two terms:
[0, 0) =
1

2
_
a

1
/2,
1
/2
a

1
/2,
1
/2

1
/
2
,
1
/
2
,
1
/
2
,
1
/
2
[
1
/
2
,
1
/
2
, 0, 0) +a

1
/2,
1
/2
a

1
/2,
1
/2

1
/
2
,
1
/
2
,
1
/
2
,
1
/
2
[
1
/
2
,
1
/
2
, 0, 0)
_
[0)
(2.9.3)
The Clebsch-Gordan (C-G) coecients in (2.9.3) were evaluated using Mathematica, according to the
same convention as Merzbacher. The results are:

1
/
2
,
1
/
2
,
1
/
2
,
1
/
2
[
1
/
2
,
1
/
2
, 0, 0) =
1

1
/
2
,
1
/
2
,
1
/
2
,
1
/
2
[
1
/
2
,
1
/
2
, 0, 0) =
1

2
Substituting these back in to (2.9.3),
[0, 0) =
1
2
_
a

1
/2,
1
/2
a

1
/2,
1
/2
a

1
/2,
1
/2
a

1
/2,
1
/2
_
[0) (2.9.4)
If our neutrons are in the (p3
/2
)
2
conguration, things get a little more tedious. First of all, our quantum
numbers become:
j
1
=
3
/
2
, m
1
= 0,
1
/
2
,
3
/
2
j
2
=
3
/
2
, m
2
= 0,
1
/
2
,
3
/
2
_
J = 0, 1, 2, 3
Luckily, since we require that our total angular momentum operator be even, we only must contend with
J = 0 and J = 2. The analogous form of (2.9.2) for our case is

From here forward, I am dropping explicit inclusion of .


CHAPTER 2: QUANTUM MECHANICS II 215
[0, 0) =
1

m1,m2
a

3
/2,m2
a

3
/2,m1

3
/
2
,
3
/
2
, m
1
, m
2
[
3
/
2
,
3
/
2
, 0, 0) [0)

m1,m2
a

3
/2,m2
a

3
/2,m1
m
1
, m
2
[0, 0) [0) (2.9.5)
Where I have dropped the inclusion of j
1
and j
2
in (2.9.5), which is standard practice. We now expand
the sum in (2.9.5), which yields
[0, 0) =
1

2
_
a

3
/2,
3
/2
a

3
/2,
3
/2

3
/
2
,
3
/
2
[0, 0) + a

3
/2,
1
/2
a

3
/2,
1
/2

1
/
2
,
1
/
2
[0, 0)+
a

3
/2,
1
/2
a

3
/2,
1
/2

1
/
2
,
1
/
2
[0, 0) +a

3
/2,
3
/2
a

3
/2,
3
/2

3
/
2
,
3
/
2
[0, 0)
_
[0) (2.9.6)
We can tabulate the relevant C-G coecients by making a table,
[J, M) m
1
m
2
J M m
1
, m
2
[J, M)
[0, 0) +
3
/
2

3
/
2
0 0 +
3
/
2
,
3
/
2
[0, 0) = +(2)
1
+
1
/
2

1
/
2
0 0 +
1
/
2
,
1
/
2
[0, 0) = (2)
1

1
/
2
+
1
/
2
0 0
1
/
2
, +
1
/
2
[0, 0) = +(2)
1

3
/
2
+
3
/
2
0 0
3
/
2
, +
3
/
2
[0, 0) = (2)
1
We now substitute these values into (2.9.6),
[0, 0) =
1
2

2
_
a

3
/2,
3
/2
a

3
/2,
3
/2
+ a

3
/2,
1
/2
a

3
/2,
1
/2
+ a

3
/2,
1
/2
a

3
/2,
1
/2
+a

3
/2,
3
/2
a

3
/2,
3
/2
_
[0)
We must now repeat this process for J = 2, and M = 0, 1, 2.
[J, M) m
1
m
2
J M m
1
, m
2
[J, M)
[2, 2)
1
/
2

3
/
2
2 2
1
/
2
,
3
/
2
[2, 2) = +(2)

1
/2

3
/
2

1
/
2
2 2
3
/
2
,
1
/
2
[2, 2) = (2)

1
/2
[2, 1) +
1
/
2

3
/
2
2 1 +
1
/
2
,
3
/
2
[2, 1) = +(2)

1
/2

3
/
2
+
1
/
2
2 1
3
/
2
, +
1
/
2
[2, 1) = (2)

1
/2
[2, 0) +
3
/
2

3
/
2
2 0 +
3
/
2
,
3
/
2
[2, 0) = +(2)

1
/2
+
1
/
2

1
/
2
2 0 +
1
/
2
,
1
/
2
[2, 0) = (2)

1
/2

1
/
2
+
1
/
2
2 0
1
/
2
, +
1
/
2
[2, 0) = +(2)

1
/2

3
/
2
+
3
/
2
2 0
3
/
2
, +
3
/
2
[2, 0) = (2)

1
/2
[2, 1) +
3
/
2

1
/
2
2 1 +
3
/
2
,
1
/
2
[2, 1) = +(2)

1
/2

1
/
2
+
3
/
2
2 1
1
/
2
, +
3
/
2
[2, 1) = (2)

1
/2
[2, 2) +
3
/
2
+
1
/
2
2 2 +
3
/
2
, +
1
/
2
[2, 2) = +(2)

1
/2
+
1
/
2
+
3
/
2
2 2 +
1
/
2
, +
3
/
2
[2, 2) = (2)

1
/2
Expanding the sum in (2.9.5) and substituting in the appropriate values, we have
W. Erbsen HOMEWORK #10
[2, 2) = (2)
1
_
a

3
/2,
3
/2
a

3
/2,
1
/2
a

3
/2,
3
/2
a

1
/2,
3
/2
_
[0)
[2, 1) = (2)
1
_
a

3
/2,
3
/2
a

3
/2,
1
/2
a

3
/2,
1
/2
a

3
/2,
3
/2
_
[0)
[2, 0) = (2)
1
_
a

3
/2,
3
/2
a

3
/2,
3
/2
a

3
/2,
1
/2
a

3
/2,
1
/2
+ a

3
/2,
1
/2
a

3
/2,
1
/2
a

3
/2,
3
/2
a

3
/2,
3
/2
_
[0)
[2, 1) = (2)
1
_
a

3
/2,
1
/2
a

3
/2,
3
/2
a

3
/2,
3
/2
a

3
/2,
1
/2
_
[0)
[2, 2) = (2)
1
_
a

3
/2,
1
/2
a

3
/2,
1
/2
a

3
/2,
3
/2
a

3
/2,
1
/2
_
[0)
If, as the prompt suggests, we have both a neutron and a proton, things change somewhat. We are
told that the quantum numbers are the same, so in this respect the subsequent calculations are easier,
however we must now introduce a creation operator, b

, which acts on the other particle.


We can build on the results shown in the previous parts of this problem with the addition of this
additional creation operator b

. For the case of (p1


/2
) (p1
/2
), our possible quantum numbers are:
j
1
=
1
/
2
, j
2
=
1
/
2
m
1
=
1
/
2
, m
2
=
1
/
2
_
J = 0, 1
We now use this in the same way we did before, with a table or otherwise. The results are:
[0, 0) = (2)
1
_
a

1
/2,
1
/2
b

1
/2,
1
/2
a

1
/2,
1
/2
b

1
/2,
1
/2
_
[0)
[1, 1) = (2)

1
/2
_
a

1
/2,
1
/2
b

1
/2,
1
/2
_
[0)
[1, 0) = (2)
1
_
a

1
/2,
1
/2
b

1
/2,
1
/2
a

1
/2,
1
/2
b

1
/2,
1
/2
_
[0)
[1, 1) = (2)

1
/2
_
a

1
/2,
1
/2
b

1
/2,
1
/2
_
[0)
And similarly, for the case of (p3
/2
) (p3
/2
), we allow for the following quantum numbers:
j
1
=
3
/
2
, j
2
=
3
/
2
m
1
=
1
/
2
,
3
/
2
, m
2
=
1
/
2
,
3
/
2
_
J = 0, 1, 2, 3
Which comes as no surprise. The only dierence here is that we can no longer exclude the odd contri-
butions of the total angular momentum quantum number J.
[0, 0) = (8)

1
/2
_
a

3
/2,
3
/2
b

3
/2,
3
/2
a

3
/2,
1
/2
b

3
/2,
1
/2
+a

3
/2,
1
/2
b

3
/2,
1
/2
a

3
/2,
3
/2
b

3
/2,
3
/2
_
[0)
[2, 2) = (2)
1
_
a

3
/2,
1
/2
b

3
/2,
3
/2
a

3
/2,
3
/2
b

3
/2,
1
/2
_
[0)
[2, 1) = (2)
1
_
a

3
/2,
3
/2
b

3
/2,
1
/2
a

3
/2,
1
/2
b

3
/2,
3
/2
_
[0)
[2, 0) = (8)

1
/2
_
a

3
/2,
3
/2
b

3
/2,
3
/2
+ a

3
/2,
1
/2
b

3
/2,
1
/2
a

3
/2,
1
/2
b

3
/2,
1
/2
+a

3
/2,
3
/2
b

3
/2,
3
/2
_
[0)
[2, 1) = (2)
1
_
a

3
/2,
1
/2
b

3
/2,
3
/2
a

3
/2,
3
/2
b

3
/2,
1
/2
_
[0)
[2, 2) = (2)
1
_
a

3
/2,
1
/2
b

3
/2,
3
/2
a

3
/2,
3
/2
b

3
/2,
1
/2
_
[0)
I am not calculating the rest; I am not a monkey. There is no more physics here. No more fun.
CHAPTER 2: QUANTUM MECHANICS II 217
Exercise 22.6
Use the symmetry relations for the Clebsch-Gordan coecients to show that a conguration (n)
2
can only give
rise to spin-orbit coupled two-electron states for which L + S is even, i.e., states
1
S,
3
P,
1
D and so on.
Solution
We rst recall (22.2) from Merzbacher, which reads

(2)
JM
= C

m1m2
a

j2m22
a

j1m11
j
1
j
1
m
1
m
2
[j
1
j
2
JM)
(0)
From which follows directly (22.17):
[
(2)
n11n22
) =

m1m1

2
m
1
m
2
[
1

2
LM
L
)

1
m

1
/
2
1
/
2
m

1
m

2
[
1
/
2
1
/
2
SM
S
)a

n22m
2
m

2
a

n11m
1
m

1
[0) (2.9.7)
In our specic case, we are interested in investigating when
1
=
2
and when n
1
= n
2
n, and so
(2.9.7) becomes
[
(2)
nn
) =

m1m1
m
1
m
2
[LM
L
)

1
m

1
/
2
1
/
2
m

1
m

2
[
1
/
2
1
/
2
SM
S
)a

nm
2
m

2
a

nm
1
m

1
[0) (2.9.8)
We also recall the symmetry relations for Clebsch-Gordan coecients, which from (17.61) reads
j
1
j
2
m
1
m
2
[j
1
j
2
jm) = (1)
jj1j2
j
2
j
1
m
2
m
1
[j
2
j
1
jm) = j
2
j
1
, m
2
, m
1
[j
2
j
1
j, m) (2.9.9)
Where in our case we let j
1
= j
2
, and for the total angular momentum j L and m M.
Accordingly, (2.9.9) becomes
m
1
m
2
[LM) = (1)
L2
m
2
m
1
[LM
L
) = , m
2
, m
1
[L, M
L
) (2.9.10)
We now substitute (2.9.10) back into (2.9.8),
[
(2)
nn
) =

m1m1
(1)
L2
m
2
m
1
[LM
L
)

1
m

1
/
2
1
/
2
m

1
m

2
[
1
/
2
1
/
2
SM
S
)a

nm
2
m

2
a

nm
1
m

1
[0) (2.9.11)
While at this point, we recall (21.35), which was used in the last homework assignment,
a

k
a

+a

k
= 0 (2.9.12)
a
k
a

+ a

a
k
= 0
a
k
a

+a

a
k
=
k
1
We note that we can use (2.9.12) to rewrite the product of the ladder operators in (2.9.11):
a

nm
2
m

2
a

nm
1
m

1
= a

nm
1
m

1
a

nm
2
m

2
(1)
1
a

nm
1
m

1
a

nm
2
m

2
(2.9.13)
Substituting (2.9.13) back into (2.9.11),
W. Erbsen HOMEWORK #10
[
(2)
nn
) =

m1m1
(1)
L2+1
m
2
m
1
[LM
L
)

1
m

1
/
2
1
/
2
m

1
m

2
[
1
/
2
1
/
2
SM
S
)a

nm
1
m

1
a

nm
2
m

2
[0) (2.9.14)
(1)
L+S2
[
(2)
nn
) (2.9.15)
The required condition is satised, i.e. unless L + S is even, (2.9.14) is violated .
Exercise 22.10
Show that the conguration space wave function corresponding to the independent particle state (22.22) can
be expressed as the Slater determinant
(r
1

1
, , r
N

N
) =
1

N!

1
(r
1

1
)
1
(r
2

2
)
1
(r
N

N
)

2
(r
1

1
)
2
(r
2

2
)
2
(r
N

N
)
.
.
.
.
.
.
.
.
.
.
.
.

n
(r
1

1
)
n
(r
2

2
)
n
(r
N

N
)

(2.9.16)
Solution
We rst recall that (22.22) from Merzbacher reads
[

) = a

N
a

N1
a

2
a

1
[0) (2.9.17)
While we must always obey the symmetrixation requirement:
(r
1
, r
2
) = (r
2
, r
1
)
Treating this in a more explicit manner,

(r
1
, r
2
) =
1

2
[
a
(r
1
)
b
(r
2
)
b
(r
1
)
a
(r
2
)] (2.9.18)
Where the upper sign represents Bosons, while the lower is for Fermions. Directly from (2.9.18) follows
the Pauli Exclusion Principle, which applies only to Fermions. For two particles that occupy the same
state
a
=
b
,

(r
1
, r
2
) =
1

2
[
a
(r
1
)
a
(r
2
)
a
(r
2
)
a
(r
1
)] 0
In other words, the resultant wave function must be anti-symmetric with the exchange of any two
particles. We note that while the previous equations do not contain an explicit spin dependence, one
may be included without loss of generality. Accordingly, (2.9.18) becomes

(r
1

1
, r
2

2
) =
1

2
[
a
(r
1

1
)
b
(r
2

2
)
a
(r
2

2
)
b
(r
1

1
)] (2.9.19)
CHAPTER 2: QUANTUM MECHANICS II 219
In this special two-particle case, it is not hard to see that we can rewrite (2.9.19) in terms of a determinant:

(r
1

1
, r
2

2
) =
1

a
(r
1

1
)
a
(r
2

2
)

b
(r
1

1
)
b
(r
2

2
)

(2.9.20)
The extension of (2.9.19) to (2.9.20) may be generalized to a N-particle system without too much trouble;
all we have to do is recognize that the odd exchanges in the wave function product terms must include
a factor of 1. Explicitly, we rewrite (2.9.20) for N particles as:

(r
1

1
, r
2

2
, , r
N

N
) =
1

2
[
a
(r
1

1
)
b
(r
2

2
)
N
(r
N

N
)
a
(r
2

2
)
b
(r
1

1
)
N
(r
N

N
)]
(2.9.21)
Where we note that the complete set of wave function denominations is
a
,
b
, ...,
N
. We can
immediately rewrite (2.9.21) in the form of a determinant, as suggested by the prompt:
(r
1

1
, , r
N

N
) =
1

N!

a
(r
1

1
)
a
(r
2

2
)
a
(r
N

N
)

b
(r
1

1
)
b
(r
2

2
)
b
(r
N

N
)
.
.
.
.
.
.
.
.
.
.
.
.

N
(r
1

1
)
N
(r
2

2
)
N
(r
N

N
)

Which is the same as (2.9.16).


Problem 22.6
Apply the Hartree-Fock method to a system of two electrons which are attracted to the coordinate origin
by an isotropic harmonic oscillator potential m
2
r
2
/2 and which interact with each other through a potential
V = C(r

)
2
. Solve the Hartree-Fock equations for the ground state and compare with the exact result and
with rst-order perturbation theory.
Solution
Due to the phrasing of the problem, we are inclined to utilize the symmetry around the origin. We also
know that the Hamiltonian for this system is just the sum of the Harmonic oscillator Hamiltonian plus
the interaction term:
H =

2
2m

2
+
m
2
2
r
2
V (2.9.22)
Applying V from the prompt to (2.9.22),

H =

2
2m
_

2
1
+
2
2
_
+
m
2
2
_
r
2
1
+r
2
2
_
C(r
1
r
2
)
2
(2.9.23)

I have replace Merzbachers silly prime notation with my selected subscripts to deter ambiguity between various notations.
W. Erbsen HOMEWORK #10
At this point, we seemingly arbitrarily, introduce the relative and center of mass coordinate variables
respectively:
r =
1

2
(r
1
r
2
) , R =
1

2
(r
1
+ r
2
) (2.9.24)
Rewriting (2.9.23) according to (2.9.24),
H =

2
2m
_

2
1
+
2
2
_
+
m
2
2
R
2
Cr
2
(2.9.25)
If we introduce the following similar coordinates for the momenta,
p =
1

2
(p
1
p
2
) , P =
1

2
(p
1
+ p
2
) (2.9.26)
Rewriting the kinetic energy term from (2.9.25) in terms momentum variables and utilizing (2.9.26), we
have

H =
1
2
_
P
2
+ R
2
_
+
1
2
_
p
2
+r
2

2C + 1
_
(2.9.27)
From this, we can see that by the use of our chosen transformations from (2.9.24) and (2.9.26), we see
that we have turned one Hamiltonian into two independent 3-dimensional harmonic oscillators.
This problem has been presented before; the (exact) results are
(r, R) =
_
2C + 1

4
_3
/8
exp
_

R
2
2
_
exp
_

2C + 1
2
r
2
_
(2.9.28a)
E
0
=
3
2
_
1 +

2C + 1
_
(2.9.28b)
To see how the Hartree-Fock (HF) perspective compares to (2.9.28a) and (2.9.28b), we see that

HF
=
1

(r
1
)
+
(r
2
)
+
(r
1
)

(r
2
)

(2.9.29)
And the HF equation for our Hamiltonian becomes
1
2
_
p
2
1
+r
2
1

(r
1
) +
1
2
__

(r
2
) C (r
1
r
2
)
2
(r
2
) dr
2
_
(r
1
) = E(r
1
) (2.9.30)
If we expand the central term in (2.9.30), we nd that
(r
1
r
2
)
2
= r
2
1
+ r
2
2
2r
1
r
2
(2.9.31)
The last term in (2.9.31) vanishes in the integral due to parity. Therefore, the integral becomes trivial
and (2.9.30) yields

0
(r
1
) =
_
C + 1

2
_3
/8
exp
_

C + 1
2
r
2
1
_

Atomic units and also = 1.


CHAPTER 2: QUANTUM MECHANICS II 221
E
0
=
3
2
_
3C + 2
2C + 2
_

C + 1
And so the Hartree-Fock ground state wave function becomes

HF
0
(r, R) =
_
C + 1

4
_3
/8
exp
_

C + 1
2
_
r
2
+ R
2
_
_
(2.9.33)
And now, the energy eigenvalues according to (2.9.33) can be found by taking the expectation value of
our Hamiltonian from (2.9.25) with the HF state from (2.9.33). The result is ?
E
HF
0
=
HF
0
[H[
HF
0
) E
HF
0
= 3

C + 1 (2.9.34)
Please see attached Mathematica printout with plots.
W. Erbsen *
Appendix A *
Creation and Annihilation Operators
We rst recall from class that the action of the creation and annihilation operators on an arbitrary vector [
n
)
acts like:
a

[
n
) =

n + 1 [
n+1
) (A.1a)
a[
n
) =

n [
n1
) (A.1b)
We also recall that the position and momentum operators are given in terms of the creation and annihilation
operators as:
x =
_

2m
_
a

+a
_
(A.2a)
p =
_
m
2
_
a

a
_
(A.2b)
We can now nd the expectation values of various combinations of ladder operators, namely a

), a), a

a), aa

),
a
2
), and a
2
):
a

) =
n
(x)[a

[
n
(x))
=
_

n
(x)a

n
(x) dx
=
_

n
(x)
_
n + 1
n+1
(x)
_
dx
=

n + 1
n

,n+1
(A.3)
a) =
n
(x)[a[
n
(x))
=
_

n
(x)a
n
(x) dx
=
_

n
(x)
_
n
n1
(x)
_
dx
=

n
n

,n1
(A.4)
a

a) =
n
(x)[a

a[
n
(x))
=
_

n
(x)a

a
n
(x) dx
=
_

n
(x)a

_
n
n1
(x)
_
dx
=
_

n
(x) (n
n
(x)) dx
=n
n

,n
(A.5)
aa

) =
n
(x)[aa

[
n
(x))
CHAPTER 2: QUANTUM MECHANICS II 223
=
_

n
(x)aa

n
(x) dx
=
_

n
(x)a
_
n + 1
n+1
(x)
_
dx
=
_

n
(x) ((n + 1)
n
(x)) dx
=(n + 1)
n

,n
(A.6)
a
2
) =
n
(x)[a
2
[
n
(x))
=
_

n
(x)a

n
(x) dx
=
_

n
(x)a

_
n + 1
n+1
(x)
_
dx
=
_

n
(x) ((n + 1)
n+2
(x)) dx
=(n + 1)
n

,n+2
(A.7)
a
2
) =
n
(x)[a
2
[
n
(x))
=
_

n
(x)aa
n
(x) dx
=
_

n
(x)a
_
n
n1
(x)
_
dx
=
_

n
(x) (n
n2
(x)) dx
=n
n

,n2
(A.8)
We now wish to nd the expectation values of the position and momentum operators from (A.2a) and (A.2b), and
also their square. We can do this by using (A.3)-(A.8):
x) =
_

2m
(a

+ a))
=
_

2m
_
a

) + a)
_
=
_

2m
_
n + 1
n

,n+1
+

n
n

,n1
_
=0 (A.9)
x
2
) =

2m
(a

+ a)
2
)
=

2m
(a
2
+a

a +aa

+a
2
))
=

2m
_
a
2
) +a

a) +aa

) + a
2
)
_
W. Erbsen *
=

2m
((n + 1)
n

,n+2
+ n
n

,n
+ (n + 1)
n

,n
+ n
n

,n2
)
=

2m
(n(n + 1))
=

2m
(2n + 1) (A.10)
p) =i
_
m
2
(a

+ a))
=i
_
m
2
_
a

) + a)
_
=i
_
m
2
_
n + 1
n

,n+1
+

n
n

,n1
_
=0 (A.11)
p
2
) =
m
2
(a

a)
2
)
=
m
2
(a
2
a

a aa

+a
2
))
=
m
2
_
a
2
) a

a) aa

) + a
2
)
_
=
m
2
((n + 1)
n

,n+2
n
n

,n
(n + 1)
n

,n
+n
n

,n2
)
=
m
2
(n (n + 1))
=
m
2
(2n + 1) (A.12)
CHAPTER 2: QUANTUM MECHANICS II 225
Appendix B *
Hermite Polynomial Recursion Relation
Equation (5.35) from Merzbacher reads:
H
n
() =
_
d
n
ds
n
e

2
(s)
2
_
s=0
=(1)
n
e

2 d
n
d
n
e

2
(B.1)
Which is known as Rodrigues formula for Hermite Polynomials. To derive the recursion relation, we rst dier-
entiate H
n
():
d
d
H
n
() =
d
d
_
(1)
n
e

2
_
d
d
_
n
e

2
_
=(1)
n
_
d
d
e

2
__
d
d
_
n
+ (1)
n
e

2
_
d
d
_
n+1
e

2
=2x(1)
n
e

2
_
d
d
_
n
(1)
n+1
e

2
_
d
d
_
n+1
e

2
=2xH
n
() H
n+1
() (B.2)
And from Eq. (5.32) in Merzbacher we know that
d
d
H
n
= 2nH
n1
()
So that (B.2) becomes:
2nH
n1
() = 2xH
n
() H
n+1
() (B.3)
And from (B.3) we nally arrive at the recursion relation,
H
n+1
() = 2xH
n
() 2nH
n1
() (B.4)
Which agrees with Abramowitz and Stegun, pg. 782 22.7.13 ?.
W. Erbsen *
clearemptydoublepage
Chapter 3
Statistical Mechanics
3.1 Homework #1
Problem 1
We discussed in class thermal equilibrium of a closed system which is divided into two parts with a barrier
inbetween (see gure). This barrier allows energy exchange between the two parts of the system. Show that:
a) If the barrier is allowed to move but does not allow particle exchange, then at thermal equilibrium the
pressures of the two parts are equal, i.e. P
1
= P
2
.
b) If the barrier is not allowed to move but allows for particle exchange, then at thermal equilibrium the chemical
potentials of the two parts are equal, i.e.
1
=
2
.
Figure 3.1: Figure for Problem 3.
Solution
Where applicable, we apply the principle of conservation to the extensive variables N, V and E:
N =N
1
+N
2
V =V
1
+V
2
E =E
1
+ E
2
(= Const.)
W. Erbsen HOMEWORK #1
As noted, the prompt states that the barrier allows for exchange in energy. We also recall that
dE =
_
E
S
_
V,N
dS +
_
E
V
_
S,N
dV +
_
E
N
_
S,V
dN (3.1.1)
Which was shown in class, but is easy enough to see.
a) If the barrier is allowed to move, but does not allow for particle exchange, we can deduce two im-
portant things. First, if the barrier moves but leaves the individual systems intact, while conserving
the total volume, then we can claim that:
dV = V
1
+ V
2
= 0 V
1
= V
2
(3.1.2)
We can also claim that since the particles are not transferred from system to system, we can say
dN = N
1
+ N
2
= 0 (3.1.3)
We can apply (3.1.3) to (3.1.1)
dE =
_
E
S
_
V,N
dS +
_
E
V
_
S,N
dV +
= 0
..
_
E
N
_
S,V
dN
=
_
E
S
_
V,N
dS +
_
E
V
_
S,N
dV (3.1.4)
We now recall (1.3.10), which states that:
_
S
E
_
V,N
=
1
T
,
_
S
V
_
E,N
=
P
T
,
_
S
N
_
V,E
=

T
From this, we take the denition which depends on the intensive parameters P and T, which can
be manipulated as
_
S
V
_
E,N
=
P
T
T
_
S
V
_
E,N
= P (3.1.5)
Now, following the form of (3.1.4), we have for the entropy S
dS =
_
S
E
_
V,E
dE +
_
S
V
_
E,N
dV +
= 0
..
_
S
N
_
E,V
dN
=
_
S
E
_
V,E
dE +
_
S
V
_
E,N
dV (3.1.6)
We also know that the steady-state entropy as t approaches zero, and (3.1.6) becomes
0 =
_
S
E
_
V,E
dE +
_
S
V
_
E,N
dV (3.1.7)
Holding E and N constant for both systems, (3.1.7)
0 =
= 0
..
S
1
E
1
dE
1
+
S
2
E
2
dE
2
+
S
1
V
1
dV
1
+
S
2
V
2
dV
2
(3.1.8)
CHAPTER 3: STATISTICAL MECHANICS 229
Recalling (3.1.5), it is easy to see that
0 =
S
1
V
1
dV
1

S
2
V
2
dV
1

P
1
T
=
P
2
T
P
1
= P
2
(3.1.9)
b) If now we disallow the barrier to move, then the change in volume becomes:
dV = V
1
+V
2
= 0
While since we are allowing for particle exchange, we can claim that the net change in the ux of
particulates is described by
dN = N
1
+ N
2
= 0 N
1
= N
2
We can now immediately pick up from (3.1.6), applying it to our current scenario:
dS =
_
S
E
_
V,E
dE +
= 0
..
_
S
V
_
E,N
dV +
_
S
N
_
E,V
dN
=
_
S
E
_
V,E
dE +
_
S
N
_
E,V
dN (3.1.10)
Recalling the denition of the intrinsic property from (1.4.10),
_
S
N
_
V,E
=

T
T
_
S
N
_
V,E
= (3.1.11)
Using (3.1.11), at thermal equilibrium (3.1.10) becomes
0 =
S
1
N
1
dN
1
+
S
2
N
2
dN
1


1
T
dN
1
=

2
T
dN
1

1
=
2
Problem 1.2
Assuming that the entropy S and the statistical number of a physical system are related through an arbi-
trary functional form
S = f () (3.1.12)
Show that the additive character of S and the multiplicative character of necessarily require that the function
f () be of the form
S = k
B
log (3.1.13)
Solution
The additive character of S can be quantitatively described using (3.1.12) and (3.1.13) as
S
(0)
(S
1
, S
2
) = S
1
(
1
) +S
2
(
2
) S
(0)
(
1
,
2
) = f (
1
) +f (
2
) (3.1.14)
W. Erbsen HOMEWORK #1
While the multiplicative property of can be written like:

(0)
(
1
,
2
) =
1

2
(3.1.15)
We now recall that both
1
and
2
represent independent variables, and accordingly a function of both
quantities may be dierentiated with respect to each variable, yielding the same number of equations.
In our case, we can dierentiate (3.1.14) as:
dS
(0)
(
1
,
2
)
d
1
=
dS
(0)
(
1
,
2
)
d

d
d
1

dS
(0)
(
1
,
2
)
d

1
(3.1.16)
Using (3.1.16) as a template, we can retrieve our two equations:
dS
(0)
(
1
,
2
)
d
1
=
dS
(0)
(
1
,
2
)
d

1
(3.1.17a)
dS
(0)
(
1
,
2
)
d
2
=
dS
(0)
(
1
,
2
)
d

2
(3.1.17b)
The full derivatives in (3.1.17a) and (3.1.17b) may be equated and shued,
1

dS
(0)
(
1
,
2
)
d
1
=
1

dS
(0)
(
1
,
2
)
d
2
(3.1.18)
We now allow ourselves to be reminded of the method of separation of variables in PDEs, which says
that if we have a function of two independent variables, which can be completely separated, then then
changing one of the variables by some amount then the other must also change by . We recall that
is, of course, a constant, which we can call whatever we want. Apparently, in our case our constant is
Boltzmanns Constant k
B
.
Armed with this knowledge, we can now rewrite (3.1.17a) and (3.1.17b), as
df (
1
)
d
1
=
dS
(0)
(
1
,
2
)
d

2
(3.1.19a)
df (
2
)
d
2
=
dS
(0)
(
1
,
2
)
d

1
(3.1.19b)
The full derivatives in (3.1.19a) and (3.1.19b) may also be equated, similar to in (3.1.18), as
1

df (
2
)
d
2
=
1

df (
1
)
d
1

df (
2
)
d
2
=
1

df (
1
)
d
1
(3.1.20)
Setting each side of (3.1.20) equal to our new constant,

df (
2
)
d
2
= k
B
,
1

df (
1
)
d
1
= k
B
(3.1.21)
The solutions to these trivial dierential equations is of course
f () = k
B
log () S () = k
B
log ()
CHAPTER 3: STATISTICAL MECHANICS 231
Problem 1.7
Study the statistical mechanics of an extreme relativistic gas characterized by the single-particle energy states
(n
x
, n
y
, n
z
) =
hc
2L
_
n
2
x
+ n
2
y
+n
2
z
_
1
/2
(3.1.22)
Instead of (1.4.5) in Pathria, along the lines in 1.4. Show that the ratio C
P
/C
V
in in this case is 4/3, instead of
5/3.
Solution
Following Pathrias logic in 1.4, we know that we need not explicitly evaluate (N, E, V ), rather simply
the product following the form of (1.4.8), where in the relativistic case, I claim that:
V
1
/3
E = const. (3.1.23)
Where E in our case is the relativistic energy, given by:
E =
_
(pc)
2
+ (mc
2
)
2
(3.1.24)
Assuming that we are in the extreme relativistic limit, we assume that v c, and in this limit (3.1.24)
becomes
E pc (3.1.25)
Plotting (3.1.24) and (3.1.25) on the same scale indeed veries that the energy may be readily approxi-
mated using (3.1.25) at very large values of v (see gure

).
Figure 3.2: Figure for Problem 3.

We note that (3.1.24) red/dashed, while (3.1.25) black/thick.


W. Erbsen HOMEWORK #1
Furthermore, we can build on (1.4.6) by noting that the expression for (N, E, V ) depends on the
number of independent, positive-integral solutions as prescribed by:
2L
hc
=
_
n
2
x
+n
2
y
+ n
2
z
_
1
/2
(3.1.26)
From the Maxwell-Boltzmann Distribution Function, and (3.1.25). Furthermore, we recall that V = L
3
,
and so (3.1.26) becomes
2
hc
V
1
/3
(n
x
, n
y
, n
z
) =
_
n
2
x
+ n
2
y
+n
2
z
_1
/2
(n
x
, n
y
, n
z
) =
hc
2L
_
n
2
x
+ n
2
y
+n
2
z
_1
/2
(3.1.27)
Where we can see from (3.1.27) where (3.1.23) was derived from. Furthermore, using (1.4.25), we can
say that:
P =
_
E
V
_
N,S
=

V
_
Const. V

1
/3
_
=Const.
1
3
V
4
/3
=Const. V
4
/3
(3.1.28)
It is easy to see from (3.1.28) that our condition for a reversible adiabatic process is given by
PV
4
/3
= Const. (3.1.29)
Where the raised power of V in (3.1.29) is none other than , so we can nally say:
=
C
P
C
V
=
4
3
Problem 1.13
If the two gases considered in the mixing process of 1.5 were initially at dierent temperatures, say T
1
and
T
2
, what would the entropy of mixing be in that case? Would the contribution arising from this cause depend on
whether the two gases were dierent or identical?
Solution
We rst recall from (1.5.13) that
S
i
= N
i
k
B
log [V
i
] +
3
2
N
i
k
B
_
1 + log
_
2m
i
k
B
T
h
2
__
; i = 1, 2 (3.1.30)
Which represents the entropy before the mixing ever took place. If we allow our systems to start at
dierent temperatures, (3.1.30) becomes
CHAPTER 3: STATISTICAL MECHANICS 233
S
i
= N
i
k
B
log [V
i
] +
3
2
N
i
k
B
_
1 + log
_
2m
i
k
B
T
i
h
2
__
; i = 1, 2 (3.1.31)
Where it should be mentioned that the only thing changed is the addition of an index to the intensive
variable T. Regrouping and realigning (3.1.31), we have
S
i
=
3
2
k
B
N
i
_
2
3
log [V
i
] + log [m
i
T
i
] + 1
_
(3.1.32)
Where I have assigned the constant to be dened by
=
2k
B
h
2
(3.1.33)
Following the convention of (3.1.32), we can say that the post-mixing entropy is (analogous to (1.5.2)):
S
T
=
3
2
k
B
2

i=1
N
i
_
2
3
log [V ] + log [m
i
T] + 1
_
(3.1.34)
The increase in S, called the entropy of mixing, is then given by
S = S
T

i=1
S
i
(3.1.35)
The rst term in (3.1.35) is found from (3.1.34) to be
S
T
=
3
2
k
B
_
N
1
_
2
3
log [V ] + log [m
1
T] + 1
_
+N
2
_
2
3
log [V ] + log [m
2
T] + 1
__
(3.1.36)
While the sum in (3.1.35) is found from (3.1.32) to be
2

i=1
=
3
2
k
B
_
N
1
_
2
3
log [V
1
] + log [m
1
T
1
] + 1
_
+ N
2
_
2
3
log [V
2
] + log [m
2
T
2
] + 1
__
(3.1.37)
Before we take the dierence of (3.1.36) and (3.1.37), thus completing the saga of (3.1.35), we note that
this page width is nite (countable, har har), we drop the terms which obviously cancel. I will also
consolidate a few things. Continuing,
S =
3
2
k
B
_
N
1
_
2
3
log
_
V
V
1
_
+ log
_
T
T
1
__
+N
2
_
2
3
log
_
V
V
2
_
+ log
_
T
T
2
___
(3.1.38)
Jostling things about a bit, (3.1.38) becomes
S = k
B
N
1
_
log
_
V
1
+ V
2
V
1
_
+ log
_
T
T
1
_3
/2
_
+ k
B
N
2
_
log
_
V
1
+ V
2
V
2
_
+ log
_
T
T
2
_3
/2
_
(3.1.39)
The contribution of entropy in (3.1.39) most certainly depends on whether the gases are distinguishable
or indistinguishable. This is the setup for Gibbs Paradox. The general idea is that if the particles in
both systems are distinguishable, then after equilibrium is reached, the gas particles may be separated
back into their original compartments, assuming that we can reinsert the partition quickly enough. This
W. Erbsen HOMEWORK #1
would correspond to a reversible process, where we know that the change in entropy, or the entropy of
mixing, must be zero.
If the particles are indistinguishable, then there is no hope of separating them, and so we have an
irreversible process, and the entropy must increase.
Problem 1.16
Establish thermodynamically the formulae
V
_
P
T
_

=S (3.1.40a)
V
_
P

_
T
=N (3.1.40b)
Express the pressure P of an ideal classical gas in terms of the variables and T, and verify the above formulae.
Solution
We rst recall that within the framework of the Grand Canonical Ensemble, where particle exchange is
allowed, we can dene the following thermodynamic potential:
= F dN d = SdT Nd PdV (3.1.41)
Where we must recognize that this new thermodynamic potential is not the same as the multiplicity.
For our case, (3.1.40a) becomes
d = SdT
..
= 0
Nd
. .
= 0
PdV = PdV (3.1.42)
Where, following the example of other thermodynamic potentials, we may deduce that
S =
_

T
_

(3.1.43a)
N =
_

_
T
(3.1.43b)
Substituting (3.1.42) into (3.1.43a) and (3.1.43b) yields
S =
_
(PV )
T
_

S = V
_
P
T
_

(3.1.44a)
N =
_
(PV )

_
T
N = V
_
P

_
T
(3.1.44b)
Where we can see that (3.1.44a) is the same as (3.1.40a), while (3.1.44b) is identical to (3.1.40b).
CHAPTER 3: STATISTICAL MECHANICS 235
We now recall that the Gibbs Sum, which is sort of like the partition function for the Grand Canonical
Ensemble, is given by
(, T) =

N=0

s(N)
exp
_
N E
s,N
k
B
T
_
(3.1.45)
While the pressure is
P =
k
B
T
V
log [(, T)] (3.1.46)
Substituting (3.1.45) into (3.1.46), and then plugging the results in to (3.1.40a) and (3.1.40b) in turn
causes the terms to collapse, which is readily seen qualitatively.
3.2 Homework #2
Problem 1
In this problem, you will nd an expression for entropy in terms of the canonical distribution function
s
=
e
Es
Z
where the partition function Z is given by Z =

s
e
Es/kBT
.
Solution
We rst recall that the entropy is dened in terms of the free energy as:
S =
_
F
T
_
V,N
(3.2.1)
While the free energy is given by
F = k
B
T log [Z] (3.2.2)
Substituting (3.2.2) into (3.2.1), we have:
S =

T
k
B
T log [Z] (3.2.3)
And since Z of course depends on T, we carry the partial derivative out as:
S =k
B
log [Z] +k
B
T
_

T
log [Z]
_
=k
B
log [Z] +k
B
T
1
Z
_
Z
T
_
(3.2.4)
The bracketed term in (3.2.4) becomes:
W. Erbsen HOMEWORK #2
Z
T
=

T

s
exp
_

E
s
k
B
T
_
=
1
k
B
T
2

s
E
s
exp
_

E
s
k
B
T
_
(3.2.5)
Substituting (3.2.5) back into (3.2.4) yields
S =k
B
log [Z] +k
B
T
1
Z
_
1
k
B
T
2

s
E
s
exp
_

E
s
k
B
T
_
_
=k
B
_
log [Z] +
1
k
B
T

s
E
s
1
Z
exp
_

E
s
k
B
T
_
. .
s
_
=k
B
_
log [Z] +
1
k
B
T

s
E
s

s
_
= k
B
_
log [Z] +

s
_

E
s
k
B
T
__
= k
B
_
log [Z] +

s
log
_
exp
_

E
s
k
B
T
__ _
= k
B
_

s
log
_
1
Z
exp
_

E
s
k
B
T
_
. .
s
__
(3.2.6)
At this point, we can trivially rewrite (3.2.6) as:
S = k
B
_

s
log [
s
]
_
Problem 2
The expression S = k
B

r
P
r
log P
r
is accepted as the general denition of the entropy of a system. Now
imagine that a system A
1
has probability P
(1)
r
of being found in a state r and a system A
2
has probability P
(2)
s
of
being found in a state s. Then one has
S
1
= k
B

r
P
(1)
r
log P
(1)
r
S
2
= k
B

s
P
(2)
s
log P
(2)
s
Each state of the composite system A consisting of A
1
and A
2
can then be labeled by the pair of numbers r, s.
Let the probability of A being found in this state be denoted by P
rs
. Then its entropy is dened as
S = k
B

s
P
rs
log P
rs
(3.2.7)
CHAPTER 3: STATISTICAL MECHANICS 237
All the probabilities are normalized so that

r
P
(1)
r
=1

s
P
(2)
s
=1

s
P
rs
=1
a) If A
1
and A
2
are weakly interacting so that they are statistically independent, then P
rs
= P
(1)
r P
(2)
s . Show
that under these circumstances the entropy is additive, i.e. S = S
1
+S
2
.
b) If A
1
and A
2
are not weakly interacting P
rs
,= P
(1)
r P
(2)
s , but the general solutions still hold:
P
(1)
r
=

s
P
rs
P
(2)
s
=

r
P
rs
Show that
S (S
1
+S
2
) = k
B

r,s
P
rs
log
_
P
(1)
r
P
(2)
s
P
rs
_
(3.2.8)
and use the inequality log x (x 1) to show that S (S
1
+ S
2
). This means that the existence of
correlations between the systems, due to the interactions between them, leads to a situation less random than
that where the systems are completely independent of each other.
Solution
a) Applying our assumption to (3.2.7), we have:
S = k
B

s
P
r
P
s
log [P
r
P
s
]
= k
B

s
P
r
P
s
log [P
r
] + log [P
s
]
= k
B

s
P
r
P
s
log [P
r
] +P
r
P
s
log [P
s
]
= k
B
_

s
P
r
P
s
log [P
r
] +

s
P
r
P
s
log [P
s
]
_
= k
B
_

s
P
s

r
P
r
log [P
r
] +

r
P
r

s
P
s
log [P
s
]
_
(3.2.9)
We recall from the previous problem that S = k
B

i
log [
i
], and applying this to (3.2.9) leads
us to:
S = k
B
_

s
P
s
_

S
1
k
B
_
+

r
P
r
_

S
2
k
B
_
_
W. Erbsen HOMEWORK #2
=S
1

s
P
s
+S
2

r
P
r
(3.2.10)
Applying the fact that the probabilities are normalized, (3.2.10) leads us to
S = S
1
+S
2
b) In order to show (3.2.8), we begin by evaluating the LHS with our previous denitions of the
respective entropies:
S (S
1
+S
2
) = k
B

s
P
rs
log P
rs

_
k
B

r
P
r
log P
r
k
B

s
P
s
log P
s
_
=k
B
_

s
P
rs
log P
rs
+

r
P
r
log P
r
+

s
P
s
log P
s
_
(3.2.11)
We can expand (3.2.11) given the provided summation identities:
S (S
1
+ S
2
) =k
B
_

s
P
rs
log P
rs
+

s
P
rs
log P
r
+

r
P
rs
log P
s
_
=k
B
_

s
P
rs
log P
rs
+ P
rs
log P
r
+ P
rs
log P
s

_
=k
B
_

s
P
rs
log P
rs
+ log P
r
+ log P
s

_
(3.2.12)
It is easy to see that (3.2.12) may be readily rewritten as:
S (S
1
+S
2
) = k
B

s
P
rs
log
_
P
r
P
s
P
rs
_
(3.2.13)
Problem 3.13
a) Evaluate the partition function and the major thermodynamic properties of an ideal gas consisting of N
1
molecules of mass m
1
and N
2
molecules of mass m
2
, conned to a space of volume V at temperature
T. Assume that the molecules of a given kind are mutually indistinguishable, while those of one kind are
distinguishable from those of the other kind.
b) Compare your results with the ones pertaining to an ideal gas consisting of (N
1
+N
2
) molecules, all of one
kind, of mass m, such that m(N
1
+N
2
) = m
1
N
1
+ m
2
N
2
.
Solution
a) We know that the volume and temperature of the system are held constant, and the system consists
of two unique gases, each distinguishable from the other type, but indistinguishable from its own
CHAPTER 3: STATISTICAL MECHANICS 239
type. Since the two gases are independent of one another, this lends the idea that the partition
function may be expressed as the product of two distinct partition functions:
Q(N
1
, N
2
, V, T) = Q(N
1
, V, T) Q(N
2
, V, T) (3.2.14)
Where we recall that
Q(N, V, T) =
1
N!h
3N
_
__
exp
_

N
i
E
i
k
B
T
_
dp
3N
dq
3N
_
N
(3.2.15)
Where the subscript N has been suppressed. Assuming that there is no interaction between the
particles, then there is no potential energy term to the Hamiltonian, and the energy is due to the
kinetic energy alone. Accordingly, (3.2.15) becomes:
Q(N, V, T) =
1
N!h
3
_
V
_

exp
_

p
2
2mk
B
T
_
dp
3N
_
N
=
1
N!h
3
_
V [2mk
B
T]
3
/2
_
N
(3.2.16)
Applying the general prescription from (3.2.16) to (3.2.14) yields
Q(N
1
, N
2
, V, T) =
1
N
1
!h
3
_
V [2m
1
k
B
T]
3
/2
_
N1

1
N
2
!h
3
_
V [2m
2
k
B
T]
3
/2
_
N2
(3.2.17)
We start by nding the major thermodynamic properties by nding the Helmholtz Free Energy:
F = k
B
T log [Q(N
1
, N
2
, V, T)]
= k
B
T log
_
1
N
1
!h
3
_
V [2m
1
k
B
T]
3
/2
_
N1

1
N
2
!h
3
_
V [2m
2
k
B
T]
3
/2
_
N2
_
(3.2.18)
One major property is the pressure, which can be found by dierentiating the free energy with
respect to V :
P =
_
F
V
_
T,N
=

V
log
_
(...)V
N1

+

V
log
_
(...)V
N2

=N
1

V
log [V ] + N
2

V
log [V ]
Carrying out this simple dierentiation, we are left with
P =
N
1
+N
2
V
Another important thermodynamic property is the internal energy, which is easy enough to nd:
U =

log [Q(N
1
, N
2
, V, T)]
=

log
_
_
1
N
1
!h
3
_
V
_
2m
1

_3
/2
_
N1

1
N
2
!h
3
_
V
_
2m
2

_3
/2
_
N2
_
_
W. Erbsen HOMEWORK #2
= N
1

log
_
(...)
1

3
/2
_
N
2

log
_
(...)
1

3
/2
_
=
3N
1
2

log
_
(...)
1

3N
2
2

log
_
(...)
1

_
(3.2.19)
From (3.2.19) we can gather
U =
3
2
k
B
T (N
1
+ N
2
)
We can also calculate the entropy from (3.2.18) as
S =
_
F
T
_
N,V
=

T
_
k
B
T log
_
1
N
1
!h
3
_
V [2m
1
k
B
T]
3
/2
_
N1

1
N
2
!h
3
_
V [2m
2
k
B
T]
3
/2
_
N2
__
(3.2.20)
Dierentiating (3.2.20) leads us to
S = k
B
_
log
_
1
N
1
!h
3
_
V [2m
1
k
B
T]
3
/2
_
N1

1
N
2
!h
3
_
V [2m
2
k
B
T]
3
/2
_
N2
__
+
3
2T
(N
1
+ N
2
)
b) If the gas is no longer a mixture of two unique species, then the partition function becomes
Q(N
1
, N
2
, V, T) =
1
N
1
!N
2
!h
6
_
V [2mk
B
T]
3
/2
_
N1+N2
The pressure and internal energy remain unchanged, while the entropy becomes:
S = k
B
_
log
_
1
N
1
!N
2
!h
6
_
V [2mk
B
T]
3
/2
_
N1+N2
__
+
3
2T
(N
1
+ N
2
)
Problem 3.15
Show that the partition function Q
N
(V, T) of an extreme relativistic gas consisting of N monatomic molecules
with energy-momentum relationship E = pc, c being the speed of light, is given by
Q
N
(V, T) =
1
N!
_
8V
_
k
B
T
hc
_
3
_
N
(3.2.21)
Study the thermodynamics of this system, checking in particular that
PV =
1
3
U, U/N = 3k
B
and =
4
3
Next, using the inversion formula (3.4.7), derive an expression for the density of states g(E) of this system.
Solution
CHAPTER 3: STATISTICAL MECHANICS 241
We rst recall that the partition function can be expressed as:
Q
N
(V, T) =
1
N!h
3N
_
__
exp
_

N
i
E
i
k
B
T
_
dp
3N
dq
3N
_
N
(3.2.22)
Assuming that our relativistic gas is in 3-D, then (3.2.22) becomes:
Q
N
(V, T) =
1
N!h
3
_
__
exp
_

N
i
p
i
c
k
B
T
_
d
3
p d
3
q
_
N
=
1
N!h
3
_
V
_
exp
_

N
i
p
i
c
k
B
T
_
d
3
p
_
N
=
1
N!h
3
_
V
___
exp
_

pc
k
B
T
_
p
2
sin dp d d
_
N
=
1
N!h
3
_
4V
_

0
p
2
exp
_

pc
k
B
T
_
dp
_
N
=
1
N!h
3
_
4V 2
_
k
B
T
c
_
3
_
N
(3.2.23)
Rearranging (3.2.23) somewhat, we are left with:
Q
N
(V, T) =
1
N!
_
8V
_
k
B
T
hc
_
3
_
N
Which is the same as (3.2.21). We now recall that the internal energy is given by:
E =

log [Q
N
] (3.2.24)
Substituting (3.2.21) into (3.2.24), we have:
E =

log
_
_
1
N!
_
8V
_
k
B
T
hc
_
3
_
N
_
_
= N

log
_
_
1
N!
_1
/N
8V
_
1
hc
_
3
_
= 3N

log
_
_
1
N!
_1
/3N
8V
_
1
hc
_
_
= 3N
_

_
(3.2.25)
From (3.2.25) it is easy to see that:
E
N
= 3k
B
T (3.2.26)
W. Erbsen HOMEWORK #2
We now recall that the free energy is given by:
F = k
B
T log [Q
N
] (3.2.27)
Substituting (3.2.21) into (3.2.27), we have:
F = k
B
T log
_
_
1
N!
_
8V
_
k
B
T
hc
_
3
_
N
_
_
= Nk
B
T log
_
_
1
N!
_1
/N
8V
_
k
B
T
hc
_
3
_
(3.2.28)
We innocently recall that the pressure is dened in terms of the free energy as:
P =
_
F
V
_
T,N
(3.2.29)
And so, putting (3.2.28) into (3.2.29):
P =

V
_
Nk
B
T log
_
_
1
N!
_1
/N
8V
_
k
B
T
hc
_
3
__
=Nk
B
T

V
log
_
_
1
N!
_1
/N
8V
_
k
B
T
hc
_
3
_
=
Nk
B
T
V
(3.2.30)
We can now manipulate (3.2.26), such that:
Nk
B
T =
E
3
(3.2.31)
And now substituting (3.2.31) into (3.2.30),
P =
E
3

1
V
PV =
E
3
(3.2.32)
We now recall that the specic heat at constant volume is dened by:
C
V
=
_
E
T
_
V
(3.2.33)
And substituting (3.2.26) into (3.2.33), we are left with:
C
V
=

T
3Nk
B
T C
V
= 3Nk
B
(3.2.34)
While the specic heat at constant pressure is given from (1.3.18) as:
C
P
=
_
(E + PV )
T
_
N,P
(3.2.35)
CHAPTER 3: STATISTICAL MECHANICS 243
Evaluating the internal energy in terms of PV from (3.2.32) and substituting into (3.2.36):
C
P
=
(3PV + PV )
T
=4

T
PV (3.2.36)
Recalling the from the ideal gas law that PV = Nk
B
T, (3.2.36) becomes:
C
P
= 4

T
Nk
B
T C
P
= 4Nk
B
(3.2.37)
Using (3.2.34) and (3.2.37), we can now nd :
=
C
P
C
V

4Nk
B
T
3Nk
B
T
=
4
3
(3.2.38)
Problem 3.22
The restoring force of an anharmonic oscillator is proportional to the cube of the displacement. Show that
the mean kinetic energy of the oscillator is twice its mean potential energy.
Solution
We begin our journey by recalling that the average of some quantity x) can be found from the corre-
sponding distribution function f(x) according to:
x) =
_
x f(x) dx (3.2.39)
Now, in order to nd the mean kinetic energy, we recall that in general, the 1-D kinetic energy is given
by:
T =
1
/
2
mv
2
(3.2.40)
So, the mean value of the kinetic energy is then:
T) =
1
/
2
mv
2
) (3.2.41)
We can now nd v
2
) from (3.2.41) by use of (3.2.39), recalling that the distribution function in this
case is none other than the Maxwell-Boltzmann Velocity Distribution:
f(v) =
_
m
2k
B
T
exp
_

mv
2
2k
B
T
_
(3.2.42)
Using (3.2.42), v
2
) becomes:
v
2
) =
_

v
2
f(v) dv
W. Erbsen HOMEWORK #2
=
_
m
2k
B
T
_

v
2
exp
_

mv
2
2k
B
T
_
dv (3.2.43)
At this point, we recall that integrals in the form of (3.2.43) can be evaluated as:
_

x
2
exp
_
ax
2

dx =

2a
3
/2
Applying this to (3.2.43), we have:
v
2
) =
_
m
2k
B
T

2

_
2k
B
T
m
_3
/2
=
k
B
T
m
(3.2.44)
Substituting (3.2.44) back into (3.2.41):
T) =
m
2

k
B
T
m

k
B
T
2
(3.2.45)
We now recall that according to Boltzmann statistics, the mean energy is given by:
E) =
1
Z

s
E
s
exp
_

E
s
k
B
T
_
(3.2.46)
And now, remembering the relationship between force and potential energy:
F(x) =
V (x)
x
(3.2.47)
We are also told that the restoring force is proportional to the cube of the displacement, so:
F(x) Cx
3
(3.2.48)
Where C is some constant. Accordingly, substituting (3.2.48) into (3.2.47), we have:
V (x) Cx
4
(3.2.49)
Using Boltzmann statistics according to (3.2.46), we nd that the mean potential energy is then given
by:
V ) =
1
Z

V (x) exp
_

V (x)
k
B
T
_
(3.2.50)
Converting the sum in (3.2.50) into an integral,
V ) =
1
Z
_

V (x) exp
_

V (x)
k
B
T
_
dx
=
1
Z
_

Cx
4
exp
_

Cx
4
k
B
T
_
dx
=
1
Z

_
5
/
4
_
2

_
k
B
T
C
_5
/4
(3.2.51)
CHAPTER 3: STATISTICAL MECHANICS 245
We now wish to evaluate the partition function Z:
Z =
_

exp
_

Cx
4
k
B
T
_
dx
=2
_
5
/
4
_

_
k
B
T
C
_1
/4
(3.2.52)
Substituting (3.2.52) back into (3.2.51),
V ) =
1
2

1
(
5
/
4
)

_
C
k
B
T
_1
/4

_
5
/
4
_
2

_
k
B
T
C
_5
/4
=
k
B
T
4
(3.2.53)
So, in summary, from (3.2.45) and (3.2.53), we have:
T) =
k
B
T
2
, V ) =
k
B
T
4
2T) = V )
Problem 3.42
Consider the system of N magnetic dipoles, studied in 3.10, in the microcanonical ensemble. Enumerate the
number of microstates, (N, E), accessible to the system at energy E, and evaluate the quantities S(N, E) and
T(N, E). Compare your results with (3.10.8) and (3.10.9).
Solution
Within the framework of the microcanonoical ensemble, we note that the energy in the system is xed
and therefore conserved. If we assume that our magnetic dipoles can have only one of two unique states
(i.e. aligned or anti-aligned), then the total number of dipoles N can be expressed in terms of the
individual components as:
N = N

+ N

(3.2.54)
It will also be useful to dene:
n = N

(3.2.55)
We can solve (3.2.54) and (3.2.55) giving expressions in terms of the number in each state:
N

=
N + n
2
(3.2.56a)
N

=
N n
2
(3.2.56b)
Using (3.2.56a) and (3.2.56b), we can enumerate the number of microstates as:
W. Erbsen HOMEWORK #2
(N, E) =
N!
N

!N

!
(N, E) =
N!
[(N +n)/2]! [(N n)/2]!
(3.2.57)
We could take the natural log of both sides of (3.2.57),
log [(N, E)] =log
_
N!
_
N+n
2
_
!
_
Nn
2
_
!
_
=log [N!] log
__
N +n
2
_
!
_
log
__
N n
2
_
!
_
(3.2.58)
We can also apply Stirlings Approximation to each of the three terms in (3.2.58). The rst terms is
log [N!] = N log [N] N (3.2.59)
The second term can now be expressed as
log
__
N +n
2
_
!
_
=
_
N +n
2
_
log
__
N +n
2
__

_
N +n
2
_
=
1
2
_
(N + n) log
__
N +n
2
__
N n
_
=
1
2
_
N log
__
N +n
2
__
+nlog
__
N + n
2
__
N n
_
(3.2.60)
While the third is
log
__
N n
2
_
!
_
=
_
N n
2
_
log
__
N n
2
__

_
N n
2
_
=
1
2
_
(N n) log
__
N n
2
__
N + n
_
=
1
2
_
N log
__
N n
2
__
nlog
__
N n
2
__
N +n
_
(3.2.61)
Substituting (3.2.59)-(3.2.61) into (3.2.58), we have
log [(N, E)] =N log [2] + N log [N]
_
N n
2
_
log [N n]
_
N +n
2
_
log [N +n] (3.2.62)
To nd S(N, E), we rst recall that S = k
B
log [], and also that (3.2.62) is in a convienent form:
S(N, E) = k
B
_
N log [2] + N log [N]
_
N n
2
_
log [N n]
_
N + n
2
_
log [N + n]
_
(3.2.63)
We now recall the thermodynamic identity
dU = TdS Pdv (3.2.64)
At constant volume, (3.2.64) becomes
dU = TdS
dS
dU
=
1
T
(3.2.65)
CHAPTER 3: STATISTICAL MECHANICS 247
Applying the chain rule to (3.2.65),
dS
dU
=
dS
dU

dU
dn

dn
dU

dS
dn

dn
dU
=
1
T
Also recalling that in our case
dU = E dn
dn
dE
=
1
E
Substituting this back into the chain rule, we have
dS
dn

1
E
=
1
T

dS
dn
=
E
T
(3.2.66)
Where we recall that E =
B
H. Substituting (3.2.63) into (3.2.66),

B
H
T
=k
B
d
dn
_
N log [2] + N log [N]
_
N n
2
_
log [N n]
_
N +n
2
_
log [N +n]
_
=
k
B
2
log
_
N n
N + n
_
(3.2.67)
It is easy to see that (3.2.67) can be solved, yielding
T (N, E) =
2
B
H
k
B
log
_
N +n
N n
_
3.3 Homework #3
Problem 1
Consider a system of N non-interacting spins of angular momentum J in the presence of an external eld H.
The energy levels for each spin are g
B
Hm

, where m

= J, J + 1, ..., J.
a) Compute the partition function for this system (evaluate all sums)
b) Compute the average magnetic moment for this system
c) Compare your results to the J =
1
/
2
example from lecture and the J = classical case (take the limit J
and g 0 such that
B
gJ retains the constant value
0
)
d) Evaluate the Curie constant for the generic spin J case
Solution
W. Erbsen HOMEWORK #3
a) The single-particle canonical partition function is dened by
Q
1

i
exp
_

E
i
k
B
T
_
(3.3.1)
In our case, the energy is due to the interaction between the spins and the magnetic eld, whose
relationship was given in the prompt. Accordingly, (3.3.1) becomes
Q
1
=
m=+J

m=J
exp
_

g
B
Hm

k
B
T
_
(3.3.2)
At this point, we dene the following quantity:
x =
g
B
H
k
B
T
Rewriting (3.3.2) in terms of x and expanding the sum,
Q
1
=
m=+J

m=J
e
xm
=e
x(J)
+e
x(J+1)
+e
x(J+2)
+ ... +e
x(J2)
+ e
x(J1)
+e
xJ
=e
xj
+ e
(J1)x
+e
(J2)x
+ ... + e
(J2)x
+e
(J1)x
+ e
xJ
=e
xJ
_
e
2xJ
+ e
(2J1)x
+ e
(2J2)x
+ ... +e
2x
+ e
x
+ 1
_
=e
xJ
_
1 + e
x
+e
2x
+ ... +e
(2J2)x
+e
(2J1)x
+e
2xJ
_
=e
xJ
_
1 + e
x
+e
2x
+ ... +e
2(J1)x
+e
2(J
1
/2)x
+ e
2xJ
_
(3.3.3)
Taking a slight detour, we recall that we can express a nite sum in terms of two innite sums like
1 + y +y
2
+ y
3
+ ... +y
x
=
_
1 +y + y
2
+ y
3
+...
_

_
y
x+1
+ y
x+2
+...
_
=
_
1 +y + y
2
+ y
3
+...
_
y
x+1
_
1 + y +y
2
+y
3
+ ...
_
=
_
1 +y + y
2
+ y
3
+...
_ _
1 y
x+1
_
(3.3.4)
We can simplify this further by recalling that
_
1 +y +y
2
+y
3
+...
_
=

y
1
1 y
(3.3.5)
Using (3.3.5), we can now rewrite (3.3.4) as
1 +y +y
2
+y
3
+... + y
x
=
1 y
x+1
1 y
(3.3.6)
Confusingly, we recognize that in the notation of (3.3.6), we let y e
x
, and x 2J. Accordingly,
(3.3.3) becomes
Q
1
=e
xJ
_
1 e
(2J+1)x
1 e
x
_
=
e
xJ
e
(J+1)x
1 e
x
CHAPTER 3: STATISTICAL MECHANICS 249
=
e
xJ
e
(J+1)x
1 e
x

e
x
/2
e
x
/2
=
e
xJ
x
/2
e
(J+1)x
x
/2
e
x
/2
e
x
x
/2
=
e
(J+
1
/2)x
e
(J+
1
/2)x
e

x
/2
e
x
/2
=
e
(J+
1
/2)x
e
(J+
1
/2)x
e
x
/2
e

x
/2
=
e
(J+
1
/2)x
e
(J+
1
/2)x
1

1
e
x
/2
e

x
/2
(3.3.7)
Recalling that 2 sinh (x) = e
x
e
x
, (3.3.7) becomes
Q
1
=
sinh
__
J +
1
/
2
_
x

sinh [
x
/
2
]
(3.3.8)
b) To nd the average magnetic moment, we rst recall that the relationship between the magnetization
(M) and the magnetic moment () is ?
M =
N
V

Such that the mean magnetic moment can now be dened in terms of the mean magnetization as
M) = N) (3.3.9)
We also recall that the magnetization is dened in terms of the free energy as
M) =
_
F
H
_
T
(3.3.10)
Where the free energy is of course
F = k
B
T log [Q
N
] (3.3.11)
Recalling that for distinguishable, non-interacting particles the full partition function is dened in
terms of the single-particle partition function as Q
N
= Q
N
1
. Keeping this in mind, and substituting
(3.3.11) into (3.3.10),
M) = k
B
T

H
log
_
Q
N
1

M) = Nk
B
T
1
Q
1
Q
1
H
(3.3.12)
It will be much easier to evaluate (3.3.12) if we convert the partial derivative of Q with respect to
H into the product of two other partial derivatives using the chain rule:
M) = Nk
B
T
1
Q
1
Q
1
H
x
Q
1
Q
1
x
M) = k
B
T
1
Q
1
x
H
Q
1
x
(3.3.13)
We can solve the rst partial derivative in (3.3.13) by recalling how we originally dened x:
x
H
=

H
_
g
B
H
k
B
T
_

g
B
k
B
T
(3.3.14)
We can also solve the second partial derivative in (3.3.13) by use of (3.3.8),
W. Erbsen HOMEWORK #3
Q
1
x
=

x
_
sinh
__
J +
1
/
2
_
x

sinh [
x
/
2
]
_
=sinh
__
J +
1
/
2
_
x


x
_
1
sinh [
x
/
2
]
_
+
1
sinh [
x
/
2
]

x
_
sinh
__
J +
1
/
2
_
x

=sinh
__
J +
1
/
2
_
x

1
2
coth [
x
/
2
]
sinh [
x
/
2
]
_
+
1
sinh [
x
/
2
]
__
J +
1
/
2
_
cosh
__
J +
1
/
2
_
x

=
1
2

sinh
__
J +
1
/
2
_
x

sinh [
x
/
2
]
coth [
x
/
2
] +
cosh
__
J +
1
/
2
_
x

sinh [
x
/
2
]
_
J +
1
/
2
_
=
Q
2
coth [
x
/
2
] +
cosh
__
J +
1
/
2
_
x

sinh [
x
/
2
]
_
J +
1
/
2
_
=
Q
2

cosh [
x
/
2
]
sinh [
x
/
2
]
+
cosh
__
J +
1
/
2
_
x

sinh [
x
/
2
]
_
J +
1
/
2
_
=
1
sinh [
x
/
2
]
_
cosh
__
J +
1
/
2
_
x
_
J +
1
/
2
_

Q
2
cosh [
x
/
2
]
_
(3.3.15)
Substituting (3.3.14) and (3.3.15) back into (3.3.13), we have
M) =Nk
B
T
1
Q
1
_
g
B
k
B
T
__
1
sinh [
x
/
2
]
_
cosh
__
J +
1
/
2
_
x
_
J +
1
/
2
_

Q
2
cosh [
x
/
2
]
__
=N
g
B
sinh [
x
/
2
]
_
1
Q
1
cosh
__
J +
1
/
2
_
x
_
J +
1
/
2
_

1
2
cosh [
x
/
2
]
_
=N
g
B
sinh [
x
/
2
]
_
sinh [
x
/
2
]
sinh [(J +
1
/
2
) x]
cosh
__
J +
1
/
2
_
x
_
J +
1
/
2
_

1
2
cosh [
x
/
2
]
_
=Ng
B
_
cosh
__
J +
1
/
2
_
x

sinh [(J +
1
/
2
) x]
_
J +
1
/
2
_

1
2
cosh [
x
/
2
]
sinh [
x
/
2
]
_
=Ng
B
_
coth
__
J +
1
/
2
_
x
_
J +
1
/
2
_

1
2
coth [
x
/
2
]
_
(3.3.16)
Applying (3.3.16) to (3.3.9), we can say that the mean magnetic moment is
) =
M)
N
) = g
B
_
coth
__
J +
1
/
2
_
x
_
J +
1
/
2
_

1
2
coth [
x
/
2
]
_
(3.3.17)
c) From lecture, in the case of J =
1
/
2
, we came to the conclusion that
) =
B
tanh
_

B
H
k
B
T
_
(3.3.18)
Whereas in the classical case, we came to the conclusion that
) =

2
B
H
3k
B
T
(3.3.19)
We can take the limit of (3.3.18) as J easily if we recall that coth[] 1,
) = g
B
_
_
J +
1
/
2
_

1
2
_
) = g
B
J (3.3.20)
CHAPTER 3: STATISTICAL MECHANICS 251
Figure 3.3: Plot of ) for small J Figure 3.4: Plot of ) for large J
We can verify (3.3.20) graphically if we plot ) from (3.3.17) against both x and J. We see from
Fig. (3.3) that the dependence of ) for small values of J is not clear, however if we plot the same
thing for very large values of J, as in Fig. (3.4) it is easy to see that ) carries a linear dependence
on J, and seems to be independent of x, thus verifying (3.3.20).
d) In order to nd Curies Constant, we rst recall that Curies Law is valid only under conditions of
low magnetization (
B
T k
B
T), and does not apply in the high-eld/low-temperature regime. ?
We recall that x is directly proportional to H, while inversely proportional to T. Therefore, we can
expand the coth[x] terms in (3.3.17) as:
coth[ax]
1
ax
+
ax
3

(ax)
3
45
+
2 (ax)
5
945
...
Taking only the rst two terms in this expansion and plugging in to (3.3.17),
) g
B
__
1
(J +
1
/
2
) x
+
_
J +
1
/
2
_
x
3
_
_
J +
1
/
2
_

1
2
_
2
x
+
x
6
_
_
g
B
__
1
x
+
_
J +
1
/
2
_
2
x
3
_

_
1
x
+
x
12
_
_
g
B
_
_
J
2
+J +
1
/
4
_
x
3

x
12
_
g
B
_
_
J
2
+J
_
x
3
+
x
12

x
12
_

g
B
3
J (J + 1) x (3.3.21)
Recalling how we chose to dene x at the beginning of the problem, (3.3.21) becomes
)
g
B
3
J (J + 1)
g
B
H
k
B
T
)
g
2

2
B
H
3k
B
T
J (J + 1) (3.3.22)
Curies Constant is dened by
W. Erbsen HOMEWORK #3
M =
C
T
H C = M
T
H
(3.3.23)
Where C is Curies Constant. Recalling that M = N, and using (3.3.22), we can nd Curies
Constant from (3.3.23) as
C =
Ng
2

2
B
3k
B
J (J + 1)
Problem 2
Consider a liquid in equilibrium (both thermal and diusive) with its vapor (treated as an ideal gas). For the
liquid, we will apply the following crude model: we treat the liquids as if the molecules still formed a gas of
molecules moving independently, but with the following considerations. 1) each molecule is assumed to have a
constant potential energy due to its interaction with the other molecules, and 2) each molecule is assumed
free to move throughout a total volume N

v
0
, where v
0
is the average volume available per molecule in the liquid
phase.
a) With these assumptions, write down the partition function for a liquid consisting of N

molecules.
b) Now set the chemical potential of the liquid equal to the chemical potential of the vapor phase and nd an
expression for the vapor pressure in terms of the temperature and other constants like v
0
and .
Solution
a) We begin our journey by recalling that the partition function can be expressed in integral form as
Q
N
(N, V, T) =
1
N!h
3N
_
__
exp
_

N
i
H
i
k
B
T
_
dp
3N
dq
3N
_
N
(3.3.24)
The Hamiltonian, H, in our case is a combination of the kinetic energy term, and the potential energy
, which is one of our assumptions in dening our crude model for the liquid. The Hamiltonian is
then given by
H =

i
T
i
+ V
i

N

i
_
p
2
i
2m

_
(3.3.25)
Substituting (3.3.25) into (3.3.24),
Q
()
N
(N, V, T) =
1
N!h
3N
_
__
exp
_

N
i
_
p
2
i
/2m
_
k
B
T
_
dp
3
dq
3
_
N
=
1
N!h
3N
___
exp
_

p
2
2mk
B
T
+

k
B
T
_
dp
3
dq
3
_
N
=
1
N!h
3N
_
exp
_

k
B
T
_ _
exp
_

p
2
2mk
B
T
_
dp
3
_
dq
3
_
N
CHAPTER 3: STATISTICAL MECHANICS 253
=
1
N!h
3N
_
V exp
_

k
B
T
___

exp
_

p
2
2mk
B
T
_
dp
_
3
_
N
=
1
N!h
3N
_
V exp
_

k
B
T
_
_
_
2mk
B
T
_
3
_
N
=
1
N!
_
V exp
_

k
B
T
_ _
2mk
B
T
h
2
_3
/2
_
N
(3.3.26)
Recalling that the total volume, V , is related to the average volume per molecule, v
0
, through the
number of molecules N
()
, as V = N
()
v
0
, and so (3.3.26) becomes
Q
()
N
(N, V, T) =
1
N!
_
N
()
v
0
exp
_

k
B
T
_ _
2mk
B
T
h
2
_3
/2
_
N
(3.3.27)
b) The chemical potential is given by:

_
F
N
_
V,T

_
k
B
T log [Q]
N
_
V,T
(3.3.28)
So, lets nd the chemical potential of the liquid rst. Substituting (3.3.27) into (3.3.28),

()
= k
B
T
log
_
Q
()
N
_
N
= k
B
T

N
log
_
_
1
N!
_
Nv
0
exp
_

k
B
T
__
2mk
B
T
h
2
_3
/2
_
N
_
_
= k
B
T

N
_
_
_
log
_
_
_
Nv
0
exp
_

k
B
T
_ _
2mk
B
T
h
2
_3
/2
_
N
_
_
log [N!]
_
_
_
= k
B
T

N
_
N log
_
Nv
0
exp
_

k
B
T
__
2mk
B
T
h
2
_3
/2
_
log [N!]
_
= k
B
T
_

N
_
N log
_
Nv
0
exp
_

k
B
T
_ _
2mk
B
T
h
2
_3
/2
__
. .
I


N
log [N!]
. .
II
_
(3.3.29)
We rst evaluate I from (3.3.29):
I =

N
_
N log
_
Nv
0
exp
_

k
B
T
_ _
2mk
B
T
h
2
_3
/2
__
=log
_
Nv
0
exp
_

k
B
T
_ _
2mk
B
T
h
2
_3
/2
_

N
[N] + N

N
_
log
_
Nv
0
exp
_

k
B
T
_ _
2mk
B
T
h
2
_3
/2
__
=log
_
Nv
0
exp
_

k
B
T
_ _
2mk
B
T
h
2
_3
/2
_
+ 1 (3.3.30)
We now carry out the remaining partial derivative in (3.3.29), employing Stirlings approximation:
W. Erbsen HOMEWORK #3
II =

N
log [N!]
=

N
N log [N] N
=log [N]

N
N +N

N
log [N]

N
N
=log [N] + 1 1
=log [N] (3.3.31)
Substituting (3.3.30) and (3.3.31) back into (3.3.29),

()
= k
B
T
_
log
_
Nv
0
exp
_

k
B
T
_ _
2mk
B
T
h
2
_3
/2
_
+ 1 log [N]
_
= k
B
T
_
log
_
v
0
exp
_

k
B
T
_ _
2mk
B
T
h
2
_3
/2
_
+ 1
_
= k
B
T
_
log
_
v
0
exp
_

k
B
T
_ _
2mk
B
T
h
2
_3
/2
_
+ log [exp [1]]
_
= k
B
T
_
log
_
v
0
exp
_

k
B
T
_ _
2mk
B
T
h
2
_3
/2
exp [1]
__
= k
B
T log
_
v
0
exp
_

k
B
T
+ 1
__
2mk
B
T
h
2
_3
/2
_
(3.3.32)
In order to nd the chemical potential of the gas, we can simply rewrite (3.3.32), where in the new
case 0, and we also recall that v
0
= V/N:

(g)
= k
B
T log
_
V
N
_
2mk
B
T
h
2
_3
/2
_
(3.3.33)
Recalling that our gas is in fact an ideal gas, we notice that we can manipulate the ideal gas law as
PV = Nk
B
T
V
N
=
k
B
T
P
(3.3.34)
Substituting (3.3.34) back into (3.3.33),

(g)
= k
B
T log
_
k
B
T
P
_
2mk
B
T
h
2
_3
/2
_
(3.3.35)
And now, to nd the vapor pressure at thermal and diusive equilibrium, we set
()
=
(g)
, and
from (3.3.32) and (3.3.35) we have
k
B
T log
_
v
0
exp
_

k
B
T
+ 1
__
2mk
B
T
h
2
_3
/2
_
= k
B
T log
_
k
B
T
P
_
2mk
B
T
h
2
_3
/2
_
v
0
exp
_

k
B
T
+ 1
_ _
2mk
B
T
h
2
_3
/2
=
k
B
T
P
_
2mk
B
T
h
2
_3
/2
v
0
exp
_

k
B
T
+ 1
_
=
k
B
T
P
(3.3.36)
CHAPTER 3: STATISTICAL MECHANICS 255
We see that (3.3.36) may be trivially solved for P, yielding
P =
k
B
T
v
0
exp
_

_

k
B
T
+ 1
__
Problem 3
A zipper has N links, each link has a state in which it is closed with energy 0 and a state in which it is open with
energy . We require that the zipper only unzip from the left end, and that the link number s can only open if all
the links to the left(1, s, ..., s 1) are already open.
a) Show that the partition function can be summed in the form:
Q
N
=
1 e
(N+1)
1 e

(3.3.37)
b) Find an expression for the average number of open links at some temperature T.
Solution
a) The standard form of the single-particle partition function is
Q
1
=

s=0
exp
_

E
s
k
B
T
_
(3.3.38)
Because the particles are distinguishable, then we can gain the full partition from (3.3.38) quite
easily
Q
s
=

s=0
exp
_

E
s
k
B
T
_
s
(3.3.39)
We now take a eld trip. Lets take a close look at the following series:
N

s=0
x
s
= 1 +x + x
2
+ x
3
+ ... +x
N
(3.3.40)
We now multiply both sides of (3.3.40) by x. This looks like
x
N

s=0
x
s
= x + x
2
+ x
3
+ x
4
+ ... +x
N+1
(3.3.41)
We now take (3.3.40) and subtract from it (3.3.41). This yields
(1 x)
N

s=0
x
s
= 1 x
N+1
(3.3.42)
W. Erbsen HOMEWORK #3
If we were to rearrange (3.3.42), then it would look like
N

s=0
x
s
=
1 x
N+1
1 x
(3.3.43)
Noticing that this is the exact sum is what we need in order to evaluate (3.3.39)!
Q
s
=
1 exp [(N + 1) E]
1 exp [E]
(3.3.44)
b) In general, the way to nd the average number of open links, we use the form
s) =
1
Q
s
N

s=0
s exp[sE] (3.3.45)
Now, notice that we can arrive at (3.3.45) if we are very tricky and take a derivative of the following
form:
s) =
1
Q
s

(E)
N

s=0
exp [sE]
. .
Q
s
(3.3.46)
Rewriting (3.3.46) once more,
s) =
1
Q
s

x
Q
s
(3.3.47)
Where I have made the temporary substitution x = E for convenience. Carrying out the rst
derivative in (3.3.47),

x
Q
s
=

x
_
1 exp [(N + 1) x]
1 exp [x]
_
=
1
1 exp [x]
_

x
[1 exp [(N + 1) x]]
_
+ [1 exp [(N + 1) x]]
_

x
1
1 exp [x]
_
=
1
1 exp [x]
[(N + 1) exp [(N + 1)x]] [1 exp [(N + 1)x]]
_
exp [x]
(1 exp [x])
2
_
=(N + 1)
exp [(N + 1)x]
1 exp [x]
exp [x]
_
1 exp [(N + 1)x]
(1 exp [x])
2
_
=(N + 1)
exp [(N + 1)x]
1 exp [x]

exp [x]
(1 exp [x])
2
+
exp [x] exp [(N + 1)x]
(1 exp [x])
2
=(N + 1)
exp [(N + 1)x]
1 exp [x]

1
1 exp [x]
_

exp [x]
1 exp [x]
+
exp [(N + 1)x] exp [x]
1 exp [x]
_
=
1
1 exp [x]
_
(N + 1) exp [(N + 1)x]
exp [x]
1 exp [x]
+
exp [(N + 1)x] exp [x]
1 exp [x]
_
=
1
1 exp [x]
_
(N + 1) exp [(N + 1)x] exp [x]
_
1 exp [(N + 1)x]
1 exp [x]
__
(3.3.48)
Where we note that the expression in brackets in (3.3.48) is none other than Q
s
. Rewriting,
CHAPTER 3: STATISTICAL MECHANICS 257

x
Q
s
=
1
1 exp [x]
[(N + 1) exp [(N + 1)x] exp [x] Q
s
] (3.3.49)
At this point we are ready to substitute (3.3.49) and (3.3.44) into (3.3.47):
s) =
1
Q
s

1
1 exp [x]
[(N + 1) exp [(N + 1)x] exp [x] Z
s
]
=
(N + 1) exp [(N + 1)x] exp [x]
1 exp [x]
=
(N + 1) exp [(N + 1)x]
1 exp [x]

exp [x]
1 exp [x]
(3.3.50)
Putting the numbers back in to (3.3.50) leads us to
s) =
(N + 1) exp [(N + 1)E]
1 exp [E]

exp [E]
1 exp [E]
(3.3.51)
We are asked to evaluate the average number of open links from (3.3.51) given the following stipu-
lation:
E k
B
T E 1
If E 1, then all the exponential terms approach zero, and so the average number of open links
is
s) = 0
Problem 4.5
Show that expression (4.3.20) for the entropy of a system in the grand canonical ensemble can also be written as:
S = k
B
_

T
(Tq)
_
,V
(3.3.52)
Solution
We see that (4.3.20) from Pathria reads
S = k
B
T
_
q
T
_
z,V
Nk
B
log [z] + k
B
q (3.3.53)
We now recall that the dierential of the q-potential has the form
dq =
_
q
T
_
z,V
dT +
_
q
V
_
z,T
dV +
_
q
z
_
V,T
dz (3.3.54)
Now that we are inspired, we may now rewrite the partial derivative in (3.3.53) as
W. Erbsen HOMEWORK #3
_
q
T
_
z,V

_
q
T
_
V,T
+
_
q
T
_
,V

q
T

T
z

z
T
+
_
q
T
_
,V

q
z

z
T
+
_
q
T
_
,V
(3.3.55)
We now recall (4.3.17), which reads
N(z, V, T)z
_

z
q(z, V, T)
_
V,T

_
q
z
_
V,T
=
N
z
(3.3.56)
Applying (3.3.56) to (3.3.55),
_
q
T
_
z,V

N
z
z
T
+
_
q
T
_
,V
(3.3.57)
Substituting (3.3.57) back into (3.3.53),
S =k
B
T
_
N
z
z
T
+
_
q
T
_
,V
_
Nk
B
log [z] + k
B
q
=k
B
_
N
T
z
z
T
+T
_
q
T
_
,V
N log [z] +q
_
(3.3.58)
We now innocently notice that
T
z
z
T
= log [z] (3.3.59)
Accordingly, we can now write (3.3.58) as
S =k
B
_
T
_
q
T
_
,V
+q
_
=k
B
_
T
_
q
T
_
,V
+q
_
T
T
_
,V
_
S = k
B
_

T
(qT)
_
,V
Problem 4.8
Determine the grand partition function of a gaseous system of magnetic atoms (with J =
1
/
2
and g = 2)
which can have, in addition to the kinetic energy, a magnetic potential energy equal to
B
H or
B
H, depending
upon their orientation with respect to the applied magnetic eld H. Derive an expression for the magnetization of
the system, and calculate how much heat will be given o by the system when the magnetic eld is reduced from
H to zero at constant volume and constant temperature.
CHAPTER 3: STATISTICAL MECHANICS 259
Solution
The (canonical) single-particle partition function is dened by
Q
1
=
1
h
3
__
exp
_

N
i
H
i
k
B
T
_
d
3
p d
3
q (3.3.60)
The Hamiltonian in this case is
H =
p
2
2m

B
H (3.3.61)
Applying (3.3.61) to (3.3.60),
Q
1
=
1
h
3
__
exp
_
p
2
/2m
B
H
k
B
T
_
d
3
p d
3
q
=
1
h
3
__
exp
_
p
2
2mk
B
T


B
H
k
B
T
_
d
3
p d
3
q
=
1
h
3
exp
_

B
H
k
B
T
_ _
d
3
q
_
exp
_
p
2
2mk
B
T
_
d
3
p
=
V
h
3
exp
_

B
H
k
B
T
_ __

exp
_
p
2
2mk
B
T
_
dp
_
3
=
V
h
3
exp
_

B
H
k
B
T
_
_
_
2mk
B
T
_
3
(3.3.62)
We now notice that the exponential term can be rewritten as
exp
_

B
H
k
B
T
_
=exp
_
+

B
H
k
B
T
_
+ exp
_

B
H
k
B
T
_
2 cosh
_

B
H
k
B
T
_
(3.3.63)
Substituting (3.3.63) back into (3.3.62),
Q
1
=2
V
h
3
cosh
_

B
H
k
B
T
_
[2mk
B
T]
3
/2
(3.3.64)
Using (3.3.64), we can now say that the full partition function is
Q
N
=
1
N!
_
2
V
h
3
cosh
_

B
H
k
B
T
_
[2mk
B
T]
3
/2
_
N
(3.3.65)
We now recall that the equation linking the grand partition function to the canonical partition function
is
Q =

N=0
z
N
Q
N
(3.3.66)
Where z is the fugacity. Substituting (3.3.65) into (3.3.66),
Q =

N=0
z
N
1
N!
_
2
V
h
3
cosh
_

B
H
k
B
T
_
[2mk
B
T]
3
/2
_
N
W. Erbsen HOMEWORK #3
=

N=0
1
N!
_
2z
V
h
3
cosh
_

B
H
k
B
T
_
[2mk
B
T]
3
/2
_
N
(3.3.67)
We now innocently recall that the series expansion for e
x
is
e
x
=

N=0
x
N
N!
We can clearly see that (3.3.67) follows this same form, and we can now write
Q = exp
_
2z
V
h
3
cosh
_

B
H
k
B
T
_
[2mk
B
T]
3
/2
_
(3.3.68)
The magnetization, M, is dened as
M =
_
F
H
_
T
(3.3.69)
We must now nd the free energy F, which is given by
F = k
B
T log [Q
N
] (3.3.70)
Substituting in the canonical partition function from (3.3.65) into (3.3.70),
F = k
B
T log
_
1
N!
_
2
V
h
3
cosh
_

B
H
k
B
T
_
[2mk
B
T]
3
/2
_
N
_
= Nk
B
T log
_
1
N!
1
/N
2
V
h
3
cosh
_

B
H
k
B
T
_
[2mk
B
T]
3
/2
_
(3.3.71)
Now substituting (3.3.71) into (3.3.69), we can nd the magnetization:
M =

H
_
Nk
B
T log
_
1
N!
1
/N
2
V
h
3
cosh
_

B
H
k
B
T
_
[2mk
B
T]
3
/2
__
=Nk
B
T

H
log
_
1
N!
1
/N
2
V
h
3
cosh
_

B
H
k
B
T
_
[2mk
B
T]
3
/2
_
=Nk
B
T

H
_
log
_
cosh
_

B
H
k
B
T
__
+ log
_
1
N!
1
/N
2
V
h
3
[2mk
B
T]
3
/2
__
=Nk
B
T
_

H
log
_
cosh
_

B
H
k
B
T
__
+

H
log
_
1
N!
1
/N
2
V
h
3
[2mk
B
T]
3
/2
__
=Nk
B
T
_

B
k
B
T
tanh
_

B
H
k
B
T
__
(3.3.72)
Cancelling terms in (3.3.72), we are left with
M = N
B
tanh
_

B
H
k
B
T
_
(3.3.73)
At constant temperature and volume, the entropy can be quantied in terms of heat by
CHAPTER 3: STATISTICAL MECHANICS 261
S =
Q
T
Q = TS (3.3.74)
We now note that S in (3.3.74) is simply the change in entropy, i.e. the dierence before and after
H 0. The entropy is dened as
S =
_
F
T
_
V,N
(3.3.75)
Substituting (3.3.71) into (3.3.75),
S =

T
_
Nk
B
T log
_
1
N!
1
/N
2
V
h
3
cosh
_

B
H
k
B
T
_
[2mk
B
T]
3
/2
__
=Nk
B

T
_
T log
_
1
N!
1
/N
2
V
h
3
[2mk
B
]
3
/2
. .

cosh
_

B
H
k
B
. .

1
T
_
T
3
/2
__
(3.3.76)
Rewriting (3.3.76) in terms of a new constant and ,
S =Nk
B

T
_
T log
_
cosh
_

T
_
T
3
/2
__
=Nk
B
_
log
_
cosh
_

T
_
T
3
/2
_
+T

T
_
log
_
cosh
_

T
_
T
3
/2
__
=Nk
B
_
log
_
cosh
_

T
_
T
3
/2
_
+T
_
3
2T


T
2
tanh
_

T
___
=Nk
B
_
log
_
cosh
_

T
_
T
3
/2
_
+
3
2


T
tanh
_

T
__
(3.3.77)
Substituting back into (3.3.77) the value of ,
S =Nk
B
_
log
_
cosh
_

B
H
k
B
T
_
T
3
/2
_
+
3
2


B
H
k
B
T
tanh
_

B
H
k
B
T
__
(3.3.78)
The initial entropy is given simply by (3.3.78), however the nal entropy (when H = 0) can be found by
setting H = 0 in (3.3.78):
S
f
=Nk
B
_
log
_
T
3
/2
_
+
3
2
_
(3.3.79)
Then using (3.3.78) and (3.3.79), we can now nd S,
S =S
f
S
i
=Nk
B
_
log
_
T
3
/2
_
+
3
2
_
Nk
B
_
log
_
cosh
_

B
H
k
B
T
_
T
3
/2
_
+
3
2


B
H
k
B
T
tanh
_

B
H
k
B
T
__
=Nk
B
_
log
_
T
3
/2
_
+
3
2
log
_
cosh
_

B
H
k
B
T
_
T
3
/2
_

3
2
+

B
H
k
B
T
tanh
_

B
H
k
B
T
__
=Nk
B
_

B
H
k
B
T
tanh
_

B
H
k
B
T
_
log
_
cosh
_

B
H
k
B
T
_ __
(3.3.80)
W. Erbsen HOMEWORK #4
Substituting (3.3.80) back into (3.3.74),
Q = Nk
B
T
_

B
H
k
B
T
tanh
_

B
H
k
B
T
_
log
_
cosh
_

B
H
k
B
T
_ __
3.4 Homework #4
Problem 1
a) Starting from the expression for the grand partition function, compute N) and N
2
). Now show that

2
N
= N
2
) N)
2
= k
B
T
N)

(3.4.1)
b) The above expression for
2
N
can be written in terms of the isothermal compressibility dened as: K
T
=
V
1
(V/P)
T
. To do this, rst show that = (G/N)
T,P
where G is the Gibbs free energy.
c) Then show that the Gibbs free energy is given by G = N, i.e. the chemical potential is the Gibbs free energy
per particle.
d) Now express d in terms of dP and dT and show that
_

N
_
V,T
=
V
2
N
2
_
P
V
_
T,N
(3.4.2)
e) Finally, show that

2
N
N
2
=
k
B
T
V

T
(3.4.3)
Solution
a) The Grand Partition Function, Q, is dened by
Q =

N,s
exp
_
N E
s,N
k
B
T
_
(3.4.4)
We also recall that in general, the average of a given quantity x can be dened by
x) =

i
x
i
P
i
Where P
i
is the probability distribution function. In our case, we are asked to nd N), and so
applying this principle to N,
CHAPTER 3: STATISTICAL MECHANICS 263
N) =

s,N
N
_
1
Q
exp
_
N E
s,N
k
B
T
__
=
1
Q

s,N
N exp
_
N E
s,N
k
B
T
_
(3.4.5)
Similarly, we can say for N
2
) that
N
2
) =
1
Q

s,N
N
2
exp
_
N E
s,N
k
B
T
_
(3.4.6)
We now wish to nd
2
N
the rst step is to take the partial derivative of Q with respect to the
chemical potential, :
Q

_
_
_

N,s
exp
_
N E
s,N
k
B
T
_
_
_
_
=

N,s

exp
_
N E
s,N
k
B
T
_
=

N,s
N
k
B
T
exp
_
N E
s,N
k
B
T
_
(3.4.7)
Moving the factor of k
B
T to the LHS of (3.4.7),
k
B
T
Q

N,s
N exp
_
N E
s,N
k
B
T
_
(3.4.8)
We now note that we can rewrite (3.4.5) using (3.4.8),
N) =
1
Q
k
B
T
Q

(3.4.9)
Similarly, we now wish to nd the second partial derivative of Q from (3.4.4), or equivalently, the
rst derivative of (3.4.9):

2
Q

2
=

2

2
_
_
_

N,s
exp
_
N E
s,N
k
B
T
_
_
_
_
=

_
_
_

N,s
N
k
B
T
exp
_
N E
s,N
k
B
T
_
_
_
_
=

N,s
N
k
B
T

exp
_
N E
s,N
k
B
T
_
=

N,s
N
2
(k
B
T)
2
exp
_
N E
s,N
k
B
T
_
(3.4.10)
Moving the thermal factor on the RHS of (3.4.10) to the LHS,
(k
B
T)
2

2
Q

2
=

N,s
N
2
exp
_
N E
s,N
k
B
T
_
(3.4.11)
W. Erbsen HOMEWORK #4
Using (3.4.11), we can rewrite (3.4.6),
N
2
) =
1
Q
(k
B
T)
2

2
Q

2
(3.4.12)
Substituting (3.4.9) and (3.4.12) into (3.4.1),

2
N
=N
2
) N)
2
=
1
Q
(k
B
T)
2

2
Q

2

_
1
Q
k
B
T
Q

_
2
=
1
Q
(k
B
T)
2

2
Q

2

1
Q
2
(k
B
T)
2
_
Q

_
2
=
k
B
T
Q
_
k
B
T

2
Q

2

1
Q
k
B
T
_
Q

_
2
_
=
k
B
T
Q
_
k
B
T

2
Q

2

1
Q
k
B
T
Q

. .

_
(3.4.13)
Noticing that the underbraced term in (3.4.13) is N) from (3.4.9),

2
N
=
k
B
T
Q
_
k
B
T

2
Q

2
N)
Q

_
=
k
B
T
Q
_

k
B
T
Q

. .
N)
Q

_
(3.4.14)
Solving (3.4.5) for the underbraced term in (3.4.14) and making the substitution,

2
N
=
k
B
T
Q
_

(QN)) N)
Q

_
(3.4.15)
We not notice that the rst partial derivative may be carried out to be

(QN)) = Q
N)

+N)
Q

(3.4.16)
Substituting (3.4.16) back into (3.4.15), we nd that

2
N
=
k
B
T
Q
_
Q
N)

+N)
Q

N)
Q

_

2
N
= k
B
T
N)

(3.4.17)
b) To show that we can write the chemical potential in terms of the Gibbs Free Energy as suggested
in the prompt, we rst recall that the Gibbs Free Energy is dened by (1.3.15) as
G = E TS + PV N (3.4.18)
Where the RHS can be used if the intensive parameters T, p, and remain constant while the
extensive parameters N, V , and E grow proportionately with one another (See Footnote 8 on pg.
28). Therefore, the chemical potential can be dened in terms of the Gibbs Free Energy as
=
_
G
N
_
T,P
(3.4.19)
CHAPTER 3: STATISTICAL MECHANICS 265
c) Recalling that the non-dierential analogue to (1.3.4) reads
E = TS PV + N (3.4.20)
Substituting (3.4.20) into the LHS of (3.4.18), we see that
G = (TS PV +N) TS + PV G = N (3.4.21)
d) Extending the Gibbs Free Energy to a multi-particle system, we can nd the dierential as
dG = dN +Nd (3.4.22)
We also note that the dierential of G from (3.4.18) is
dG = SdT + V dP + dN (3.4.23)
Equating (3.4.22) and (3.4.23), we nd that
dN +Nd = SdT + V dP +dN d =
S
N
dT +
V
N
dP (3.4.24)
Letting dT 0, (3.4.24) becomes
d =
V
N
dP

P
=
V
N
(3.4.25)
Applying the chain rule to (3.4.25),

P
P
V
V
P
=
V
N

V
V
P
=
V
N


V
=
V
N
P
V
(3.4.26)
Applying the chain rule once again to the LHS of (3.4.26),

V
V
N
N
V
=
V
N
P
V

N

N
V
=
V
N
P
V
(3.4.27)
We now note that from Maxwells relations, we can write
_
N
V
_
T,
=
_
P

_
T,V
(3.4.28)
Substituting (3.4.28) into (3.4.27),
_

N
_
T,V
_
P

_
T,V
=
V
N
_
P
V
_
T,N
(3.4.29)
From a previous homework set, we were asked to establish
N = V
_
P

_
T,V

_
P

_
T,V
=
N
V
(3.4.30)
Substituting (3.4.32) into (3.4.31),
_

N
_
T,V
N
V
=
V
N
_
P
V
_
T,N

_

N
_
T,V
=
V
2
N
2
_
P
V
_
T,N
(3.4.31)
W. Erbsen HOMEWORK #4
e) The rst thing we must do is recognize that the isothermal compressibility,
T
, can be quantied
as

T
=
1
V
_
V
P
_
T,N
(3.4.32)
For a system being isothermally compressed (both T and N constant), we note that we can rewrite
(3.4.31) as
_

N
_
T,V
=
V
2
N
2
_
P
V
_
T,N

_
N)

_
T,V
=
N
2
V
2
_
V
P
_
T,N
=
N
2
V
_

1
V
_
V
P
_
T,N
_
(3.4.33)
Substituting (3.4.32) into the RHS of (3.4.33),
_
N)

_
T,V
=
N
2
V

T
k
B
T
_
N)

_
T,V
= k
B
T
N
2
V

T
(3.4.34)
Substituting (3.4.1) into (3.4.34) and rearranging, we see that

2
N
= k
B
T
N
2
V

T


2
N
N
2
=
k
B
T
V

T
Which is the same as (3.4.3).
Problem 2
A surface with N adsorption sites is in equilibrium with a mixed ideal gas containing molecules of type A and B.
Each site can take either one single A molecules with energy
A
or a single B molecule with energy
B
. Com-
pute the grand canonical partition function for this adsorbed system. Then nd relations between the fractional
occupations (
A
= n
A
/N and
B
= n
B
/N) and the partial pressures p
A
and p
B
.
Solution
Recalling that the Grand Canonical Partition Function (i.e. the Gibbs Sum) is given by
Q =

N,s
exp
_

s
N E
s,N
k
B
T
_
(3.4.35)
In order to nd the PT for the adsorbed system, we assume that there are only two states available to
the system, one where molecule A is adsorbed and the other where molecule B is adsorbed. There is no
state where neither molecule is connected - this is deduced from the wording of the problem. We recall
that the Grand PT can be dened in terms of the canonical PT from (4.3.15) as
Q =

n
z
n
Q
n
(3.4.36)
CHAPTER 3: STATISTICAL MECHANICS 267
Where z is the fugacity (= e
s/kBT
) and Q
n
is the canonical PT. Following the example in class of a gas
adsorbed onto a lattice, , we note that the canonical PT with degeneracy is
Q
n
= g
n
exp
_

E
s,n
k
B
T
_
n
(3.4.37)
Where the degeneracy is
g
N
=
N(N 1)...(N n + 1)
n!

N!
(N n)!n!
(3.4.38)
Substituting (3.4.38) into (3.4.37),
Q
n
=
N!
(N n)!n!
exp
_

E
s,n
k
B
T
_
n
(3.4.39)
And now substituting (3.4.39) back into the Grand PT from (3.4.36),
Q =

n,s
exp
_

s
k
B
T
_

N!
(N n)!n!
exp
_

E
s,n
k
B
T
_
n
=

n,s
N!
(N n)!n!
exp
_
(
s
E
s,n
) n
k
B
T
_
(3.4.40)
Using the Binomial Expansion, (3.4.40) becomes,
Q =

s
_
1 + exp
_

s
E
s
k
B
T
__
N
(3.4.41)
In our case, since we have two possible energies, the sum is carried out to A and B and hence has only
two terms. We now see that (3.4.41) becomes
Q =
_
1 + exp
_
(
A
+
A
)
k
B
T
_
+ exp
_
(
B
+
B
)
k
B
T
__
N
(3.4.42)
We now recall from class that the thermodynamic potential, , is dened in terms of the Grand Canonical
PT as
= k
B
T log [Q] (3.4.43)
Substituting (3.4.36) into (3.4.43),
= k
B
T log
_

n
z
n
Q
n
_
k
B
T log [Q] (3.4.44)
We can now take the partial derivative of (3.4.44) with respect to , which yields

= z

z
log [Q] (3.4.45)
Noticing that the RHS of (3.4.45) is equal to n, or the average number of adsorbed molecules, we now
can say
W. Erbsen HOMEWORK #4
n =

(3.4.46)
Substituting the thermodynamic potential from (3.4.44) into (3.4.46),
n =

k
B
T log [Q] (3.4.47)
At this point, we remember that it is important to be very careful, as both n and are a function of the
specic states in the Gibbs sum. Rewriting (3.4.48) to make this more explicit,
n
s
= k
B
T

s
log [Q] (3.4.48)
And now substituting the grand canonical PT from (3.4.42) into (3.4.48),
n
s
= k
B
T

s
log
_
_
1 + exp
_
(
A
+
A
)
k
B
T
_
+ exp
_
(
B
+
B
)
k
B
T
__
N
_
= Nk
B
T

s
log
_
1 + exp
_
(
A
+
A
)
k
B
T
_
+ exp
_
(
B
+
B
)
k
B
T
__
(3.4.49)
We can go no further until we explicitly dene which state we are evaluating n
s
for. Starting with A,
(3.4.49) becomes
n
A
=Nk
B
T

A
log
_
1 + exp
_
(
A
+
A
)
k
B
T
_
+ exp
_
(
B
+
B
)
k
B
T
__
=
N

A
log [1 + exp [(
A
+
A
) ] + exp [(
B
+
B
) ]]
=
N

exp [(
A
+
A
) ]
1 + exp [(
A
+
A
) ] + exp [(
B
+
B
) ]
_
=N
exp [(
A
+
A
) ]
1 + exp [(
A
+
A
) ] + exp [(
B
+
B
) ]
(3.4.50)
The average number of adsorbed molecules per site is given by
s
= n
s
/N, and we are of course interested
in the fractional coverage of each type of molecule to each site. Accordingly,
A
can be found from (3.4.50)
to be

A
=
exp [(
A
+
A
) ]
1 + exp [(
A
+
A
) ] + exp [(
B
+
B
) ]
(3.4.51)
And similarly, for
B
,

B
=
exp [(
B
+
B
) ]
1 + exp [(
A
+
A
) ] + exp [(
B
+
B
) ]
(3.4.52)
The time has now come to nd the partial pressure p
A
and p
B
. The rst step is to isolate from
in either (3.4.51) or (3.4.52). Following the example from lecture, we rst wish to nd the chemical
potential of the surface (
surf
), and equate it with the chemical potential of the gas (
gas
). Starting with

surf
, we recognize that
A
=
B
, and also
A
=
B
, and so
CHAPTER 3: STATISTICAL MECHANICS 269
1

=
1 + exp [(
surf
+
A
) ] + exp [(
surf
+
B
) ]
exp [(
surf
+
A
) ]
=
1
exp [(
surf
+
A
) ]
+
exp [(
surf
+
A
) ]
exp [(
surf
+
A
) ]
+
exp [(
surf
+
B
) ]
exp [(
surf
+
A
) ]
=1 +
1
exp [(
surf
+
A
) ]
+ exp [(
B

A
) ] (3.4.53)
If we dene =
B

B
, then (3.4.53) becomes
exp [(
surf
+
A
) ] =
1

1 exp []
surf
=
1

log
_
1

1 exp []
_

A
(3.4.54)
From class, we know that the chemical potential of an ideal gas is given, in general by

gas
=
1

log [P f(T)] (3.4.55)


Equating (3.4.54) and (3.4.56),

log
_
1

1 exp []
_

A
=
1

log [P f(t)]
log
_
1

1 exp []
_
+
A
=log
_
1
P f(t)
_
_
1

1 exp []
_
exp [
A
] =
1
P f(t)
(3.4.56)
Recalling that =
B

A
, (3.4.56) becomes
_
1

_
exp [
A
] exp [
B
] =
1
P f(t)
P =
1
f(T)
_

1
_
1
exp [
A
] exp [
B
]
(3.4.57)
If we assume that the total gas mixture is given by P from (3.4.57), then the partial pressure is dened
by p
i
= n
i
P. To nd n
A
, we follow this logic and from (3.4.50) and (3.4.57), we can nd p
A
:
p
A
= N
exp [(
A
+
A
) ]
1 + exp [(
A
+
A
) ] + exp [(
B
+
B
) ]

1
f(T)
_

1
_
1
exp [
A
] exp [
B
]
(3.4.58)
And similarly, we can say that the partial pressure of molecule B is given by
p
B
= N
exp [(
B
+
B
) ]
1 + exp [(
A
+
A
) ] + exp [(
B
+
B
) ]

1
f(T)
_

1
_
1
exp [
A
] exp [
B
]
(3.4.59)
It might be possible to reduce (3.4.58)(3.4.59) to a slightly nicer form, however it will likely still be
quite ugly.
Problem 3
W. Erbsen HOMEWORK #4
Suppose that N molecules of H
2
O gas (assumed to behave like a classical ideal gas) are introduces into a container
of xed volume V at a temperature low enough that virtually all the gas remains as molecular water vapor. At
higher temperatures dissociation can take place according to the reaction
2H
2
O 2H
2
+ O
2
(3.4.60)
Let x be the fraction of H
2
O molecules dissociated at some temperature T corresponding to the total gas pressure
P. Write an equation relating x, P, and F(t) where F(t) is some function of temperature (that can include the
binding energy of the water molecule).
Solution
The dynamic reaction in (3.4.60) can be rewritten as
2H
2
O 2H
2
O
2
= 0 (3.4.61)
We can express (3.4.61) in terms of coecients and species according to

i
a
i
A
i
, and in our case the
coecients are
A
1
= H
2
, A
2
= O
2
, A
3
= H
2
O
a
1
= 2, a
2
= 1, a
3
= 2
We also recall that the condition for chemical equilibrium can be dened in terms of the coecients a
i
as

i
a
i

i
= 0 (3.4.62)
Where this condition is met at the equilibrium pressure and temperature. We can express (3.4.61) in
the language of (3.4.62) as
2(H
2
O) 2(H
2
) (O
2
) = 0 (3.4.63)
The chemical potential for each species,
i
, can be described by (1.5.7), which reads

i
(N
i
, V, T) = k
B
T log
_
N
i
V
_
h
2
2m
i
k
B
T
_
3
/2
_
(3.4.64)
We now dene the fractional occupancy N
i
, which we quantify as N

i
= N
i
/N. If we assume that the
gas phase of each species can be treated as ideal, then we can write PV = Nk
B
T N/V = P/ (k
B
T),
and we can rewrite (3.4.64) as

i
(N
i
, V, T) =k
B
T log
_
P
k
B
T
N

i
_
h
2
2m
i
k
B
T
_
3
/2
_
=k
B
T log
_
P
k
B
T
_
h
2
2m
i
k
B
T
_
3
/2
_
+k
B
T log [N

i
] (3.4.65)
Using (3.4.62), we may rewrite (3.4.65) as
CHAPTER 3: STATISTICAL MECHANICS 271
k
B
T

i
a
i
_
log
_
P
k
B
T
_
h
2
2m
i
k
B
T
_
3
/2
_
+ log [N

i
]
_
= 0 (3.4.66)
Killing the factor in front of the sum and rearranging, (3.4.66) becomes

i
a
i
log
_
P
k
B
T
_
h
2
2m
i
k
B
T
_
3
/2
_
=

i
a
i
log [N

i
]

i
log
_
k
B
T
P
_
2m
i
k
B
T
h
2
_3
/2
_
ai
=

i
log [N

i
]
ai

i
log
_
V
N
i
_
2m
i
k
B
T
h
2
_3
/2
_
ai
=

i
log [N

i
]
ai

i
log
_
q
i
N
i
_
ai
=

i
log [N

i
]
ai
(3.4.67)
From lecture, we know that
N
a1
1
N
a2
2
N
a3
3
... = q
a1
1
q
a2
2
q
a3
3
... K(T, V ) (3.4.68)
And, simply by visual inspection of (3.4.67), we can see that K(T, V ) = q
ai
i
makes sense, recognizing
that at equilibrium

i
K
i
(T, V ) = 1 so the equation reduces readily to 0 = 0. We now recall that we
can alternatively dene the equilibrium constant in terms of (3.4.60) as
K(T, V ) =
[H
2
O]
2
[H
2
]
2
[O
2
]

N (H
2
O)
2
N (H
2
)
2
N (O
2
)
(3.4.69)
We also recall from lecture that

i
_
N
i
q
i
_
ai
= 1 (3.4.70)
Applying (3.4.70) to the case of (3.4.60),
_
N
H2O
q
H
2
O
_
2

_
N
H2
q
H
2
_
2

_
N
O2
q
O
2
_
1
=1 (3.4.71)
The total number of species at any time, N

, can be quantied by
N

= N
H2O
+N
H2
+N
O2
(3.4.72)
We are told to let x denote the fraction of water molecules dissociated after some period of time, and also
that initially, the total number of free particulates are entirely water molecules, so that N = N
H2O
.
Accordingly, we see that in my notation x = 1 N
H2O
/N. With this, (3.4.72) becomes
N

= N (1 x) + N
H
2
+N
O
2
(3.4.73)
Following the same logic, we see that N
H2
grows proportionally with N
H2O
, whereas N
O2
grows half -
proportionally with respect to N
H
2
O
(according to the stoichiometric coecients from (3.4.60)). We see
that (3.4.73) becomes
W. Erbsen HOMEWORK #4
N

= N (1 x) + xN +x
N
2
1 =
N
N

(1 x) +
N
N

x +
N
N

x
2
(3.4.74)
Substituting the appropriate fractional coecients into (3.4.71),
_
N (1 x)
q
H2O
_
2
I

_
xN
q
H2
_
2
II

_
1
/
2
xN
q
O2
_
1
III
=1 (3.4.75)
We now recall that q
i
is dened by
q
i
=
p
i
k
B
T

P
p
i

p
i
P
_
h
2
2m
i
k
B
T
_
3
/2
=
P
k
B
T
N
i
N
_
h
2
2m
i
k
B
T
_
3
/2
=
P
m
3
/2
i
N
i
N
_
h
2
2 (k
B
T)
3
_3
/2
. .
F(T)
q
i
=
N
i
N

P
m
3
/2
i
F(T) (3.4.76)
In the language of (3.4.76), we can rewrite each of the terms in (3.4.75). Starting with I,
I =
_
N (1 x)
q
H
2
O
_
2
=
_
_
_
(1 x)
m
3
/2
H2O
F(T)P
_
_
_
2
=(1 x)
2

m
3
H2O
[F(T)P]
2
(3.4.77a)
II =
_
xN
q
H2
_
2
=
_
_
_
x
m
3
/2
H2
F(T)P
_
_
_
2
=
_
x
2

m
3
H2
[F(T)P]
2
_
1
=
1
x
2

[F(T)P]
2
m
3
H2
(3.4.77b)
III =
_
1
/
2
xN
q
O2
_
1
=
_
_
_
x
2

m
3
/2
O2
F(T)P
_
_
_
1
CHAPTER 3: STATISTICAL MECHANICS 273
=
2
x

F(T)P
m
3
/2
O2
(3.4.77c)
Substituting (3.4.77a)(3.4.77c) back into (3.4.75),
_
(1 x)
2

m
3
H2O
[F(T)P]
2
_

_
1
x
2

[F(T)P]
2
m
3
H2
__
2
x

F(T)P
m
3
/2
O2
_
=1
(1 x)
2
x
3
2
_
m
2
H2O
m
2
H2
m
O
2
_
3
/2
F(T)P =1 (3.4.78)
Rearranging (3.4.78), we are left with
F(T)P =
x
3
2 (1 x)
2
_
m
2
H2
m
O
2
m
2
H2O
_
3
/2
Problem 4
Evaluate the density matrix
mn
of an electron spin in the representation which makes
x
diagonal. Next, show
that the value of
z
), resulting from this representation, is precisely the same as the one obtained in 5.3. Hint :
the representation needed here follows from the one used in 5.3 by carrying out a transformation with the help
of the unitary operator

U =
_
1/

2 1/

2
1/

2 1/

2
_
(3.4.79)
Solution
The Pauli spin matrices are given by

x
=
_
0 1
1 0
_
,
y
=
_
0 i
i 0
_
,
z
=
_
1 0
0 1
_
We now note that we can straightforwardly rewrite our unitary operator from (3.4.79) as

U =
1

2
_
1 1
1 1
_
(3.4.80)
We also recognize that by using this matrix, we may transform our spin matrices to a basis in which
x
is diagonal via a unitary transformation:

x
=

U

U (3.4.81)
Where

x
denotes
x
in the basis in which it is diagonal. Substituting (3.4.80) into (3.4.81),
W. Erbsen HOMEWORK #4

x
=
1

2
_
1 1
1 1
_

_
0 1
1 0
_
1

2
_
1 1
1 1
_
=
1
2
_
1 1
1 1
__
0 1
1 0
__
1 1
1 1
_
=
1
2
_
1 1
1 1
__
1 1
1 1
_
=
1
2
_
2 0
0 2
_
=
_
1 0
0 1
_
(3.4.82)
So, the provided matrix does indeed diagonalize
x
. We must now see how it aects
z
,

z
=
1

2
_
1 1
1 1
_

_
1 0
0 1
_
1

2
_
1 1
1 1
_
=
1
2
_
1 1
1 1
__
1 0
0 1
__
1 1
1 1
_
=
1
2
_
1 1
1 1
__
1 1
1 1
_
=
1
2
_
0 2
2 0
_
=
_
0 1
1 0
_
(3.4.83)
The Hamiltonian operator,

H, is given by (5.3.1),

H =
B
B

z
(3.4.84)
Which includes our new Pauli Matrix

z
in the basis in which
x
is diagonal. Substituting (3.4.83) into
(3.4.84),

H =
B
B
_
0 1
1 0
_
(3.4.85)
We now recall the expression for the density matrix in the canonical ensemble is given by (5.3.3) as
( ) =
_
e


H
_
Tr
_
e


H
_
=
1
e
BB
+e
BB
_
e
BB
0
0 e
BB
_
(3.4.86)
But this is not the answer. We must now express this density matrix in our new basis, in the same way
we did with our Pauli spin matrices before. Performing the unitary transformation on (3.4.86),
CHAPTER 3: STATISTICAL MECHANICS 275
(

) =
1

2
_
1 1
1 1
_

_
1
e
BB
+ e
BB
_
e
BB
0
0 e
BB
__
1

2
_
1 1
1 1
_
=
1
2
1
e
BB
+ e
BB
_
1 1
1 1
__
e
BB
0
0 e
BB
__
1 1
1 1
_
=
1
2
1
e
BB
+ e
BB
_
e
BB
e
BB
e
BB
e
BB
__
1 1
1 1
_
=
1
e
BB
+e
BB
_
e
BB
+ e
BB
e
BB
e
BB
e
BB
e
BB
e
BB
+ e
BB
_
=
1
2 cosh [
B
B]
_
cosh [
B
B] sinh [
B
B]
sinh [
B
B] cosh [
B
B]
_
(3.4.87)
We may now absorb the cosh term out front of (3.4.87) to inside the matrix, leaving us with
(

) =
1
2
_
1 tanh[
B
B]
tanh [
B
B] 1
_
(3.4.88)
We can nd the expectation value of

x
by generalizing the form of (5.3.4),

x
) = Tr [

] (3.4.89)
Substituting

z
from (3.4.83) and

from (3.4.88) into (3.4.89),

x
) =Tr
_
1
2
_
1 tanh [
B
B]
tanh [
B
B] 1
__
0 1
1 0
__
=Tr
_
1
2
_
tanh [
B
B] 1
1 tanh [
B
B]
__
(3.4.90)
Recalling that the trace of a matrix is simply the sum of the diagonal elements, (3.4.90) becomes

x
) = tanh [
B
B]
Which is the same as found in 5.3.
3.5 Homework #7
Problem 1
Extension of the Debye Model of a Solid: Assuming the dispersion relation = Ak
s
, where is the angular
frequency and k is the wave-number of a vibrational mode existing in a solid, show that the respective contribu-
tion towards the specic heat of the solid at low temperature is proportional to T
3
/s
.
W. Erbsen HOMEWORK #7
Solution
This problem is essentially the same as if we were trying to nd the specic heat of a Debye solid at
low temperatures, the only dierence is that the dispersion relation has an extra term s in the exponent
of the wave-number: = Ak
s
. We can carry through the arithmatic in very much the same way then,
following Pathrias work in 7.2 and 7.3.
We rst recall that the specic heat in terms of internal energy is given by
C
V
(T) =
_
U
T
_
V
(3.1.1)
So our goal then, is to nd the internal energy, U, as a function of our dispersion relation. We recall
Pathrias denition of the Debye function from (7.3.18):
D(x
0
) =
3
x
3
0
_
x0
0
x
3
e
x
1
dx (3.1.2)
Where we have dened x
0
to be
x
0
=

D
k
B
T
(3.1.3)
And substituting (3.1.3) back into (3.1.2),
D(x
0
) = 3
_
k
B
T

D
_
3
_
x0
0
x
3
e
x
1
dx (3.1.4)
We also realize that, to make things more transparent, Pathria has made the substitution x =
E
/k
B
T.
Recalling our denition of x
0
from (3.1.3), we note that in the low temperature limit, as suggested in
the prompt, we see that x
0
1/T , and so (3.1.4) becomes
D(x
0
) =3
_
k
B
T

D
_
3
_

0
x
3
e
x
1
dx
=3
_
k
B
T

D
_
3
_

4
15
_
(3.1.5)
The only dierence in our case is that we make the following change to (3.1.3),
x
0
=
_

D
k
B
T
_
s
(3.1.6)
Which we can see makes no dierence in the integrand of (3.1.5). Therefore, the result from (3.1.5) using
(3.1.6) instead of (3.1.3) is
D(x
0
) =3
_
k
B
T

D
_3
/s
_

4
15
_
(3.1.7)
From (3.1.7) we can nd the specic heat at low temperature by using the following approximation:
C
V
(T) = 3Nk
B
E(x) (3.1.8)
CHAPTER 3: STATISTICAL MECHANICS 277
Which is (7.3.9) from Pathria. This is actually what determined the factor in our integral, which was
solved in (3.1.5). So, our specic heat is then:
C
V
(T) = D(x
0
) =9N
_
k
B
T

D
_3
/s
_

4
15
_
C
V
T
3
/s
Problem 2
Stability of a white dwarf against gravitational collapse: Consider a star to be made up of an approximately
equal number N of electrons and protons (otherwise the Coulomb repulsion would overcome the gravitational in-
teraction). Somewhat arbitrarily we would also assume that there is an equal number of neutrons and protons.
Now, it is energetically favorable for a body held together by gravitational forces to be as compact as possible. On
Earth, the gravitational force is not large enough to overcome the repulsive forces between atoms and molecules.
Inside the sun, matter does not exist in the form of atoms and molecules, where the radiation pressure from the
nuclear fusion keeps the star from collapsing. We will consider the case of a white dwarf, where the fusion has
completed and there is no longer radiation pressure to prevent collapse. Assume that the temperature of the
star is low enough, compared to the electron Fermi temperature, that the electrons can be approximated by a
T = 0 Fermi gas (because of their large mass, the kinetic energy of protons and neutrons are assumed to be small
compared to that of the electrons).
a) If the electron gas is non-relativistic, show that the electron kinetic energy (KE) of the star is given by
E
kin
=
3
2
10m
e
_
9
4
_2
/3
N
5
/3
R
2
(3.1.9)
where m
e
is the electron mass and R is the radius of the star.
b) The gravitational potential energy is dominated by neutrons and protons. Let m
N
be the nucleon mass.
Assume the mass density is approximately constant inside the star. Show that, if there is an equal number
N of protons and neutrons, then the gravitational potential energy is given by
E
pot
=
12
5
m
2
N
G
N
2
R
(3.1.10)
where G = 6.67 10
11
Nm
2
/kg
2
is the gravitational constant.
c) Find the radius for which the gravitational potential energy plus the kinetic energy is a minimum for a white
dwarf with the same mass as the sun (1.99 10
30
Kg) in units of solar radii (6.96 10
8
m).
d) If the density is very large, the Fermi velocity of the electrons becomes comparable to the speed of light and
we should use the relativistic formula for the relationship between energy and momentum. Compute the
electron kinetic energy in the ultra relativistic limit cp.
e) You have just shown in d) that in the ultra relativistic limit, the electron kinetic energy is proportional to
N
4
/3
/R, i.e. the R dependence is the same as the gravitational potential energy computed in part b) (which
remains unchanged). Since for large N, N
2
N
4
/3
, we nd that if the mass of the star is large enough, the
gravitational potential energy will dominate. The star will then collapse. Show taht the critical value of N
for this to happen is
W. Erbsen HOMEWORK #7
N
crit
=
_
5c
36m
2
N
G
_3
/2
_
9
4
_
(3.1.11)
Substituting numbers, one nd that this corresponds to about 1.71 solar masses. This is the so-called
Chandrasekhar limit.
Solution
a) We recall that the total energy of a degenerate electron gas is given by
E
deg
=
3
5
NE
F
(3.1.12)
Where the Fermi Energy, E
F
, is given by
E
F
=

2
2m
e
_
3
2
N
V
_2
/3
(3.1.13)
Substituting (3.1.13) into (3.1.12),
E
deg
=
3
5
N
_

2
2m
e
_
3
2
N
V
_2
/3
_
=
3
2
10m
e
N
_
3
2
N
V
_2
/3
=
3
2
10m
e
_
3
2
_2
/3

1
V
2
/3
N
5
/3
(3.1.14)
Recalling that V =
4
/
3
R
3
, (3.1.14) becomes
E
deg
=
3
2
10m
e
_
3
2
_
2
/3

_
3
4R
3
_2
/3
N
5
/3
=
3
2
10m
e
_
9
4R
3
_2
/3
N
5
/3
(3.1.15)
We note that (3.1.15) may be straightforwardly rewritten as
E
deg
=
3
2
10m
e
_
9
4
_2
/3
N
5
/3
R
2
(3.1.16)
b) To nd the gravitational potential energy, we assume the star to be a perfect sphere of mass density
, where we dene = m/V m = V . Also recalling the volume of a sphere, we nd that
m =
4
3
r
3
(3.1.17)
The dierential element of (3.1.17) is of course
dm = 4r
2
dr (3.1.18)
We also recall that the potential energy of a celestial body is dened by
CHAPTER 3: STATISTICAL MECHANICS 279
dU =
Gm
r
dm (3.1.19)
We can nd the gravitational potential energy by integrating (3.1.19), and substituting in to this
(3.1.17) and (3.1.18) we have
E
grav
=
_
R
0
Gm
r
dm
= G
_
R
0
1
r
_
4r
3

3
_
_
4r
2

_
dr
=
16
2
G
2
3
_
R
0
r
4
dr
=
16
2
G
2
3
_
R
5
5
_
=
16
2
G
2
15
R
5
(3.1.20)
Substituting in the density in terms of the total mass, (3.1.21) becomes
E
grav
=
16
2
G
15
_
M
4
/
3
R
3
_
2
R
5
=
16
2
G
15
_
9M
2
16
2
R
6
_
R
5
=
3
5
GM
2
R
(3.1.21)
If we want to express this in terms of the nucleon mass, we note that M = 2m
N
N, and so from
(3.1.21) we have
E
grav
=
3
5
G(2m
N
N)
2
R
E
grav
=
12
5
Gm
2
N
N
2
R
(3.1.22)
c) In order to nd the equilibrium radius, R
e
, we must minimize the total energy of the star, which is
the sum of the results from the previous two parts of this problem:
E
tot
= E
deg
+E
grav

3
2
10m
e
_
9
4
_2
/3
N
5
/3
R
2

12
5
Gm
2
N
N
2
R
(3.1.23)
Minimizing (3.1.23) with respect to R,
E
tot
R
=

R
_
3
2
10m
e
_
9
4
_2
/3
N
5
/3
R
2

12
5
Gm
2
N
N
2
R
_
=
3
2
10m
e
_
9
4
_2
/3
N
5
/3

R
_
1
R
2
_

12
5
Gm
2
N
N
2

R
_
1
R
_
=
3
2
10m
e
_
9
4
_2
/3
N
5
/3
_

2
R
3
_

12
5
Gm
2
N
N
2
_

1
R
2
_
=
3
2
5m
e
_
9
4
_2
/3
N
5
/3
1
R
3
+
12
5
Gm
2
N
N
2
1
R
2
(3.1.24)
W. Erbsen HOMEWORK #7
To minimize (3.1.24), we set the derivative with respect to R equal to zero and simplify:
12
5
Gm
2
N
N
2
1
R
2
=
3
2
5m
e
_
9
4
_2
/3
N
5
/3
1
R
3
R =
1
4
_
9
4
_2
/3

2
Gm
e
m
2
N
1
N
1
/3
(3.1.25)
We now wish to eliminate N, where we use our previous denition of the total mass M to declare
that M = 2m
N
N N = M/(2m
N
), and (3.1.25) becomes
R =
1
4
_
9
4
_2
/3

2
Gm
e
m
2
N
_
2m
N
M
_1
/3

(2)
1
/3
4
_
9
4
_2
/3

2
Gm
e
m
5
/2
N
M
1
/3
(3.1.26)
Substituting in the appropriate values into (3.1.26),
R
e
=
(2)
1
/3
4
_
9
4
_2
/3
_
1.055 10
34
J s
_
2
_
1.990 10
30
Kg
_1
/3
_
1.674 10
11
m
3
Kg
1
s
2
_
(9.109 10
31
Kg) (1.673 10
27
Kg)
5
/2
=7.162 10
6
m (3.1.27)
We can re-express (3.1.27) in terms of solar radii as
R
e
R
=
7.162 10
6
m
6.960 10
8
m

R
e
R
= 1.029 10
2
d) In the relativistic limit, we let cp, and we can use the density of states formula from (7.2.16):
a() d =
8V
h
3
c
3

2
d (3.1.28)
Recalling that h = 2, (3.1.28) becomes
a() d =
V

3
c
3

2
d (3.1.29)
We can integrate the density of states from (3.1.29) to nd the number of particles N:
N =
_
F
0
a() d
=
_
F
0
V

3
c
3

2
d
=
V

3
c
3

3
F
3
(3.1.30)
We can now substitute in to (3.1.30) the equation for volume,
N =
1

3
c
3

3
F
3

4
3
R
3

4R
3
9
3
c
3

3
F
(3.1.31)
Similarly, we can integrate the density of states from (3.1.29) to nd the internal energy:
U =
_
F
0
a() d
=
_
F
0
V

3
c
3

3
d
=
V

3
c
3

4
F
4
(3.1.32)
CHAPTER 3: STATISTICAL MECHANICS 281
Substituting in our value for V into (3.1.32),
U =
1

3
c
3

4
F
4
4
3
R
3

R
3
3
3
c
3

4
F
(3.1.33)
We can now solve (3.1.31) for
F
, which we will later substitute into (3.1.33). We nd that:
N =
1

3
c
3

3
F
3

4
3
R
3

4R
3
9
3
c
3

3
F

F
=
_
9
4
N
_1
/3
c
R
(3.1.34)
Substituting (3.1.34) into (3.1.33),
U =
R
3
3
3
c
3
_
_
9
4
N
_1
/3
c
R
_
4

R
3
3
3
c
3
_
9
4
_4
/3
N
4
/3
(c)
4
R
4
(3.1.35)
From (3.1.35) it is easy to see that the energy is then
U =
c
3
_
9
4
_4
/3
N
4
/3
R
(3.1.36)
e) To nd the critical value of N, we evaluate (3.1.36) set equal to (3.1.22):
c
3
_
9
4
_4
/3
N
4
/3
c
R
=
12
5
Gm
2
N
N
2
c
R
N
c
=
_
5c
36
_
9
4
_4
/3
1
m
2
N
G
_
3
/2
(3.1.37)
From (3.1.37) we can simplify our expression for N
c
and we arrive at
N
c
=
_
5c
36m
2
N
G
_3
/2
_
9
4
_
2
Problem 3
Let the Fermi distribution at low temperatures be represented by a straight line (shown in Fig. 8.11) that is
tangent to the actual curve at = . Show that this approximate representation yields a correct result for the
low-temperature specic heat of a Fermi gas, except that this numerical factor turns out to be smaller by a factor
of 4/
2
. Discuss, in a qualitative manner, the origin of the numerical discrepancy.
Solution
First of all, I dont really understand this problem. From Pathria, the low temperature specic heat of
a gas is given from (8.1.39) as
C
V
Nk
B
=

2
2
k
B
T
E
F
+ ... (3.1.38)
Recalling that the Fermi Energy can be dened in terms of the Fermi Temperature as T
F
= E
F
/k
B
,
(3.1.38) may be rewritten as
W. Erbsen HOMEWORK #7
C
V
Nk
B
=

2
2
T
T
F
+... (3.1.39)
While the classical value is just given by
C
V
=
3
2
NK
B

C
V
Nk
B
=
3
2
(3.1.40)
Dividing (3.1.38) by (3.1.40) we nd that
C
F-D
V
C
Class
V
=
_

2
2
T
T
F
_

_
3
2
_
1


2
3
T
T
F
(3.1.41)
The factor relating the classical to the quantum result for C
V
at low temperature seems is
2
/3 . The
reason why the quantum factor is larger than the classical factor has to do with the behavior of gases
as T 0. In the classical case, the barrier is assumed to be a step function, yielding the factor of
3/2. In the quantum case, things are not so easy we must integrate the energy from the Fermi-Dirac
distribution function, which yields a numerical result which ends up being attributed to the factor of

2
/2 out front.
Chapter 4
Mathematical Methods
4.1 Homework #3
Problem 1
Determine if the following series converge:
a)

n=10
1
n(lnn)(ln(lnn))
(4.1.1a)
b)

n=2
ln
_
[ n]lnn (4.1.1b)
c)

n=0
4
n
+ 1
3
n
1
(4.1.1c)
d)

n=1
(n!)
2
(n
2
)!
(4.1.1d)
e)

n=0
(log

2)
n
(4.1.1e)
Solution
a) Since a
n+1
a
n
it is to our advantage to use the integral test to see if (4.1.1a) converges. Graph-
ically this just means that if we plot (4.1.1a) in terms of n we will get a curve that has negative
instantaneous slopes at all (nite) points. Rewriting (4.1.1a),

n=10
1
n(ln n)(ln(lnn))

_

10
1
x(lnx)(ln(lnx))
dx
_
u = lnx, du = 1/x dx
=
_

ln 10
1
u lnu
du
_
v = lnu, dv = 1/u du
=
_

ln ln 10
1
v
dv
W. Erbsen HOMEWORK #3
=[ln v[

ln ln 10
(4.1.2)
From (4.1.2) we can see that since v diverges, so does u and so does x, so from the integral test it
is clear that (4.1.1a) diverges .
b) This problem begs to be solved by way of the comparison test, however a more straightforward,
qualitative explanation looms overhead. We know that lnn where n 2 is always a positive
number, so all we must do is analyze whats left of (4.1.1b) and if that diverges then so does the
entire function. We can see that
_
[ n]lnn diverges since for signicant values of n the square root
overcomes the enveloping ln function. Therefore, since
_
[ n]lnn diverges , so too must (4.1.1b).
c) This is a fairly straightforward; recall that if
lim
n
a
n
,= 0, then

n=0
a
n
diverges. (4.1.3)
In our case,
lim
n
4
n
+ 1
3
n
1
lim
n
4
n
3
n
= lim
n
_
4
3
_
n
(4.1.4)
So that (4.1.1c) diverges .
d) This can be analyzed using the ratio test. Letting a
n
= (n!)
2
/(n
2
)! and a
n+1
= ((n+1)!)
2
/((n+1)
2
)!,
we have
a
n+1
a
n
=
(n + 1)!(n + 1)!
[(n + 1)(n + 1)]!
[n n]!
n! n!
=
(n + 1)(n + 1)[n n]!
[(n + 1)(n + 1)]!
=
(n + 1)
2
(n + 1)
2
= 1 (4.1.5)
The fact that (4.1.5) equals 1 signies that the ratio test failed; we must nd another way to test for
convergence. A method that is not immediately apparent would be the integral test. The behavior
of (4.1.1d) is complex near the origin, but eventually the denominator wins, and it dominates the
behavior of the function. Subsequently, this implies that a
n+1
a
n
and we can therefore use the
integral test.

n=1
(n!)
2
(n
2
)!

_

1
(n!)
2
(n
2
)!
dx
0.685292 (< 1) (4.1.6)
Which was computed numerically using Mathematica. Since (4.1.6) is less than 1 then we can say
that (4.1.1d) converges .
e) Here we can straightforwardly apply the ratio test, where a
n
= (log

2)
n
and a
n+1
= (log

2)
n+1
.
So,
a
n+1
a
n
=
(log

2)
n+1
(log

2)
n
CHAPTER 4: MATHEMATICAL METHODS 285
=log

2
0.605512 (< 1) (4.1.7)
From (4.1.7) we can see that since a
n+1
/a
n
< 1 then we know that (4.1.1e) converges .
Problem 2
Find the circle of convergence of the following series:
a)

n=0
nz
n
(4.1.8a)
b)

n=0
n!
n
n
z
n
(4.1.8b)
c)

n=0
(z + 5i)
2n
(n + 1)
2
(4.1.8c)
Solution
a) To nd the radius of convergence, we rst must use a test to see for what values (4.1.8a) is convergent.
In this case we should use the ratio test, where a
n
= nz
n
and a
n+1
= (n + 1)z
n+1
:
lim
n

a
n+1
a
n

= lim
n

(n + 1)z
n+1
nz
n

= lim
n

(n + 1)z
n

=[z[ lim
n

n + 1
n

=[z[ lim
n

1 +
1
n

=[z[ (4.1.9)
Since we are looking for the radius of convergence we require (4.1.9) to be a positive number less
than 1, so that we have 0 [z[ 1 .
b) Again, we need to go ahead and implement the ratio in order to attack this problem. Let a
n
=
n!z
n
/n
n
and a
n+1
= (n + 1)!z
n+1
/(n + 1)
n+1
. Then
lim
n

a
n+1
a
n

= lim
n

(n + 1)!z
n+1
(n + 1)
n+1
n
n
n!z
n

= lim
n

(n + 1)n
n
z
(n + 1)
n+1

=[z[ lim
n

n
n
(n + 1)
n

W. Erbsen HOMEWORK #3
=[z[ lim
n

_
n
n + 1
_
2

=[z[ lim
n

_
1
1 + 1/n
_
2

=[z[
1
e
(4.1.10)
The last step of (4.1.10) was achieved by noticing that the limit was nothing more than the inverse
of Eulers famous limit denition of e. Analogous to what we did in the last problem, to determine
the radius of convergence you say that (4.1.10) is between 0 and 1, such that 0 [z[/e 1
0 [z[ e .
c) We can nd the radius of convergence for (4.1.8c) by way of the ratio test. As usual, go ahead and
let a
n
= (z + 5i)
2n
(n + 1)
2
and a
n+1
= (z + 5i)
2(n+1)
(n + 2)
2
. Then
lim
n

a
n+1
a
n

= lim
n

(z + 5i)
2(n+1)
(n + 2)
2
(z + 5i)
2n
(n + 1)
2

= lim
n

(z + 5i)
2
(n + 2)
2
(n + 1)
2

=[z + 5i[
2
lim
n

(n + 2)
2
(n + 1)
2

(4.1.11)
To nd the limit in (4.1.11) we need to use the comparison test. We know that
[z + 5i[
2
lim
n

(n + 2)
2
(n + 1)
2

[z + 5i[
2
lim
n

(n + 2)
2
(n)
2

=[z + 5i[
2
lim
n

_
(n + 2)
(n)
_
2

=[z + 5i[
2
lim
n

_
1 + 1/n
1
_
2

. .
1
=[z + 5i[
2
(4.1.12)
And, as before, for convergence we require that (4.1.12) be between 0 and 1, so 0 [z + 5i[
2
1
0 [z + 5i[ 1 .
Problem 3
Sum the following series:
a)

n=0
(n + 1)(n + 2)
2
n
(4.1.13a)
CHAPTER 4: MATHEMATICAL METHODS 287
b)

n=0
(1)
n
n!
(4.1.13b)
c)

n=0
()
n
(2n)!
(4.1.13c)
Solution
a) The easiest way by far to evaluate this sum is to write out some of the terms to see what value it
approaches. To make our job easier, lets rewrite (4.1.13a):

n=0
(n + 1)(n + 2)
2
n
=

n=0
n
2
+ 3n + 2
2
n
=

n=0
n
2
2
n
. .

1
+3

n=0
n
2
n
. .

2
+2

n=0
1
2
n
. .

3
(4.1.14)
Where I have dened
1
,
2
and
3
to be:

1
=

n=0
n
2
2
n
= 0 +
1
2
+ 1 +
9
8
+ 1 +
25
32
+
9
16
+
49
128
+
1
4
+
81
512
+
25
256
5.85742

2
=

n=0
n
2
n
= 0 +
1
2
+
1
2
+
3
8
+
1
4
+
5
32
+
3
32
+
7
128
+
1
132
+
9
512
+
5
512
1.98828

3
=

n=0
1
2
n
= 0 + 1 +
1
2
+
1
4
+
1
8
+
1
16
+
1
32
+
1
64
+
1
128
+
1
256
+
1
512
+
1
1024
1.99902
So that we now have

n=0
(n + 1)(n + 2)
2
n

1
+ 3
2
+ 2
3
= 5.85742 + 3 1.98828 + 2 1.99902 = 15.8203 16
b) This is rather straightforward to solve if we recognize that (4.1.13b) is similar in form to the series
expansion of e
x
,
e
x
= 1 +x +
x
2
2!
+
x
3
3!
+ ... +
x
n
n!
=

n=0
x
n
n!
(4.1.15)
Where at this point it should be noticed that x in (4.1.15) is equal to 1 in (4.1.13b). Making this
substitution into (4.1.15) we are left with,

n=0
(1)
n
n!
= e
1

1
e
(4.1.16)
c) We approach this problem in the same way we did for the last one; look at a table of Taylor
expansions for common functions and see if we can use one to solve (4.1.13c). In this case, we are
interested in the expansion of cos x:
cos x = 1
x
2
2!
+
x
4
4!

x
6
6!
+ ... +
(1)
n
(2n)!
=

n=0
(1)
n
(2n)!
x
2n
(4.1.17)
W. Erbsen HOMEWORK #4
Recognizing that x in (4.1.17) is

in (4.1.13c) and substituting it into the right side of (4.1.17),

n=0
(1)
n
(2n)!
(

)
2n
=

n=0
(1)
n
()
n
(2n)!
=

n=0
()
n
(2n)!
= cos

(4.1.18)
4.2 Homework #4
Problem 1
Evaluate the integral:
_
CR
cot (z)
z
4
dz (4.2.1)
Where C is a circle of large radius R centered at z = 0, and prove that (4) =

4
90
.
Hint 1: As R approaches innity, the integral approaches zero.
Hint 2: Series expansion of the cotangent function near zero is cot (x) =
1
x

x
3

x
3
45
+ ...
Solution
The rst step is to substitute the series expansion given in the 2nd hint into (4.2.1):
_
CR
cot (z)
z
4
dz =
_
CR

z
4
_
1
z

z
3

(z)
3
45
+...
_
dz
=
_
CR
_
1
z
5


2
3z
3


4
45z
+ ...
_
dz (4.2.2)
And also recall that
_
C
1
z
n
dz = 0 for n ,= 1 (4.2.3)
According to (4.2.3), the only surviving term in (4.2.2) is the 3
rd
term:
_
CR
cot (z)
z
4
dz =
_
CR

4
45z
dz (4.2.4)
And we can see that the residue at z
0
= 0 is just
4
/45, and due to the residue theorem the integral
ends up being
_
CR
cot (z)
z
4
dz =2i
_

4
45
_

2
5
45
(4.2.5)
To nd (4), rst take the taylor expansion of sinx/x:
CHAPTER 4: MATHEMATICAL METHODS 289
sin x
x
=1
x
2
3!
+
x
4
5!

x
6
7!
+... (4.2.6)
And also recall that the Euler-Wallis Sine Product has the form
sin x
x
=

n=1
_
1

2
x
2
n
2
_
=
_
1
2
x
2
_
_
1

2
x
2
4
__
1

2
x
2
8
_
... (4.2.7)
By multiplying out the rst few terms of (4.2.7) and equating it with (4.2.6), it might be possible to
nd (4). Mathews and Walker go through a signicantly dierent method to solve the problem on pg.
53, however I did not nd it to be intuitive and decided to investigate other ways to solve the problem.
Unfortunately, I ran out of time!
Problem 2
Evaluate the integral:
I(a) =
_

0
e
ax
1 + x
2
dx (4.2.8)
By obtaining a dierential equation for I(a) and solving it by variation of parameters. Express the answer using
Ci(x) and Si(x) integrals.
Solution
To solve this integral in the way suggested, the rst step would be to combine several derivatives of our
integral I(a) to form a dierential equation, which we can then solve. So, lets nd the derivatives of
(4.2.8) with respect to a:
I

(a) =
_

0
xe
ax
1 +x
2
dx (4.2.9)
I

(a) =
_

0
x
2
e
ax
1 + x
2
dx (4.2.10)
Now, using (4.2.8), (4.2.9), and (4.2.10) we can create a dierential equation. Following Mathews and
Walker p. 61, lets add I(a) and I

(a):
I

(a) + I(a) =
_

0
x
2
e
ax
1 + x
2
dx +
_

0
e
ax
1 +x
2
dx
=
_

0
e
ax
1 +x
2
_
x
2
+ 1
_
dx
=
_

0
e
ax
dx
W. Erbsen HOMEWORK #4
=
1
a
So, our dierential equation is then:
I

(a) + I(a) =
1
a
(4.2.11)
Which takes the standard form of an inhomogeneous second order dierential equation,
P(x)y

+ Q(x)y

+ R(x)y = S(x) (4.2.12)


At this point we use variation of parameters to solve (4.2.11). To start, rst look at the general solution
to the homogeneous analogue to (4.2.12):
y(x) = c
1
y
1
(x) + c
2
y
2
(x) (4.2.13)
If our equation was homogeneous, then we would have I

(a) +I(a) = 0, which has solutions I


1
(a) = e
ia
and I
2
(a) = e
ia
. But in the case of our inhomogeneous dierential equation, the constants in front of
the terms in (4.2.13) are also dependent on a, which we now refer to as u
i
(a) instead of c
i
:
I(a) = u
1
(a)I
1
(a) +u
2
(a)I
2
(a) (4.2.14)
Where from now on I will drop the explicit a dependence for convenience (eg I(a) I). We must take
the rst and second derivatives of (4.2.14) and insert them into (4.2.11). Taking the rst derivative,
I

= u
1
I

1
+u
2
I

2
+u

1
I
1
+u

2
I
2
(4.2.15)
Where, following Mathews and Walkers lead on pg. 9, we impose the condition that the last two terms
in (4.2.15) sum to zero:
u

1
I
1
+ u

2
I
2
= 0 (4.2.16)
This is one of two equations which allows us to nd u
1
and u
2
. Continuing, (4.2.15) becomes
I

= u
1
I

1
+u
2
I

2
(4.2.17)
Now nd the second derivative of (4.2.17):
I

= u
1
I

1
+ u
2
I

2
+ u

1
I

1
+ u

2
I

2
(4.2.18)
Substituting (4.2.14) and (4.2.18) into (4.2.12):
PI

+ QI

+RI =P[u
1
I

1
+u
2
I

2
+u

1
I

1
+u

2
I

2
] + Q[u
1
I

1
+ u
2
I

2
] + R[u
1
I
1
+ u
2
I
2
]
=P[u

1
I

1
+u

2
I

2
] + u
1
[PI

1
+ QI

1
+RI
1
]
. .
0
+u2 [PI

2
+QI

2
+ RI
2
]
. .
0
(4.2.19)
Where the last two terms vanish since both I
1
and I
2
are solutions to the homogeneous analogue to
(4.2.12). So, remembering that P = 1 and S = 1/a, we now have:
u

1
I

1
+ u

2
I

2
= 1/a (4.2.20)
Now we must use (4.2.16) and (4.2.20) to nd out what u
1
and u
2
are. Solving (4.2.16) for u

1
:
CHAPTER 4: MATHEMATICAL METHODS 291
u

1
=
u

2
I
2
I
1
(4.2.21)
And substituting this into (4.2.20):
_

2
I
2
I
1
_
I

1
+ u

2
I

2
=
1
a
u

2
_
I

I
2
I

1
I
1
_
=
1
a
u

2
=
1
a
_
I

I
2
I

1
I
1
_
1
(4.2.22)
At this point it should be remembered that I
1
= e
ia
,I

1
= ie
ia
,I
2
= e
ia
, and I

2
= ie
ia
. Continuing,
u

2
=
1
a
_
(ie
ia
)
(e
ia
)(ie
ia
)
e
ia
_
1
=
1
a
_
ie
ia
ie
ia

1
=
e
ia
2ia
=
1
2a
_

cos a
i
sin a
_
(4.2.23)
And integrating,
u
2
=
1
2
_

1
i
_
cos a
a
da
_
sin a
a
da
_
=
1
2
_

1
i
__
cos a
a
da
_

__
sin a
a
da
__
=
1
2
_

1
i
Ci(a) Si(a)
_
=
Ci(a)
2i

Si(a)
2
(4.2.24)
Where I used the fundamental denitions of Ci(x) and Si(x). To nd u

1
substitute (4.2.23) into (4.2.16):
u

1
=
_

e
ia
2ia
_
I
2
I
1
=
_
e
ia
2ia
_
e
ia
e
ia
=
e
ia
2ia
=
1
2a
_
cos a
i
sin a
_
(4.2.25)
And integrating here as well,
u
1
=
1
2
_
1
i
_
cos a
a
da
_
sin a
a
da
_
W. Erbsen HOMEWORK #4
=
1
2
_
1
i
__
cos a
a
da
_

__
sin a
a
da
__
=
1
2
_
+
1
i
Ci(a) Si(a)
_
=
Ci(a)
2i

Si(a)
2
(4.2.26)
At this point we substitute (4.2.24) and (4.2.26) into (4.2.14):
I =u
1
I
1
+ u
2
I
2
=
_
Ci(a)
2i

Si(a)
2
_
e
ia
+
_

Ci(a)
2i

Si(a)
2
_
e
ia
=
Ci(a)
2i
e
ia

Si(a)
2
e
ia

Ci(a)
2i
e
ia

Si(a)
2
e
ia
=Ci(a)
_
e
ia
2i

e
ia
2i
_
+ Si(a)
_

e
ia
2

e
ia
2
_
=Ci(a)(sin a) Si(a)(cos a) (4.2.27)
But (4.2.27) is not the answer. I apologize but I did not have time to go back to change everything
recall that the denition for Ci(x) and Si(x) are:
Ci(x) =
_
x

cos t
t
dt (4.2.28)
and
Si(x) =
_
x
0
sin t
t
dt (4.2.29)
So in order to make the substitution, we must account for the fact that the integration limits of Ci(x)
are dierent from Si(x). In fact, we must require that I(a) and all its derivatives vanish at a = , so
we need to substitute the following correction in place of Si(a) in (4.2.27):
_
a

sina
a
da =
_
a
0
sin a
a
da
_

0
sin a
a
da
=Si(a)

2
(4.2.30)
So, substituting this into (4.2.27),
I =Ci(a)(sin a)
_
Si(a)

2
_
(cos a)
= sin aCi(a) + cos a
_

2
Si(a)
_
(4.2.31)
Problem 3
CHAPTER 4: MATHEMATICAL METHODS 293
Evaluate
_
+

1 cos x
x
2
dx (4.2.32)
Solution
First recall Eulers equation e
ix
= cos x + i sin x. Since (4.2.32) has a cos x in the numerator, we may
simply refer to it as the real part of e
ix
: 'e
ix
= cos x. Rewriting (4.2.32), and also moving it over to
the complex plane, we have
_
+

1 cos x
x
2
dx ='
__
+

1 e
ix
x
2
dx
_
=
_
C
1 e
iz
z
2
dz (4.2.33)
There is a singularity at z = 0, and recall that if we have a pole of order n,
a
1
=
1
(n 1)!
d
n1
dz
(z z
0
)
n
f(z) (4.2.34)
Since n = 2 in our case,
Res(z
0
= 0) =
d
dz
z
2
_
1 e
iz
z
2
_

z0
= ie
iz

z0
= i (4.2.35)
Since our residue is on the negative imaginary axis, we should integrate over the lower hemisphere.
Futhermore, since our singularity is located at z = 0 and is on the path of the contour, we must detour
around the origin. To do this subtract from the result upon applying the Residue Theorem:
_
+

1 cos x
x
2
dx =2i(i) = 2 = (4.2.36)
Problem 4
Show that:
_
+

cos x
x
2
+ a
2
dx =

a
e
a
(4.2.37)
Solution
W. Erbsen HOMEWORK #5
Noticing that we can rewrite (4.2.37) in much the same way that we did in Problem 3,
_
+

cos x
x
2
+a
2
dx = '
__
+

e
ix
x
2
+ a
2
dx
_
(4.2.38)
At this point we can solve (4.2.38) using the Residue Theorem rather straightforwardly. Notice that we
can rewrite the denominator in (4.2.38) as (x + ia)(x ia). Making this substitution and moving the
integral to the complex plane,
'
__
+

e
ix
x
2
+ a
2
dx
_
=
_
C
e
iz
(z +ia)(z ia)
dz (4.2.39)
From (4.2.39) it is obvious that there are residues at both z
0
= ia and z
0
= ia. Evaluating the integral
over the uer hemisphere we would take only the positive residue,
Res(z
0
= ia) = (z ia)
_
e
iz
(z +ia)(z ia)
_

zia
=
e
a
2ia
(4.2.40)
And by the residue theorem,
_
+

cos x
x
2
+ a
2
dx =2i
_
e
a
2ia
_
=

a
e
a
(4.2.41)
4.3 Homework #5
Problem 1
Prove Parsevals Theorem:
If f(x)
a
0
2
+

n=1
(a
n
cos nx + b
n
sin nx) (4.3.1a)
Then
_

f
2
(x) dx =

2
a
2
0
+

n=1
(a
2
n
+b
2
n
) (4.3.1b)
Solution
This can be solved via straightforward substitution, where we insert (4.3.1a) into the left hand side of
(4.3.1b). This yields,
CHAPTER 4: MATHEMATICAL METHODS 295
_

f
2
(x) dx =
_

_
a
0
2
+

n=1
(a
n
cos nx +b
n
sin nx)
_
2
dx
=
_

_
a
0
4
2
+ a
0

n=1
(a
n
cos nx +b
n
sin nx) +

n=1
(a
n
cos nx + b
n
sin nx)
2
_
dx
=
a
0
4
2
_

dx + a
0

n=1
_

(a
n
cos nx + b
n
sin nx) dx +

n=1
_

(a
n
cos nx + b
n
sin nx)
2
dx
=
a
2
0
2
+

n=1
_

_
a
2
n
cos
2
nx + 2a
n
b
n
sin nx cos nx +b
2
n
sin
2
nx
_
dx
=
a
2
0
2
+

n=1
_
a
2
n
_

cos
2
nx dx + b
n
_

sin
2
nx dx
_
=
a
2
0
2
+

n=1
_
a
2
n
2
_

(1 + cos 2nx) dx +
b
2
n
2
_

(1 cos 2nx) dx
_
=
a
2
0
2
+

n=1
_
a
2
n
2
_
2 +
_

cos 2nx dx
_
+
b
2
n
2
_
2
_

cos 2nx dx
__
=
a
2
0
2
+

n=1
_
a
2
n
2
_
2 +
sin2n
2n
_
+
b
2
n
2
_
2
sin 2n
2n
__
=
a
2
0
2
+

n=1
_
a
2
n
2
(2) +
b
2
n
2
(2)
_
=
a
2
0
2
+

n=1
_
a
2
n
+ b
2
n

Which is the same as the right hand side of (4.3.1b). Therefore, we have shown that given (4.3.1a), that:
_

f
2
(x) dx =

2
a
2
0
+

n=1
_
a
2
n
+ b
2
n

Problem 2
Show that for a symmetric function
f
_

2
+ x
_
= f
_

2
x
_
, (4.3.2)
The Fourier series has coecients b
n
= 0 and a
2n+1
= 0.
Solution
Recall that
W. Erbsen HOMEWORK #5
f(x) =
a
0
2
+

n=1
(a
n
cos nx +b
n
sin nx) (4.3.3)
Fourier series expands a given function as a sum of cosine and sine terms, where cosine is even (symmetric)
and sine is odd (anti-symmetric). If the function is purely even, then it is clear that it cannot be expanded
in terms of any odd terms, which in (4.3.3) corresponding to b
n
= 0 :
f(x) =
a
0
2
+

n=1
a
n
cos nx (4.3.4)
To see that a
2n+1
= 0, recall how we dene a
n
:
a
n
=
1

f(x) cos nx dx
Such that we have for a
2n+1
:
a
2n+1
=
1

f(x) cos [(2n + 1)x] dx


=
1

f(x) (cos 2nxcos x sin 2nxsin x) dx


=
1

__

f(x) cos 2nx cos x dx


_

f(x) sin 2nxsin x dx


_
(4.3.5)
We recall from (4.3.4) that f(x) is an even function, and also that the domain is symmetric. Therefore,
the rst integral in (4.3.5) is zero since it is an even function integrated over a symmetric domain. The
second integral is also zero since it is an even function multiplying an odd function over a symmetric
domain. Therefore, a
2n+1
= 0 .
Problem 3
Consider the step function
f(x) =
_
for 0 x <
for x < 0
(4.3.6)
Find
a) The Fourier series for f(x)
b) The integral of the Fourier series
c) Show that the dierential of the Fourier series does not exist.
Solution
CHAPTER 4: MATHEMATICAL METHODS 297
a) We begin by noting that the Fourier Series is dened by
f(x) =
a
0
2
+

n=1
(a
n
cos nx +b
n
sin nx) (4.3.7)
Where the coecients a
0
, a
n
, and b
n
are dened by
a
0
=
1

f(x) dx (4.3.8)
And,
a
n
=
1

f(x) cos nx dx (4.3.9)


And also
b
n
=
1

f(x) sin nx dx (4.3.10)


For the function dened by (4.3.6), we can immediately see that the function is purely odd, and
consequently a
n
= 0. To nd the Fourier Series (4.3.7) we start by calculating a
0
from (4.3.8):
a
0
=
1

f(x) dx
=
1

__
0

() dx +
_

0
() dx
_
=
1

_
[x[
0

+ [x[

0
_
=
1

[(0 +) +( 0)]
=
1

2
+
2

= 0
And similarly we nd b
n
from (4.3.10):
b
n
=
1

f(x) sin nx dx
=
1

_
0

sin nx dx +
_

0
sin nx dx
_
=
1

cos nx
n

+
_

cos nx
n

0
_
=
1

_
1
n

cos n
n
_

_
cos n
n

1
n
__
=
1

_
2
_
1
n

cos n
n
__
=2
_
1 cos n
n
_
Substituting the values for a
0
, a
n
, and b
n
into (4.3.7):
W. Erbsen HOMEWORK #5
f(x) = 2

n=1
_
1 cos n
n
_
sin nx (4.3.11)
b) To nd the integral of the Fourier series, integrate (4.3.11):
_

f(x) dx =2

n=1
_
1 cos n
n
__

sin nx dx
=2

n=1
_
1 cos n
n
_
_

cos nx
n

= 0 (4.3.12)
I do believe that I misinterpreted this problem.
c) A quick, qualitative solution to this problem would be to notice that we have a discontinuous
piecewise function, and therefore the derivative is not dened over the entire domain. A slightly
less quick solution would be to calculate f

(x) and see if the resultant series converges or diverges


at each point, which should determine unequivocally whether or not the derivative exists.
So, taking the derivative of (4.3.11):
f

(x) =2

n=1
d
dx
_
1 cos n
n
_
sinnx
=2

n=1
(1 cos n) cos nx
A point of interest being x = 0, so:
f

(x) =2

n=1
(1 cos n)
A test for convergence is not necessary; we can clearly see that f

(0) does not converge, so that


f

(x) is not dierentiable .


Problem 4
Apply a constant external force to a damped harmonic oscillator, starting at time t = 0 and keeping it on.
F(t) =
_
0 for t < 0
F for t > 0
What is the resulting motion?
Solution
The dierential equation describing the damped, forced harmonic oscillator is:
x

(t) + x

(t) +
2
0
x(t) = F(t) (4.3.13)
CHAPTER 4: MATHEMATICAL METHODS 299
Where F(t) in this problem is dened by the heaviside step function. For t < 0, (4.3.13) becomes
x

(t) + x

(t) +
2
0
x = 0
Which is easy enough to solve, start by nding the roots r
1
and r
2
:
r
2
+r +
2
0
= 0
Which is clearly quadratic, letting a = 1, b = and c =
2
0
:
r
1,2
=

_

2
4
2
0
2
So the motion is described by
x(t) = C
1
exp
_
i
_
+
_

2
4
2
0
2
_
t
_
+ C
2
exp
_
i
_

_

2
4
2
0
2
_
t
_
(4.3.14)
Where C
1
and C
2
are determined via initial conditions, which are not given. If we say that initially the
system is not in motion, e.g. x(0) = 0 and x

(0) = 0, then x(t) = 0 for all times. Otherwise it will


oscillate in a predictable way.
We should now investigate the second condition of the heaviside step function, where our dierential
equation picks up a constant term:
x

(t) + x

(t) +
2
0
x(t) = F
0
Which we can rearrange as
x

(t) + x

(t) +
2
0
_
x(t)
F
0

2
0
_
= 0
The addition of this term necessitates the inclusion of a particular solution to our homogeneous solution
(4.3.15):
x(t) = C
1
exp
_
i
_
+
_

2
4
2
0
2
_
t
_
+ C
2
exp
_
i
_

_

2
4
2
0
2
_
t
_
+
F
0

2
0
(4.3.15)
The eect of adding a constant external force to a damped harmonic oscillator only acts to oset the
equilibrium position, so that the system oscillates around a dierent point, which is given by F
0
/
2
0
and
is independent of time.
Problem 5
Show that the Fourier sine transforms of y

(x) and y

(x) are
F
s
[y

] = y
c
() (4.3.16a)
F
s
[y

] =
2
y
s
() +

y(0) (4.3.16b)
W. Erbsen HOMEWORK #5
Solution
We rst note the denitions of the Fourier sine and cosine transforms:
F
s
[f(x)] =f
s
() =
_

0
f(x) sin x dx
F
c
[f(x)] =f
c
() =
_

0
f(x) cos x dx
So, for the rst derivative, f(x) = y

(x) and:
F
s
[y

(x)] =
_

0
y

(x) sin x dx
=
_

0
y(x)
x
sin x dx
At this point, and this is a historic moment since I have never done this before, we want to do a sort of
inverse integration by parts. We begin by noting that
_

0
y(x) cos x dx (4.3.17)
Doing integration by parts,
u = y(x) du =
y(x)
x
dv = cos x v =
1

sin x
Such that (4.3.17) becomes:
_

0
y(x) cos x dx =
1

_
y(x) sin x

_

0
y(x)
x
sinx dx
Solving this for
_
y(x)
x
sin x dx,
_

0
y(x)
x
sin x dx =
_
y(x) sin x

0

_

0
y(x) cos x dx
= y
c
() (4.3.18)
=F
s
[y

(x)]
The process of nding the second derivative is very much the same, recognizing that in this case f(x) =
y

(x) and
F
s
[y

(x)] =
_

0
y

(x) sin x dx
=
_

0
y

(x)
x
sin x dx
Where this time we look at:
_

0
y

(x) cos x dx
CHAPTER 4: MATHEMATICAL METHODS 301
Where we have
u = y

(x) du =
y

(x)
x
dv = cos x v =
1

sin x
Such that
_

0
y

(x) cos x dx =
1

_
y

(x) sin x

_

0
y

(x)
x
sin x dx
And solving for the relavent term,
_

0
y

(x)
x
sin x dx =
_
y

(x) sin x

0

_

0
y

(x) cos x dx
=
_

0
y

(x) cos x dx (4.3.19)


We repeat this process yet another time, but this time around we are looking at
_

0
y

(x) cos x dx =
_

0
y(x)
x
cos x dx
And now we look at
_

0
y(x) sin x dx
Where we let
u = y(x) du =
y(x)
x
dv = sin x v =
1

cos x
So
_

0
y(x) sin x dx =
1

_
y(x) cos x

0
+
1

_

0
y(x)
x
dx
And now
_

0
y(x)
x
cos x dx =
_
y(x) cos x

0
+
_

0
y(x) sin x dx
= y(0) +y
s
()
Substituting this back into (4.3.19),
_

0
y

(x)
x
sin x dx = (y
s
() y(0))
=
2
y
s
() +y(0) (4.3.20)
W. Erbsen HOMEWORK #5
In summary, we have shown that:
F
s
[y

] = y
c
()
F
s
[y

] =
2
y
s
() + y(0)
You might have noticed that what I found for F
s
[y

] diers from yours by a factor of 1/ in the last


term. I have no idea where that could have come from.
Problem 6
Show that the solution of
y

+ 2y

+ (
2
+
2
)y = g(x), > 0, > 0, y() = y() = 0 (4.3.21)
With g() = 0 is
y =
1

_
x

g()e
(x)
sin [(x )] d (4.3.22)
Hint: Find Greens function of the equation rst.
Solution
As per the hint, I am going to go ahead and nd Greens function for (4.3.21). Substituting G(x ) for
y assuming that x > , we solve the homogeneous analogue of our given dierential equation:
G

(x ) + 2G

(x ) + (
2
+
2
)G(x ) = 0
Finding the roots,
r
2
+ 2r + (
2
+
2
) = 0
Letting a = 1, b = 2 and c =
2
+
2
:
r
1,2
=
2
_
4
2
4(
2
+
2
)
2
= i
So that we have
G(x ) =C
1
cos[( + i)(x )] + C
2
sin[( i)(x )]
=
1

sin[( i)(x )] (4.3.23)


Due to the boundary conditions of the problem. At this point we should note that this problem represents
a driven, damped harmonic oscillator, whose solution was discussed in an earlier problem. The solution
CHAPTER 4: MATHEMATICAL METHODS 303
to this type of problem using Greens function is given in M&W in the prompt of problem 9-7. Tailoring
it to our case,
y(x) =
_
x

G(x )g() d (4.3.24)


The nite upper limit comes from the fact that x in this problem really should be t, and up to this point
we have been assuming that for x 0, G(x ) = 0. To make this more transparent, we note that
for a harmonic oscillator, we would have likely chosen x to be t, and to be t

. In this case our


solution for y(x) (or y(t), rather) is represented by an integral over past times t.
Substituting (4.3.23) into (4.3.24),
y(x) =
1

_
x

sin[( i)x]g() d (4.3.25)


Where we can say
sin[( i)x] =
1
2i
exp[( i)(x )] exp[( + i)(x )]
=
1
2i
exp[x ix +i] exp[x + +ix i]
=
1
2i
exp[(x ) +i(x )] exp[(x ) + i(x )]
=
exp[(x )]
2i
exp[i(x )] exp[i(x )]
=exp[(x )] sin [(x )] (4.3.26)
Now substituting (4.3.26) back into (4.3.25):
y(x) =
1

_
x

exp[(x )] sin[(x )] g() d


In case you couldnt tell I worked backwards a bit, sorry for the missteps in logic. I am very ignorant to
Greens functions (I nd the entire thing very confusing), however it is an invaluable mathematical tool,
one I hope to master soon.
4.4 Homework #6
Problem 1
Find the Laplace transform of f(t) = te
t
, and determine the region of existence.
Solution
W. Erbsen HOMEWORK #6
We know that the Laplace Transform of some function f(t) is given by
Lf(t) = F(s) =
_

0
f(t)e
st
dt
And in our case,
Lf(t) =
_

0
te
t
e
st
dt
=
_

0
te
(s1)t
dt
_
u = t, du = 1
dv = e
(s1)t
, v =
1
s1
e
(s1)t
=
_

t
s 1
e
(s1)t

0
+
1
s 1
_

0
e
(s1)t
dt
=
_

1
(s 1)
2
e
(s1)t

0
=
1
(s 1)
2
We can see that in the last limit, the result will be convergent only if s 1 < 0, or s > 1, so
F(s) =
1
(s 1)
2
for s > 1
Problem 2
Find the inverse Laplace Transform of
a)
1
s
3
s
2
(4.4.1a)
b)
s
2
+s 1
s
3
2s
2
+s 2
(4.4.1b)
Solution
a) As per the hint, lets go ahead and employ partial fractions on (4.4.1a):
1
s
3
s
2
=
1
s
2
(s 1)
=
A
s
2
+
B
s
+
C
s 1
1 =A(s 1) + Bs(s 1) + Cs
2
Let s = 0: 1 = A A = 1
Let s = 1: 1 =Cs
2
C = 1
Let A = 1, C = 1: 1 =1 s +Bs(s 1) +s
2
B = 1
So, (4.4.1a) becomes
CHAPTER 4: MATHEMATICAL METHODS 305
1
s
3
s
2
=
1
s 1

1
s
2

1
s
(4.4.2)
Where we note that
L
_
e
t
_
=
_

0
e
t
e
st
dt =
_

0
e
(s1)t
dt =
1
s 1
(4.4.3a)
Lt =
_

0
te
st
dt =
1
s
_

0
e
st
dt =
1
s
2
(4.4.3b)
L1 =
_

0
e
st
dt =
1
s
(4.4.3c)
Substituting (4.4.3a)-(4.4.3c) into (4.4.2),
1
s
3
s
2
= e
t
t 1
b) This time we only need to factor; no partial fractions is needed:
s
2
+ s 1
s
3
2s
2
+s 2
=
(s
2
+ 1)
(s
2
+ 1)(s 2)
+
(s
2
+ 1) + (s 2)
(s
2
+ 1)(s 2)
=
1
s
2
+ 1
+
1
s 2
(4.4.4)
We note the Laplace Transforms of the following functions
Lsin t =
_

0
sin te
st
dt
=
1
2i
_

0
_
e
it
e
it

e
st
dt
=
1
2i
_

0
_
e
(is)t
e
(i+s)t
_
dt
=
1
2i
_

e
(is)t
s i

e
(i+s)t
s + i

0
=
1
2i
_
1
s i

1
s +i
_
=
1
2i
_
2i
s
2
+ 1
_
=
1
s
2
+ 1
(4.4.5)
L
_
e
2t
_
=
_

0
e
2t
e
st
dt =
_

0
e
(s2)t
dt =
1
s 2
(4.4.6)
Putting (4.4.5) and (4.4.6) into (4.4.4),
s
2
+s 1
s
3
2s
2
+s 2
= sint +e
2t
Problem 3
W. Erbsen HOMEWORK #6
Solve the following dierential equations using Laplace Transforms:
a) y

+y = cos t, for t > 0, y(0) = 1 (4.4.7a)


b) y

+ y = cos (2t), for t > 0, y(0) = 0, y

(0) = 0 (4.4.7b)
Solution
a) First lets nd the Laplace Transform of cos t, since it will be needed later:
Lcos t =
_

0
cos te
st
dt
=
1
2
_

0
_
e
it
+e
it

e
st
dt
=
1
2
_

0
_
e
(is)t
+ e
(i+s)t
_
dt
=
1
2
_

e
(is)t
s i
+
e
(i+s)t
s +i

0
=
1
2
_
1
s i
+
1
s +i
_
=
1
2
_
2s
s
2
+ 1
_
=
s
s
2
+ 1
(4.4.8)
Taking the Laplace Transform of both sides of (4.4.7a),
Ly

+ Ly =Lcos t
sY (s) y(0) +Y (s) =
s
s
2
+ 1
Applying the given initial condition and solving for Y (s),
Y (s) =
s
(s
2
+ 1)(s + 1)
+
1
s + 1
=
1 + s
2(s
2
+ 1)

1
2(s + 1)
+
1
s + 1
=
1
2
_
1
s
2
+ 1
_
+
1
2
_
s
s
2
+ 1
_
+
_
1
s + 1
_
Now we must nd the inverse Laplace Transform of Y (s):
L
1
Y (s) =
1
2
L
1
_
1
s
2
+ 1
_
+
1
2
L
1
_
s
s
2
+ 1
_
+L
1
_
1
s + 1
_
Where we recall that the inverse Laplace Transform of 1/(s
2
+ 1) was found to be sint in (4.4.5),
and s/(s
2
+ 1) was found to be cos t in (4.4.8). Finding 1/(s + 1),
L
_
e
t
_
=
_

0
e
t
e
st
dt =
_

0
e
(s+1)t
dt =
1
s + 1
CHAPTER 4: MATHEMATICAL METHODS 307
So we now have
L
1
Y (s) =
sin t
2
+
cos t1
2
+e
t
And nally
y(t) =
1
2
(sin t + cos t) +e
t
b) We rst nd the Laplace Transform of cos (2t), since it will be needed later:
Lcos (2t) =
_

0
cos (2t)e
st
dt
=
1
2
_

0
_
e
2it
+ e
2it

e
st
dt
=
1
2
_

0
_
e
(2is)t
+e
(2i+s)t
_
dt
=
1
2
_

e
(2is)t
s 2i
+
e
(2i+s)t
s + 2i

0
=
1
2
_
1
s 2i
+
1
s + 2i
_
=
1
2
_
2s
s
2
+ 4
_
=
s
s
2
+ 4
(4.4.9)
Now, applying the Laplace Transform to both sides of (4.4.7b):
Ly

+ Ly =Lcos (2t)
s
2
Y (s) sy(0) y

(0) + Y (s) =
s
s
2
+ 4
Applying the initial conditions,
s
2
Y (s) + Y (s) =
s
s
2
+ 4
Y (s) =
s
(s
2
+ 4)(s
2
+ 1)
=
1
3
_
s
s
2
+ 1

s
s
2
+ 4
_
Taking the inverse Laplace Transform of Y (s),
L
1
Y (s) =
1
3
_
L
1
_
s
s
2
+ 1
_
L
1
_
s
s
2
+ 4
__
Where again the inverse Laplace Transform of s/(s
2
+ 1) is just cos t as per (4.4.5). Similarly,
s/(s
2
+ 4) is just cos (2t), which was found in (4.4.9). So, we are nally left with:
y(t) =
1
3
[cos t cos (2t)]
W. Erbsen HOMEWORK #6
Problem 4
An electric circuit gives rise to the system
L
dI
1
dt
+RI
1
+
q
C
= E
0
(4.4.10a)
L
dI
2
dt
+RI
2

q
C
= 0 (4.4.10b)
dq
dt
= I
1
I
2
(4.4.10c)
With initial conditions
I
1
(0) = I
2
(0) =
E
0
2R
, q(0) = 0 (4.4.11)
Solve the system by Laplace Transform methods and show that
I
1
=
E
0
2R
+
E
0
2L
e
t
sin (t), where =
R
2L
and
2
=
2
LC

2
(4.4.12)
Solution
We start by taking the inverse Laplace Transform of (4.4.10c):
Lq

(t) =LI
1
(t) LI
2
(t)
sq(s) q(0) =I
1
(s) I
2
(s)
q(s) =
I
1
(s)
s

I
2
(s)
s
(4.4.13)
We now do the same for (4.4.10a):
LLI
1
(t) + RLI
1
+
1
C
Lq(t) =E
0
L1
L[sI
1
(s) I
1
(0)] + RI
1
(s) +
1
C
q(s) =
E
0
s
L
_
sI
1
(s)
E
0
2R
_
+ RI
1
(s) +
1
C
_
I
1
(s)
s

I
2
(s)
s
_
=
E
0
s
sI
1
(s) +
R
L
I
1
(s) +
2
0
I
1
(s)
s

2
0
I
2
(s)
s
=
E
0
Ls
+
E
0
2R
_
s +
R
L
+

2
0
s
_
I
1
(s)
2
0
I
2
(s)
s
=
E
0
Ls
+
E
0
2R
_
s
2
+
2
0
s
+
R
L
_
I
1
(s)
2
0
I
2
(s)
s
=
E
0
Ls
+
E
0
2R
(4.4.14)
Where I have dened
2
0
= 1/

LC . And similarly for (4.4.10b):


LLI
2
(t) + RLI
2

1
C
Lq(t) =0
L[sI
2
(s) I
2
(0)] + RI
2
(s)
1
C
q(s) =0
CHAPTER 4: MATHEMATICAL METHODS 309
L
_
sI
2
(s)
E
0
2R
_
+ RI
2
(s)
1
C
_
I
1
(s)
s

I
2
(s)
s
_
=0
sI
2
(s) +
R
L
I
2
(s)
2
0
I
1
(s)
s
+
2
0
I
2
(s)
s
=
E
0
2R
_
s +
R
L
+

2
0
s
_
I
2
(s)
2
0
I
1
(s)
s
=
E
0
2R
_
s
2
+
2
0
s
+
R
L
_
I
2
(s)
2
0
I
1
(s)
s
=
E
0
2R
(4.4.15)
At this point we have two equations, (4.4.14) and (4.4.15), and two unknowns. Finding the s-domain
solutions (I
1
(s) and I
2
(s)) is trivial, but time consuming. The most tedious steps in what follows were
left up to Mathematica.
The obvious rst step would be to solve (4.4.14) for I
2
(s), inserting into (4.4.15), and solving for
I
1
(s). The result is
I
1
(s) =
E
0
2
_
1
Rs
+
1
s(Ls + R) + 2L
2
0
_
=
E
0
2
_
1
Rs
+
1
Ls(s + R/L) + 2/C
_
=
E
0
2
_
1
Rs
+
1
L
_
1
s
2
+sR/L + 2/LC
__ _
Let = R/2L
=
E
0
2
_
1
Rs
+
1
L
_
1
s
2
+ 2s + 2/LC
__
=
E
0
2
_
1
Rs
+
1
L
_
1
s
2
+ 2s +
2
+ 2/LC
2
__ _
Complete the square
=
E
0
2
_
1
Rs
+
1
L
_
1
(s + )
2
+ 2/LC
2
__ _
Let = 2/LC
2
=
E
0
2
_
1
Rs
+
1
L
_

(s +)
2
+
2
__
=
E
0
2R
1
s
+
E
0
2L
_

(s +)
2
+
2
_
Taking the inverse Laplace Transform,
L
1
I
1
(s) =
E
0
2R
L
1
_
1
s
_
+
E
0
2L
L
1
_

(s + )
2
+
2
_
=
E
0
2R
+
E
0
2L
e
t
sin (t)
Which was mercilessly stolen from a table. We nally have
I
1
(t) =
E
0
2R
+
E
0
2L
e
t
sin (t)
Repeating this process for I
2
(s), Mathematica gives
I
2
(s) =
E
0
2
_
1
Rs

1
s(Ls +R) + 2L
2
0
_
W. Erbsen HOMEWORK #7
Which we note is identical to our expression for I
1
(s) sans the minus sign, therefore we can go ahead
and conclude that:
I
2
(t) =
E
0
2R

E
0
2L
e
t
sin (t)
4.5 Homework #7
Problem 1
For the vectors in three dimensions,
v
1
= x + y, v
2
= y + z, v
3
= z + x (4.5.1)
Use the Gram-Schmidt procedure to construct an orthonormal basis starting from v
1
.
Solution
It might be most instructive if we explicitly dene each of our vectors in terms of x, y, and z. e.g.,
v
1
=
_
_
1
1
0
_
_
, v
2
=
_
_
0
1
1
_
_
, v
3
=
_
_
1
0
1
_
_
And now recall that the Gramm-Schmidt Orthonormalization procedure is
u
n
= v
n

n1

i=1
u
i
[v
n
)
u
i
[u
i
)
u
i
, e
1
=
u
n
|u
n
|
Where u
n
represents a orthogonal set of vectors, and e
n
represents an orthonormal set of vectors. Ap-
plying this to our case,
u
1
=v
1
=
_
_
1
1
0
_
_
e
1
=
1

2
_
_
1
1
0
_
_
Where I used the fact that |u
1
| =

2 . Moving on to the second vector,


u
2
=v
2

v
2
[u
1
)
u
1
[u
1
)
u
1
Where
u
1
[u
1
) = u
T
1
u
1
= (1 1 0)
_
_
1
1
0
_
_
= 2, v
2
[u
1
) = v
T
2
v
1
= (0 1 1)
_
_
1
1
0
_
_
= 1
CHAPTER 4: MATHEMATICAL METHODS 311
Such that we now have
u
2
=
_
_
0
1
1
_
_

1
2
_
_
1
1
0
_
_
=
_
_
1
/
2
1
/
2
1
_
_
e
2
=
_
2
3
_
_
1
/
2
1
/
2
1
_
_
And now the last vector is
u
3
=v
3

v
3
[u
1
)
u
1
[u
1
)
u
1

v
3
[u
2
)
u
2
[u
2
)
u
2
Where we have
v
3
[u
1
) =(1 0 1)
_
_
1
1
0
_
_
= 1
v
3
[u
2
) =(1 0 1)
_
_

1
/
2
1
/
2
1
_
_
=
1
/
2
u
2
[u
2
) =(
1
/
2
1
/
2
1)
_
_

1
/
2
1
/
2
1
_
_
=
3
/
2
And now
u
3
=
_
_
1
0
1
_
_

1
2
_
_
1
1
0
_
_

1
3
_
_

1
/
2
1
/
2
1
_
_
=
_
_
2
/
3

2
/
3
2
/
3
_
_
e
3
=
1

3
_
_
1
1
1
_
_
Problem 2
For the vector space of polynomials in x, use the scalar product dened as
f[g) =
_

dxe
x
2
f(x) g(x) (4.5.2)
Start from the vectors
v
0
= 1, v
1
= x, v
2
= x
2
, v
3
= x
3
(4.5.3)
And use the Gram-Schmidt procedure to construct an orthonormal basis starting from v
0
. Repeat this calculation
for the scalar product
f[g) =
_
1
0
dxx
2
f(x) g(x) (4.5.4)
Solution
W. Erbsen HOMEWORK #7
Using the same logic as before, we are looking to ortho-normalize v
0
, v
1
, v
2
, and v
3
using the scalar
product dened by (4.5.2). The rst vector is trivial,
u
0
= v
0
= e
0
= 1
And the second is
u
1
=v
1

v
1
[u
0
)
u
0
[u
0
)
u
0
=x
x[1)
1[1)
1
=x
_

xe
x
2
dx
_

e
x
2
dx
=x e
1
=
_
4

_1
/4
x
And the third is
u
2
=v
2

v
2
[u
0
)
u
0
[u
0
)
u
0

v
2
[u
1
)
u
1
[u
1
)
u
1
=x
2

x
2
[1)
1[1)

x
2
[x)
x[x)
x
=x
2

x
2
e
x
2
dx
_

e
x
2
dx

x
3
e
x
2
dx
_

x
2
e
x
2
x
=x
2

2
1

=x
2

1
2
e
2
=
_
4

_1
/4
(x
2

1
/
2
)
And lastly, the forth is
u
3
=v
3

v
3
[u
0
)
u
0
[u
0
)
u
0

v
3
[u
1
)
u
1
[u
1
)
u
1

v
3
[u
2
)
u
2
[u
2
)
u
2
=x
3

x
3
[1)
1[1)

x
3
[x)
x[x)
x
x
3
[
_
x
2

1
/
2
_
)
(x
2

1
/
2
) [(x
2

1
/
2
))
_
x
2

1
/
2
_
=x
3

x
3
e
x
2
dx
_

e
x
2
dx

x
4
e
x
2
dx
_

x
2
e
x
2
dx
x
_

x
3
_
x
2

1
/
2
_
e
x
2
dx
_

(x
2

1
/
2
)
2
e
x
2
dx
_
x
2

1
/
2
_
=x
3

4
2

x
=x
3

3
2
x e
3
=
_
8
9
_1
/4
(x
2

1
/
2
)
Now we repeat this calculation for (4.5.4), in much the same way we did for (4.5.3). The rst vector is
then
CHAPTER 4: MATHEMATICAL METHODS 313
u
0
= v
0
= e
0
= 1
While the second is given by
u
1
=v
1

v
1
[u
0
)
u
0
[u
0
)
u
0
=x
x[1)
1[1)
=x
_
1
0
x
3
dx
_
1
0
x
2
dx
=x
1
/
4
1
/
3
=x
3
4
e
1
=

80 (x
3
/
4
)
And now the third
u
2
=v
2

v
2
[u
0
)
u
0
[u
0
)
u
0

v
2
[u
1
)
u
1
[u
1
)
u
1
=x
2

_
1
0
x
4
dx
_
1
0
x
2
dx

_
1
0
x
4
(x
3
/
4
) dx
_
1
0
x
2
(x
3
/
4
) dx
(x
3
/
4
)
=x
2

1
/
5
1
/
3

1
/
60
1
/
80
(x
3
/
4
)
=x
2

4
3
x +
2
5
e
2
=

1575 (x
2

4
/
3
x +
2
/
5
)
And the forth
u
3
=v
3

v
3
[u
0
)
u
0
[u
0
)
u
0

v
3
[u
1
)
u
1
[u
1
)
u
1

v
3
[u
2
)
u
2
[u
2
)
u
2
=x
3

_
1
0
x
5
dx
_
1
0
x
2
dx

_
1
0
x
5
(x
3
/
4
) dx
_
1
0
x
2
(x
3
/
4
) dx
(x
3
/
4
)
_
1
0
x
5
(x
2

4
/
3
x +
2
/
5
) dx
_
1
0
x
2
(x
2

4
/
3
x +
2
/
5
)
2
dx
(x
2

4
/
3
x +
2
/
5
)
=x
3

1
/
6
1
/
3

1
/
56
1
/
224
(x
3
/
4
)
1
/
840
1
/
1575
(x
2

4
/
3
x +
2
/
5
)
=x
3

15
8
x
2

3
2
x +
7
4
e
3
=
_
20160
/
1667
(x
3

15
/
8
x
2

3
/
2
x +
7
/
4
)
Problem 3
On the vector space of quadratic polynomials, of degree 3, the operator d/dx is dened. Use the basis of
the Legendre Polynomials

P
0
(x) = 1,

P
1
(x) = x,

P
2
(x) =
3
2
x
2

1
2
,

P
3
(x) =
5
2
x
3

3
2
x (4.5.5)
W. Erbsen HOMEWORK #7
And compute the components of this operator. Repeat this exercise for the operator d
2
/dx
2
.
Solution
Let our set of quadratic polynomials be given by f(x) = a
0
+ a
1
x + a
2
x
2
+ a
3
x
3
(so, v
0
= a
0
, v
1
=
a
1
x, v
2
= a
2
x
2
, v
3
= a
3
x
3
). We can also dene our dierential operator to be D[f(x)] = df(x)/dx. So,
D[v
0
] =0
D[v
1
] =a
1
=a
1

P
0
(x)
D[v
2
] =2a
2
x = 2a
2

P
1
(x)
D[v
3
] =3a
3
x
2
= a
3
(2

P
2
(x) +

P
0
)
And if we let D
2
[f(x)] = df(x)
2
/dx
2
,
D[v
0
] =0
D[v
1
] =0
D[v
2
] =2a
2
=2a
2

P
0
(x)
D[v
3
] =3a
3
x
2
= 6a
3

P
1
This is my wild stab at the problem, however I dont believe it is a very accurate one, since if it were
the problem would be trivial (clearly). Whoops!
Problem 4
Diagonalize each of the Pauli spin matrices. Find their eigenvalues and specify the respective eigenvectors as
the basis in which they are diagonal.
Solution
The Pauli matrices are given by

x
=
_
0 1
1 0
_
,
y
=
_
0 i
i 0
_
,
z
=
_
1 0
0 1
_
Starting with
x
,

1
1

= 0
2
1 = 0
= 1
For = +1 :
_
1 1
1 1
__
a
b
_
=
_
0
0
_

a +b = 0
a b = 0
_
a = b, so
_
1
1
_
CHAPTER 4: MATHEMATICAL METHODS 315
For = 1 :
_
1 1
1 1
__
a
b
_
=
_
0
0
_

a + b = 0
a + b = 0
_
a = b, so
_
1
1
_
And now for
y
:

i
i

= 0
2
1 = 0
= 1
For = +1 :
_
1 i
i 1
__
a
b
_
=
_
0
0
_

a ib = 0
ia b = 0
_
a = ib, so
_
1
i
_
For = 1 :
_
1 i
i 1
__
a
b
_
=
_
0
0
_

a ib = 0
ia +b = 0
_
a = ib, so
_
1
i
_
In analyzing
z
we immidiately recognize that = 1, since it is diagonal. So, we have
For = +1 :
_
0 0
0 2
__
a
b
_
=
_
0
0
_
2b = 0, so
_
1
0
_
For = 1 :
_
2 0
0 0
__
a
b
_
=
_
0
0
_
2a = 0, so
_
0
1
_
4.6 Homework #8
Problem 1
Find the eigenvlaues and eigenfunctions of the boundary value problem
y

+

(x + 1)
2
y = 0 (4.6.1)
On the interval a x 2 with boundary conditions y(1) = y(2) = 0. Write the equation in terms of a regular
Sturm-Liouville eigenvalue problem, and nd the coecients in the expansion of an arbitrary function f(x) in a
series of the eigenfunctions.
Solution
In order to express (4.6.1) within the formalism of Sturm-Lioville theory, we recall that if we are given
some dierential equation of the form
a
2
(x)y

+a
1
(x)y

+ a
0
(x)y = f(x) (4.6.2)
W. Erbsen HOMEWORK #8
then we can dene
p(x) =exp
__
a
1
(x)
a
2
(x)
dx
_
q(x) =p(x)
a
0
(x)
a
2
(x)
F(x) =p(x)
f(x)
a
2
(x)
Such that (4.6.2) is transformed into
d
dx
(p(x)y

) +q(x)y = F(x) (4.6.3)


We can see form (4.6.1) that in our case, a
2
(x) = 1, a
1
(x) = 0, a
0
(x) = /(x + 1)
2
, and f(x) = 0. From
these, it is clear that p(x) = 1, q(x) = /(x + 1)
2
, and F(x) = 0, such that we can now express (4.6.1)
in the form of (4.6.3):
y

+

(x + 1)
2
y = 0 (4.6.4)
Which is identical to (4.6.1). So, apparently our DE was already in S-L form. To nd the eigenvalues
and eigenfunctions, we wish to make the substitution and corresponding derivatives
y =(x + 1)

=(x + 1)
1
y

=( 1)(x + 1)
2
Now (4.6.4) becomes
( 1)(x + 1)
2
+

(x + 1)
2
(x + 1)

= 0
( 1)(x + 1)
2
+ (x + 1)
2
= 0

2
+ = 0
And now
=
1

1 4
2
(4.6.5)
At this point we split our solutions according to the three distinct outcomes from (4.6.5), which are if
1 4 < 0, 1 4 = 0, and 1 4 > 0:
<
1
/
4
: then =
1
/
2
_
1

1 4
_
with real, distinct roots, which admits solutions of
the form
y(x) = c
1
e
x
+c
2
e
x
Which only satises the rst boundary condition, y(1) = 0 for the trivial case where y = 0.
This cannot be our solution.
CHAPTER 4: MATHEMATICAL METHODS 317
=
1
/
4
: then =
1
/
2
, with real, repeated roots and solutions
y(x) = c
1
e
x
+ c
2
xe
x
Which also fails to satisfy the rst boundary condition, therefore we must ignore this case
as well.
>
1
/
4
: then =
1
/
2
_
1 i

4 1
_
with complex, distinct roots and
y
1
(x) =c
1
(x + 1)
1
/2
sin
_
4 1
2
log
_
x + 1
2
__
y
2
(x) =c
2
(x + 1)
1
/2
cos
_
4 1
2
log
_
x + 1
2
__
Which does work. Applying the rst boundary condition
y(1) =c
1
(2)
1
/2
sin
_
4 1
2
log
_
2
2
__
+ c
2
(2)
1
/2
cos
_
4 1
2
log
_
2
2
__
=c
1
0 +c
2
2
1
/2
c
1
= 1, c
2
= 0
So, our solution takes the form
y(x) = y
1
(x) = (x + 1)
1
/2
sin
_
4 1
2
log
_
x + 1
2
__
(4.6.6)
Applying our second boundary condition,
y(2) = (3)
1
/2
sin
_
4 1
2
log
_
3
2
__
sin
_
4 1
2
log
_
3
2
__
= 0
And in order for this to hold...

4 1
2
log
_
3
/
2
_
=n
4 1 =
_
2n
log (
3
/
2
)
_
2
=
_
n
log (
3
/
2
)
_
2
+
1
4
And to nd the corresponding eigenfunction we substitute this into (4.6.6):

n
(x) = (x + 1)
1
/2
sin
_
2n
log (
3
/
2
)
log
_
x + 1
2
__
Problem 2
W. Erbsen HOMEWORK #8
Find the eigenvalues and eigenfunctions of the boundary value problem
x
2
y

+xy

+y = y (4.6.7)
on the interval 1 x 2 with boundary conditions y(1) = y(2) = 0. Write the equation in terms of a regular
Sturm-Liouville eigenvalue problem, and nd the coecients in the expansion of an arbitrary function f(x) in a
series of the eigenfunctions.
Solution
In the same spirit as the last problem, we must rst put (4.6.7) into Sturm-Lioville form. We note that
a
2
(x) = x
2
, a
1
(x) = x, a
0
(x) = 1 , and f(x) = 0. From this we can nd
p(x) =exp
__
1
x
dx
_
= exp [logx] = x
q(x) =x
1
x
2
=
1
x
F(x) =0
Which allows us to write
d
dx
(xy

) +
1
x
y = 0 (4.6.8)
Try a solution of the form (and corresponding derivatives)
y =x

=x
1
y

=( 1)x
2
Substituting these into (4.6.8),
d
dx
_
xx
1
_
+
1
x
x

= 0
d
dx
(x

) + (1 )x
1
= 0

2
x
1
+ (1 )x
1
= 0

2
+ 1 = 0
Solving for ,
=

4
2
Following the same logic as before, we nd that in the case that < 4 that we have
y
1
(x) =sin
_
4
2
log x
_
y
2
(x) =cos
_
4
2
log x
_
CHAPTER 4: MATHEMATICAL METHODS 319
Where we must choose y
1
(x) since it satises the rst boundary condition. For the second,
y(2) = 2

1
/2
sin
_
4
2
log 2
_
sin
_
4
2
log 2
_
= 0
And so

4
2
log 2 =n
_
4 =
2n
log 2
=
_
2n
log 2
_
2
+ 4
And nally

n
(x) = sin
_
n
log 2
log x
_
Problem 2
Using Rodriques formula show that the P
n
(x) are orthogonal and that
_
1
1
[P
n
(x)]
2
dx =
2
2n + 1
(4.6.9)
Solution
Rodrigues formula for the Legendre Polynomials is given by (12.65) in A&W:
P
n
(x) =
1
2
n
n!
_
d
dx
_
n
_
x
2
1
_
n
(4.6.10)
To prove that P
n
(x) given by (4.6.10) are orthogonal functions, we evaluate:
_
1
1
P
n
(x)P
m
(x) dx =
1
2
n+m
n!m!
_
1
1
_
d
dx
_
n
_
x
2
1
_
n
_
d
dx
_
m
_
x
2
1
_
m
dx (4.6.11)
For convenience lets set C = 1/2
n+m
n!m!. At this point we integrate by parts m times, with
u =
_
d
dx
_
n
(x
2
1)
n
, du =
_
d
dx
_
n+m
(x
2
1)
n
dv =
_
d
dx
_
m
(x
2
1)
m
, v = (x
2
1)
m
W. Erbsen HOMEWORK #8
Such that (4.6.11) becomes
_
1
1
P
n
(x)P
m
(x) dx =C
_
(x
2
1)
m
_
d
dx
_
n
(x
2
1)
n

1
1
+ C(1)
m
_
1
1
(x
2
1)
m
_
d
dx
_
n+m
(x
2
1)
n
dx
=C(1)
m
_
1
1
(x
2
1)
m
_
d
dx
_
n+m
(x
2
1)
n
dx
At this point we note that in the case that n < m, the above integral goes to zero. Eg if, for instance,
n = 1 and m = 2:
_
1
1
P
1
(x)P
2
(x) dx =C
_
1
1
(x
2
1)
2
_
d
dx
_
3
(x
2
1)
2
dx = C
_
1
1
2(x
2
1)
2
d
dx
dx = 0
Which works for any n < m. This can be straightforwardly extended to the case in which n ,= m when
we recall that our choice of m and n was arbitrarily chosen in integrating by parts. So, we have just
shown that if n ,= m then (4.6.11) is equal to zero.
If, on the other hand, m = n, then we have
_
1
1
[P
n
(x)]
2
dx =
(1)
n
2
2n
(n!)
2
_
1
1
(x
2
1)
n
_
d
dx
_
2n
(x
2
1)
n
dx
=
(1)
n
(2n)!
2
2n
(n!)
2
_
1
1
(x
2
1)
n
dx (4.6.12)
At this point we make the substitution u =
1
/
2
(x + 1) x = 2u 1, du =
1
/
2
dx dx = 2 du, such
that (4.6.12) becomes:
_
1
1
[P
n
(x)]
2
dx =
(1)
n
(2n)!
2
2n
(n!)
2
_
1
0
(4u
2
4u)
n
2 du
=
2(1)
n
(2n)!
2
2n
(n!)
2
_
1
0
[4u(u 1)]
n
du
=
2(1)
n
(2n)!
(n!)
2
_
1
0
u
n
(u 1)
n
du
=
2(1)
n
(2n)!
(n!)
2
(1)
n
(n + 1)
2
(2n + 2)
=
2(2n)!
(n!)
2
(n!)
2
(2n + 1)!
=2
(2n)!
(2n + 1)!
=
2
2n + 1
Where I integrated by parts n times to evaluate the integral, and also the fact that (n) = (n 1)!:
CHAPTER 4: MATHEMATICAL METHODS 321
_
1
1
[P
n
(x)]
2
dx =
2
2n + 1
Problem 4
Using a generating function, evaluate the sum

n=0
x
n+1
n + 1
P
n
(x) (4.6.13)
Solution
We note that from (7 10) in M&W that
f(x) =
1

1 2hx +h
2
=

n=0
h
n
P
n
(x) (4.6.14)
Which denes the generating function. We wish to manipulate (4.6.14) until it resembles (4.6.13). Start
by squaring both sides,
[f(x)]
2
=

n=0
[h
n
P
n
(x)]
2
And now integrating,
_
1
1
[f(x)]
2
dx =
_
1
1

n=0
[h
n
P
n
(x)]
2
dx (4.6.15)
Not sure where to go from here. h
n
must be dependent on x, otherwise the integral on the RHS of
(4.6.15) does not make sense. Otherwise, I was unable to integrate-by-parts to solve it either.
Summing by hand and simplifying results in the following series:
f(x) =x +
x
3
3
+
x
5
5
+
x
7
7
+...
Which is clearly odd, so this leads me to believe f(x) is a sin function. It also seems as though the
sum diverges, however it does indeed converge, since we required that 0 < x < 1, so 0 < f(x) < 4.52
(approximately). Ugh.
Problem 5
Prove the trigonometric identity using vector methods:
W. Erbsen HOMEWORK #8
cos = cos
1
cos
2
+ sin
1
sin
2
cos(
1

2
) (4.6.16)
Where is the angle between two directions (
1
,
1
) and (
2
,
2
).
Solution
We dene two vectors on the units sphere,

A and

B, each normlized to unity. Each is then described by
only two coordinates, and . To prove the trigonometric identity (4.6.16), we take the dot product of
the two vectors. But rst, we dene the components of our vectors in terms of Cartesian coordinates,
with
x =sin cos
y =sin sin
z =cos
Yielding a dot product of

A

B =x
1
x
2
+ y
1
y
2
+z
1
z
2
=sin
1
cos
1
sin
2
cos
2
+ sin
1
sin
1
sin
2
sin
2
+ cos
1
cos
2
=(sin
1
+ sin
2
)(cos
1
cos
2
+ sin
1
sin
2
. .
cos(12)
) + cos
1
cos
2
=cos
1
cos
2
+ sin
1
sin
2
cos(
1

2
) (4.6.17)
And recall

A

B = |A|
..
1
|B|
..
1
cos = cos .
So (4.6.17) is now
cos = cos
1
cos
2
+ sin
1
sin
2
cos(
1

2
)
Problem 6
The amplitude of a scattered wave is given by
f() =
1
k

=0
(2 + 1)e
i
sin

(cos ) (4.6.18)
Where is the angle of scattering, is the angular momentum eigenvalue, k is the incident momentum, and

is the phase shift produced by the central potential that is doing the scattering. The total cross section is

tot
=
_
[f()[
2
d. Show that

tot
=
4
k
2

=0
(2 + 1) sin
2

(4.6.19)
CHAPTER 4: MATHEMATICAL METHODS 323
Solution
We begin by rst evaluating [f()[
2
:
[f()[
2
=
_
1
k

=0
(2 + 1)e
i
sin

(cos )
_

_
1
k

=0
(2 + 1)e
i
sin

(cos )
_
=
1
k
2

=0
(2 + 1)e
i
sin

(cos )(2 + 1)e


i
sin

(cos )
=
1
k
2

=0
(2 + 1)
2
sin
2

P
2

(cos )
And now to nd the total cross section,

tot
=
_
[f()[
2
d
=
_
2
0
d
_

0
[f()[
2
sin d
=2
_

0
_
1
k
2

=0
(2 + 1)
2
sin
2

P
2

(cos )
_
sin d
=
2
k
2

=0
(2 + 1)
2
sin
2

_

0
[P

(cos )]
2
sin d
=
2
k
2

=0
(2 + 1)
2
sin
2

_

0
_
1
2

!
_
d
d cos
_

_
cos
2
1
_

_
2
sin d
=
2
k
2

=0
(2 + 1)
2
sin
2

1
2
2
(!)
2
_

0
_
d
d cos
_
2
_
cos
2
1
_
2
sin d
=
2
k
2

=0
(2 + 1)
2
sin
2

1
2
2
(!)
2
_

0
_
d
dx
_
2
_
x
2
1
_
2
sin d
=
2
k
2

=0
(2 + 1)
2
sin
2

1
2
2
(!)
2
_

0
__
d
dx
_
_
x
2
1
_
_
2
sin d
=
2
k
2

=0
(2 + 1)
2
sin
2

1
(!)
2
_

0
(cos )
2
sin d
=
2
k
2

=0
(2 + 1)
2
sin
2

1
(!)
2
_
1 + (1)
2
2 + 1
_
=
4
k
2

=0
(2 + 1) sin
2

1
(!)
2
(4.6.20)
Where I used the fact that d = sin dd and also the results from problem 3. Therefore we have
shown that

You might have noticed that there is an extra factor of ()


2
in (4.6.20). I was not able to get rid of it. So when I say that we
have shown that I suppose I really mean we have nearly shown that.
W. Erbsen HOMEWORK #9

tot
=
4
k
2

=0
(2 + 1) sin
2

4.7 Homework #9
Problem 1
By dierentiating the Legendre polynomial generating function
g(x, t) =
1

1 2xt +t
2
(4.7.1)
with respect to t and with respect to x, obtain the recurrence relations for the Legendre polynomials, and prove
that P
n
(x) satises the Legendre dierential equation.
Solution
We know that the generating function g(x, t) can be expressed as
g(x, t) =
1

1 2xt + t
2
=

n=0
t
n
P
n
(x) (4.7.2)
So, as per the suggestion, we dierentiate (4.7.1) rst with respect to t:
dg(x, t)
dt
=
d
dt
1

1 2xt + t
2
=
x t
(1 2xt +t
2
)
3
/2
=
x t
(1 2xt +t
2
)
g(x, t) (4.7.3)
Doing the same for the RHS of (4.7.2),
dg(x, t)
dt
=
d
dt

n=0
t
n
P
n
(x) =

n=0
nt
n1
P
n
(x)
Inserting this and (4.7.2) into the (4.7.3),

n=0
nt
n1
P
n
(x) =
x t
(1 xt +t
2
)

n=0
t
n
P
n
(x)
(1 xt + t
2
)

n=0
nt
n1
P
n
(x) =(x t)

n=0
t
n
P
n
(x)

n=0
nt
n1
P
n
(x) 2x

n=0
nt
n
P
n
(x) +

n=0
nt
n+1
P
n
(x) =x

n=0
t
n
P
n
(x)

n=0
t
n+1
P
n
(x)
CHAPTER 4: MATHEMATICAL METHODS 325

n=0
..
n 1 =
nt
n1
P
n
(x) +

n=0
..
n+1=
(n + 1)t
n + 1
P
n
(x) =

n=0
..
n =
x(1 + 2n)t
n
P
n
(x)
Shifting the indices as indicated:

=0
( + 1)t

P
+1
(x) +

=0
t

P
1
(x) =

=0
x(1 + 2)t

(x)
( + 1)P
+1
(x) x(1 + 2)P

(x) = P
1
(x) (4.7.4)
Since we require that each coecient should vanish. We can nd a second recursion relation by similarly
integrating (4.7.2) by x:
dg(x, t)
dx
=
d
dx
1

1 2xt +t
2
=
t
(1 2xt +t
2
)
3
/2
=
t
(1 2xt +t
2
)
g(x, t) (4.7.5)
And the the derivative of the RHS of (4.7.2) is
dg(x, t)
dx
=
d
dx

n=0
t
n
P
n
(x) =

n=0
t
n
P

n
(x)
And now

n=0
t
n
P

n
(x) =
t
(1 2xt + t
2
)

n=0
t
n
P
n
(x)

n=0
t
n
P

n
(x) 2x

n=0
t
n+1
P

n
(x) +

n=0
t
n+2
P

n
(x) =

n=0
t
n+1
P
n
(x)

n=0
..
n=
t
n
P

n
(x) +

n=0
..
n+2=
t
n+2
P

n
(x) =

n=0
..
n+1=
t
n+1
[2xP

n
(x) +P
n
(x)]
Shifting the indices once more,

=0
t

(x) +

=0
t

2
(x) =

=0
t

_
2xP

1
(x) + P
1
(x)

(x) +P

2
(x) =2xP

1
(x) + P
1
(x)
P

+1
(x) 2xP

(x) = P

(x) P

1
(x) (4.7.6)
We can combine (4.7.4) and (4.7.6) as follows. First, moving all the terms of (4.7.4) to the left and
taking the derivative with respect to x yields
( + 1)P

+1
(x) x(1 + 2)P

(x) (1 + 2)P

(x) +P

1
(x) = 0 (4.7.7)
W. Erbsen HOMEWORK #9
Now moving all the terms of (4.7.6) to the left, and multiplying by
P

+1
(x) + 2xP

(x) + P

(x) P

1
(x) = 0 (4.7.8)
Adding (4.7.7) and (4.7.8),
P

+1
(x) = xP

(x) + (1 + )P

(x)
Many more (in fact, innitely more) recursion relations may be derived, in addition to the three I have
found here, there are two more that can be found in M&W that become useful:
xP

(x) P

(x) P

1
(x) =0 (4.7.9)
P

(x) xP

1
(x) P
1
(x) =0 (4.7.10)
Multiplying (4.7.9) by x:
x
2
P

(x) xP

(x) xP

1
(x) = 0 (4.7.11)
(4.7.12)
Now take (4.7.11) and subtract (4.7.10):
x
2
P

(x) xP

(x) xP

1
(x) P

(x) +xP

1
(x) +P
1
(x) = 0
(x
2
1)P

(x) xP

(x) +P
1
(x) = 0 (4.7.13)
Now taking the derivative of (4.7.13) with respect to x:
2xP

(x) +x
2
P

(x) P

(x) P

(x) xP

(x) + P

1
(x) = 0
x
2
P

(x) P

(x) + (2 )xP

(x) P

(x) + P
1
(x) = 0 (4.7.14)
At this point we take (4.7.9), solve for P

1
(x), and substitute into (4.7.14):
x
2
P

(x) P

(x) + (2 )xP

(x) P

(x) +xP

(x)
2
P

(x) = 0
(x
2
1)P

(x) + 2xP

(x) xP

(x) P

(x) +xP

(x)
2
P

(x) = 0
(x
2
1)P

(x) + 2xP

(x) ( + 1)P

(x) = 0
Which is the Legendre dierential equation.
Problem 2
By dierentiating the Bessel generating function
g(x, t) = e
x
2
(t
1
/t)
(4.7.15)
with respect to t and with respect to x, obtain the recurrence relations for the Bessel functions, and prove that
J
n
(x) satises the Bessel dierential equation.
CHAPTER 4: MATHEMATICAL METHODS 327
Solution
In very much the same spirit as the last problem, we are given that the generating function is
g(x, t) = e
x
2
(t
1
/t)
=

n=0
t
n
J
n
(x) (4.7.16)
Dierentiating (4.7.15) rst with respect to t,
dg(x, t)
dt
=
d
dt
e
x
2
(t
1
/t)
=
x
2
_
1
1
/
t
2
_
e
x
2
(t
1
t
)
=
x
2
_
1
1
/
t
2
_
g(x, t)
=

n=0
nt
n1
J
n
(x)
Playing with this a little bit, we have

n=0
nt
n1
J
n
(x) =
x
2
_
1
1
t
2
_

n=0
t
n
J
n
(x)
2

n=0
..
n1=
nt
n1
J
n
(x) =

n=0
..
n=
xt
n
J
n
(x)

n=0
..
n2=
xt
n2
J
n
(x)
Shift the indices,
2

=0
( + 1)t

J
+1
(x) =

=0
xt

(x) +

=0
t

J
+2
2( + 1)J
+1
(x) =xJ

(x) + xJ
+2
(x)

2
x
J

(x) = J
1
(x) + J
+1
(x) (4.7.17)
Now dierentiating (4.7.15) with respect to x,
dg(x, t)
dx
=
d
dx
e
x
2
(t
1
/t)
=
1
2
_
t
1
t
_
e
x
2
(t
1
/t)
=
1
2
_
t
1
t
_
g(x, t)
=

n=0
t
n
J

n
(x)
And now
W. Erbsen HOMEWORK #9
2

n=0
t
n
J

n
(x) =
_
t
1
t
_

n=0
t
n
J
n
(x)
2

n=0
..
n=
t
n
J

n
(x) =

n=0
..
n+1=
t
n+1
J
n
(x)

n=0
..
n1=
t
n1
J
n
(x)
Shifting,
2

=0
t

(x) =

=0
t

J
1
(x)

=0
t

J
+1
(x)
2J

(x) = J
1
(x) J
+1
(x) (4.7.18)
If we subtract (4.7.17),
J

(x) =

x
J

(x) J
+1
(x) (4.7.19)
We can arrive at one more recursion relation if we take (4.7.19) and subtract (4.7.17):
J

(x) = J
1
(x)

x
J

(x) (4.7.20)
To prove that J
n
(x) satises the Bessel dierential equation (4.7.15), we start by multiplying (4.7.20) by
x:
xJ

(x) xJ
1
(x) +J

(x) = 0 (4.7.21)
We now dierentiate (4.7.21) with respect to x:
d
dx
[xJ

(x) xJ
1
(x) +J

(x)] =J

(x) + xJ

(x) J
1
(x) xJ

1
(x) + J

(x)
=xJ

(x) + ( + 1)J

(x) xJ

1
(x) J
1
(x) (4.7.22)
Multiplying (4.7.22) by x,
xJ

(x) + ( + 1)J

(x) xJ

1
(x) J
1
(x) =0
x
2
J

(x) + x( + 1)J

(x) x
2
J

1
(x) xJ
1
(x) =0 (4.7.23)
And multiply (4.7.21) by :
xJ

(x) xJ
1
(x) +
2
J

(x) = 0 (4.7.24)
Now take (4.7.22) and subtract (4.7.21):
x
2
J

(x) + x( + 1)J

(x) x
2
J

1
(x) xJ
1
(x) xJ

(x) + xJ
1
(x)
2
J

(x) =0
x
2
J

(x) + xJ

(x) x
2
J

1
(x) + ( 1)J
1
(x)
2
J

(x) =0 (4.7.25)
At this point we take (4.7.19) and shift the index from to 1,
CHAPTER 4: MATHEMATICAL METHODS 329
J

1
(x) =
1
x
J
1
(x) J

(x)
And multiplying this by x and solving for J
1
(x),
xJ

1
(x) = ( 1)J
1
(x) xJ

(x) J
1
(x) =
x
1
J

1
(x) +
x
1
J

(x)
Substituting this into (4.7.25),
x
2
J

(x) + xJ

(x) x
2
J

1
(x) + ( 1)
_
x
1
J

1
(x) +
x
1
J

(x)
_

2
J

(x) =0
x
2
J

+xJ

(x) x
2
J

1
(x) +x
2
J
1
(x) + x
2
J

(x)
2
J

= 0
x
2
J

(x) + xJ

(x) +
_
x
2

2
_
J

(x) = 0
Which is the Bessel dierential equation.
Problem 3
Prove the normalization condition for the Bessel functions
_

0
J
n
(kr) J
n
(k

r) drr =
1
k
(k

k) (4.7.26)
Solution
It was shown in class, and additionally can be found in M&W that:
_
b
a
J
n
(r)J
n
(r)rdr =
1

2

2
[rJ
n
(r)J

n
(r) rJ
n
(r)J

n
(r)[
b
a
(4.7.27)
Where I dispensed k and k

in favor of and , respectively in order to avoid confusion with the


derivatives. First lets choose a 0 as in (4.7.26):
_
b
0
J
n
(r)J
n
(r)rdr =
1

2

2
[rJ
n
(r)J

n
(r) rJ
n
(r)J

n
(r)[
b
0
=
1

2

2
[rJ
n
(b)J

n
(b) rJ
n
(b)J

n
(b)] (4.7.28)
And since both J

(r) and J

(r) evaluated at the endpoints equal zero, then (4.7.36) too must vanish.
To see that this is the case at x = 0, it is illuminating to recall that J

(0) = J
+1
(0) = 0. We should
also recognize that J

() 0. Both of these properties become very evident when plotting the relevant
functions and recognizing the respective behaviors. Therefore, (4.7.28) becomes
W. Erbsen HOMEWORK #9
_
b
0
J
n
(r)J
n
(r)rdr = 0 (4.7.29)
For the case in which and are equal, we must evaluate
_
b
0
J
n
(r)J
n
(r)rdr =
_
b
0
J
n
(r)J
n
(r)rdr =
_
b
0
[J
n
(r)]
2
rdr
Which we now set equal to the RHS of (4.7.28)
_
b
0
[J
n
(r)]
2
rdr =
rJ
n
(b)J

n
(b) rJ
n
(b)J

n
(b)

2

2
(4.7.30)
Evaluating the RHS of (4.7.30) is not so easy; we must take the limit as , but since this is
indeterminate we must use LH ospitals Rule:
_
b
0
[J
n
(r)]
2
r dr = lim

_
rJ
n
(b)J

n
(b) rJ
n
(b)J

n
(b)

2

2
_
= lim

_
rJ
n
(b)J

n
(b) + rJ
n
(b)J

n
(b) rJ

n
(b)J

n
(b)
2
_
=
r [J

n
(b)]
2
rJ
n
(b)J

n
(b) rJ
n
(b)J

n
(b)
2
(4.7.31)
We now solve Bessels dierential equation for J

n
(b):

2
J

n
(b) +J

n
(b) + (
2
n
2
)J
n
(b) = 0 J

n
(b) =
J

n
(b) (
2
n
2
)J
n
(b)

2
(4.7.32)
Substituting (4.7.32) into (4.7.31),
_
b
0
[J
n
(r)]
2
rdr =
r [J

n
(b)]
2
rJ
n
(b)J

n
(b) rJ
n
(b)
_

n
(b)(
2
n
2
)Jn(b)

2
_
2
=
r [J

n
(b)]
2
rJ
n
(b)J

n
(b) +rJ
n
(b)J

n
(r) + r
_
1
n
2

2
_
[J
n
(b)]
2
2
=
r
2
_
[J

n
(b)]
2
+
_
1
n
2

2
_
[J
n
(b)]
2
_
(4.7.33)
So that if and represent two unique roots of order n then (4.7.33) must vanish.
Problem 4
A disk of radius R in the xy-plane (z = 0) is kept at a constant potential
0
and the rest of the plane z = 0 is
kept at zero potential. As shown in class, the potential for z > 0 is given by
(r, z) =
0
R
_

0
dkJ
1
(kR)J
0
(kr)e
kz
(4.7.34)
CHAPTER 4: MATHEMATICAL METHODS 331
Hence, the potential above the center of the disk (r = 0) is given by the integral
(0, z) =
0
R
_

0
dkJ
1
(kR)e
kz
(4.7.35)
Using the Bessel function recurrence relations and the integral representation of the Bessel function, show that the
last expression can be reduced to
(0, z) =
0
_
1
z

z
2
+R
2
_
(4.7.36)
Solution
According to (4.7.19),
J
1
(x) = J

0
(x)
And (4.7.35) becomes
(0, z) =
0
R
_

0
J

0
(kR)e
kz
dk
And at this point we wish to undergo a change of variables x = kR k =
x
/
R
:
(0, z) =
0
R
_

0
J

0
(x)e

xz
/R
dx
R
=
0
_

0
J

0
(x)e

xz
/R
dx
Where we must integrate by parts:
u = e

xz
/R
, du =
z
R
e

xz
/R
dv = J

0
(x), v = J
0
(x)
So we now have
(0, z) =
0
_
_
J
0
(x)e

xz
/R

0
+
z
R
_

0
J
0
(x)e

xz
/R
dx
_
=
0
_
1 +
z
R
_

0
J
0
(x)e

xz
/R
dx
_
(4.7.37)
At this point we pause, try a myriad of obscure substitutions, get frustrated, have a beer, and then
realize that the integral in (4.7.37) has actually been tabulated by the likes of Gradshteyn and Ryzhik
as Eq. 6.611.1: ?
_

0
e
x
J

(x) dx =

_
_

2
+
2

2
+
2
Where in our case =
z
/
R
, = 1 and = 0:
W. Erbsen HOMEWORK #10
_

0
e

xz
/R
J
0
(x) dx =
1
_
(
z
/
R
)
2
+ 1
=
1
1
/
R

z
2
+R
2
=
R

z
2
+ R
2
(4.7.38)
And substituting (4.7.38) into (4.7.37),
(0, z) =
0
_
1 +
z
R
_
R

z
2
+ R
2
__
From which we nally arrive at (4.7.36), thus completing the problem:
(0, z) =
0
_
1
z

z
2
+ R
2
_
4.8 Homework #10
Problem 1
In class we shows that the lowest order spherical Bessel function is given by
j
0
(x) =
sin x
x
(4.8.1)
Using the recurrence relations, nd j
1
(x) and j
2
(x), and also prove that
j

(x) = (1)

_
1
x

x
_

j
0
(x) (4.8.2)
Solution
Recall that one of the recurrence relations for Bessel functions reads
j
+1
(x) =

x
j

(x) j

(x) (4.8.3)
Which was rigorously proven in Homework #9, and subsequently will be treated as an axiom in this
assignment. We are given j
0
(x), so we need to calculate j
1
(x) and j
2
(x) using our recursion relation
(4.8.3). We start by nding the rst derivative of (4.8.1):
j

0
(x) =
cos x
x

sin x
x
2
We now substitute this into (4.8.3),
j
1
(x) =
0
x
j
0
(x) j

0
(x)
=
sin x
x
2

cos x
x
(4.8.4)
CHAPTER 4: MATHEMATICAL METHODS 333
Taking the rst derivative of j
1
(x) yields
j

1
(x) =
2 cos x
x
2

2 sinx
x
3
+
sinx
x
From which we can now nd j
2
(x):
j
2
(x) =
1
x
j
1
(x) j

1
(x)
=
sin x
x
3

cos x
x
2

2 cos x
x
2
+
2 sin x
x
3

sin x
x
=
3 sinx
x
3

3 cos x
x
2

sin x
x
(4.8.5)
In order to prove (4.8.2), it will be most convenient to show that (4.8.2) works when substituted into
(4.8.3). I will employ proof by induction. So the rst step is to show that it works for = 0,
j
1
(x) = j

0
(x)
=
d
dx
_
(1)
0
x
0
_
1
x

x
_
0
j
0
(x)
_
=
d
dx
sin x
x
=
sin x
x
2

cos x
x
=j
1
(x)
So it works for = 0. Assuming that it holds for all , we now show that it holds for + 1. First nd
the derivative of j

(x):
j

(x) =
d
dx
_
(x)

_
1
x
d
dx
_

sin x
x
_
=
_
d
dx
(x)

_
_
_
1
x
d
dx
_

sin x
x
_
+ (x)

d
dx
_
_
1
x
d
dx
_

sin x
x
_
=(x)
1
_
_
1
x
d
dx
_

sinx
x
_
+ (x)

d
dx
_
_
1
x
d
dx
_

sin x
x
_
=
(x)

x
_
_
1
x
d
dx
_

sin x
x
_
+ (x)

d
dx
_
_
1
x
d
dx
_

sin x
x
_
=

x
(x)

_
_
1
x
d
dx
_

sin x
x
_
+ (x)

d
dx
_
_
1
x
d
dx
_

sin x
x
_
=

x
j

(x) + (x)

d
dx
_
_
1
x
d
dx
_

sin x
x
_
(4.8.6)
So, lets nd j
+1
(x):
j
+1
(x) =(x)
+1
_
1
x
d
dx
_
+1
sin x
x
W. Erbsen HOMEWORK #10
=(x)

(x)
_
1
x
d
dx
_

_
1
x
d
dx
_
sin x
x
= (x)

d
dx
_
_
1
x
d
dx
_

sin x
x
_
=

x
j

(x) j

(x)
Where I used (4.8.6). Since (4.8.2) works for = 0 (the base step) and also + 1 (the inductive step),
then it follows that it works for (the hypothesized step).
Problem 2
Using the denition of Neumann functions in terms of Bessel functions, show that for the spherical Neumann
functions,

0
(x) =
cos x
x
(4.8.7)
Using the recurrence relations, nd
1
(x) and
2
(x).
Solution
Analogous to (4.8.3), the recursion relation for the Neumann functions takes the form

+1
(x) =

x

(x)

(x) (4.8.8)
So, from = 0 we can nd
1
(x):

1
(x) =

x

0
(x)

0
(x) ==

0
(x) (4.8.9)
So, in nding the rst derivative of
0
(x), we nd
1
(x):

1
(x) =

0
(x) =
d
dx
cos x
x
=
sin x
x

cos x
x
2
(4.8.10)
Applying the same process letting = 1,

1+1
(x) =
1
x

1
(x)

1
(x)
=
sin x
x
2

cos x
x
3

1
(x) (4.8.11)
Where we nd that

1
(x) =
d
dx
_

sin x
x

cos x
x
2
_
=
cos x
x
+
2 sinx
x
2
+
sin x
x
2
+
2 cos x
x
3
CHAPTER 4: MATHEMATICAL METHODS 335
=
cos x
x
+
2 sinx
x
2
+
2 cos x
x
3
(4.8.12)
Substituting (4.8.12) into (4.8.11), we nd that

2
(x) =
sin x
x
2

cos x
x
3
+
cos x
x

2 sinx
x
2

2 cos x
x
3
=
cos x
x

3 sinx
x
3

3 cos x
x
3
(4.8.13)
Problem 3
Transform the hyperbolic equation
y
2

xx
x
2

yy
= 0 (4.8.14)
into the canonical form of

= f(...)
Solution
The general formalismof the Method of Characteristics entails taking some arbitrary dierential equation,
Au
2
xx
+ 2Bu
xy
+ Cu
2
yy
= 0

Au
2

+ 2

Bu

+

Cu
2

+ R = 0 (4.8.15)
Where

A =A
2
x
+ 2B
x

y
+C
2
y

B =A
x

x
+ B(
x

y
+
y

x
) +C
y

C =A
2
x
+ 2B
x

y
+ C
2
y
R =(A
xx
+ 2B
xy
+C
yy
) u

+ (A
xx
+ 2B
xy
+C
yy
) u

So, from (4.8.14), it is apparent that A = y


2
, B = 0, and C = x
2
. Since (4.8.14) is hyperbolic, the
characteristics can be found by
dy
dx
=
B

B
2
4AC
2A
=

4AC
2A
=
x
y
Solving and integrating this we arrive at
x
2
2

y
2
2
= 0 x
2
y
2
= 0 (4.8.16)
Where we now dene
(x, y) = = x
2
+y
2
, and (x, y) = = x
2
y
2
(4.8.17)
Taking some partial derivatives,

x
=2x,
y
= 2y,
xx
=
yy
= 2,
xy
= 0
W. Erbsen HOMEWORK #10

x
=2x,
y
= 2y,
xx
= 2,
yy
= 2,
xy
= 0
Now nding the coecients in (4.8.15),

A =A
2
x
+ 2B
x

y
+C
2
y
=y
2

2
x
x
2

2
y
=y
2
(2x)
2
x
2
(2y)
2
= 0

B =A
x

x
+ B(
x

y
+
y

x
) +C
y

y
=y
2

x
x
2

y
=y
2
(2x)(2x) x
2
(2y)(2y)
=4x
2
y
2
+ 4x
2
y
2
=8x
2
y
2

C =A
2
x
+ 2B
x

y
+ C
2
y
=y
2

2
x
x
2

2
y
=y
2
(2x)
2
x
2
(2x)
2
=0
R =(A
xx
+ 2B
xy
+C
yy
) u

+ (A
xx
+ 2B
xy
+C
yy
) u

=
_
y
2

xx
x
2

yy
_
u

+
_
y
2

xx
x
2

yy
_
u

=2
_
y
2
x
2
_
u

+ 2
_
y
2
+ x
2
_
u

And putting all these into (4.8.15),

Au
2

+ 2

Bu

+

Cu
2

+ R = 0
2(8x
2
y
2
)u

+ 2
_
y
2
x
2
_
u

+ 2
_
y
2
+x
2
_
u

= 0
8x
2
y
2
u

+
_
y
2
x
2
_
u

+
_
y
2
+x
2
_
u

= 0 (4.8.18)
We can combine our original denitions of and from (4.8.17) to obtain:
x
2
=
+
2
, and y
2
=

2
So that we can now rewrite (4.8.18) as
8
_
+
2
__

2
_
u

+u

= 0
2
_

2
_
u

+u

= 0
And the canonical form of (4.8.14) becomes
u

=
u

2 (
2

2
)
CHAPTER 4: MATHEMATICAL METHODS 337
Problem 4
Transform the elliptical equations
y
2

xx
+ x
2

yy
= 0 (4.8.19a)

xx
+ (1 + y)
2

yy
= 0 (4.8.19b)
into the canonical form of

= f(...).
Solution
Starting from (4.8.19a), it is apparent to the casual observer that A = y
2
, B = 0 and C = x
2
. Further-
more, we can go on to nd the characteristics of this equation:
dy
dx
=
B

B
2
4AC
2A
=

AC
A
=
ix
y
(4.8.20)
Solving and integrating, we nd that
ix
2
2

y
2
2
= 0 ix
2
y
2
= 0 (4.8.21)
At this point we set
(x, y) = = ix
2
+y
2
, and (x, y) = = ix
2
y
2
(4.8.22)
With corresponding partial derivatives

x
=2ix,
y
= 2y,
xx
= 2i,
yy
= 2,
xy
= 0

x
=2ix,
y
= 2y,
xx
= 2i,
yy
= 2,
xy
= 0
And now nding the coecients,

A =A
2
x
+ 2B
x

y
+C
2
y
=y
2
(2ix)
2
+x
2
(2y)
2
= 4x
2
y
2
+ 4x
2
y
2
=0

B =A
x

x
+ B(
x

y
+
y

x
) +C
y

y
=y
2
(2ix)(2ix) + x
2
(2y)(2y)
= 4x
2
y
2
4x
2
y
2
= 8x
2
y
2

C =A
2
x
+ 2B
x

y
+ C
2
y
=y
2
(2ix)
2
+x
2
(2y)
2
= 4x
2
y
2
+ 4x
2
y
2
W. Erbsen HOMEWORK #10
=0
R =(A
xx
+ 2B
xy
+C
yy
) u

+ (A
xx
+ 2B
xy
+C
yy
) u

=
_
y
2
(2i) + x
2
(2)
_
u

+
_
y
2
(2i) +x
2
(2)
_
u

=2
_
iy
2
+x
2
_
u

+ 2
_
iy
2
x
2
_
u

Substituting these in we have


2

Bu

+ R = 0
16x
2
y
2
u

+ 2
_
iy
2
+ x
2
_
u

+ 2
_
iy
2
x
2
_
u

= 0 (4.8.23)
And it is possible to eliminate x and y by making the following substitution:
x
2
=
+
2i
, and y
2
=

2
Putting this in (4.8.23) and simplifying,
16
_

2

2
4i
_
u

2iu

+ 2iu

2
_

2
_
u

+ u

= 0
From which we nally arrive at
u

=
u

2 (
2

2
)
In the same spirit for (4.8.19b), we note that A = 1, B = 0, C = (1 +y)
2
. Finding the characteristics,
dy
dx
=
B

B
2
4AC
2A
=

AC
A
= i(1 +y) (4.8.24)
Solving and integrating,
1
1 +y
dy = i dx log (1 + y) = ix
Assigning our two independent variables,
(x, y) = = ix + log (1 + y), and (x, y) = = ix log (1 +y) (4.8.25)
Lets take some partial derivatives:

x
=i,
y
=
1
1 + y
,
xx
= 0,
yy
=
1
(1 + y)
2
,
xy
= 0

x
=i,
y
=
1
1 +y
,
xx
= 0,
yy
=
1
(1 + y)
2
,
xy
= 0
And the coecients are
CHAPTER 4: MATHEMATICAL METHODS 339

A =A
2
x
+ 2B
x

y
+C
2
y
=A
2
x
+ (1 + y)
2

2
y
=(i)(i) + (1 + y)
2
1
(1 + y)
2
= 1 + 1
=0

B =A
x

x
+ B(
x

y
+
y

x
) +C
y

y
=
x

x
+ (1 +y)
2

y
=(i)(i) + (1 + y)
2
_
1
1 +y
__

1
1 + y
_
= 1 1
= 2

C =A
2
x
+ 2B
x

y
+ C
2
y
=
2
x
+ (1 +y)
2

2
y
=(i)(i) + (1 + y)
2
_
1
1 +y
_
2
= 1 + 1
=0
R =(A
xx
+ 2B
xy
+C
yy
) u

+ (A
xx
+ 2B
xy
+C
yy
) u

=(1 + y)
2

yy
u

+ (1 + y)
2

yy
u

=(1 + y)
2
_

1
(1 + y
2
)
2
_
u

+ (1 + y)
2
_
1
(1 + y)
2
_
u

= u

+ u

And now we can nd the canonical form of (4.8.19b):


2

Bu

+R = 0
4u

+u

= 0
And nally
u

=
u

4
W. Erbsen HOMEWORK #10
Problem 5
Classify the following equations in appropriate regions and transform them into canonical form:

xx
y
2

yy
+
x
+ x
2
= 0 (4.8.26a)

xx
+ x
yy
= 0 (4.8.26b)
Solution
For (4.8.26a) it is important that the elements that characterize the dominant behavior are the second
order terms, so analyzing this (the principle part), we see that A = 1, B = 0, and C = y
2
. Furthermore,
we can see that:
B
2
AC = y
2
> 0 hyperbolic
Solving, integrating:
dy
dx
= y x log y = 0
And now we can say
(x, y) = = x + log y, and (x, y) = = x log y
Take some partials:

x
=1,
y
=
1
y
,
xx
= 0,
yy
=
1
y
2
,
xy
= 0

x
=1,
y
=
1
y
,
xx
= 0,
yy
=
1
y
2
,
xy
= 0
And the coecients are

A =A
2
x
+ 2B
x

y
+C
2
y
=
2
x
y
2

2
y
=1 y
2
1
y
2
=1 1
=0

B =A
x

x
+ B(
x

y
+
y

x
) +C
y

y
=
x

x
y
2

y
=1 y
2
_
1
y
__
1
y
_
=1 + 1
=2
CHAPTER 4: MATHEMATICAL METHODS 341

C =A
2
x
+ 2B
x

y
+ C
2
y
=
2
x
y
2

2
y
=1 y
2
_

1
y
_
2
=1 1
=0
R =(A
xx
+ 2B
xy
+C
yy
) u

+ (A
xx
+ 2B
xy
+C
yy
) u

=
_

xx
y
2

yy
_
u

+
_

xx
y
2

yy
_
u

=
_
1 y
2
_
1
y
__
u

+
_
1 y
2
_

1
y
__
u

=(1 y) u

+ (1 + y) u

And we now have


4u

+ (1 y) u

+ (1 +y) u

= 0
And consequently
4u

+
_
1 exp
_

2
__
u

+
_
1 +
_

2
__
u

= 0
This last equation yields A = 1, B = 0, C = x, which is elliptical, assuming that x > 0:
B
2
AC = x < 0 elliptical
Find the characteristics:
dy
dx
=

x = i

x
Solving and integrating, we arrive at
(x, y) = =
2
3
ix
3
/2
+y, and (x, y) = =
2
3
ix
3
/2
y
With partial derivatives

x
=i

x ,
y
= 1,
xx
=
1
2

x
,
yy
= 0,
xy
= 0

x
=

x ,
y
= 1,
xx
=
1
2

x
,
yy
= 0,
xy
= 0
Finding the coecients

A =A
2
x
+ 2B
x

y
+C
2
y
=
2
x
+x
2
y
W. Erbsen HOMEWORK #11
=(i

x )
2
+x
= x +x
=0

B =A
x

x
+ B(
x

y
+
y

x
) +C
y

y
=(i

x )(i

x ) + x(1)(1)
= x x
= 2x

C =A
2
x
+ 2B
x

y
+ C
2
y
=(

x )
2
+ x(1)
2
=x x
=0
R =(A
xx
+ 2B
xy
+C
yy
) u

+ (A
xx
+ 2B
xy
+C
yy
) u

=(
xx
+ x
yy
) u

+ (
xx
+ x
yy
) u

=
_
i
2

x
_
u

+
_
i
2

x
_
u

Our equation becomes


4xu

+
i
2

x
(u

+u

) = 0
And nally
4
_
3
4i
( )
_2
/3
u

+
i
2
_
4i
3
1

_1
/3
(u

+ u

) = 0
4.9 Homework #11
Problem 1
Consider a thin half pipe of unit radius laying on the ground. It is heated by radiation from above. We take
the initial temperature of the pipe and the temperature of the ground to be zero. We model this problem with a
heat equation with a source term.
u
t
= u
xx
+ Asin x, u(0, t) = u(, t) = 0, u(x, 0) = 0 (4.9.1)
CHAPTER 4: MATHEMATICAL METHODS 343
Solution
We begin by assuming that our solution can be written in terms of a steady-state solution and a transient
one
u(x, t) = (x) +w(x, t) (x) : Steady-state solution
w(x, t) : Transient solution
To nd the equilibrium solution, we note that
t
(x) = 0, and substitute (x) into (4.9.1):

t
=
xx
+Asin x
xx
=
A

sin x, (0) = () = 0 (4.9.2)


Integrating (4.9.2) once,
_
x
0

2
x
2
(x) dx =
A

_
x
0
sin x dx

x
(x) =
A

cos x +C
1
(4.9.3)
And now integrating (4.9.3),
_
x
0

x
(x) dx =
A

_
x
0
cos x dx +
_
x
0
C
1
dx
(x) =
A

sinx + C
1
x + C
2
(4.9.4)
Applying our rst boundary condition to (4.9.4), we nd that
(0) =C
2
,= 0 C
2
= 0
And similarly, applying th4e second boundary condition,
() =C
1
,= 0 C
1
= 0
And (4.9.4) becomes
(x) =
A

sin x (4.9.5)
And now we do the same for w(x, t):
w
t
= w
xx
+Asin x w
t
= w
xx
(4.9.6)
We can see that (4.9.6) can be expressed as an eigenvalue problem, the explicit details of which are shown
in subsequent problems and I will not repeat them here. Applying all our given conditions, we arrive at
w
n
(x, t) =
_
2

sin (nx) e
n
2
t
(4.9.7)
And in order for (4.9.7) to work, we require that it corresponds to the Fourier coecient a
n
from
w(x, t) =

a
n
w
n
(x, t):
W. Erbsen HOMEWORK #11
2 4 6 8 10
Time

2
3
4

Position
Figure 4.1: Contour plot for Problem 1.
a
n
=
_
2

_

0
sin(nx) (x) dx
=
_
2

_

0
sin (nx) sin(x) dx
=
_
2

sin (n)
n
2
1
(4.9.8)
Using (4.9.7) and (4.9.8), we have
w(x, t) =

n=1
a
n
w(x, t) =

n=1
_
_
2

sin (n)
n
2
1
__
_
2

sin (nx) e
n
2
t
_
=
2A

n=1
sin
2
(n) e
n
2
t
n
2
1
=
A

sin(x)e
t
(4.9.9)
Which was arrived at via the following Mathematica code:
V = Sum[-Sqrt[2/Pi] Integrate[Sin[n*x]*A/kappa Sin[x],{x,0,Pi}]*Sqrt[2/Pi]*Sin[n*x]
*Exp[-kappa*n^2*t],{n,1,Infinity}]
>>Out[331] = -(A*Exp[-t*kappa]*Sin[x])/kappa
CHAPTER 4: MATHEMATICAL METHODS 345
And nally, to get the full solution for u(x, t) we combine our steady-state solution from (4.9.5) and our
transient solution from (4.9.9):
u(x, t) = w(x, t) +(x) =
A

sin(x)e
t
+
A

sin(x)
Which may be rewritten as
u(x, t) =
A

sin(x)
_
1 e
t

A contour plot of my solution can be seen in Fig. (4.1), where I set A = = 1.


Problem 2
Obtain Poissons formula to solve the Dirichlet problem for the circular region 0 r < R, 0 < 2. That is,
determine a solution (r, ) to solve Laplaces equation
2
= 0 in polar coordinates given (R, ). Show that
(r, ) =
1
2
_
2
0
(R, )
R
2
r
2
R
2
+ r
2
2Rr cos ( )
d (4.9.10)
Solution
We begin by noting that Laplaces equation in polar coordinates is

2
f =
1
r

r
_
r
f
r
_
+
1
r
2

2
f

2
= 0 (4.9.11)
Which should be solvable using separation of variables, where we assume that the solution can take the
form of f(r, ) = R(r)(). Substituting this into (4.9.11) and separating the variables,
r
R
d
dr
_
r
dR
dr
_
=
1

d
2

d
2
=
2
(4.9.12)
Where we set each equation equal to the square of some equilibrium constant . We rst take the
equation for R(r),
r
R
d
dr
_
r
dR
dr
_
=
2
r
R
_
r
d
2
R
dr
2
+
dR
dr
_
=
2
r
2
d
2
R
dr
2
+ r
dR
dr

2
R =0 (4.9.13)
Now we must solve the equation for () from (4.9.12):
W. Erbsen HOMEWORK #11

d
2

d
2
=
2
d
2

d
2
+
2
=0 (4.9.14)
We require our solution for () to be periodic, such that ( + 2) = (). Therefore we set = n
where n is an integer, and (4.9.14) becomes
d
2

d
2
+ n
2
=0
Which has solutions
() =A
n
e
in
+B
n
e
in
= A
n
cos(n) + B
n
sin(n) (4.9.15)
And similarly we can rewrite (4.9.13) as
r
2
d
2
R
dr
2
+r
dR
dr
n
2
R =0
Which has solutions of the form
R(r) = C
n
r
n
+D
n
r
n
(4.9.16)
Combining our solutions from (4.9.15) and (4.9.16),
f
n
(r, ) = R(r)() =
_
C
n
r
n
+ D
n
r
n
_
(A
n
cos(n) + B
n
sin(n))
Choosing D
n
= 0 for boundary conditions,
f
n
(r, ) = C
n
r
n
(A
n
cos(n) + B
n
sin(n))
And more generally,
f
n
(r, ) =

n=0
C
n
r
n
(A
n
cos(n) + B
n
sin(n))
=C
0
r
0
(A
0
cos(0) +B
0
sin(0)) +

n=1
C
n
r
n
(A
n
cos(n) +B
n
sin(n))
=C
0
A
0
+

n=1
C
n
r
n
(A
n
cos(n) +B
n
sin(n)) (4.9.17)
The solution of (4.9.17) is most easily found if we rst assume that R = r = 1,
f(1, ) =C
0
A
0
+

n=1
C
n
(A
n
cos(n) +B
n
sin(n))
=a
0
+

n=1
(a
n
cos(n) +b
n
sin(n)) (4.9.18)
CHAPTER 4: MATHEMATICAL METHODS 347
Where I have set C
0
A
0
= a
0
, C
n
= 1, A
n
= a
n
and B
n
= b
n
. In doing this, we see that (4.9.18) is just
a Fourier series. The coecients are readily determined with
a
0
=
1
2

1

_
2
0
f(1, ) d (4.9.19a)
a
n
=
1

_
2
0
f(1, ) cos(n) d (4.9.19b)
b
n
=
1

_
2
0
f(1, ) sin(n) d (4.9.19c)
Substituting (4.9.19a), (4.9.19b) and (4.9.19c) into (4.9.18):
f(1, ) =a
0
+

n=1
(a
n
cos(n) +b
n
sin(n))
=
1
2
_
2
0
f(1, ) d +

n=1
_
1

_
2
0
f(1, ) cos(n) dcos(n) +
1

_
2
0
f(1, ) sin(n) dsin(n)
_
=
1

_
2
0
_
1
2
f(1, ) +

n=1
(f(1, ) cos(n) cos(n) +f(1, ) sin(n) sin(n))
_
d
=
1

_
2
0
f(1, )
_
1
2
+

n=1
(cos(n) cos(n) + sin(n) sin(n))
_
d
=
1

_
2
0
f(1, )
_
1
2
+

n=1
cos [n( )]
_
d (4.9.20)
Where I used the identity cos cos + sin sin = cos ( ) = cos ( ). The bracketed term in
(4.9.20) can be simplied, and I used Mathematica:
FullSimplify[1/2 + Sum[r^n*Cos[n*(alpha - theta], {n, 1, Infinity}]]
>>Out[157]=-((-1 + r^2)/(2*(1 + r^2 - 2*r*Cos[alpha - theta])))
With this, (4.9.20) becomes
f(1, ) =
1

_
2
0
f(1, )
_
1 r
2
2 (1 +r
2
2r cos ( ))
_
d
=
1
2
_
2
0
f(1, )
_
1 r
2
1 +r
2
2r cos ( )
_
d (4.9.21)
And we can extend (4.9.21) to the more general case of f(r, ) by replacing r with r/R:
f(r, ) =
1
2
_
2
0
f(R, )
_
1 (r/R)
2
1 + (r/R)
2
2(r/R) cos ( )
_
d
=
1
2
_
2
0
f(R, )
1/R
2
1/R
2
_
R
2
r
2
R
2
+ r
2
2rRcos ( )
_
d (4.9.22)
From (4.9.22) we are nally left with
W. Erbsen HOMEWORK #11
f(r, ) =
1
2
_
2
0
f(R, )
_
R
2
r
2
R
2
+ r
2
2rRcos ( )
_
d
Problem 3
For 0 < x < , solve
u
t
= a
2
u
xx
+ w(x, t); u(0, t) = 0, u
x
(, t) = 0, u(x, 0) = f(x) (4.9.23)
By means of a series expansion involving the eigenfunctions of
d
2
(x)
dx
2
+ (x) = 0; (0) =

() = 0 (4.9.24)
Solution
We begin our journey by assuming that the solution of (4.9.23) can be separated in the form u(x, t) =
X(x)T(t). Assuming that this is the case, we plug it into the homogeneous analogue to (4.9.23) and take
the corresponding partial derivatives:
XT
t
= a
2
X
xx
T
And separating the variables,
T
t
a
2
T
=
X
xx
X
= (4.9.25)
Where I have chosen to be the separation constant. Taking the equation for X(x) rst,
X
xx
x
= X
xx
+ X = 0 (4.9.26)
We immediately recognize that (4.9.26) is identical in form to (4.9.24). Furthermore, we also see that
these are Sturm-Lioville boundary value problems, and the solution of which was derived in class (and
commonly used in introduction quantum mechanics courses). The corresponding eigenvalues and eigen-
functions are given by

n
=
_
n

_
2
, and X
n
(x) = sin
_
nx

_
Now, taking our separated solution and plugging it into (4.9.23),

n=1
T

n
(t)X
n
(x) =a
2

n=1
T
n
(t)X

n
(x) + f(x, t)

n=1
T

n
(t)X
n
(x) =a
2

n=1
T
n
(t)X

n
(x) +

n=1
f
n
(t)X
n
(x) (4.9.27)
CHAPTER 4: MATHEMATICAL METHODS 349
And since the equation for X satises the Sturm-Liouville problem, we know that X

n
(x) +X
n
(x) = 0,
and (4.9.27) becomes

n=1
T

n
(t)X
n
(x) = a
2

n=1
T
n
(t)
n
X
n
(x) +

n=1
f
n
(t)X
n
(x)

n=1
T

n
(t) = a
2

n=1
T
n
(t)
n
+

n=1
f
n
(t) (4.9.28)
Which is satised if
T

n
(t) = a
2
T
n
(t)
n
+ f
n
(t)
Whose solution takes the form
T
n
(t) = e
a
2
nt
__
t
0
e
a
2
n
f
n
() d +T
n
(0)
_
(4.9.29)
And at this point we note that u
n
(x, 0) = f
n
(x), which implies that
f
n
(x) =

n=0
T
n
(0) sin
_
nx

_
And accordingly
T
n
(0) =
2

_

0
f
n
(x) sin
_
nx

_
dx
So that (4.9.18) becomes
T
n
(t) = e
a
2
nt
_
_
t
0
e
a
2
n
f
n
() d +
2

_

0
f
n
(x) sin
_
nx

_
dx
_
(4.9.30)
Combining (4.9.20) with our solution for X(x) (and substituting in the corresponding eigenvalues), we
arrive at
u
n
(x, t) =

n=1
_
e
(
na

)
2
t
sin
_
nx

_
_
_
t
0
e
(
na

)
2

f
n
() d +
2

_

0
f
n
(x) sin
_
nx

_
dx
__
Problem 4
Find the solution of Laplaces equation subject to the following boundary conditions

2
u = 0, 0 < < , a < r < b (4.9.31)
u(r, 0) = u(r, ) = 0, u(a, ) = 0, u(b, ) = f() (4.9.32)
W. Erbsen HOMEWORK #11
Solution
On this problem we will build on what was done in Problem 3, specically for our separated solutions
for () and R(r) from (4.9.15) and (4.9.16), respectively:
() =Acos() +B
n
sin() (4.9.33a)
R(r) =Cr

+ Dr

(4.9.33b)
We now take our solution for () from (4.9.33a) and apply our boundary conditions:
(0) =A ,= 0 A = 0
Which yields
() = Bsin()
And applying our second angular boundary condition,
() = Bsin() ,= 0
This is an eigenvalue problem, where the corresponding eigenvalue and eigenvector are given by

n
=
n

,
n
() = sin
_
n

_
(4.9.34)
Moving on to our equation for R(r) from (4.9.33b), and applying our rst boundary condition
R(a) = Ca

+Da

,= 0
For this we choose C and D such that we have
R
n
(r) =
_
r
a
_n

_
r
a
_

Which we can see satises our rst radial boundary condition. Before applying our last boundary
condition, we express our solution in the form of a superposition of the separated solutions:
u
n
(r, ) = R
n
(r)
n
() =

n=1
E
n
_
_
r
a
_n

_
r
a
_

_
sin
_
n

_
And now applying our last boundary condition,
u
n
(b, ) = f() =

n=1
E
n
_
_
b
a
_n

_
b
a
_

_
sin
_
n

_
(4.9.35)
And we see from (4.9.35) that the boundary condition u(b, ) = f() is satised if it coincides with the
Fourier Sine series

b
n
sin
_
n
/

_
. Therefore, we require that
E
n
=
b
n
(
b
/
a
)
n
/
(
b
/
a
)

n
/
With this, (4.9.35) becomes
CHAPTER 4: MATHEMATICAL METHODS 351
f() =

n=1
b
n
sin
_
n

_
And also
b
n
=
1

_

0
f() sin
_
n

_
d
And we can nally express our solution as
u
n
(r, ) =
1

n=1
__
_
r
a
_n

_
r
a
_

_
sin
_
n

__

0
f() sin
_
n

_
d
_
Problem 5
Use transformation methods to nd an integral representation of the solution u(x, y) of
u
xx
+ u
yy
= 0 for < x < , 0 < y < (4.9.36)
subject to the boundary conditions
u(x, 0) = f(x), < x < , u(x, y) 0 as x
2
+y
2
(4.9.37)
Solution
We start by applying the Fourier transform in x, where we denote Fu(x, y) = u(, y):
Fu
xx
+ Fu
yy
= 0 (4.9.38)
And, since y is independent of x,
Fu
yy
=
1

2
_

2
y
2
u(x, y)e
ix
dx
=

2
y
2
_
1

2
_

u(x, y)e
ix
dx
_
=

2
y
2
u(, y) (4.9.39)
And similarly,
Fu
xx
=
2
u(, y) (4.9.40)
See next problem for derivation of (4.9.40). Using (4.9.39) and (4.9.40), we can rewrite (4.9.38):
W. Erbsen HOMEWORK #11

2
y
2
u(, y)
2
u(, y) = 0
Which has a trivial solution of the form
u(, y) = A()e
y
+B()e
y
(4.9.41)
In order for our equation to remain nite according to our boundary conditions from (4.9.37), we must
require that if 0 and y , then we must require that A() = 0. Conversely, if 0 and y ,
we must have B() = 0. With this, our solution becomes
u(, y) = C()e
||y
And from the next problem in this assignment, we have C() =

f(), and the solution is nally
u(, y) =

f()e
||y
(4.9.42)
To nd the real solution, we must take the inverse Fourier transform of u(, y) in (4.9.42):
u(x, y) =Fu(, y) =
1

2
_

u(, y)e
ix
d
=
1

2
_

f()e
||y
e
ix
d (4.9.43)
And now

f() can be found by taking the Fourier transform with respect to some other variable :

f() =
1

2
_

f()e
i
d
Substituting this into (4.9.43),
u(x, y) =
1

2
_

_
1

2
_

f()e
i
d
_
e
||y
e
ix
d
=
1
2
_

f()e
i
e
||y
e
ix
dd
=
1
2
_

__

e
i
e
||y
e
ix
d
_
f() d
=
1
2
_

__

e
||y
e
i(x)
d
_
f() d (4.9.44)
The integral in parenthesis is solved via Mathematica, with the following code:
Integrate[Exp[-Abs[omega]*y]*Exp[-i*omega*(alpha - x)], {omega, -Infinity, Infinity}]
>>Out[75] = (2 y)/(-i^2*x^2 + y^2 + 2*i^2*x*alpha - i^2*alpha^2)
In keeping with Mathematicas typically nonsensical outputs, we can evaluate the output as follows:
_

e
||y
e
i(x)
d =
2y
x
2
+y
2
2x +
2
CHAPTER 4: MATHEMATICAL METHODS 353
=
2y
y
2
+ (x )
2
Substituting this back into (4.9.44),
u(x, y) =
1
2
_

_
2y
y
2
+ (x )
2
_
f() d
And we are nally left with
u(x, y) =
y

f()
y
2
+ (x )
2
d
Problem 6
Solve the Cauchy problem for the one-dimensional heat equation in the domain < x < , t > 0
u
t
= u
xx
, u(x, 0) = f(x) (4.9.45)
with the Fourier transform.
Solution
We begin by taking the Fourier Transform of both sides of (4.9.45):
Fu
t
= Fu
xx
(4.9.46)
In future arguments we note that Fu(x, t) = u(, t), and the terms in (4.9.46) are
Fu
t
=
1

2
_

u
t
e
ix
dx
=
1

2
_

u
t
e
ix
dx
=

t
_
1

2
_

ue
ix
dx
_
=

t
u(, t) (4.9.47)
And similarly,
Fu
xx
=

2
_

u
xx
e
ix
dx
=

2
_

2
u
x
2
e
ix
dx
_
u = e
ix
, du = ie
ix
dv =
d
2
u
dx
2
, v =
du
dx
=

2
_
du
dx
e
ix

+ i
_

du
dx
e
ix
dx
_
u = e
ix
, du = ie
ix
dv =
du
dx
, v = u
W. Erbsen HOMEWORK #11
=
i

2
_
_
ue
ix

+ i
_

ue
ix
dx
_
=
2
_
1

2
_

ue
ix
dx
_
=
2
u(, t) (4.9.48)
Using (4.9.47) and (4.9.48), we can rewrite (4.9.46):
d
dt
u(, t) +
2
u(, t) = 0
With solutions
u(, t) = C()e

2
t
(4.9.49)
Applying our boundary condition, we nd that
u(, t) = Fu(, t) =

f()e

2
t
(4.9.50)
Because
C() =

f() = u(, 0) =
1

2
_

u(x, 0)e
ix
dx =
1

2
_

f(x)e
ix
dx
So, from (4.9.50), we know what u(, t) is, and to nd u(x, t) we must nd the inverse Fourier Transform:
u(x, t) =F
1
u(, t)
=
1

2
_

u(, t)e
ix
d
=
1

2
_

f()e

2
t
e
ix
d (4.9.51)
And taking the Fourier transform of

f() with respect to yields

f() =
1

2
_

f()e
i
d
And (4.9.51) becomes
u(x, t) =
1

2
_

_
1

2
_

f()e
i
d
_
e

2
t
e
ix
d
=
1
2
_

f()e
i
e

2
t
e
ix
dd
=
1
2
_

f()e

2
t
e
i(x)
dd
=
1
2
_

__

2
t
e
i(x)
d
_
f() d (4.9.52)
The term in parenthesis can be evaluated with Mathematica,
CHAPTER 4: MATHEMATICAL METHODS 355
Integrate[Exp[-kappa*omega^2*t]*Exp[-i*omega*(alpha-x)], {omega, -Infinity, Infinity}]
>>Out[102]= e^((i^2*(x - alpha)^2)/(4*t*kappa))*Sqrt[Pi]/Sqrt[t*kappa]
This translates to
_

2
t
e
i(x)
d =
_

t
exp
_

(x )
2
4t
_
And (4.9.52) becomes
u(x, t) =
1
2
_

__

t
exp
_

(x )
2
4t
__
f() d
And we are nally left with
1

4t
_

f() exp
_

(x )
2
4t
_
d
4.10 Homework #12
Problem 1
Find the solution of the equations subject to the following boundary conditions
8x
xx
6

x
xy
+
yy
+ 4
x
= 0, x > 0, y > 0
[
y=0
=
x
/
2
,
y
[
y=0
= 0
(4.10.1)
Solution
We start o by determining the nature of our partial dierential equation, by rst noticing that A = 8x,
B = 6

x , and C = 1. Accordingly,
B
2
4AC =
_
6

x
_
2
4 (8x) = 36x 32x = 4x > 0 Hyperbolic
Which is a fair assumption, since the prompt states that x > 0. Furthermore, we nd the characteristics
with
8x
_
dy
dx
_
2
+ 6

x
_
dy
dx
_
+ 1 = 0
dy
dx
=
6

4x
16x
=
3 1
8

x
(4.10.2)
We note that there are two solutions (characteristics) that may be found from (4.10.2), one corresponding
to the + and the other with the .
W. Erbsen HOMEWORK #12
+ :
dy
dx
=
3 + 1
8

x
=
1
4

x
dy =
1
4

x
dx y =

x
2
y +

x
2
= 0
:
dy
dx
=
3 1
8

x
=
1
2

x
dy =
1
2

x
dx y =

x y +

x = 0
And I choose the characteristics to be assigned as follows:
(x, y) = = y +

x , and (x, y) = = y +

x
2
And now taking some partial derivatives,

x
=
1
2

x
,
y
= 1,
xx
=
1
4x
3
/2
,
yy
= 0,
xy
= 0

x
=
1
4

x
,
y
= 1,
xx
=
1
8x
3
/2
,
yy
= 0,
xy
= 0
And at this point we must nd
x
,
y
,
xx
,
xy
, and
yy
:

x
=
x

+
x

=
1
2

+
1
4

(4.10.3a)

y
=
y

+
y

(4.10.3b)

xx
=
2
x

+ 2
x

+
2
x

+
xx

+
xx

=
1
4x

+
1
4x

+
1
16x


1
4x
3
/2

1
8x
3
/2

(4.10.3c)

xy
=
x

+ (
x

y
+
y

x
)

+
x

+
xy

+
xy

=
1
2

+
_
1
2

x
+
1
4

x
_

+
1
4

=
1
2

+
3
2

+
1
4

(4.10.3d)

yy
=
2
y

+ 2
y

+
2
y

+
yy

+
yy

+ 2

(4.10.3e)
And now, we use (4.10.3a)-(4.10.3e) to nd

and

. I choose to use a table to do this.

xx

xy

yy

x

y

8x
_
1
4x
_
6

x
_
1
2

x
_
1 0 0 0

8x
_
1
4x
_
6

x
_
3
4

x
_
2 0 0
1
2

8x
_
1
16x
_
6

x
_
1
4

x
_
1 0 0 0

8x
_
1
4x
3
/
2
_
0 0 4
_
1
2

x
_
0 0

8x
_
1
8x
3
/
2
_
0 0 4
_
1
4

x
_
0 0
So, it turns out that our equation is,

1
2

= 0

=f() + g()
CHAPTER 4: MATHEMATICAL METHODS 357
=f
_
y +

x
_
+ g
_
y +

x
2
_
(4.10.4)
Which is our arbitrary solution. In order to apply our rst boundary condition, we let y = 0, and then
take the derivative:
(x, 0) =f
_
x
_
+ g
_
x
2
_
=
x
2
(4.10.5)

(x, 0) =
1
2

x
f

_
x
_
+
1
4

x
g

_
x
2
_
=
1
2
(4.10.6)
And now we apply the second boundary condition,

y
(x, 0) =f

_
x
_
+g

_
x
2
_
= 0 (4.10.7)
Now, we solve (4.10.7) for f

x ),
f

_
x
_
= g

_
x
2
_
(4.10.8)
Now, we rewrite (4.10.6) and substitute in (4.10.8):
1
2

x
f

_
x
_
+
1
4

x
g

_
x
2
_
=
1
2
=2f

_
x
_
+ g

_
x
2
_
= 2

x
2g

_
x
2
_
+g

_
x
2
_
= 2

x
g

_
x
2
_
= 2

x (4.10.9)
We can now solve (4.10.9),
g

_
x
2
_
= 2

x g
_
x
2
_
=
4x
3
/2
3
+ C (4.10.10)
By substituting (4.10.10) into (4.10.5), we can nd g

x /2):
f
_
x
_

4x
3
/2
3
+ C =
x
2
f
_
x
_
=
4x
3
/2
3
+
x
2
C (4.10.11)
Now, using (4.10.10) and (4.10.11), we can say
(x, y) =
4x
3
/2
3
+
x
2
C +
4x
3
/2
3
+ C
Which leads us to the nal answer:
(x, y) =
x
2
W. Erbsen HOMEWORK #12
Problem 2
Show that the Euler-Lagrange equation can be written in the form
d
dx
_
L y

L
y

L
x
= 0 (4.10.12)
Solution
I will work backwards from (4.10.12), and hopefully arrive at the Euler-Lagrange equation. But rst, we
must nd what the total derivative of the Lagrangian is, since it will be needed later. First recognizing
that L L(x, y(x), y

(x)), we nd that
dL
dx
=

x
L +

y
dy
dx
L +

y

dy

dx
L
=
L
x
+
L
y
y

+
L
y

Using this, we can now work backwards from (4.10.12):


d
dx
_
L y

L
y

L
x
=0
dL
dx
y

L
y

d
dx
L
y


L
x
=0
L
x
+
L
y
y

+
L
y

L
y

d
dx
L
y


L
x
=0
L
y
y

d
dx
L
y

=0
y

_
L
y

d
dx
L
y

_
=0 (4.10.13)
From (4.10.13) it is easy to see that we arrive at the Euler Lagrange equation:
L
y

d
dx
L
y

= 0
Problem 3
The equations for water waves with free surface y = h(x, t) and bottom at y = 0 are

xx
+
yy
= 0, for 0 < y < h(x, t)

t
+
1
2

2
x
+
1
2

2
y
+ gy = 0, for y = h(x, t)
h
t
+
x
h
x

y
= 0, for y = h(x, t)

y
= 0, for y = 0
(4.10.14a)
CHAPTER 4: MATHEMATICAL METHODS 359
Where the uid motion is described by (x, y, t) and g is the acceleration due to gravity. Show that all these
equations may be obtained by varying the functions (x, y, t) and h(x, t) in the variational principle

_ _
R
_
_
h(x,t)
0
_

t
+
1
2

2
x
+
1
2

2
y
+ gy
_
dy
_
dxdt = 0 (4.10.15)
Where R is an arbitrary region in the (x, t) plane.
Solution
We begin by choosing a smaller chunk of (4.10.21a), which will be more manageable:
L =
_
h(x,t)
0
_

t
+
1
2

2
x
+
1
2

2
y
+gy
_
dy (4.10.16)
And what we want to do is vary by some small amount , and then doing the same with h(x, t). The
prescribed formula for this is

L = L( ) L() (4.10.17)
If we apply (4.10.17) to (4.10.16) (only the variational part rst, and we will subtract the rest later), we
have
L =
_
h(x,t)
0
_

t
+
t
+
1
2
(
x
+
x
)
2
+
1
2
(
y
+
y
)
2
+gy
_
dy
=
_
h(x,t)
0
_

t
+
t
+
1
2
_

2
x
+ 2
x

x
+ (
x
)
2
_
+
1
2
_

2
y
+ 2
y

y
+ (
y
)
2
_
+ gy
_
dy
=
_
h(x,t)
0
_

t
+
t
+
1
2

2
x
+
x

x
+
1
2

2
y
+
y

y
+ gy
_
dy (4.10.18)
One thing I did was remove, or more accurately, neglect the terms including a , a standard tool in the
calculus of variations. We assume to be a small change, so must indeed be a very small change, and
is therefore neglected. We now take (4.10.18) and subtract from it the original Lagrangian, (4.10.16) as
follows:
L =
_
h(x,t)
0
_

t
+
t
+
1
2

2
x
+
x

x
+
1
2

2
y
+
y

y
+gy
t

1
2

2
x

1
2

2
y
gy
_
dy
=
_
h(x,t)
0
(
t
+
x

x
+
y

y
) dy
=
_
h(x,t)
0
_

t
+

x

x
+

y

y
_
dy
=
_
h(x,t)
0

t
dy +
_
h(x,t)
0

x
dy +
_
h(x,t)
0

y
dy (4.10.19)
At this point we apply Leibniz rule, and do integration by parts over and over again. Im not showing
all my work on this one:
[h
t
[
h
0
+

t
_
h
0
dy [
x
h
x
[
h
0
+

x
_
h
0

x
dy + [
y
[
h
0

_
h
0

xx
dy
_
h
0

yy
dy
W. Erbsen HOMEWORK #12
[h
t
[
h
+

t
_
h
0
dy [
x
h
x
[
h
+

x
_
h
0

x
dy + [
y
[
h
0

_
h
0

xx
dy
_
h
0

yy
dy
_

t
_
h
0
dy +

x
_
h
0

x
dy
_

_
h
0
(
xx
+
yy
) dy [h
t
[
h
[
x
h
x
[
h
+ [
y
[
h
+ [
y
[
0
_

t
_
h
0
dy +

x
_
h
0

x
dy
_

_
h
0
(
xx
+
yy
)
. .
dy [h
t
+
x
h
x

y
[
h
. .
+[
y
[
0
. .
The three underbraced equations represent three out of the four equations we were asked to nd. The
forth is just (4.10.16).
Problem 4
Find a minimum for the functional
I(y) =
_
m
o
_
y + h
_
1 + (y

)
2
dx for h > 0, y(0) = 0, , y(m) = M > h (4.10.20)
Solution
Within the Euler-Lagrangian formalism, we wish to extract the Lagrangian from (4.10.20), and put it
into the Euler-Lagrange equation. The form most convenient for our purposes will be that given by
(4.10.12). Consequently, we must evaluate all the terms within the Euler-Lagrange equation rst:
L =
_
y +h
_
1 + (y

)
2
(4.10.21a)
L
y

=

y

_
y +h
_
1 + (y

)
2
=
y

y +h
_
1 + (y

)
2
(4.10.21b)
L
x
= 0 (4.10.21c)
We now substitute (4.10.21a)-(4.10.21c) into (4.10.12):
d
dx
_
_
_
y +h
_
1 + (y

)
2
y

y + h
_
1 + (y

)
2
_
_
=0
d
dx
_
_
_
y + h
_
_
_
1 + (y

)
2
y

_
1 + (y

)
2
_
_
_
_
=0
d
dx
_
_
_
y + h
_
_
__
1 + (y

)
2
_

_
1 + (y

)
2
_
1 + (y

)
2
y

_
1 + (y

)
2
_
_
_
_
=0
CHAPTER 4: MATHEMATICAL METHODS 361
d
dx
_
_
_
y +h
_
_
1 + (y

)
2
_
1 + (y

)
2

(y

)
2
_
1 + (y

)
2
_
_
_
_
=0
d
dx
_
_
_
y + h
_
_
1
_
1 + (y

)
2
_
_
_
_
=0
d
dx
_
_

y +h
_
1 + (y

)
2
_
_
. .
constant
=0 (4.10.22)
So, the bracketed term in (4.10.22) is a constant of motion:

y + h
_
1 + (y

)
2
=C (4.10.23)
We are now tasked to nd a solution of y(x) corresponding to the given initial conditions:
y +h
1 + (y

)
2
=C
2
y +h =C
2
_
1 + (y

)
2
_
dy
dx
=
_
y + h
C
2
1
dy
dx
=
_
y + h C
2
C
dx =
C
_
y + h C
2
dy
x =2C
_
y +h C
2
+ C

(x C

)
2
=4C
2
(y + h C
2
) (4.10.24)
In order for this problem to work, I have to assume that the second integration constant goes to zero.
We now apply the initial condition on (4.10.24) to nd C:
0 =4C
2
(h C
2
) C =

h
Substituting this back into (4.10.24),
x
2
=4h(y + h h)
From this it is easy to see that the solution is...
y(x) =
x
2
4h
W. Erbsen HOMEWORK #12
Problem 5
A rocket is propelled vertically upward so as to reach a prescribed height h in a minimum time while using a
given xed quantity of fuel. The vertical distance x(t) above the surface satises,
mx

= mg + mU(t), with x(0) = 0, x

(0) = 0 (4.10.25)
where U(t) is the acceleration provided by the engine thrust. We impose the terminal constant x(T) = h, and we
wish to nd the particular thrust function U(t) which will minimize T assuming that the total thrust of the rocket
engine over the entire thrust time is limited by the condition,
_
T
0
U
2
(t) dt = k
2
(4.10.26)
Where k is a given positive constant which measures the total amount of available fuel.
Solution
The rst thing we need to do is solve (4.10.25) with the given initial conditions.
x

(t) = g + U(t)
_
t
0
x

(t) dt = g
_
t
0
dt +
_
t
0
U() d
x

(t) = gt +
_
t
0
U() d
_
t
0
x

(t) dt = g
_
t
0
t dt +
_
t
0
_
_

0
U() d
_
d
x(t) =
gt
2
2
+
_
t
0
_
_

0
U() d
_
d
=
gt
2
2
+
_
t
0
U() d
_
t

d
=
gt
2
2
+
_
t
0
(t )U() d (4.10.27)
Where I eliminated the integration constants en route. We now apply our boundary condition, x(T) = h,
to (4.10.27):
x(T) =
gT
2
2
+
_
T
0
(T )U() d = h (4.10.28)
And we dene two new equations,
F
1
=
gT
2
2
+
_
T
0
(T t)U(t) dt (4.10.29a)
F
2
=
_
T
0
U
2
(t) dt (4.10.29b)
CHAPTER 4: MATHEMATICAL METHODS 363
Where I changed variables from to t in (4.10.29a). Starting with (4.10.29a), we continue with our
variational adventures by adding some small element to our to-be-varied functional:
F
1
(T +T) =
g
2
(T + T)
2
+
_
T
0
(T +T t)(U + U) dt
=
g
2
(T
2
+ 2TT + (T)
2
) +
_
T
0
(TU +TU tU + TU +TU tU) dt
=
gT
2
2
gTT +
_
T
0
(TU + TU +TU tU tU) dt (4.10.30)
We now subtract the original functional from (4.10.30),
F
1
(T +T) F
1
(T) =
gT
2
2
gTT +
_
T
0
(TU + TU +TU tU tU) dt +
gT
2
2

_
T
0
(T t)U dt
= gTT +
_
T
0
(TU + TU tU) dt (4.10.31)
We repeat this process with F
2
in (4.10.29b):
F
2
(T +T) =
_
T
0
(U + U)
2
dt
=
_
T
0
_
U
2
+ 2UU + (U)
2
_
dt
=
_
T
0
_
U
2
+ 2UU
_
dt (4.10.32)
We now take (4.10.32) and subtract (4.10.29b):
F
2
(T + T) F
2
(T) =
_
T
0
_
U
2
+ 2UU
_
dt
_
T
0
U
2
dt
=2
_
T
0
UU dt (4.10.33)
And now bringing all the pieces together,
T =
1
F
1
(T)
2
F
2
(t)
=
1
_
gTT +
_
T
0
(TU +TU tU) dt
_
+
2
_
2
_
T
0
UU dt
_
=
1
gTT +
_
T
0
(
1
TU +
1
TU
1
tU +
2
2UU) dt (4.10.34)
Now we take (4.10.34) such that T = 0, eg no variation. This gives
0 =
_
T
0
(
1
TU
1
tU +
2
2UU) dt
0 =
_
T
0
(
1
T
1
t + 2
2
U)
. .
dt
W. Erbsen HOMEWORK #12
0 =
1
T
1
t + 2
2
U

1
t
1
T = 2
2
U
1
(t T) = 2
2
U (4.10.35)
And remember F
1
and F
2
? Sure you do - heres where we use them. Solve (4.10.35) for U(t) and
substitute into these equations and integrate:
F
1
=
gT
2
2
+
_
T
0
(T t)
_


1
2
2
(T t)
_
dt
=
gT
2
2


1
2
2
_
T
0
(T t)
2
dt
=
gT
2
2


1
2
2
_
T
3
3
T
3
+ T
3
_
=
gT
2
2


1
6
2
T
3
= h (4.10.36)
And now for F
2
:
F
2
=
_
T
0
_


1
2
2
(T t)
_
2
dt =

2
1
4
2
2
T
3
3
= k
2
(4.10.37)
We have three independent variables,
1
,
2
, and T, so we need another equation. In addition to these
we can one more as suggested by Donald Smith in his book Variational Methods in Optimization. We
do this by again varying T by a small amount, other than zero. Lets do just that:
1 =
1
gT +
_
T
0
(
1
TU +
1
U
1
tu + 2
2
uu) dt
=
1
gT +
_
T
0
_

1
(T t)
. .
22
UU +
1
U + 2
2
UU
_
dt
=
1
gT +
_
T
0
(2
2
UU +
1
U + 2
2
UU) dt
=
1
gT +
_
T
0

1
U dt (4.10.38)
Now insert (4.10.35) into (4.10.38)!
1 =
1
gT +
1
_
T
0
_

1
(T t)
2
2
_
dt
=
1
gT

2
1
2
2
_
_
T
0
T dt
_
T
0
t dt
_
=
1
gT

2
1
2
2
_
T
2

T
2
2
_
=
1
gT

2
1
2
2
T
2
2
=
1
gT

2
1
4
2
T
2
(4.10.39)
CHAPTER 4: MATHEMATICAL METHODS 365
From here forward is just plugging in equations and solving; I left this up to Mathematica. There seem
to be many solutions, here are two:
U(t) =
3(t T)
_
gT
2
2h
_
2T
2
, U(t) =

3 k(T t)
T
3
/2
W. Erbsen HOMEWORK #12
Chapter 5
Departmental Examinations
5.1 Quantum Mechanics
Problem 1
Consider two dierent spin
1
/
2
whose Hamiltonian is completely specied as H = cs(1) s(2) where c is a real
constant.
a) What are the constants of motion? Obtain the eigenvalues of H for both singlet and triplet states.
b) Consider at time t = 0 the spin of particle (1) along the z-axis is up and the spin of particle (2) along the
z-axis is down. What is the wave function of the system at a later time t?
c) Calculate the probability that at a later time t the spins of both particles are aligned as they were at t = 0.
Useful information:

m
1
=
_

_
(1)(2) m = 1
(1)(2) m = 1
1

2
[(1)(2) + (1)(2)] m = 0

0
0
=
1

2
[(1)(2) (1)(2)]
Solution
a) The Hamiltonian is given by H = cs
1
s
2
,

and the total spin is just s = s


1
+ s
2
. We can nd an
alternate form of the Hamiltonian by doing the following:
s
2
=(s
1
+ s
2
)
2
= s
2
1
+ s
2
2
+ 2s
1
s
2
s
1
s
2
=
1
2
_
s
2
s
2
1
s
2
2

Parenthetic indicies have been shamelessly transformed into subscripts to avoid confusion.
W. Erbsen QUANTUM MECHANICS
Such that our Hamiltonian becomes
H =
c
2
_
s
2
s
2
1
s
2
2

(5.1.1)
To nd the constants of motion, we test to see whether or not the quantity in question commutes
with the Hamiltonian (5.1.1). The contenders are s
1
, s
2
, and s. Starting with s
1
,
[H, s
1
] =
c
2
__
s
2
s
2
1
s
2
2
_
, s
1

=
c
2
_
[s
2
, s
1
]
. .
i
[s
2
1
, s
1
]
. .
ii
, [s
2
2
, s
1
]
. .
iii
_
= 0 (5.1.2)
It is immediately obvious that i and ii equal zero, and since the operators in iii act on two dierent
particles, it commutes to. Therefore, s
1
is a constant of motion. Similarly, for s
2
we have:
[H, s
2
] =
c
2
__
s
2
s
2
1
s
2
2
_
, s
2

=
c
2
_
[s
2
, s
2
] [s
2
1
, s
2
], [s
2
2
, s
2
]
_
= 0 (5.1.3)
For the same reasons applied to (5.1.2), we see from (5.1.3) that s
2
is also a constant of motion.
Furthermore, for s, we have
[H, s] =
c
2
__
s
2
s
2
1
s
2
2
_
, s

=
c
2
_
[s
2
, s] [s
2
1
, s], [s
2
2
, s]
_
= 0 (5.1.4)
So, from (5.1.2)-(5.1.5), we see that the constants of motion are s
1
, s
2
, and s .
For the second part of this problem, we wish to nd the eigenvalues of (5.1.1) for both singlet s = 0
and triplet s = 1 states. For the singlet state, we recognize that s = 0, so s
1
and s
2
must be in
opposite spin states. Lets let s
1
=
1
/
2
, and s
2
=
1
/
2
.
H
0
0
=
c
2
_
s
2
s
2
1
s
2
2

0
0
=
c
2
_
s
2

0
0
s
2
1

0
0
s
2
2

0
0
_
=
c
2
_
s(s + 1)
2

0
0
s
1
(s
1
+ 1)
2

0
0
s
2
(s
2
+ 1)
2

0
0
_
=
c
2
_
0(0 + 0)
2

1
/
2
(
1
/
2
+ 1)
2

1
/
2
(
1
/
2
+ 1)
2
_

0
0
=
c
2
_
0
3
/
4

3
/
4

2
_

0
0
=
3c
4

0
0
(5.1.5)
For the triplet state we have s = 1, so following the steps from (5.1.5),
=
c
2
_
1(1 + 1)
2

1
/
2
(
1
/
2
+ 1)
2

1
/
2
(
1
/
2
+ 1)
2
_

m
1
=
c
2
_
2
3
/
4

3
/
4

2
_

m
1
=
c
4

m
1
(5.1.6)
So from (5.1.5) and (5.1.6), our eigenvalues for the provided Hamiltonian for both singlet and triplet
states are, respectively:
H
0
0
=
3c
4

2

0
0
, H
m
1
=
c
4

m
1
CHAPTER 5: DEPARTMENTAL EXAMINATIONS 369
b) The general wave function for particle one spin up and particle two spin down is given by
(0) =
1

2
_

0
0
+
0
1

(5.1.7)
If we wish to obtain the wave function at some later time t, we need to apply the time evolution
operator U = exp
_

iHt

to (5.1.7)
(t) =
1

2
_
exp
_

iHt

0
0
+ exp
_

iHt

0
1
_
=
1

2
_
exp
_

it

3c
4

2
__

0
0
+ exp
_

it

_
c
4

2
_
_

0
1
_
Which readily simplies to
(t) =
1

2
_
exp
_
3c
4
it
_

0
0
+ exp
_

c
4
it
_

0
1
_
c) To calculate the probability that at some later time t both electrons will return to their original
state, we simply take the modulus squared of the matrix element between these two states. To
simplify notation, I make the following substitutions:
E
0
=
3c
4
, E
1
=
c
4
So, the probabilty is then:
[(0)[(t))[
2
=

__
1

2
_

0
0
+
0
1

_
1

2
_
e
E0it

0
0
+e
E1it

0
1
_
__

2
=
1
4

0
0
+
0
1

e
E0it

0
0
+e
E1it

0
1
_

2
=
1
4

e
E0it

0
0
[
0
0
)
. .
1
+e
E1it

0
0
[
0
1
)
. .
0
+e
E0it

0
1
[
0
0
)
. .
0
+e
E1it

0
1
[
0
1
)
. .
1

2
=
1
4

e
E0it
+e
E1it

2
=
1
4

e
3c
4
it
+ e

c
4
it

2
=
1
4

e
c
4
it
_
e
c
2
it
+ e

c
2
it
_

2
=
1
4

e
c
4
it
2 cos
_
c
2
t
_

2
(5.1.8)
From (5.1.8) it is clear that the probability of realignment is
P = cos
2
_
c
2
t
_
W. Erbsen QUANTUM MECHANICS
Problem 2
a) If
_
2 1
1 1
_
, what are the eigenvalues of this Hamiltonian?
b) Prove that [H, exp (H)] = 0, that is, the two operators commute.
c) What are the eigenvalues of exp (H)?
d) If a small perturbation H

= (
0 1
1 0
), where is a small positive number, is applied to the system, calculate
the change in the eigenenergy of the ground state, to rst order in .
e) Carry out the change in the ground state energy to second order in .
Solution
a) Lets diagonalize this mofo:

2 1
1 2

= (2 )(2 ) 1 =
2
5 = 0 =

5
b) We rst recall that the series expansion for e
x
is
e
x
=

n=0
x
n
n!
= 1 + x +
x
2
2
+
x
3
6
+...
So, our commutator is
[H, exp[H]] = [H, (1 + H + h
2
/2 +h
3
/6 +...)] = [H, 1] + [H, H] + [H, H
2
/2] + [H, H
3
/6] (5.1.9)
Each of the terms in (5.1.9) vanish, so we can only conclude that H and exp[H] commute, and so
[H, exp[H]] = 0
c) The eigenvalues of exp [H] are just = exp [

5 ] .
d) The rst thing we need to do is nd the eigenvectors associated with the eigenvalues that we found
in part a):
For = +

5 :
_
2

5 1
1 1

5
_ _

_
=
_
0
0
_
(2

5 ) + = 0
fes
=
_
1
2

5
_
For =

5 :
_
2 +

5 1
1 1 +

5
_ _

_
=
_
0
0
_
(2 +

5 ) + = 0
gs
=
_
1
2 +

5
_
The ground state is associated with the eigenvalue =

5 , with eigenvector
gs
. The rst order
energy correction is given by

gs
[H

[
gs
) =
_
1 (2 +

5 )
_

_
0 1
1 0
__
1
2 +

5
_
=
_
1 (2 +

5 )
_
_
2 +

5
1
_
CHAPTER 5: DEPARTMENTAL EXAMINATIONS 371
From which it is clear that the rst order shift in energy is simply
E
(1)
gs
=
_
4 + 2

5
_
e) To carry out the change in the ground state energy to second order in , we rst recall that the
second order energy shift from time-independent perturbation theory is
E
(2)
n
=

m,n
m=n
[
m
[H

[
n
)[
2
E
n
E
m
(5.1.10)
In our case
m
=
fes
,
n
=
gs
, E
m
= E
1
and E
n
= E
0
. We compute the matrix element in
(5.1.10) rst:

fes
[H

[
gs
) =
_
1 (2

5 )
_

_
0 1
1 0
__
1
2 +

5
_
=
_
1 (2

5 )
_
_
2 +

5
1
_
=4 (5.1.11)
The denominator of (5.1.10) is just
E
0
E
1
=

5 = 2

5 (5.1.12)
We now substitute (5.1.11) and (5.1.12) into (5.1.10) to obtain the energy shift:
E
(2)
n
=

m,n
m=n
[4[
2
2

5
E
(2)
n
=
8
2

5
Problem 3
Consider two spin
1
/
2
particles described by the Hamiltonian
H =
p
2
1
2m
+
p
2
2
2m
+V (x
1
) + V (x
2
) (5.1.13)
Where V (x) = for x < 0 and for x > a; V (x) = 0 for 0 < x < a. Assume that the electrons are in the opposite
spin state, that is, the total S = 0.
a) Write down the spin wave function(s). Use the standard notations: for spin up and for spin down.
b) Find the energy and wavefunctions of the ground state of this Hamiltonian.
c) Find the energy and wavefunction of the lowest state for S = 1.
d) Find the energy and wavefunction of the second S = 0 state. Show that the energy is the same as in (c).
W. Erbsen QUANTUM MECHANICS
e) If the two particles have a small interaction W(x
1
, x
2
) = bx
1
x
2
where b is small and positive, show that the
degeneracy in (c) and (d) is removed. Which one has the lower energy?
Solution
We rst note that since s
1
=
1
/
2
and s
2
=
1
/
2
then s
1
+s
2
= 0. The eigenfunctions and eigenenergies
are

n
(x) =
_
2
L
sin
_
nx
a
_
, E
n
=
n
2

2
2ma
2
a) The spin wave function is
=
1

2
[(1)(2) (2)(1)] (5.1.14)
b) The full wave function is just the product of the space part and the spin part:
(x) =
1
(x
1
)
1
(x
2
) (x) =

2
L
sin
_
x
1
L
_
sin
_
x
2
L
_
[(1)(2) (2)(1)] (5.1.15)
While the energy is just the sum:
E
gs
= E
1
+ E
1
E
gs
=

2

2
2mL
2
c) For s = 1 the spin wave function is symmetric, so the space part must be anti-symmetric:

a,b
(x
1
, x
2
) =
1

2
[
a
(x
1
)
b
(x
2
)
a
(x
2
)
b
(x
1
)]
Since s = 1, this implies that they cannot both be in the same state, so the lowest state would be
for one of the electrons to be at n = 1, and the rst at n = 2. The total energy is just the sum of
these:
E =
5
2

2
2mL
2
d) Now that s = 0 this implies that the spin wave function is anti-symmetric, so the space part should
now be symmetric:

a,b
(x
1
, x
2
) =
1

2
[
a
(x
1
)
b
(x
2
) +
a
(x
2
)
b
(x
1
)]
And since we are now looking for the second lowest state, and we know that since s = 0 implies
that the electrons are in opposite states this means they can occupy the same energy level, so the
ground state is n
1
= 1 and n
2
= 1, and the second state is n
1
= 1 and n
2
= 2 (or vice versa). The
total energy is the same as the ground state for the case of s = 1:
CHAPTER 5: DEPARTMENTAL EXAMINATIONS 373
E =
5
2

2
2mL
2
e) To do this part we must compute the rst order energy shift in time-independent perturbation
theory. Lets dene

a
(x
1
, x
2
) =
1

2
[
a
(x
1
)
b
(x
2
)
a
(x
2
)
b
(x
1
)]

b
(x
1
, x
2
) =
1

2
[
a
(x
1
)
b
(x
2
) +
a
(x
2
)
b
(x
1
)]
Then the rst order shift for s = 1 is then
E
(1)
=
a
[H

[
a
)
=
b
2
__
x
1
x
2
[
a
(x
1
)
b
(x
2
)
a
(x
2
)
b
(x
1
)]
2
dx
1
dx
2
=
b
2
__
_
x
1
x
2
(
a
(x
1
)
b
(x
2
))
2
+ x
1
x
2
(
a
(x
2
)
b
(x
1
))
2
2x
1
x
2

a
(x
1
)
b
(x
2
)
a
(x
2
)
b
(x
1
)] dx
1
dx
2
(5.1.16)
And also the rst order shift for s = 0 is
E
(1)
=
b
2
__
x
1
x
2
[
a
(x
1
)
b
(x
2
) +
a
(x
2
)
b
(x
1
)]
2
dx
1
dx
2
=
b
2
__
_
x
1
x
2
(
a
(x
1
)
b
(x
2
))
2
+ x
1
x
2
(
a
(x
2
)
b
(x
1
))
2
+2x
1
x
2

a
(x
1
)
b
(x
2
)
a
(x
2
)
b
(x
1
)] dx
1
dx
2
(5.1.17)
It is clear from just looking at (5.1.16) and (5.1.17) that the degeneracy will be broken, since they
dier in the sign of one of the cross terms. The case of s = 1 has a lower energy.
Problem 4
The result of a measurement shows that the electron spin is along the +x direction at t = 0. For t > 0, the
electron enters in a uniform magnetic eld that is parallel to the +z direction. Calculate the quantum mechanical
probability as a function of time for nding the electron in each of the following states ( = 1):
a) S
x
= +1/2
b) S
x
= 1/2
c) S
z
= +1/2
d) S
z
= 1/2
The Pauli matrices are given by

x
=
_
0 1
1 0
_
,
y
=
_
0 i
i 0
_
,
z
=
_
1 0
0 1
_
W. Erbsen QUANTUM MECHANICS
Solution
Initially at time t = 0 the spin is along the x-direction, so it will be most convenient to express the spinor
in the S
z
basis. We rst recall that

(x)
+
=
1

2
_
1
1
_
,
(x)

=
1

2
_
1
1
_

(z)
+
=
_
1
0
_
,
(z)

=
_
0
1
_
And also that we can apply the time-evolution operator to a state at t = 0 as
[(t)) = U[(0)), with U = exp [iHt]
The Hamiltonian for a dipole in a magnetic eld (the dipole in this case is provided by the spin) is
H = B =
eS
z
m
B
0
= S
z
So the time evolution operator becomes
U = exp [iS
z
t] (5.1.18)
We now wish to express our current spinor in terms of the S
z
basis:
[(0)) =
_

(x)
+
_

(z)
+
+
_

(x)
+
_

(z)

=
1

2
_
1 1
_
_
1
0
_
+
1

2
_
1 1
_
_
0
1
_
=
1

2
_
1
0
_
+
1

2
_
0
1
_
=
1

2
[) +
1

2
[) (5.1.19)
We apply the time evolution operator from (5.1.18) to our state at time t = 0 from (5.1.19):
[(t)) =e
iSzt
1

2
[) + e
iSzt
1

2
[)
=
1

2
_
e
i

2
t
[) +e
i

2
t
[)
_
(5.1.20)
Where we have set = 1. Now that we have our time-dependent wave function, we can calculate the
time-dependent probabilities for the spin to ip:
a) The probability for nding the particle to be in S
x
=
1
/
2
is found by
P =[S
x
()[(t))[
2
=
1
2

S
x
()[
_
e
i

2
t
[) +e
i

2
t
[)
_
)

2
=
1
2

S
x
()[e
i

2
t
[) +S
x
()[e
i

2
t
[)

2
CHAPTER 5: DEPARTMENTAL EXAMINATIONS 375
=
1
2

e
i

2
t
S
x
()[) + e
i

2
t
S
x
()[)

2
(5.1.21)
Where the expectation values are given by
S
x
()[) =
_

(x)
+
_

(z)
+
=
1

2
_
1 1
_
_
1
0
_
=
1

2
(5.1.22a)
S
x
()[) =
_

(x)
+
_

(z)

=
1

2
_
1 1
_
_
1
0
_
=
1

2
(5.1.22b)
Substituting (5.1.22a) and (5.1.22b) into (5.1.21),
P =
1
2

2
e
i

2
t
+
1

2
e
i

2
t

2
P = cos
2
_
t
2
_
b) And for the probability for nding the particle in S
x
= 1/2,
P =
1
2

e
i

2
t
S
x
()[) + e
i

2
t
S
x
()[)

2
(5.1.23)
Where we have
S
x
()[) =
_

(x)

(z)
+
=
1

2
_
1 1
_
_
1
0
_
=
1

2
(5.1.24a)
S
x
()[) =
_

(x)

(z)

=
1

2
_
1 1
_
_
0
1
_
=
1

2
(5.1.24b)
Substituting (5.1.24a) and (5.1.24b) into (5.1.23),
P =
1
2

2
e
i

2
t

2
e
i

2
t

2
P = sin
2
_
t
2
_
c) Doing the same thing for S
z
= 1/2,
P =
1
2

e
i

2
t
S
z
()[) + e
i

2
t
S
z
()[)

2
(5.1.25)
Where
S
z
()[) =
_

(z)
+
_

(z)
+
=
_
1 0
_
_
1
0
_
= 1 (5.1.26a)
S
z
()[) =
_

(z)
+
_

(z)

=
_
1 0
_
_
0
1
_
= 0 (5.1.26b)
Putting (5.1.26a) and (5.1.26b) into (5.1.25),
P =
1
2

e
i

2
t

2
P =
1
2
d) And the mantra continues with S
z
= 1/2:
P =
1
2

e
i

2
t
S
z
()[) + e
i

2
t
S
z
()[)

2
(5.1.27)
W. Erbsen QUANTUM MECHANICS
Where we have
S
z
()[) =
_

(z)

(z)
+
=
_
0 1
_
_
1
0
_
= 0 (5.1.28a)
S
z
()[) =
_

(z)

(z)

=
_
0 1
_
_
0
1
_
=
1
2
(5.1.28b)
Slamming (5.1.28a) and (5.1.28b) into (5.1.27),
P =
1
2

e
i

2
t

2
P =
1
2
Problem 5 (Old Problem 2)
A quark-quark potential that is sometimes used in quark models of mesons is V (r) = g log(r/a). If the quarks are
very heavy, then non-relativistic physics is possible.
Use the uncertainty principle to estimate the size and binding energy of a pair of quarks, each of mass m (g > 0).
Solution
Before we get down to bidness we recall that the reduced mass for two particles of equal mass is
=
mm
m + m
=
m
2
m = 2
The uncertainty principle is xp /2 xp . If we assume that r r and p p,
then rp p /r. We now substitute this into the two-particle Hamiltonian, with the given
interasction term:
H =
p
2
1
2m
+
p
2
2
2m
+g log
_
r
a
_
=
p
2
2
+g log
_
r
a
_
=

2
2r
2
+ g log
_
r
a
_
(5.1.29)
The plan of action now is to minimize (5.1.29), by taking the derivative and setting it equal to zero:
dH
dt
=

t
_

2
2r
2
_
+

t
_
g log
_
r
a
__
= 0


2
r
3
+
g
r
= 0 r =

2
g
(5.1.30)
Taking r from (5.1.30) and putting it back into H from (5.1.29),
CHAPTER 5: DEPARTMENTAL EXAMINATIONS 377
H =

2
2
_
g

2
_
+g log
_
1
a

2
g
_
=
g
2
+g log
_

2
a
2
g
_
(5.1.31)
We can see that (5.1.31) is actually an eigenvalue, and so the solution is
E =
g
2
+ g log
_

2
a
2
g
_
Problem 6
Show that for a system consisting of two identical particles with spin I, the ratio of the number of states symmetric
under exchange of the two spins to the number of states antisymmetric under exchange of the two spins is equal
to (I + 1)/I. [Hint: If you do not know where to start, consider I =
1
/
2
rst.]
Solution
For a nucleus with spin quantum number I ,= 0, there are 2I + 1 dierent possible values of the z-
component of the spin angular momentum quantum number:
M
I
= I, I + 1, ..., I 1, I
For two such nuclei, the total multiplicity is (2I + 1) (2I + 1). Clearly, some of the wave functions
associated with these will be symmetric, while some will be anti-symmetric:

S
(x
1
, x
2
) =
1

2
[
a
(x
1
)
b
(x
2
) +
b
(x
1
)
a
(x
2
)] (5.1.32)

A
(x
1
, x
2
) =
1

2
[
a
(x
1
)
b
(x
2
)
b
(x
1
)
a
(x
2
)] (5.1.33)
Where
S
(x
1
, x
2
) denotes the symmetric states, and
A
(x
1
, x
2
) is for the anti-symmetric states. Lets
also denote the number of symmetric states as num(S), and the number of anti-symmetric states as
num(A). In addition to these two sources of states, there is another case for the symmetric wave
function:

(x
1
, x
2
) =
a
(x
1
)
a
(x
2
) (5.1.34)
And as before, we let num(S

) denote the number of symmetric states in the form of (5.1.34). We know


that (5.1.32), (5.1.33) and (5.1.34) represent all possible states for a nucleus with spin quantum number
I. Accordingly,
W. Erbsen QUANTUM MECHANICS
(2I + 1)
2
. .
Total # of states
= num(S) + num(S

)
. .
# of symmetric states
+ num(A)
. .
# of anti-symmetric states
We can see that the multiplicity of num(S

) is just 2I +1, while the multiplicity of num(S) and num(A)


are equal, but unknown (call this multiplicity ). So,
(2I + 1)
2
= + (2I + 1) +
(2I + 1)
2
2I 1 = 2
4I
2
+ 4I + 1 2I 1 = 2
4I
2
+ 2I = 2 = 2I
2
+ I = I(2I + 1)
So, num(S) = num(A) = I(2I + 1), and num(S

) = 2I + 1. The ratio of symmetric states to anti-


symmetric states is then
num(S) + num(S

)
num(A)
=
I(2I + 1) + 2I + 1
I(2I + 1)
=
(2I + 1)
(2I + 1)
(I + 1)
I

I + 1
I
Problem 7
A rod of length d and uniform mass distribution is pivoted at its center and constrained to rotate in an xy-plane.
The rod has mass M and charges +Q and Q xed at either end.
a) Write down the Hamiltonian, its eigenfunctions, and eigenvalues.
b) If a constant weak electric eld lies along the x-direction, calculate the perturbed energy of the ground
state and the rst excited state to the 2
nd
order in .
Solution
a) Classically, we know that the Hamiltonian for a system like this is expressed as
H =
L
2
2I
Where L is the angular momentum, and I the moment of inertia, which in our case is
I =

i
m
i
r
2
i
= m
_
d
/
2
_
2
+ m
_
d
/
2
_
2
=
1
/
2
md
2
And we also know that L
2
can be expressed as
L
2
=
2

2

2
CHAPTER 5: DEPARTMENTAL EXAMINATIONS 379
So, our Hamiltonian is:
H =

2
md
2

2
(5.1.35)
We now insert our Hamiltonian from (5.1.35) into the time-independent Schr odinger equation:


2
md
2

2
() = E(, )

2

2
() +
md
2
E

2
() = 0
With solutions
() = Asin(k) + Bcos(k), k
2
=
md
2
E

2
(5.1.36)
We require rotational periodicity, eg that () = ( + 2). Applying this to (5.1.36),
( + 2) =Asin [k( + 2)] + Bcos [k( + 2)]
=A
_
sin(k) cos(2k)
. .
1
+sin(2k)
. .
0
cos(k)

+ B
_
cos(k) cos(2k)
. .
1
sin(k) sin(2k)
. .
0

=Asin(k) +B cos(k) (5.1.37)


Two important things happened here. First, I assumed that k was an integer, so lets just say that
k = n from now on. Also, I used the following trig identities:
sin ( ) =sincos cos sin
cos ( ) =cos cos sinsin
So, our eigenvalues are then
k
2
= n
2
=
md
2
E

2
E =
n
2

2
md
2
And our eigenfunctions are
() = Asin(n) + Bcos(n) = Ae
in
+Be
in
However, if we imagine our system to be classical, then we can simply our expression for the
eigenfunctions by recognizing that our rotor can only spin in one direction concurrently (classically),
so our wave function collapses and allows only motion in one direction:
() = Ae
in
Normalizing,
A
2
_
2
0
_
e
in
_ _
e
in
_
d = A
2
2 = 1 A =
1

2
So, our eigenfunction is:
() =
1

2
e
in
W. Erbsen QUANTUM MECHANICS
b) The dipole Hamiltonian for an oscillating perturbation is
H

= pE = QdE
0
cos
The rst order energy shift is then
E
(1)
n
=
n
()[H

[
n
())
=
1
2
_
2
0
e
in
(QdE
0
cos ) e
in
d
=
QdE
0
2
_
2
0
cos d = 0
The second order energy shift has the form
E
(2)
n
=

n,m
n=m
[
m
()[H

[
n
())[
2
E
n
E
m
(5.1.38)
We start with the ground state rst. Taking the numerator,

m
()[H

[
n
()) =
1
2
e
im
[QdE
0
cos [e
in
)
=
QdE
0
2
e
im
[ cos [e
in
)
=
QdE
0
2
_
2
0
e
im
cos e
in
d
=
QdE
0
2
_
2
0
cos e
i(nm)
d
=
QdE
0
4
_
2
0
_
e
i
+e
i
_
e
i(nm)
d
=
QdE
0
4
__
2
0
e
i(nm+1)
d +
_
2
0
e
i(nm1)
d
_
=
QdE
0
2
[
n,m+1
+
n,m1
] (5.1.39)
The denominator of (5.1.38) would be
E
n
E
m
=

2
md
2
(n
2
m
2
) (5.1.40)
For the ground state, we have n = 1. Realizing this, we now take (5.1.39) and (5.1.40) and substitute
it back into (5.1.38):
E
(2)
1
=
Q
2
d
2
E
2
0
4
md
2

m=1
[
1,m+1
+
1,m1
[
2
1 m
2
(5.1.41)
The only possible value of m to sum over is m = 2, the rest of the terms vanish. Therefore, the
second order energy shift for the ground state is just
E
(2)
1
=
Q
2
d
2
E
2
0
4
md
2

2
(1)
2
1 (2)
2
E
(2)
1
=
mQ
2
d
4
E
2
0
12
2
CHAPTER 5: DEPARTMENTAL EXAMINATIONS 381
To nd the second order energy shift for the rst excited state, we start with (5.1.41) with n = 2:
E
(2)
2
=
Q
2
d
2
E
2
0
4
md
2

m=2
[
2,m+1
+
2,m1
[
2
4 m
2
(5.1.42)
In this case there are two possible values we can sum to such that (5.1.42) will not collapse: m = 1
and m = 3. Summing these, we can nd the energy:
E
(2)
2
=
Q
2
d
2
E
2
0
4
md
2

2
_
(1)
2
2 (1)
2
+
(1)
2
2 (3)
2
_
=
Q
2
d
2
E
2
0
4
md
2

2
_
1
1
7
_
Which nally leads us to
E
(2)
2
=
3mQ
2
d
4
E
2
0
14
2
c) If the eld is very strong, then the molecule will be perfectly aligned and cos 1, so that
H

= QdE
0
cos H

= QdE
0
. Inserting this into the time-independent Schr odinger equation,
_


2
md
2

2
QdE
0
_
() = E()

2
() +
md
2

2
(QdE
0
+E) () = 0
If we let

2
=
md
2

2
(QdE
0
+ E)
Then our approximate wave function is
() = Acos +B sin
Problem 8
Consider a particle in an innitely deep three-dimensional box with the length on each side equal to a.
a) Write down the Hamiltonian describing such a particle.
b) Find the rst 5 lowest eigenenergies.
c) Find the degeneracy of each energy eigenstate above.
d) If there are 10 noninteracting, indistinguishable fermions in such a box, what is the ground state energy of
the whole system? Consider separately the cases for spin
1
/
2
and spin
3
/
2
particles.
W. Erbsen QUANTUM MECHANICS
Solution
a) The Hamiltonian is for a particle in a three dimensional box is
H =

2
2m

2
+ V H =

2
2m
_

2
x
2
+

2
y
2
+

2
z
2
_
+ V (5.1.43)
Where the potential is dened by
V =
_
V
0
if 0 < x, y, z < a
0 otherwise
b) The energy eigenvalues corresponding to (5.1.43) are given by
E
n
=

2

2
2ma
2
_
n
2
x
+n
2
y
+ n
2
z
_
(5.1.44)
And the rst ve lowest energies, tabulated via (5.1.44) are
E
111
=
3
2

2
2ma
2
E
112
=E
121
= E
211
=
6
2

2
2ma
2
E
122
=E
212
= E
221
=
9
2

2
2ma
2
E
113
=E
131
= E
311
=
11
2

2
2ma
2
E
222
=
12
2

2
2ma
2
E
223
=E
232
= E
322
=
17
2

2
2ma
2
c) The degeneracies are given by the number of dierent combinations of atomic numbers that result
in the same eigenenergy; these are obvious from part b).
d) If 10 spin
1
/
2
fermions are in the box, then the ground state is determined by the fact that 2 particles
can occupy each of the lowest energy states (
1
/
2
for each):
E = 2E
111
+ 2E
211
+ 2E
121
+ 2E
112
+ 2E
122
E =
60
2

2
2ma
2
For spin
3
/
2
, then 4 can occupy each of the lowest states:
E = 4E
111
+ 4E
211
+ 2E
121
E =
48
2

2
2ma
2
Problem 9
Let
[) =
_
1
/
3
Y
10
+
_
2
/
3
Y
11
(5.1.45)
CHAPTER 5: DEPARTMENTAL EXAMINATIONS 383
a) If you were to measure S
z
, what is the probability that you would get /2?
b) If indeed you do get /2 in (a), what is the new state (or wavefunction after the measurement)?
c) If you measure L
z
after (b), what is the probability that you will get ? (Note: for spin up and for
spin down, and the Y s are the spherical harmonics).
Solution
a) The probability of measuring S
z
would simply be the square of the coecient associated with that
state. In other words,
P =
1
3
b) If you do measure /2, then you have essentially collapsed the wave function, leaving only the
part which corresponds to your measurement. In this case,
[) = Y
10
(5.1.46)
c) The chances would be zero. The current state our particle is in (5.1.46), which has = 1 and
m

= 0. In the general case, when measuring L


z
we have
L
z
Y
m

(, ) = m

Y
m

(, )
However, in our case
L
z
Y
0
1
(, ) = (0)Y
0
1
(, )
So, it is very clear that the chances that upon measuring L
z
you get is zero.
Problem 10
A spin
1
/
2
particle is placed in a uniform magnetic eld H
0
directed along z. A small rf eld H
1
cos t is ap-
plied along x. Calculate the time independent probability for the spin to ip from up to down along z. The pauli
spin matrices are

x
=
_
0 1
1 0
_
,
y
=
_
0 i
i 0
_
,
z
=
_
1 0
0 1
_
Solution
W. Erbsen QUANTUM MECHANICS
We begin our journey by restating the problem in a much more transparent way. Our particle is initially
perfectly content with beign spin-up whilst a uniform magnetic eld is being applied to it along the
z-direction. Then, a grumpy old perturbation comes along pointing in the x-direction. The question is:
what are the chances that the particle will go from begin spin-up to spin-up along the z-direction to spin
down along the same direction?
This is a time-dependent perturbation theory problem, and accordingly we require the formalism
of rst-order time-dependent perturbation theory. First things rst, however; the Hamiltonian for a
charged particle in a magnetic eld is given by
H
0
=
eB
0
S
z
mc
, where S
z
=

2
_
1 0
0 1
_
Now we recall from time-dependent perturbation theory that
c
1
(t) =
1
i
H

12
e
i0t
c
2
(t) (5.1.47a)
c
2
(t) =
1
i
H

21
e
i0t
c
1
(t) (5.1.47b)
Now, the particle is initially spin-up, so
E
b

(z)
+
c
b
(0) = 1
E
a

(z)

c
a
(0) = 0
Since at time t = 0 we are guaranteed to be in the upper state, and wish to know the probability that
we will be in the lower state as a function of time, we use (5.1.47a). We are told that the perturbation
takes the form:
H

1
= H
1
cos t =
eB

m
S
x
cos t
So, the matrix element that we use in (5.1.47a) is then
H

ab
=[
eB

m
S
z
cos t[)
=
eB

m
cos t[S
x
[)
=
eB

m
cos t
_
_
0 1
_

2
_
0 1
1 0
__
1
0
__
=
eB

m
cos t
_
_
1 0
_
_
1
0
__
=
eB

m
cos t (5.1.48)
We now take (5.1.48) and substitute it into the rst-order integral equation for the 2 1 transition:
c
a
(t) =
1
i
_
t
0
H

ab
(t

)e
i0t

dt

=
1
i
_
t
0
_
eB

m
cos t

_
e
i0t

dt

CHAPTER 5: DEPARTMENTAL EXAMINATIONS 385


=
eB

im
_
t
0
cos t

e
i0t

dt

=
eB

2im
_
t
0
_
e
it

+ e
it

_
e
i0t

dt

=
eB

2im
__
t
0
e
i(0)t

dt

+
_
t
0
e
i(0+)t

dt

_
=
eB

2im
_
_
_
_

e
i(0)t

i(
0
)

t
0
+
_

e
i(0+)t

i(
0
+ )

t
0
_
_
_
=
eB

2im
_

e
i(0)t
i(
0
)
+
1
i(
0
)

e
i(0+)t
i(
0
+ )
+
1
i(
0
+ )
. .
_
(5.1.49)
We typically neglect the underbraced terms in (5.1.49). Why? Good question. We are typically looking
at driving frequencies that are very close to the resonant frequencies, eg
0
. In this approximation,
the underbraced terms are very small compared to the rest of the expressions, so they essentially vanish.
We could carry them on through the end to get an exact solution; but this is not necessary. Continuing,
c
a
(t) =
eB

2im
_

e
i(0)t
i(
0
)
+
1
i(
0
)
_
=
eB

2im
e
i
(
0
)
2
t
i(
0
)
_
e
i
(
0
)
2
t
e
i
(
0
+)
2
t
_
=
eB

2im
e
i
(
0
)
2
t
i(
0
)
sin
_
(
0
)t
2
_
(5.1.50)
And to nd the transitional probability, we square the modulus of (5.1.50), and we obtain:
P
ba
=
e
2
B
2
m
2
1
(
0
+)
2
sin
2
_
(
0
)t
2
_
Problem 11
Calculate the transmission coecient T at any energy E for the potential
V (x) =

2
2ma
(x b) (5.1.51)
Solution
The solutions of the time-independent Schr odinger for this problem are

I
(x) =Ae
ikx
+Be
ikx
for x < b (5.1.52a)
W. Erbsen QUANTUM MECHANICS

II
(x) =Ce
ikx
+ De
ikx
for x > b (5.1.52b)
We can immediately say that D = 0 in (5.1.52b), since we dont expect any reection after the well. The
rst continuity condition is
I
(b) =
II
(b):
Ae
ikb
+Be
ikb
= Ce
ikb
A+ Be
2ikb
= C B = e
2ikb
(C B) (5.1.53)
The second continuity condition is

I
(b) =

II
(b), and in order to apply this we must go back to the
time-independent Schr odinger equation with the potential provided:
_

2
2m

2
x
2


2
2ma
(x b)
_
(x) =E(x)

2
x
2
(x) +
1
a
(x b)(x) =
2mE

2
(x)
_
b+
b

2
x
2
(x) dx +
1
a
_
b+
b
(x b)(x) dx =0
lim
0
_
_

x
(x)

b+

_

x
(x)

b
_
=
1
a
(b)

II
(b)

x

I
(b) =
1
a

II
(b)

x
_
Ce
ikx
_


x
_
Ae
ikx
+ Be
ikx
_
=
C
a
e
ikb
ikCe
ikb
ikAe
ikb
+ ikBe
ikb
=
C
a
e
ikb
C A+Be
2ikb
=
C
ika
(5.1.54)
At this point we substitute (5.1.53) into (5.1.54),
C A+
_
e
2ikb
(C A)
_
e
2ikb
=
C
ika
C A +C A =
C
ika
2C +
C
ika
=2A
C
A
_
2 +
1
ika
_
= 2 (5.1.55)
And the transmission coecient is
T =

C
A

2
=

2
2 +
1
ika

2
=
_
2
2
1
ika
__
2
2 +
1
ika
_
=
4
4 +
1
k
2
a
2
T =
1
1 +

2
8mEa
2
Problem 12
CHAPTER 5: DEPARTMENTAL EXAMINATIONS 387
Consider a two-state system with state [1), [2) and energies E
1
and E
2
, respectively. The system is in the state [1).
At t = 0 it enters a region where it is perturbed by a constant potential with matrix elements 1[V [1) = 2[V [2) = 0,
1[V [2) = 2[V [1) = V . Find P
2
(t), the probability that the system is in the state [2) as a function of time.
Solution
This problem directly reects the derivation of the time-dependent perturbation theory for a two-state
system. This is derived in great detail in Griths in Chapter 9. The general idea is that you have two
dierent states with two dierent energy levels, where each state is an eigenstate in the time-independent
Schr odinger equation:

1
(t) =c
1
e
iE1t/
[1), H
0
[1) = E
1
[1)

2
(t) =c
1
e
iE2t/
[2), H
0
[2) = E
2
[2)
Our total wave function can then be expressed as a linear combination of
1
(t) and
2
(t), and the
coecients are time-dependent :
(t) = c
1
(t)e
iE1t/
[1) +c
2
(t)e
iE2t/
[2) (5.1.56)
By substituting
1
(t) and
2
(t) into the time-dependent Schr odinger equation, with the Hamiltonian
H H
0
+H

(t), where H
0
is the unperturbed Hamiltonian and H

(t) is the time-dependent perturbation.


After simplication, we arrive at:
c
1
(t) =
1
i
H

12
e
i0t
c
2
(t) (5.1.57a)
c
2
(t) =
1
i
H

21
e
i0t
c
1
(t) (5.1.57b)
Where we have dene H

12
and H

21
to be
H

12
=
1
(t)[H

(t)[
2
(t)), H

21
=
2
(t)[H

(t)[
1
(t))
While
0
is

0
=
E
2
E
1

The hallmark result from rst-order time-dependent perturbation theory is


c
(1)
2
(t) =
1
i
_
t
0
H

21
(t

)e
i0t

dt

(5.1.58)
By inserting the appropriate values into (5.1.58) and squaring the modulus of the result, we arrive at
the probability that the system will transition from
1
(t) to
2
(t) as a function of time. The provided
perturbation matrix is
V
ij
=
_
0 V
12
V
21
0
_
(5.1.59)
Where we note that V
12
= V
21
= V . Substituting the relevant matrix element from (5.1.59) into (5.1.58),
W. Erbsen QUANTUM MECHANICS
c
(1)
2
(t) =
1
i
V
_
t
0
e
i0t

dt

=
V
i
_
e
i0t

i
0

t
0
=
V

0
_
e
i0t
1
_
=
V

0
e
i
0
t
2
_
e
i
0
t
2
e

i
0
t
2
_
=
2iV

0
e
i
0
t
2
sin
_

0
t
2
_
=
2iV
(E
2
E
1
)
e
i(E
2
E
1
)t
2
sin
_
(E
2
E
1
)t
2
_
(5.1.60)
Taking the modulus squared of (5.1.60),
P
12
= [c
1
(t)[
2
P
12
=
4V
2
(E
2
E
1
)
2
sin
2
_
(E
2
E
1
)t
2
_
Problem 13
Consider a hydrogen atom in its ground state. An electric eld, E(t), is applied in the z-direction.
E(t) =
_
0 t < 0
E
0
e
t/
t > 0
(5.1.61)
a) Apply rst-order time-dependent perturbation theory to calculate an expression for the probability that the
hydrogen atom is in an excited state.
b) What is the probability if the excited state is (i) 2s and (ii) 2p? Explain briey the justication for your
answer.
c) How do your answers to (b) behave in the limits = 0 and = ? Explain physically whether this behavior
is sensible.
Solution
Before we get down to business, lets remember a few things. For instance,

nm
(r, , ) =R
n
(r)Y
m

(, )
=

_
2
na
_
3
(n 1)!
2n[(n + )!]
3
e

r
na
_
2r
na
_

_
L
2+1
n1
_
2r
na
__
Y
m

(, ) (5.1.62)
And the rst three radial wave functions are:
CHAPTER 5: DEPARTMENTAL EXAMINATIONS 389
R
10
= 2a

3
/2
exp
_

r
a

Y
0
0
=
_
1
4
_1
/2
R
20
=
1

2
a

3
/2
_
1
r
2a
_
exp
_

r
2a

Y
0
1
=
_
3
4
_1
/2
cos
R
21
=
1

24
a

3
/2
r
a
exp
_

r
2a

Y
1
1
=
_
3
8
_1
/2
sin e
i
So, the rst excited state has n = 2 with = 0, 1, and admits the following states:

200
(r, , ) =R
20
(r)Y
0
0
(, )
=
_
1

2
a

3
/2
_
1
r
2a
_
exp
_

r
2a
_
__
1

4
_
=
1

8
a

3
/2
_
1
r
2a
_
exp
_

r
2a
_
(5.1.63)

210
(r, , ) =R
21
(r)Y
0
1
(, )
=
_
1

24
a

3
/2
r
a
exp
_

r
2a
_
_
_
_
3
4
cos
_
=
1
4

2
a

3
/2
r
a
exp
_

r
2a
_
cos (5.1.64)
We also recall the time-dependent coecients,
c
a
(t) =
1
i
H

ab
e
i0t
c
b
(t)
c
b
(t) =
1
i
H

ba
e
i0t
c
a
(t)
And in the spirit of rst order time-dependent perturbation theory,
c
(1)
b
=
1
i
_
t
0
H

ba
(t

)e
i0t

dt

(5.1.65)
The matrix element in (5.1.65) is given by
H

ba
=
b
[H

[
a
)
The perturbation in our case is E(t) = E
0
e

t
/
, and for dipole radiation the perturbation becomes
H

= pE = qzE
0
e

t
/
If we recall that in spherical coordinates z = r cos , then the full matrix element becomes
H

ba
(t) =
b
[
_
qE
0
e

t
/
r cos
_
[
a
)
= qE
0
e

t
/

b
[r cos [
a
) (5.1.66)
For the most general expression for the probability that hydrogen is in an excited state, we carry on
calculating our matrix element from (5.1.66) using the full wave functions for hydrogen:
H

ba
= qE
0
e

t
/

nm
(r, , )[r cos [
100
(r, , ))
= qE
0
e

t
/
a

3
/2

R
nl
(r)Y
m
l
(, )[r cos [e

r
/a
)
W. Erbsen QUANTUM MECHANICS
= qE
0
e

t
/
a

3
/2

_
2
0
_

0
_

0
(R
n
(r)Y
m

(, )) (r cos )
_
e

r
/a
_
r
2
sin drdd
= qE
0
e

t
/
a

3
/2

_
2
0
_

0
Y
m

(, ) cos sin dd
_

0
r
3
R
n
(r)e

r
/a
dr (5.1.67)
To nd the time-dependent probability, we would take the matrix element (5.1.67) and substitute it into
the integral (5.1.65) and take the modulus squared. Since We do not know what state we are calculating
the probability of jumping to, things will get very messy very quickly. Although they do call me messy
wessy, this does not have anything to do with messy equations. Yuck.
a) The dipole selection rules prevent the 1s 2s transition ( 1), so lets nd the probability of
exciting to the 2p state. The matrix element would be
H

ba
(t) = qE
0
e

t
/

210
(r, , )[r cos()[
100
(r, , ))
= qE
0
e

t
/
_
1
4

2
a

3
/2
r
a
e

r
/2a
cos

r cos

3
/2
e

r
/a
_
= qE
0
e

3
/2
1
a
3
1
4

2
1
a
_
re

r
/2a
cos()

r cos()

r
/a
_
=
qE
0
e

t
/
4

2 a
4
_
re

r
/2a
cos()

r cos()

r
/a
_
=
_
2
0
_

0
_

0
_
re

r
/2a
cos()
_
(r cos())
_
e

r
/a
_
r
2
sin() drdd
=
_
2
0
d
_

0
cos
2
() sin() d
_

0
r
4
e

3r
/4a
dr (5.1.68)
Where I have temporarily clumped all constants infront of the integrals as . To solve the integral,
we require the following trig identities:
sin
2
() =
1
2
(1 cos(2)) , cos
2
() =
1
2
(1 + cos(2))
sin() cos() =
1
2
[sin (

) + sin ( )]
So the integral becomes
_

0
cos
2
sin() d =
_

0
1
2
(1 + cos(2)) sin() d
=
1
2
_

0
sin() d +
1
2
_

0
sin() cos(2) d
=
1
2
[cos()[

0
+
1
4
_

0
[sin(3) + sin() d]
=
1
2
[1 + 1] +
1
4
1
3
[1 + 1]
1
4
[1 + 1]
=1 +
1
6

1
2
=
2
3
(5.1.69)
For the r integral, we recall the following integral
CHAPTER 5: DEPARTMENTAL EXAMINATIONS 391
_

0
x
n
e
ax
dx =
n!
a
n+1
Then the r integral becomes
_

0
r
4
e

3r
/4a
dr =
4!
(
3
/
4a
)
5
=
4
5
a
5
4!
3
5
(5.1.70)
Combining (5.1.69) and (5.1.70) into (5.1.68) (lets throw back in there too),
H

ba
(t) =
qE
0
e

t
/
4

2 a
4
(2)
_
2
3
__
4
5
a
5
4!
3
5
_
= qE
0
e

t
/
a
4
5
4!
3
6

2
(5.1.71)
Substituting (5.1.71) into (5.1.65),
c
(1)
b
=
qE
0
a
i
4
5
4!
3
6

2
_
t
0
e

/
e
i0t

dt

=
qE
0
a
i
4
5
4!
3
6

2
_
t
0
e
t

(i0
1
/)
dt

=
qE
0
a
i
4
5
4!
3
6

2
_
e
i0t

/
(i
0

1
/

t
0
=
qE
0
a
i
4
5
4!
3
6

2
_
e
(i0
1
/)t
(i
0

1
/

)

1
(i
0

1
/

)
_
=
qE
0
a
i
4
5
4!
3
6

2
1
(i
0

1
/

)
_
e
(i0
1
/)t
1
_
=
qE
0
a
i
4
5
4!
3
6

2
e
(i0
1
/)
t
2
(i
0

1
/

)
_
e
(i0
1
/)
t
2
e
(i0
1
/ )
t
2
_
=
qE
0
a

2
11
4!
3
6

2
e
i
0
t
2
e
t
2
(i
0

1
/

)
sin
_
(i
0

1
/

)t
2
_
(5.1.72)
The probability of transition is then
P(1s 2p) =

qE
0
a

2
11
4!
3
6

2
e
i
0
t
2
e
t
2
(i
0

1
/

)
sin
_
(i
0

1
/

)t
2
_

2
Which nally leads to
P(1s 2p) =
q
2
E
2
0
a
2

2
2
21
(4!)
2
3
12
e
t
/2

2
0
+ (
1
/

)
2
sin
2
_
(i
0

1
/

)t
2
_
(5.1.73)
b) If we let = 0, then (5.1.73) leads to
P(1s 2p) =
q
2
E
2
0
a
2

2
2
21
(4!)
2
3
12
1

2
0
sin
2
_
i
0
t
2
_
W. Erbsen QUANTUM MECHANICS
Problem 14
Deuterons have spin-1 (bosons).
a) In general, what are the possible total spin S and total spin angular momentum J of two deuterons in an
arbitrary (relative) orbital angular momentum state L? Assume the state vector is a product of a spin part
and an orbital part. Determine just the allowed S, J, not the state vector.
b) In particular, for L = 0 and L = 1, what S and J are allowed?
Solution
a) For bosons, m
s
= 1, so S = 0, 1, 2, and the possible arbitrary J values are just J = L + S =
[L S[, ..., [L+S[. The (arbitrary) total angular momentum quantum numbers are given by
L S J
L 2 [L 2[, ..., [L+ 2[
L 1 [L 1[, ..., [L+ 1[
L 0 L
b) Lets make a table and nd out!
L S J
0 2 2
1 2 1, 2, 3
0 1 1
1 1 0, 1, 2
0 0 0
1 0 1
Problem 15 (Old Problem 16)
A particle of mass m is conned to a one-dimensional potential (x), > 0. Show that there is one bound
state. Calculate its binding energy and its eigenfunction.
Solution
We imagine our potential to consist of three regions: I is before the potential, II is at the potential, and
III is after the potential. Plugging our given potential into the time-dependent Schr odinger equation,
_

2
2m

2
x
2
(x)
_
(x) = E(x) (5.1.74)
We now solve (5.1.74) for each of the regions. For the rst region (region I),


2
2m

2
x
2
(x) = E(x)

2
x
2
(x) k
2
(x) = 0, k =
_
2m[E[

(5.1.75)
CHAPTER 5: DEPARTMENTAL EXAMINATIONS 393
The sign changed because we know that E is negative. The solution to (5.1.75) is rather obvious,

I
(x) = Ae
kx
+ Be
kx

I
(x) = Ae
kx
(5.1.76)
Where we found that B = 0 due to boundary conditions. The solution for region III is similar,

III
(x) = Ce
kx
+ De
kx

III
(x) = De
kx
(5.1.77)
And boundary conditions dictated that C = 0, as before. We now move on to the trickier region, that
of II:

2
2m

2
x
2
(x) + V (x)(x) = E(x)


2
2m
_

2
x
2
(x) dx +
_

V (x)(x) dx = E
_

(x) dx

2
2m
lim
0
_
_

x

_

x

(x)(x) dx = 0

2
2m
_

x

III
(x)

x

I
(x)
_
= (0)

x
_
De
kx
_


x
_
Ae
kx
_
=
2m

2
D
kD kA =
2m

2
D
At x = 0,
I
(0) =
III
(0) A = D, so:
2kA =
2m

2
A K =
m

2
=
_
2m[E[

Rearranging and solving,


2m[E[ =
m
2

2
[E[ =
m
2
2
2
E =
m
2
2
2
(5.1.78)
We can see that the only values in (5.1.78) are numbers; there are no indicies like n, that would infer a
dierent eigenenergy. Therefore, (5.1.78) is the only bound state. For normalization,
_

0
[
III
(x)[ dx =1
2A
2
_

0
e
2kx
dx =1
A
2
__

1
2k
e
2kx

1
2k
e
2kx

0
_
=0
A
2
k
= 1 A =

So that we have
(x) =

exp
_

2
x
_
W. Erbsen QUANTUM MECHANICS
Problem 16
Consider the isotropic harmonic oscillator where the potential is given by
V (r) =
1
/
2
m
2
r
2
(5.1.79)
a) Show that the eigenfunctions can be expressed in the form R
n
(r)Y
m

.
b) The problem is separable in Cartesian coordinates, show that the eigenenergies can be expressed as
E
n
=
_
n +
3
/
2
_
(5.1.80)
c) Find the degeneracy D(n) of E
n
for the rst 4 values of energies. You obtain the degeneracy from the
solutions in Cartesian coordinates. For each energy, identify the value(s) of orbital angular momentum (or
momenta if more than one) for each eigenenergy.
Solution
a) I dont wanna
b) The full Hamiltonian for the 3-D isotropic harmonic oscillator is
H =

2
2m

2
+
1
2
m
2
r
2
=

2
2m
_

2
x
2
+

2
y
2
+

2
z
2
_
+
1
2
m
2
_
x
2
+ y
2
+ z
2
_
(5.1.81)
We now insert (5.1.81) into the time-independent Schr odinger equation,
_

2
2m
_

2
x
2
+

2
y
2
+

2
z
2
_
+
m
2
2
_
x
2
+y
2
+z
2
_
_
(x, y, z) = E(x, y, z) (5.1.82)
We will solve this partial dierential equation by separation of variables, whereby we assume that
a separable solution exists in the form (x, y, z) = X(x)Y (y)Z(z). Substituting this into (5.1.82),
_

2
x
2
+

2
y
2
+

2
z
2
_
X(x)Y (y)Z(z)
m
2

2
_
x
2
+ y
2
+ z
2
_
X(x)Y (y)Z(z) =
2mE

2
1
X(x)

2
x
2
X(x) +
1
Y (y)

2
y
2
+
1
Z(z)

2
z
2

m
2

2
m
2
_
x
2
+ y
2
+ z
2
_
X(x)Y (y)Z(z) =
2mE

2
_
1
X(x)

2
x
2
X(x)
_
m

_
2
x
2
_
+
_
1
Y (y)

2
x
2
Y (y)
_
m

_
2
y
2
_
+
_
1
Z(z)

2
x
2
Z(z)
_
m

_
2
z
2
_
=
2mE

2
(5.1.83)
If we let E be the separation constant, where E = E
x
+E
y
+E
z
, we nd that (5.1.83) can be fully
separated into three unique equations:
1
X(x)

2
x
2
X(x)
_
m

_
2
x
2
+
2mE
x

2
= 0 (5.1.84a)
1
Y (y)

2
y
2
Y (y)
_
m

_
2
y
2
+
2mE
y

2
= 0 (5.1.84b)
1
Z(z)

2
z
2
Z(z)
_
m

_
2
z
2
+
2mE
z

2
= 0 (5.1.84c)
CHAPTER 5: DEPARTMENTAL EXAMINATIONS 395
It is clear that (5.1.84a)-(5.1.84c) represent the 1-D dierential equations for the harmonic oscillator
along the x, y, and z directions, respectively. The eigenvalues for the three 1-D solutions are well
known,
E
x
=
_
n
x
+
1
/
2
_
, E
y
=
_
n
y
+
1
/
2
_
, E
z
=
_
n
z
+
1
/
2
_
So that the total energy becomes
E
nx,ny,nz
=
_
n
x
+ n
y
+n
z
+
3
/
2
_
E
n
=
_
n +
3
/
2
_
c) We can discover the degeneracies by using a table as follows
n
x
n
y
n
z
E()
0 0 0
3
/
2
1 0 0
5
/
2
0 1 0
5
/
2
0 0 1
5
/
2
1 1 0
7
/
2
1 0 1
7
/
2
0 1 1
7
/
2
1 1 1
9
/
2
2 0 0
7
/
2
0 2 0
7
/
2
0 0 2
7
/
2
2 1 0
9
/
2
2 0 1
9
/
2
0 2 1
9
/
2
0 1 2
9
/
2
1 0 2
9
/
2
1 2 0
9
/
2
3 0 0
9
/
2
0 3 0
9
/
2
0 0 3
9
/
2
This table includes all possible values for the lowest four energy levels: E
0
=
3
/
2
, E
1
=
5
/
2
,
E
2
=
7
/
2
, E
3
=
9
/
2
. The degeneracies, as well as the possible values for the orbital angular
momentum are:
E d n
E
0
1 0 0
E
1
3 1 0
E
2
6 2 0, 1
E
3
10 3 0, 1, 2
Problem 17
W. Erbsen QUANTUM MECHANICS
a) Consider a helium ion. The two electrons are in the 2p and 3p states. The energy levels will have denite total
angular momentum (J = L + S). What J-values can occur and how many degenerate energy levels for each J
can there be?
b) How do your answers change if the electrons are both in the 3p states?
Solution
a) Lets call the electron in the 2p state 1, and the electron in the 3p state 2. Then
1
= 1, and

2
= 1. The total orbital angular momentum can then take the values
L =[
1

2
[, ..., [
1
+
2
[ = 0, 1, 2 (5.1.85)
And for electrons, m
s
=
1
/
2
, so the possible values for the total spin angular momentum are
S = 0, 1 (5.1.86)
And we also know that the possible values for the total angular momentum is determined by
J = L + S = [L S[, ..., [L+ S[ (5.1.87)
And to nd the possible values, we create a table:
L S J
0 1 1
1 1 0, 1, 2
2 1 1, 2, 3
0 0 0
1 0 1
2 0 2
So, the possible values for J are J = 0, 1, 2, 3 , each with degeneracies 2J + 1:
J D
0 1
1 3
2 5
3 7
b) If both electrons are in the 3p state, then
L =0, 1, 2
S =0
And the possible total angular momentum quantum numbers can be found by
L S J
0 0 0
1 0 1
2 0 2
So the possible values for J are J = 0, 1, 2 . The degeneracies are
CHAPTER 5: DEPARTMENTAL EXAMINATIONS 397
J D
0 1
1 3
2 5
Problem 18
A particle of spin zero is conned in a two-dimensional innite square well of dimension 2a on each side.
a) Find the energies and the wave functions of the rst two states.
b) If there are two indistinguishable noninteracting spin
1
/
2
particles inside the box, write down the total wave
function of the rst two states. Identify the degree of degeneray for each state.
c) If there are ve indistinguishable spin
1
/
2
noninteracting particles in the box, what is the total energy of the
ground state?
d) If there are ve indistinguishable spin 1 noninteracting particles in the box, what is the total energy of the
ground state?
Solution
a) In general, the eigenstates and energy eigenvalues for this problem are given by
(x, y) =
1
a
sin
_
n
x
x
2a
_
sin
_
n
y
y
2a
_
(5.1.88a)
E
nx,ny
=

2

2
8ma
2
(n
2
x
+ n
x
y
) = E
0
(n
2
x
+ n
2
y
) (5.1.88b)
The the ground state, n
x
= n
y
= 1, (5.1.88a) and (5.1.88b) become
(x, y) =
1
a
sin
_
x
2a
_
sin
_
y
2a
_
, E
11
= 2E
0
For the rst excited state, n
x
= 2, and n
y
= 1, so we have
(x, y) =
1
a
sin
_
x
a
_
sin
_
y
2a
_
, E
12
= 5E
0
So, the rst excited state is doubly degenerate.
b) We rst recall that the total wave function of a fermion (space and spin part) must be completely
anti-symmetric under the exchange of any two particles in the system.
For the case at hand, we may have to symmetrize, or anti-symmetrize the wave function
such that the nal total wave function is anti-symmetric. In other words, if the spin part is sym-
metric, then the space part must be anti-symmetric, and also vice versa.
We recall that, in general,
W. Erbsen QUANTUM MECHANICS
(x
1
, x
2
) =
1

2
[
n1
(x
1
)
n2
(x
2
)
n2
(x
1
)
n1
(x
2
)] (5.1.89)
And also
(x
1
, x
2
) =
1
a
sin
_
n
1
x
1
2a
_
sin
_
n
2
x
2
2a
_
(5.1.90)
For the ground state, n
1
= n
2
= 1 and S = 0 (singlet) so the space part is just
(x
1
, x
2
) =
1
a
sin
_
x
1
2a
_
sin
_
x
2
2a
_
(5.1.91)
Since the space part is symmetric, the spin part must be anti-symmetric:

0
0
=
1

2
[[) [)] (5.1.92)
The total wave function for the ground state for two indistinguishable spin-
1
/
2
fermions is just the
product of (5.1.91) and (5.1.92)
(x
1
, x
2
) =
1
a

2
sin
_
x
1
2a
_
sin
_
x
2
2a
_
[[) [)]
The ground state has singlet degeneracy. For the rst excited state, where are two dierent options:
electron 1 can be excited or electron 2 can be excited.
Either of the rst excited states may have by a symmetric (S = 0) or an anti-symmetric (S = 1)
spatial wave function, like (5.1.89). In our case, letting n
1
= 1 and n
2
= 2,
(x
1
, x
2
) =
1
a

2
_
sin
_
x
1
2a
_
sin
_
x
2
a
_
+ sin
_
x
1
a
_
sin
_
x
2
2a
__
(5.1.93a)
=
1
a

2
_
sin
_
x
1
2a
_
sin
_
x
2
a
_
sin
_
x
1
a
_
sin
_
x
2
2a
__
(5.1.93b)
Where (5.1.93a) is symmetric and thus has an anti-symmetric spin part, while (5.1.93b) is anti-
symmetric , and must have a symmetric spin part. The former is familiar; the latter (S = 1) has
three components, each corresponding to one of the available symmetric spin functions. So, this
gives us four wave functions:
(x
1
, x
2
) =
1
a

2
_
sin
_
x
1
2a
_
sin
_
x
2
a
_
+ sin
_
x
1
a
_
sin
_
x
2
2a
__
1

2
[[) [)]
=
1
a

2
_
sin
_
x
1
2a
_
sin
_
x
2
a
_
sin
_
x
1
a
_
sin
_
x
2
2a
__
[)
=
1
a

2
_
sin
_
x
1
2a
_
sin
_
x
2
a
_
sin
_
x
1
a
_
sin
_
x
2
2a
__
1

2
[[) +[)]
=
1
a

2
_
sin
_
x
1
2a
_
sin
_
x
2
a
_
sin
_
x
1
a
_
sin
_
x
2
2a
__
[)
We arrive at four more wave functions if we choose n
1
= 2 and n
2
= 1. Therefore, the rst excited
state is eightfold degenerate, and
E = 5E
0
CHAPTER 5: DEPARTMENTAL EXAMINATIONS 399
c) If there are ve indistinguishable spin
1
/
2
fermions in the box, the total energy of the ground state
can be determined via a table:
n
1
n
2
E d
1 1 2E
0
1
1 2 5E
0
4
2 1 5E
0
4
There can be two electrons per each set of quantum numbers, so we can t ve in the rst two rows,
so
E = 2 2E
0
+ 3 5E
0
E = 19E
0
d) For ve indistinguishable spin-1 bosons, all the particles can be in the lowest level, so the ground
state is just
E = 5 2E
0
E = 10E
0
Problem 19
Ten electrons are conned in a box of dimensions (a, 2a, 2a) on each side. Calculate the total energy of the
ground state of these ten electrons if we assume that the electrons do not interact with each other.
Solution
The separated eigenfunctions and associated eigenenergies are
(x) =
_
2
a
sin
_
n
x
x
a
_
, E
x
=
n
2
x

2
2ma
2
(5.1.94a)
(y) =
_
1
a
sin
_
n
y
y
2a
_
, E
y
=
n
2
y

2
8ma
2
(5.1.94b)
(z) =
_
1
a
sin
_
n
z
z
2a
_
, E
z
=
n
2
z

2
8ma
2
(5.1.94c)
The total energy is then
E
nxnynz
=

2

2
8ma
2
_
4n
2
x
+n
2
y
+ n
2
z
_
(5.1.95)
In order to nd out which combination of atomic numbers yield the ground state, we create a table:
W. Erbsen QUANTUM MECHANICS
n
x
n
y
n
z
4n
2
x
n
2
y
n
2
z
#
1 1 1 4 1 1 6 1
1 1 2 4 1 4 9 2
1 2 1 4 4 1 9 3
2 1 1 16 1 1 18
2 1 2 16 1 4 21
2 2 1 16 4 1 21
1 2 2 4 4 4 12 4
1 1 3 4 1 9 14 5
1 3 1 4 9 1 15
3 1 1 36 1 1 38
With fermions, two can have each energy (
1
/
2
), so we picked the ve lowest energies and now multiply
them by two and sum to nd the total ground state energy:
E =

2

2
8ma
2
[2 (6 + 9 + 9 + 12 + 14)] E =
100
2

2
8ma
2
Problem 23
Suppose a particle moves in one dimension under the potential V (x). V (x) is even, that is V (x) = V (x).
Also, all the eigenstates of the Hamiltonian H are known to be non-degenerate.
a) Prove that every eigenstate of H has either even or odd parity.
b) Consider a completely free particle in one dimension. Show that there are solutions to the eigenvalue equation
which do not have well-dened parity. Explain why this is not a contradiction to the theorem of part (a).
Solution
a) To show that every eigenstate of H has either even or odd parity, we apply the parity operator to
the time-independent Schr odinger equation:
PH(x) =P
_
p
2
2m
+ V (x)
_
(x) =
_
p
2
2m
+V (x)
_
(x) =
_
p
2
2m
+ V (x)
_
(x) (5.1.96)
If we were to apply the parity operator twice,
P
2
H(x) = P
_
p
2
2m
+V (x)
_
(x) =
_
p
2
2m
+ V (x)
_
(x) (5.1.97)
So, we can see that P
2
has an eigenvalue of 1: P
2
= 1 P = 1. Furthermore, P and H
commute, so they must share the same eigenvector. Since the eigenstates of P can have either even
or odd parity (p = 1), and H and P commute (have same eigenvector), then every eigenstate of
H must have either even or odd parity.
CHAPTER 5: DEPARTMENTAL EXAMINATIONS 401
b) A free particle is described by

+
(x) = Ae
ikx
,

(x) = Ae
ikx
And if we apply the parity operator once,
P
+
(x) =PAe
ikx
= Ae
ikx
P

(x) =PAe
ikx
= Ae
ikx
Since
+
(x) and

(x) have the same energy (they are degenerate), we cannot follow the same
logic as in a).
Problem 24
Consider an isotropic 3D spherical square well. What are the conditions on the depth V
0
and radius a such
that a particle of mass m has exactly one bound state in this well? Show your reasoning and calculations.
Solution
The radial equation that we wish to solve is in terms of a intermediary function U(r) = rR(r) One could
just imagine it to be and carry on, since it plays no important part here.
_

2
2m

r
2
+ V +

2
2m
( + 1)
r
2
_
U(r) = EU(r) (5.1.98)
However, for the ground state, we recognize that = 0 and V
e
V , and (5.1.98) becomes
_

2
2m

r
2
+ V
_
U(r) = EU(r) (5.1.99)
For our isotropic spherical square well, we dene two regions; I is inside the sphere, where V = V
0
,
and II is outside, where V = 0. Our task is to rst solve (5.1.99) for each of these two regions. Starting
with region I,
_

2
2m

r
2
V
0
_
U(r) = EU(r)

2
r
2
u(r) +
2m(E +V
0
)

2
U(r) = 0

2
r
2
U(r) +k
2
U(r) = 0, k =
_
2m(E +V
0
)

(5.1.100)
The solution of (5.1.100) is
U
I
(r) = Asin (kr) + Bcos (kr) (5.1.101)
And now for region II,
W. Erbsen QUANTUM MECHANICS


2
2m

2
r
2
U(r) = EU(r)

2
r
2
U(r) +
2
U(r) = 0, =

2mE

(5.1.102)
The solution of (5.1.102) we wish to put in exponential form,
U
II
(r) = Ce
ir
+De
ir
(5.1.103)
Due to boundary conditions our solutions for regions I and II become
U
I
(r) =Asin (kr)
U
II
(r) =De
ir
Our two boundary conditions are U
I
(a) = U
II
(a), and U

I
(a) = U

II
(a). These are, respectively:
Asin (ka) = De
ia
(5.1.104a)
K
I
Acos (ka) = iDe
ia
(5.1.104b)
If we take (5.1.104a) and divide it by (5.1.104b),
tan (ka) =
k
i
cot (ka) =
i
k
(5.1.105)
Lets dene z = ka and z
0
= a

2mV
0
/. Then
z
2
0
=
a
2

2
2mV
0
V
0
a
2
=

2
2m
z
2
0
and (5.1.105) becomes
cot(z) =
_
_
z
0
z
_
2
1 (5.1.106)
If we graph (5.1.106), it is very clear what the boundaries are that determine what the radius and depth
of the well should be. z
0
will intersect the z-axis between /2 and 3/2 if there is to be exactly one
bound state (less than /2 means there is no bound state). So,

2
< z
0
<
3
2

2
2m
_

2
_
2
< V
0
a
2
<

2
2m
_
3
2
_
2
So, the conditions on the width a and depth v
0
of the well such that there is exactly one bound state is:

2
8m
< V
0
a
2
<
9
2

2
8m
CHAPTER 5: DEPARTMENTAL EXAMINATIONS 403
Problem 25
If the spin state of an electron is prepared such that it has a spin /2 along the x-direction, what is the probability
of nding this spin to have eigenvalue /2 if the spin is measured along the y-direction? Show the steps in your
calculation. The Pauli spin matrices are:

x
=
_
0 1
1 0
_
,
y
=
_
0 i
i 0
_
,
z
=
_
1 0
0 1
_
Solution
In the spirit of showing all the steps in our calculations, our rst step would be to nd the eigenvalues
and eigenvectors of S
x
, S
y
and S
z
. We know that S = /2, where represents the Pauli spin matrices.
Starting with S
x
rst,

/2
/2

=
2


2
4
= 0 =

2
And now we nd the eigenvectors for each of the eigenvalues:
For = +/2 :
_
/2 /2
/2 /2
_ _

_
=
_
0
0
_


2
+

2
= 0
(x)
+
=
1

2
_
1
1
_
For = /2 :
_
/2 /2
/2 /2
_ _

_
=
_
0
0
_

2
+

2
= 0
(x)

=
1

2
_
1
1
_
Doing the same for S
y
,

i/2
i/2

=
2


2
4
= 0 =

2
Now the eigenvectors are
For = + /2 :
_
/2 i/2
i/2 /2
_ _

_
=
_
0
0
_
i
2


2
= 0
(y)
+
=
1

2
_
1
i
_
For = /2 :
_
/2 i/2
i/2 /2
_ _

_
=
_
0
0
_
i
2
+

2
= 0
(y)

=
1

2
_
1
i
_
And now for S
z

/2 0
0 /2

2
4
+
2
= 0 =

2
Whose eigenvectors are
For = +/2 :
_
0 0
0
_ _

_
=
_
0
0
_
= 0
(z)
+
=
_
1
0
_
For = /2 :
_
0
0 0
_ _

_
=
_
0
0
_
= 0
(z)

=
_
0
1
_
If, for the sake of exercise, we have a particle that is prepared in the z-direction with eigenvalue /2 and
we wish to know the probability of obtaining /2 in the x-direction,
W. Erbsen QUANTUM MECHANICS
_

(x)
+
_

(z)
=
1

2
_
1 1
_
_

_
=
+

2
P =

2
_

(x)

(z)
=
1

2
_
1 1
_
_

_
=

2
P =

2
And to do the same for the y-direction,
_

(y)
+
_

(z)
=
1

2
_
1 i
_
_

_
=
i

2
P =

2
_

(y)

(z)
=
1

2
_
1 i
_
_

_
=
+ i

2
P =

2
However, if the system is initially prepared in the x-direction with an eigenvalue of /2, as is asked in
the problem, the calculations are no dierent:
_

(y)
+
_

(x)
+
=
1

2
_
1 i
_
_
1
0
_
=
1

2
P =
1
2
_

(y)

(x)
+
=
1

2
_
1 i
_
_
1
0
_
=
1

2
P =
1
2
Problem 26
a) A two-dimensional harmonic oscillator has the potential V (x, y)
1
2
m
2
(x
2
+ 4y
2
). Calculate the energies of
the rst three lowest states, and identify the degrees of degeneracy for each energy.
b) If there is an additional small coupling term W(x, y) = ax
2
y present, where a is a small constant, calculate
the rst-order correction to each of the three levels.
Write down the matrix elements that you need to calculate. If the matrix element is zero, you have to say so and
explain why. If it is not zero, say so and represent each integral by a constant and proceed further.
Solution
a) We nd a solution by employing separation of variables. The Hamiltonian for our system is
H =

2
2m

2
x
2


2
2m

2
y
2
+
m
2
2
x
2
+ 2m
2
y
2
(5.1.107)
Substituting (5.1.107) into the time-independent Schr odinger equation,
_

2
2m

2
x
2


2
2m

2
y
2
+
m
2
2
x
2
+ 2m
2
y
2
_
(x, y) = E(x, y)
CHAPTER 5: DEPARTMENTAL EXAMINATIONS 405
_

2
x
2
+

2
y
2

m
2

2
x
2

2

4m
2

2
y
2

2
_
(x, y) =
2mE

2
(x, y) (5.1.108)
At this point we introduce the separable solutions (x, y) = X(x)Y (y). Substituting this into
(5.1.108),
_

2
x
2
+

2
y
2

m
2

2
x
2

2

4m
2

2
y
2

2
_
X(x)Y (y) =
2mE

2
X(x)Y (y)
_

2
x
2

m
2

2
x
2

2
_
X(x)Y (y) +
_

2
y
2

4m
2

2
y
2

2
_
X(x)Y (y) =
2mE

2
X(x)Y (y)
_
1
X(x)

2
x
2
X(x)
m
2

2
x
2

2
_
+
_
1
Y (y)

2
y
2
Y (y)
4m
2

2
y
2

2
_
=
2mE

2
(5.1.109)
In our case the separation constant is E = E
1
+E
2
, and the separated equations become

2
x
2
X(x)
_
_
m

_
2
x
2

2mE
x

2
_
X(x) = 0 E
x
= (n
x
+
1
/
2
)

2
y
2
Y (y)
_
_
2m

_
2
x
2

2mE
y

2
_
X(x) = 0 E
y
= 2(n
y
+
1
/
2
)
And we know that E = E
x
+ E
y
, so:
E
n
= (n
x
+
1
/
2
) +(2n
y
+ 1) E
n
=
_
n
x
+ 2n
y
+
3
/
2
_
To nd the degree of degeneracy for the rst three energies, lets make a table:
n
x
n
y
E() order
0 0
3
/
2
1
1 0
5
/
2
2
0 1
7
/
2
3
1 1
9
/
2
4
2 0
7
/
2
3
0 2
11
/
2
5
2 1
11
/
2
5
2 2
15
/
2
6
To display the degeneracies for the energies, lets make another table,
E() d
3
/
2
1
5
/
2
1
7
/
2
2
b) Now the game has changed; we add a perturbation to our oscillator of the form W(x, y) = ax
2
y.
Before we can calculate the rst order energy shift, we must recall some stu about harmonic
oscillators:
x =
_

2m
(a

x
+ a
x
), y =
_

4m
(a

y
+ a
y
)
a

n
(x, y) =

n + 1
n+1
(x, y), a
n
(x, y) =

n
n1
(x, y)
With this, our perturbation becomes:
W. Erbsen QUANTUM MECHANICS
W =a
_
_

2m
(a

x
+a
x
)
_
2
_
_

4m
(a

y
+ a
y
)
_
=a

2m
_

4m
(a

x
+a
x
)
2
(a

y
+a
y
) (5.1.110)
In general, the rst order shift in energy is given by
E
(1)
n
=n
x
[n
y
[W(x, y)[n
y
)[n
x
)
=
a
2m
_

4m
n
x
[n
y
[(a

x
+ a
x
)
2
(a

y
+a
y
)[n
y
)[n
x
)
=
a
2m
_

4m
_
n
x
[n
y
[(a

x
+ a
x
)
2
a

y
[n
y
)[n
x
) +n
x
[ n
y
[(a

x
+ a
x
)
2
a
y
[n
y
)[n
x
)

=
a
2m
_

4m
__
n
y
+ 1 n
x
[n
y
[(a

x
+a
x
)
2
[n
y
+ 1)[n
x
) +

n
y
n
x
[n
y
[(a

x
+a
x
)
2
[n
y
1)[n
x
)

(5.1.111)
Problem 28
A particle is in the ground state of an innite square well. That is
V (x) =
_
0 0 x L
otherwise
(5.1.112)
At t = 0 the wall at x = L is suddenly moved to x = 2L - this happens very fast, approximately instantaneously.
a) Calculate the probability that long after t = 0 the system is in the ground state of the new potential.
b) How fast must the change take place for this instantaneous assumption to be good?
Solution
The eigenstates for the ground states before and arbitrary eigenstates after the transition are

gs
(x) =
_
2
L
sin
_
x
L
_
(5.1.113a)

n
(x) =
_
1
L
sin
_
nx
2L
_
(5.1.113b)
a) We wish to express our initial ground state wave function
gs
(x) in terms of a linear combination
of
n
(x) states:

gs
=
_
2
L
sin
_
x
L
_
=

n
c
n

n
(x) =
_
1
L

n
c
n
sin
_
nx
2L
_
For the probability that we will be in the ground state, we calculate c
n
for n = 1:
CHAPTER 5: DEPARTMENTAL EXAMINATIONS 407
c
n
=
_
L
0

gs
(x)
n
(x) dx
=

2
L
_
L
0
sin
_
x
L
_
sin
_
x
2L
_
dx
=

2
L
_
L
0
1
2
_
cos
_
x
L

x
2L
_
cos
_
x
L
+
x
2L
__
dx
=

2
L
1
2
_
L
0
_
cos
_
x
2L
_
cos
_
3x
2L
__
dx
=

2
2L
_
2L

sin
_
x
2L
_

2L
3
sin
_
3x
2L
_

L
0
=

2
2L
_
2L

sin
_

2
_

2L
3
sin
_
3
2
__
=

2
2L
_
2L

+
2L
3
_
=
4

2
3
So, the probability that we will remain in the ground state is
[c
1
[
2
=
32
9
2
b) To nd out how fast this process must take place, we use the time-energy uncertainty relation:
tE . We nd the change in energy rst
E =
n
2

2
8mL
2

n
2

2
2mL
2
=
3n
2

2
8mL
2
So, the necessary time is (approximately)
t =
8mL
2
3n
2

Problem 29
The scattering amplitude for a proton on a target atom is f(). That is, if the target is at r
T
= 0 then far
from the target the wavefunction is
(r) = e
ikr
+f()
e
ikr
r
(5.1.114)
a) Suppose that there are N(N 1) target atoms located at the point r
1
= A, r
2
= 2A, r
3
= 3A and so on.
What is (r) for positions r far from the line? That is, nd (r) for [r[ N[A[. It will have the form
(r) = e
ikr
+ F()
e
ikr
r
(5.1.115)
Find F() and d/d = [F()[
2
. Is d/d N?
W. Erbsen QUANTUM MECHANICS
b) Suppose now that the N(N 1) target atoms are located at random positions, but all inside a small nite
ball of radius R. Calculate F() and d/d. Is d/d N?
Solution
a) For some point P very far from the target, then we can make the approximation that r
1
r
2

r
3
r. We start with the template provided, (5.1.115), but add to it an additional phase term:
(r) =e
ikr
+f()
N

n=0
e
ikrinkA
r
=e
ikr
+
e
ikr
r
f()
N

n=0
e
inkA
(5.1.116)
The sum in (5.1.117) is nite; to evaluate it, we let

e
inkr
=

a
n
, where a = e
ikr
. Then
N

n=0
a
n
= 1 +a + a
2
+a
3
+ ... + a
N1
1+
N

n=0
a
n+1
= 1 +a + a
2
+a
3
+ ... + a
N1
+ a
N
1+
N

n=0
a
n+1
=
N

n=0
a
n
+ a
N
1 a
N
=
N

n=0
a
n

n=0
a
n+1
1 a
N
=
N

n=0
a
n
(1 a)
N

n=0
a
n
=
1 a
N
1 a
Using this, (5.1.117) becomes
(r) = e
ikr
+
e
ikr
r
_
1 e
iNkr
1 e
ikr
_
f() (5.1.117)
From (5.1.117), it is clear that F() is simply the term in the brackets,
F() =
1 e
iNkr
1 e
ikr
f()
And now to nd the dierential cross section,
d
d
=[F()[
2
CHAPTER 5: DEPARTMENTAL EXAMINATIONS 409
=

1 e
iNkr
1 e
ikr
f()

2
=

e
iNkr/2
e
ikr/2
_
e
iNkr/2
e
iNkr/2
e
ikr/2
e
ikr/2
_
)

2
[f()[
2
=

e
iNkr
e
ikr
sin (Nkr/2)
sin (kr/2)

2
[f()[
2
From which it is clear that
d
d
=
sin
2
(Nkr/2)
sin
2
(kr/2)
[f()[
2
No, the dierential cross section is not proportional to N .
b) For the case of gathered target atoms we start in much the same way,
(r) =e
ikr
+ f()
N

n=0
e
ikriXnk
r
=e
ikr
+
e
ikriXnk
r
f()
N

n=0
e
iXnk
(5.1.118)
Where X
n
represents the displacement of the n
th
target particle. From (5.1.118) it is clear that
F() = f()
N

n=0
e
iXnk
(5.1.119)
To nd the dierential cross section,
d
d
=[F()[
2
=

f()
N

n=0
e
iXnk

2
=[f()[
2
_
N

n=0
e
iXnk
__
N

m=0
e
iXmk
_
=[f()[
2
N

n=0
m=0
e
ik(XnXm)
=[f()[
2
_
_
N

n=m=1
e
ik(XnXm)
+
N

n=m=1
e
ik(XnXm)
_
_
=[f()[
2
_
_
N

n=m=1
e
0
+
N

n=m=1
e
ik(XnXm)
_
_
W. Erbsen ELECTRODYNAMICS
=[f()[
2
_
_
N +
N

n=m=1
e
ik(XnXm)
_
_
(5.1.120)
The remaining sum is often referred to as random walk, where the trajectories are completely random
and as an ensemble, average to zero. Thus, our dierential cross section is
d
d
= N [f()[
2
There is no escaping the fact that the dierential cross section is proportional to the number of
atoms in this particular conguration.
5.2 Electrodynamics
Problem 1
A conducting sphere of radius a is placed in a uniform electrostatic eld E
0
. If the sphere is well insulated,
nd the expressions for potential and electric elds V and E at the point P(r, , ) lying outside the sphere. Also
obtain the expression for the induced surface charge density and show that it forms a dipole distribution. Speciy
your boundary conditions clearly.
Solution
The induced surface charge density is () =
0
E; the plan of action then is to calculate E, which can
be readily done by intermediately solving for the potential, V :
V (r, ) =

=0
_
A

+
B

r
+1
_
P

(cos ) (5.2.1)
Where (5.2.1) stems from (3.63) in Griths. The problem, then, is to take (5.2.1) and adjust it so that
it describes our problem. We do this by applying boundary conditions. It should be mentioned that
as a conductor, the sphere has a constant potential, but we are only interested in the potential outside
of the sphere, so we can choose it to be whatever we want. The most convenient choice would be to
let V
sphere
= 0. Additionally, we recognize that in the eld (absent from the sphere) the potential is
V (r, ) = E
0
r cos . We can now form our boundary conditions:
V (r, ) =
_
0 at r = R
E
0
r cos at r
Applying our rst boundary condition to (5.2.1),
V (R, ) =

=0
_
A

+
B

R
+1
_
P

(cos ) = 0 A

+
B

R
+1
= 0 B

= A

R
2+1
(5.2.2)
CHAPTER 5: DEPARTMENTAL EXAMINATIONS 411
Substituting our new value for B

into (5.2.1), we have


V (r, ) =

=0
_
A

R
2+1
r
+1
_
P

(cos ) (5.2.3)
If we apply the second boundary condition to (5.2.3), we nd that the second term in parenthesis
approaches 0 as r , and we are left with
V (r, ) =

=0
_
A

_
P

(cos ) = E
0
r cos (5.2.4)
From (5.2.4) we can see that the second boundary condition is satised only if = 1 and A

= A
1
= E
0
.
With these modications, (5.2.3) is now
V (r, ) =
_
E
0
r + E
0
R
3
r
2
_
cos (5.2.5)
From the potential, we can now nd E, and in turn :
() =
0

r
V (r, )
=
0

r
__
E
0
r E
0
R
3
r
2
_
cos
_
=
0
__
E
0
2E
0
R
3
r
3
_
cos

r=R
From this it is easy to see that
() = 3E
0

0
cos
Problem 2
Two insulated square parallel conducting plates, of side L and separation d, are charged with surface densi-
ties + on the upper plate and on the lower. Two dieletric slabs, each with thickness d/2 and area L L,
are inserted between the plates, one slab above the other. The dieletric constants are K
1
and K
2
. Assume that
d L. Determine:
a) D everywhere between the plates.
b) E everywhere between the plates.
c) The bound surface charge densities
b
on the three dialetric surfaces.
d) The capacitance.
Solution
W. Erbsen ELECTRODYNAMICS
a) We rst recall that Gauss Law for for dielectrics can be written as
_
D dA = Q
Where D =
0
KE is the electric displacement. In the current case, the area is just L L = L
2
, and
the surface charge density denes the total charge as Q = L
2
, so that we have
DL
2
= L
2
D = z for
d
/
2
< z <
d
/
2
(5.2.6)
b) We again use the fact that D =
0
KE, such that Gauss Law becomes:

0
K
1,2
EL
2
= L
2
z E =

0
K
1,2
z (5.2.7)
And we now have
E =
_

0
K
1
z for 0 < z <
d
/
2

0
K
2
(z) for
d
/
2
< z < 0
c) The bound surface charge density is related to the polarization as
b
= P n, where we remember
that P =
0
E. We also recall that the dielectric constant is dened as K = (1 +). The (general)
polarization is then given by
[P
1,2
[ =
0

1,2
[E
1,2
[ =
0
(K
1,2
1) E =
0
(K
1,2
1)
_

0
K
1,2
_
=
_
1
1
K
1,2
_
(5.2.8)
The three surfaces the prompt is speaking about refers to the top surface, where slab 1 interfaces
with the positive conducting plate (and analogously for slab 2 on the lower negative conducting
plate), and the interface between the two dielectrics. The only tricky bit is in the interstitial region;
but all we have to do is add the bound charge densities being contributed from both slabs. The
result is

b
=
_

_
1
1
K
1
_
for surface 1
_
1
1
K
1
_

_
1
1
K
2
_
for surface 2
_
1
1
K
2
_
for surface 3
d) In order to calculate the capacitance, we rst recall the relation Q = C[V [. So, it seems like all we
would have to do is calculate V and we are home-free. We must integrate over both dielectrics,
going from the negative terminal at the bottom of slab 2 to the positive terminal at the top of slab
1:
V =
_
0

d
/2
E
2
dz
_ d
/2
0
E
1
dz
=
_

0
K
2
__
0

d
/2
dz
_

0
K
1
__ d
/2
0
dz
CHAPTER 5: DEPARTMENTAL EXAMINATIONS 413
=
_

0
K
2
__
0
_

d
2
__
+
_

0
K
1
___
d
2
_
0
_
=
d
2
0
K
2
+
d
2
0
K
1
=
d
2
0
_
1
K
1
+
1
K
2
_
(5.2.9)
Using (5.2.9) we can now try to nd C:
C =
Q
V
=
L
2
d
20
_
1
K1
+
1
K2
_
Which leads us to
C =
2
0
L
2
d
_
1
K
1
+
1
K
2
_
Problem 3
A hollow dieletric sphere is centered on the origin. It has dielectric constant K ,= 1. Its inner radius is a
1
and its outer radius is a
2
. It is uncharged. Now a point charge q > 0 is placed at the origin.
a) Find the electric eld strength E for r < a
1
, a
1
< r < a
2
, and a
2
< r.
b) Find the surface charge density (charge/area) on the inside (at r = a
1
) and on the outside (at r = a
2
).
c) Now the situation is changed. A thin conducting coating is applied to the outside of the sphere, and this
surface is maintained at +V
0
volts relative to innity. Find E(r) for r > a
2
.
Solution
W. Erbsen ELECTRODYNAMICS
a) The electric eld in all three regions can be found from Gauss Law as
E =
_

_
1
4
0
q
r
2
for 0 < r < a
1
1
4
0
K
1
q
r
2
for a
1
r a
2
1
4
0
q
r
2
for r > a
2
b) To nd the bound surface charge density we must use the following relation
b
= P n, where
P =
0
E. The general solution can be found by,

b
=
0

1,2
E
1,2
n =
0
(K
1,2
1)
1
4
0
K
1,2
q
r
2
r n = (K
1,2
1)
1
4K
1,2
q
r
2
r n (5.2.10)
Our solution from (5.2.10) is applicable as long as we recognize that the induced charges on the
inner surface are negative, while those on the outer surface are positive. Therefore, for the inner
surface we have (r) n = 1, and for the outer r n = 1. With this, our solutions become

b
=
_

_
_
1
K
1
1
_
1
4
q
a
2
1
at r = a
1
_
1
1
K
2
_
1
4
q
a
2
2
at r = a
2
c) Since we are only interested in the eld outside the sphere, the particular conguration of all the
interior charges is irrelevant; all that is important is the total charge. We know that there is one
point charge at the center of the sphere, q, and that the conducting shell is held at V
0
. What we
dont know is how much charge exists on the exterior of the conductor; we can nd this out as
follows:
V (r = a
2
) = V
0
=
1
4
0
(Q+q)
a
2
Q = 4
0
a
2
V
0
q (5.2.11)
And now using (5.2.11) we can nd the potential at a greater distance r > a
2
:
V (r > a
2
) =
1
4
0
(Q+ q)
r
=
1
4
0
(4
0
a
2
V
0
q +q)
r
V (r) =
V
0
a
2
r
(5.2.12)
Recalling that E = V , from (5.2.12) we have
E(r) =
1
r

r
V
0
a
2
r
r =
1
r
_

V
0
a
2
r
2
_
r
From which it is easy to see that the eld is
E(r) =
V
0
a
2
r
3
r
Problem 4
CHAPTER 5: DEPARTMENTAL EXAMINATIONS 415
A linear molecule with a permanent electric dipole moment p
0
and moment of inertia I (for example, HCL)
is placed in a uniform electric eld E.
a) Make a sketch showing the charges that make up the dipole, the dipole moment and the electric eld when
the dipole is in the equilibrium orientation.
b) Derive an expression for the frequency
0
of small amplitude oscillations about the equilibrium orientation.
c) Describe the polarization and power of the radiation produced by this oscillating dipole, assuming it has been
initially displaced by an angle 90 degrees from its equilibrium orientation. Make a sketch of the angular
distribution of this power.
d) Over what time scale will it continue radiating appreciably, and therefore what range of frequencies are
emitted?
Solution
a) Picture picture picture
b) To nd an expression for
0
, we begin with Newtons Second Law applied to rotational kinematics:
= I = I

. We note that the torque is also dened by = r F, where r in our case is the
distance between the two charges in the dipole, lets call this distance d. With this, we have
= r F = d F =
q
q
d F = qd
F
q
= = P
0
E (5.2.13)
Where in (5.2.13) I have used the fact that P
0
= qd and also that F = qE. The magnitude of is
of course [[ = P
0
E sin . Using this with Newtons Second Law, we have
I

= P
0
E sin

=
P
0
E
I
sin
P
0
E
I
=

=
2
(5.2.14)
Where I have used the small angle approximation sin . We know that (5.2.14) is the dierential
equation for the harmonic oscillator, where we typically dene as:

0
=
_
P
0
E
I
(5.2.15)
c) Equation (11.22) from Griths is:
P) =

0
P
2
0

4
12c
(5.2.16)
Which represents the total time averaged power radiated from an oscillating dipole. If we imagine
our molecule rotating periodically (pendulum-like), it has a combined energy equal to
E =
1
2
mv
2
+
1
2
I
2
(5.2.17)
And also sources of potential energy, which we neglect. When the molecule is perfectly aligned with
the eld, that is, = 0, then (5.2.17) reduces to E =
1
/
2
I
2
. We also note that power is dened as
the amount of work per unit time, and therefore from (5.2.16) we can say
W. Erbsen ELECTRODYNAMICS
P) =

0
P
2
0
12c
. .
C

4
= C
4
=
ENERGY
TIME
=
dE
dt
Where the time derivative of the rotational kinetic energy is simply I . Recalling that C

= C/I,
we have
I = C
4

d
dt
= C

3
d = C

dt
Whose solution is
t =
1
2C
2
Problem 5
A current I ows into a parallel plate capacitor with circular plates of radius R separated by a distance d.
The current was 0 before t = 0 and I ,= 0 after.
a) What is the charge on the plates as a function of time?
b) What is the electric eld between the plates?
c) What is the displacement current density between the plates?
d) What is the magnetic eld between the plates at r = R/2 from the center of the plates?
e) What is the Poynting vector between the plates at R/2?
Solution
a) We note that the fundamental denition for current is
I =
dq(t)
dt
We can solve this dierential equation, noting that I(0) = 0:
_
I dt = q(t) It +A = q(t) q(t) = It (5.2.18)
b) To nd the electric eld between the plates, we note that our plates have a (time dependent) surface
charge density (t). Applying Gauss Law,
_
A
E dA =
q
enc

0
(z)
EA =
(t)A

0
CHAPTER 5: DEPARTMENTAL EXAMINATIONS 417
E =
1

0
q(t)
R
2
(z) (5.2.19)
From (5.2.19) and using (5.2.18), the electric eld is given by
E =
It

0
R
2
(z) (5.2.20)
c) We know that the displacement current density is denoted by J
d
and is given by
J
d
=
0
E
t
+
P
t
(5.2.21)
The displacement current density is obtained simply by taking the time derivative of the electric
displacement eld D =
0
E+P. In our case, we have no dielectric media so the last term in (5.2.21)
is neglected. Using (5.2.20), J
d
becomes
J
d
=

t
_
It

0
R
2
(z)
_
Which is simple enough to evaluate. The result is:
J
d
=
I

0
R
2
(z) (5.2.22)
d) The magnetic eld at a position r = R/2 between the plates can be computed using Amperes Law:
_
B d =
0
I
enc
In we are allowed to switch I
enc
with I
d
since the magnetic eld within the plates is the same
as between the plates. We also note that the displacement current density J
d
is related to the
displacement current I
d
by J
d
= I
d
/R
2
(in the following case R is actually r). So, with (5.2.22),
(d) becomes:
B2r =
0
J
d
r
2
B2r =
0
_
I
R
2
(z)
_
r
2
B =

0
Ir
2R
2
z (5.2.23)
If we recall that r = R/2, it is easy to see that (5.2.23) can be written as:
B =

0
I
4R

(5.2.24)
e) The fundamental formulation for the Poyinting Vector is
S =
1

0
EB (5.2.25)
Inserting our previously tabulated values for E from (5.2.20) and B from (5.2.24), (5.2.25) becomes:
S =
1

0
_

It

0
R
2
z
_

0
I
4R

_
W. Erbsen ELECTRODYNAMICS
Which can be rewritten as
S =
I
2
t
4
0

2
R
3

Problem 6
A sphere of radius R carries charge Q distributed uniformly over its surface. The sphere is rotated at a con-
stant angular velocity around the z-axis. Calculate the magnetic dipole moment. Find the magnetic eld at the
point (x, 0, 0) where x R.
Solution
The integral form of the magnetic dipole moment is
=
_
I dA =
_
A dI (5.2.26)
We also recall that the denition of current is I = dQ/dt, but if we want the dierential element of the
amount of current over some specied length of time (the period, T), this becomes dI = dQ/T. The
dierential charge element in our case since we have a surface charge density is dQ = dA.
So, theoretically we just insert our values into (5.2.26) and integrate. we begin by looking for the
charge element dQ for a thin strip of the sphere, rotating around at some radius r = a sin with some
angular velocity . This rotating charge, of course, produces current, and in turn a magnetic dipole
moment. The thickness of the stripe is a d, and so we have:
dQ = dA =
_
Q
4a
2
_
(a d) (2a sin ) =
1
2
Qsin d (5.2.27)
Using (5.2.27) and remembering that the temporal period at some rotational frequency is T = 2/,
our dierential current element becomes
dI =
dQ
2/
=

2
1
2
Qsin d =
Q
4
sin d (5.2.28)
The loop area is A = r
2
= (a sin )
2
, and the magnetic dipole moment becomes
=
_
AdI =
_

0
_
(a sin)
2
z
_
_
Q
4
sin d
_
sin =
4Qa
2
4
_

0
sin
3
d z (5.2.29)
Straightforward integration yields
=
4Qa
2
4
_

0
_
1 cos
2

_
sin d z
=
4Qa
2
4
__

0
sin d
_

0
cos
2
sin d
_
z
CHAPTER 5: DEPARTMENTAL EXAMINATIONS 419
=
4Qa
2
4
_
[cos [

0
+
1
3
_
cos
3

0
_
z
=
4Qa
2
4
_
(1 1) +
1
3
(1 1)
_
z
=
4Qa
2
4
_
4
3
_
z
From which we conclude that
=
Qa
2
3
z (5.2.30)
To calculate the eld B, we recount the following equation, the bastard love-child of the Biot-Sevart law:
B
dip
(r) =

0
4
1
r
3
[3 ( r) r ] (5.2.31)
In our case we use r = x, and with (5.2.30), (5.2.31) becomes
B(x) =

0
4
1
x
3
Qa
2
3
[3 (z x) x z]
Where we note that z x = 0, so we nally have
B(x) =

0
qa
2
12x
3
z
Problem 8
Two long coaxial cylindrical metal tubes (inner radius a, outer radius b) stand vertically in a dielectric oil (dielec-
tric constant , density ). The inner one is maintained at potential V and the outer one is grounded. To what
height does the oil rise in the space between the tubes? (Hints: Find the electric eld, then the electrical energy
and then the force).
Solution
To nd the maximum height that the oil reaches, we must think about what forces are acting on the
uid. The force that is lifting the oil upwards is due to the capacitance between the two cylinders. The
motion of the uid stops when this force is exactly cancelled by the opposing force due to gravity. We
can, then start from the end and work towards the beginning. Lets say the uid is currently at its
maximum height, h. Then the mass density can be used as follows:
=
MASS
VOLUME
=
m
(b
2
a
2
) h
m =
_
b
2
a
2
_
h
The force acting downward on the center of mass is then
W. Erbsen ELECTRODYNAMICS
F
g
=
g
_
b
2
a
2
_
h
2
z (5.2.32)
Where the factor of 2 in the denominator stems from the fact that we are interested in the average
height. We also know that (5.2.32) must be equal to the force pulling up on the uid. To nd out what
this opposing (lifting) force is, recall that
F =
1
2
V
2
dC
dz
z (5.2.33)
At this point we take a break to calculate what the capacitance of our system is. In general,
_
A
E dA =
Q
enc

0
E2rL =
Q

0
E =
1
2
0
Q
rL
r (5.2.34)
And from (5.2.34) we now calculate the potential:
V =
_
a
b
E rdr =
1
2
0
Q
L
_
a
b
1
r
r rdr =
1
2
0
Q
L
log
_
b
/
a
_
(5.2.35)
Again using the handy relation Q = CV , (5.2.35) becomes
C =
Q
V
=
2
0
L
log (
b
/
a
)
(5.2.36)
What we have just found in (5.2.36) is the general relation for the capacitance of a cylindrical capacitor. In
our case, part of the capacitor is lled with the dielectric, and the other with oil, so the total capacitance
would just be the sum of the two:
C
air
=
2
0
(L z)
log (
b
/
a
)
C
oil
=
2
0
zK
1
log (
b
/
a
)
=
2
0
z (1 + )
log (
b
/
a
)
So that the total capacitance is
C = C
air
+C
oil
=
2
0
(L z)
log (
b
/
a
)
+
2
0
z (1 + )
log (
b
/
a
)
=
2
0
log (
b
/
a
)
(L +z) (5.2.37)
At this point we substitute (5.2.37) into (5.2.33):
F =
1
2
V
2
d
dz
_

2
0
log (
b
/
a
)
(L +z) z

z=h
=
V
2

0
log (
b
/
a
)
z (5.2.38)
And now we set (5.2.32) equal to (5.2.38):
g
_
b
2
a
2
_
h
2
z =
V
2

0
log (
b
/
a
)
z
If we solve for the height h we nally arrive at
h =
2V
2
(
r
1)
0
g (b
2
a
2
) log (
b
/
a
)
CHAPTER 5: DEPARTMENTAL EXAMINATIONS 421
The factor of 1/2 comes from the fact that h represents the average height of the oil, which is half-way
up the vertical height.
Problem 9
A cylindrical capacitor consists of two long, concentric tubes of sheet metal of radii R
1
and R
2
, respectively.
The space between the tubes is lled with a dielectric constant .
a) Find the capacitance of this capacitor.
b) Suppose the potential dierence between the shells is V
0
, nd the electrostatic energy.
Solution
a) The typical equation relating to capacitance is Q = CV C = Q/V . We can rmly say that if
the total height of the capacitor is L, and if the plates have a line charge density , then the total
charge is Q = L. We now calculate the potential,
V =
_
R1
R2
E dr (5.2.39)
We now nd E as follows
E =
1
4
0

Q
r
=
L
4
0

1
r
(5.2.40)
Using (5.2.40), we can now calculate V from (5.2.39):
V =
L
4
0

_
R1
R2
1
r
dr =
L
4
0

(log [R
2
] log [R
1
]) =
L
4
0

log
_
R
2
R
1
_
=
L
4
0

log
_
R
1
R
2
_
(5.2.41)
From (5.2.41) we can nally say that
C =
L
V
=
2
0
L
log
_
R2
R1
_ (5.2.42)
b) We recall that the equation for the electrostatic energy is U =
1
/
2
CV
2
. From (5.2.41) and (5.2.42),
U =
1
2
_
_
4
0

log
_
R2
R1
_
_
_
_
L
4
0

log
_
R
2
R
1
__
2
=
2
0

log
_
R2
R1
_

2
L
2
16
2

2
0

2
log
_
R
2
R
1
_
2
From which it is easy to arrive at
W. Erbsen ELECTRODYNAMICS
U =

2
L
2
4
0

log
_
R
2
R
1
_
Problem 11
A long wire is bent into a hairpin-like shape shown in the gure. Find an exact expression for the magnetic
eld at the point P which lies at the center of the half circle.
Solution
It is easy to show from Amperes Law that the magnetic elds produced by an innite straight wire and
an arc are, respectively:
B =

0
I
2r
, and B =

0
I
4r
The two lengths of wire contribute each half of the magnetic eld produced by an innite straight wire,
while the arc extends through an angle = , so:
B =

0
I
2R
+

0
I
4R
B =

0
I
4R
_
2

+ 1
_
Problem 12
A copper wire with a diameter of 2 mm has a resistance of 1.6 for every 300 meters of length. It is carry-
ing a current of 25 amps. Assuming that this DC current density is uniformly distributed over a cross section,
a) What is the numerical value of the resistivity of the copper from the information given?
b) Determine the electric eld vector E, the magnetic induction vector B, and the Poynting vector S at the
surface of the wire (be sure to state their directions and actual numerical values).
0
/4 = 10
7
, 1/4
0
=
9 10
9
in MKS units.
c) How much energy (in Joules/meter) is stored in the electric eld and in the magnetic eld within the wire?
d) Suppose the current is increasing at a rate of 1 Amp/s. What statement can be made about the radial
dependence of the induced electric eld that results within the wire?
Solution
CHAPTER 5: DEPARTMENTAL EXAMINATIONS 423
a) The fundamental relation relating resistance and resistivity is:
R =
L
A
=
RA
L
(5.2.43)
In our case, A = r
2
=
_
10
3
m
_
2
, so that the resistivity is given by
=
(1.6)((10
3
m)
2
)
300m
= 1.67 10
8
m (5.2.44)
b) To nd the electric eld vector E, we must employ Gauss Law:
_
E d =
1

L
E 2r =
1

L
E =
1
2
0

r
(5.2.45)
We notice that the equaled part in (5.2.45) looks suspiciously similar to the denition of the scalar
potential V . And we can go ahead and nd the scalar potential using the given information:
V = IR = (25A)(1.6) = 40v. Using this information, and manipulating (5.2.45), we have
E =
_
1
4
0
L
r
_
2
L
=
2V
L
(5.2.46)
Substituting the given values, (5.2.46) becomes:
E =
2
15
v
m
z (5.2.47)
The magnetic eld may similarly be found using Amperes Law:
_
B d =
0
I
enc
B =

0
I
2r
=
2 10
7
25A
10
3
m
Evaluating this, we arrive at
B = 5 10
3
T

(5.2.48)
Using (5.2.47) and (5.2.48), we can calculate the Poynting Vector S:
S =
1

0
E B =
1

0
_
4
15
v
m
z
_

_
3 10
3
T

_
S = 1061
W
m
2
(5.2.49)
c) Finding the total energy stored in the both the electric and magnetic elds entails recalling the
following relations:

E
=
energy
volume
=
1
2

0
[E[
2
, and
B
=
energy
volume
=
1
2
1

0
[B[
2
Lets say that we are interested in the amount of energy in a 1m length of wire. Then the volume
is V = r
2
= (10
3
m)
2
(1 m) = 3.14 10
6
m
3
. Accordingly, the amount of energy stored in the
electric and magnetic elds is given by
1
2
_
8.887 10
12
A s
V m
_

2
15
v
m

2
_
3.14 10
6
m
3
_
Energy in E = 2.473 10
19
J
m
1
2
_
1
410
7
A m
V s
_

5 10
3
T

2 _
3.14 10
6
m
3
_
Energy in B = 1.562 10
5
J
m
W. Erbsen ELECTRODYNAMICS
d) Qualitatively, when an time-varying current is present in a conductor (eg, AC current), the current
has a greater tendency to exist near the surface of the conductor (or on the skin). This is called
the skin eect.
Problem 13
A battery with emf V , a resistor with resistance R, and a capacitor with initial capacitance C have been connected
in series as shown for a very long time. The dielectric, with permittivity , occupies
1
/
2
the gap in the parallel
plate capacitor. Now suppose that the dielectric is removed very quickly, in a time short compared to any relevant
time constants. The new capacitance is C

.
a) Give an explanation of what happens to each circuit element. After a very long time, a new equilibrium is
reached. In terms of C, C

, R and V , give expressions for:


i) The net change in energy in the capacitor
ii) The net change in energy in the battery
iii) The total energy dissipated in the resistor
iv) The amount of work that was done to remove the dielectric
b) In terms of , what is the ratio C/C

?
Solution
a) We rst recall that the capacitance for a parallel plate capacitor is given by C = A/d, where A is
the surface area of the plates and d is the distance between them, while corresponds to whatever
material lies between. To nd the capacitance of the capacitor before the dielectric is removed, we
recognize that since the dielectric does not occupy the entire interstitial space between the plates
we essentially have two capacitors in series, C
air
and C
dia
. Mathematically, these are (respectively)
C
air
=

0
A
d
/
2
=
2
0
A
d
, and C
dia
=
A
d
/
2
=
2A
d
The total capacitance is then just
CHAPTER 5: DEPARTMENTAL EXAMINATIONS 425
C =
1
C
air
+
1
C
dia
=
C
air
C
dia
C
air
+ C
dia
=
(2
0
A/d)(2A/d)
2
0
A/d + 2A/d
=
2
0
A
d( +
0
)
(5.2.50)
While after the dielectric is removed, the capacitance is simply
C

=

0
A
d
(5.2.51)
i) The amount of energy in the capacitor decreases. Adding dielectrics between the plates of
a capacitor act to decrease the magnitude of the electric eld, but increase the capacitance
(and therefore the energy stored) through the polarization of the static dielectric atoms. To
calculate this change, we just have to remember that the amount of energy in a capacitor is
just U =
1
/
2
CV
2
. So,
U
c
=
1
/
2
C

V
2

1
/
2
CV
2
U
c
=
1
/
2
V
2
(C

C) (5.2.52)
ii) When the dielectric is removed, there is a drop in capacitance and charge ows from the plates
of the capacitor towards the battery. The change in energy of the circuit occurs due to thermal
losses in the resistor; the potential energy of a charge in a potential is U = QV , so the change
in energy is then
U
b
= Q

V QV = C

V
2
CV
2
U
b
= (C

C) V
2
(5.2.53)
iii) As mentioned previously, the total amount of energy dissipated in the circuit is done through
the resistor. The basic equation to calculate this is P = I
2
R, however all we have to do is take
the dierence of (5.2.52) and (5.2.53):
U
r
= U
c
U
b
=
1
/
2
V
2
(C

C) (C

C) V
2
U
r
=
1
/
2
V
2
(C

+C) (5.2.54)
The ratio of the capacitances before and after the dielectric is removed is
C
C

=
2
0
A
d( +
0
)

d

0
A

C
C

=
2
+
0
Problem 14
A series LCR circuit with L= 2 H, C=2 F, and R=20 is driven by a generator with maximum emf of 100 V
and variable frequency. Find the resonant frequency
0
and the phase and maximum current I
max
when the
generator angular frequency is = 400 s
1
.
Solution
We rst recall that for a resonant RLC circuit, we have Z = R, = 1/

LC , X
C
= X
L
, and = 0. This
is not the case, however; the rst thing we want to do is nd the resonant frequency
0
of the circuit:

0
=
1

LC
=
1
_
(2 H)(2 10
6
F)

0
= 500
rad
sec
(5.2.55)
W. Erbsen ELECTRODYNAMICS
In order to nd the phase angle, we must rst recall that X
C
= 1/C, X
L
= L, and tan() =
(X
L
X
C
) /R. Using these, we have
= tan
1
_
L
1
C
R
_
= tan
1
_
(400 s
1
)(2 H)
1
(400 s
1
)(210
6
F)
20
_
= 87.46
o
The maximum current occurs at resonance, where R = Z =
_
R
2
+ (X
L
X
C
)
2
. Using Ohms Law
with this condition, and also the result from (5.2.55),
I
max
=
V
Z
=
V
_
R
2
+
_
L
1
C
_
2
=
100 V
_
(20 )
2
+
_
(400 s
1
) (2 H)
1
(400s
1
)(210
6
F)
_
2
I
max
= 0.222 A
Problem 15
A grounded conducting plane is connected to a conducting sphere with radius a through a battery of voltage
V
0
. The sphere is a distance d a above the plane.
a) Find the charge on the sphere.
b) Find the force between the plane and the sphere.
Solution
a) Since we are told that d a, it is fair to go ahead and assume that the sphere can be thought of
as isolated; and since it is connected to a battery of potential V
0
, we could go ahead and assume
that this is the potential of the sphere. The standard capacitance of a conducting sphere is
C = 4
0
r (5.2.56)
CHAPTER 5: DEPARTMENTAL EXAMINATIONS 427
Where in our case r = a. Furthermore, we know that Q = CV , so the total charge on our sphere
is just
Q = 4
0
aV
0
(5.2.57)
b) The force between the sphere and the grounded conducting plane can be calculated via the method
of images, where we imagine an identical, albeit oppositely charged sphere a distance 2d away. Using
(5.2.57) and Coulombs law, we have
F =
1
4
0
Q
2
(2d)
2
=
1
4
0
(4
0
aV
0
)
2
4d
2
Which can be readily simplied to
F =

0
a
2
V
2
0
d
2
Problem 17
A conducting sphere of radius a and total charge Q is surrounded by a spherical shell of dielectric material
(with permittivity ) of inner radius a and outer radius b. Find the electrostatic energy of the system.
Solution
The general form for the energy stored in any electrostatic system is
W =

2
_
E
2
d (5.2.58)
In our case, the electric eld of the conducting sphere is given by
E =
1
4
Q
r
2
r (5.2.59)
Inserting this value for E into (5.2.58), we have
W =

2
___ _
1
4
Q
r
2
r
_
2
dV =

2
1
16
2

2
Q
2
_
2
0
_

0
_

0
1
r
4
r
2
sin drdd (5.2.60)
The radial integral in (5.2.60) must be divided according to each section: from 0 a we are inside the
conductor, so E = 0. However from a b and b we must evaluate the integral, and (5.2.60)
becomes:
W =
Q
2
32
2

2
0
_
2
0
d
_

0
sin d
_

2
0

2
_
b
a
1
r
2
dr +
_

b
1
r
2
dr
_
W. Erbsen ELECTRODYNAMICS
=
Q
2
32
2

2
0
(2)(2)
_

2
0

2
_

1
r

b
a
+
_

1
r

b
_
=
Q
2
8
2
0
_

2
0

2
_

1
b
+
1
a
_
+
1
b
_
From which is may be readily deduced that
W =
Q
2
8
2
_
1
a

1
b
_
+
Q
2
8
2
0
1
b
Problem 18
The small loop of wire (radius a and resistance R) falls under gravity towards the larger loop (radius A), which has
a constant current I. The small loop is contrained to move along the axis of the large loop and remains parallel
to the large loop.
a) Explain (in words) what happens to the small loop.
b) For a particular height h and velocity v, what is the induced emf in the small loop? Assume a A.
c) What is the electromagnetic force acting on the small loop?
d) Does the system radiate? If so, describe the radiation pattern qualitatively (e.g., frequency, direction, polar-
ization).
Solution
a) Words words words
CHAPTER 5: DEPARTMENTAL EXAMINATIONS 429
b) The emf induced on the smaller loop will be due to the change of ux passing through it, changing
as it falls:
E =
d
B
dt
=
d
dt
_
B dA (5.2.61)
If we assume that the eld is homogeneous throughout the smaller loop, we can expand on (5.2.61)
as follows
E =
d
dt
B a
2
(5.2.62)
At this point all we have to do is nd B. The best way to do this is with the Biot-Sevart Law, but
before we do that we should dene some needed quantities. d, the length element of the source
loop (the bigger one) is A d. Furthermore, the radial vector r is found by subtracting by boring
trig stu: r = z

k A . The magnitude of r is just

z
2
+A
2
. Using all this in the Biot-Sevart
Law:
B =

0
I
4
d r
[r[
3
=

0
I
4
A d

(z

k A )
(z
2
+ A
2
)
3
/2
=

0
IA
4
z + Az
(z
2
+A
2
)
3
/2
d
=

0
IA
4
1
(z
2
+A
2
)
3
/2
_
z
_
2
0
d + A
_
2
0
z d
_
=

0
I
4
A
(z
2
+A
2
)
3
/2
[0 + 2A] z]
=

0
I
2
A
2
(z
2
+A
2
)
3
/2
z (5.2.63)
We can now use (5.2.63) in (5.2.62) to nd E:
E = a
2
d
dt
_

0
I
2
A
2
(z
2
+A
2
)
3
/2
_
=

0
Ia
2
A
2
2
d
dt
dt
dz
dz
dt
_
1
(z
2
+A
2
)
3
/2
_
=

0
Ia
2
A
2
2
z
d
dz
_
1
(z
2
+A
2
)
3
/2
_
=

0
Ia
2
A
2
2
z
d
dz
_

3z
(z
2
+ A
2
)
5
/2

z=h
(5.2.64)
By evaluating (5.2.64) at z = h, and recognizing that z is the downward velocity, we have
E =
3h
0
Ia
2
A
2
2(h
2
+A
2
)
5
/2
v (5.2.65)
W. Erbsen ELECTRODYNAMICS
c) Not too sure on this one
d) Yes , the system does radiate, in the direction.
Problem 19
A cylindrically shaped conductor has length L, radius a, and conductivity . It carries a uniform time-independent
current density J = Je
0
, parallel to its long axis.
a) Determine the electric and magnetic elds within the conductor.
b) Calculate the Poynting vector S within the conductor.
c) Write out Poyntings theorem for this situation. Verify that it is satised for any point within the conductor.
Solution
a) We rst recall the fundamental relation for current density: J = E. So, the electric eld is given
simply by
E =
_
J/ z for 0 a
0 elsewhere
(5.2.66)
And to nd the internal magnetic eld B, we apply Amperes Law:
_
B d =
0
I
inc
=
0
_

2
J
_
2B =
2
J
Such that we have
B =
_

0
J/2

for 0 a
0 elsewhere
(5.2.67)
b) We note that the Poynting vector takes the form
S =
1

0
EB
And since we know that E = J/ from (5.2.66) and B =
0
J/2 from (5.2.67), then the
Poynting vector is simply
S =
J
2

2
( ) (5.2.68)
CHAPTER 5: DEPARTMENTAL EXAMINATIONS 431
c) Poyntings theorem states that
u
t
+ S = J E (5.2.69)
Where u is the energy density. We can express the energy density as:
u =
1
2
_

0
E
2
+
B
2

0
_
(5.2.70)
Substituting (5.2.66) and (5.2.67) into (5.2.70),
u =
1
2
_

0
J
2

2
+
0

2
0
J
2

2
4
_
=
1
2
_

0
J
2

2
+

0
J
2

2
4
_
(5.2.71)
And we note that u is time independent, so u/t = 0. We also note that the divergence in
cylindrical coordinates takes the form
F =
1

(F

) +
1

+
F
z
z
So nally we can calculate
S =
1

(S

) +
1

+
S
z
z
=
1

(S

)
=
1

J
2

2
( )
_
=
1

J
2

( )
=
J
2

(5.2.72)
So, using (5.2.72), Poyntings theorem (5.2.69) is now

J
2

= J E = J
J

=
J
2


Problem 21
An electron beam of uniform charge density
0
inside the beam radius r
0
is traveling on the axis of a metal
tube with an inner radius R
i
and an outer radius of R
o
which is kept at a potential

.
a) Calculate the potential at the center of the electron beam.
b) Calculate the net charge on the tube per unit length .
W. Erbsen ELECTRODYNAMICS
c) Calculate the energy that is required to double the tube potential to 2

assuming that the electron beam


charge density remains constant.
Solution
a) Calculating the potential at the center of the beam poses no major obstacles. We imagine to be
dragging a unit charge (test charge) from to the center of the beam . The total potential is
how much work it took. So we start from the outside, from , and boundary by boundary calculate
the potentials and at the end add them together.
(r = 0) =
_
0

E ds
=
_
Ro

E
4
ds
. .

_
Ri
Ro
E
3
ds
_
Ra
Ri
E
2
ds
_
0
a
E
1
ds (5.2.73)
We are told that the potential of the shell is kept at

, so we neednt even calculate E


4
. The rest
are
E
1
=

0
(r
2
)
2
0
r =

0
2
0
r r (5.2.74a)
E
2
=

0
(r
2
0
)
2
0
r
r =

0
r
2
0
2
0
r
r (5.2.74b)
E
3
=0 (5.2.74c)
The reason why E
3
= 0 is because this region corresponds to within a conductor, where the internal
electric eld is guaranteed to be zero. Using (5.2.74a), (5.2.74b) and (5.2.74c), we can evaluate our
potential from (5.2.73):
(0) =


_
Ra
Ri

0
r
2
0
2
0
r
r dr
_
0
a

0
2
0
r r dr
=



0
r
2
0
2
0
_
Ro
Ri
1
r
dr

0
2
0
_
0
a
r dr
=



0
r
2
0
2
0
[logr[
Ro
Ri


0
2
0
_
r
2
2

0
a
=



0
r
2
0
2
0
log
_
R
i
R
o
_
+

0
2
0
a
2
2
(5.2.75)
From (5.2.75) it is easy to see that the potential is
(0) =

+

0
r
2
0
2
0
log
_
R
i
R
o
_
+

0
a
2
4
0
b) The net charge on the tube per unit length would just be the charge density times the area in
question where we take the length to be equal to 1,
=
0
a
2
CHAPTER 5: DEPARTMENTAL EXAMINATIONS 433
Problem 22
A charge 2q is located at (0, 0, 0), a second charge, q, is located at (0, a, 0), and a third charge, q, is lo-
cated at (0, 0, a).
a) Calculate the monopole and dipole moments.
b) Calculate all quadrupole moments.
c) What is the potential for r a using multipole approximation?
Solution
a) The monopole moment is zero, since it is just the sum of the charges. The dipole moment for an
array of charges is dened by
p =
N

i=1
r
i
q
i
(5.2.76)
Applied to our geometric arrangement of charges,
p = (a

j)(q) + (0)(2q) + (a

k)(q) p = aq(

j +

k) (5.2.77)
b) The dening equation for the quadrupole moment is
Q
0
=
1
2

i=1
q
i
_
3x
i
x
i
r
2
i

_
(5.2.78)
Applying (5.2.78) to our case,
Q
0
=
1
2
(a
2
)(q) +
1
2
(0)(2q)
1
2
(a
2
)(q) Q
0
= a
2
q (5.2.79)
c) The full equation for multipole expansion is
V (r) =
1
4
0

n=0
1
r
(n+1)
_
(r

)
n
P
n
(cos

)(r

) d

=
1
4
0
_
1
r
_
(r

) d

+
1
r
2
_
r

cos

(r

) d

+
1
r
3
_
(r

)
2
_
3
2
cos
2

1
2
_
(r

) d

+...
_
(5.2.80)
The integrals represent the various -pole terms through quadrupole. We know that the monopole
term is zero, so we simply substitute (5.2.77) and (5.2.79) into (5.2.80):
V (r)
1
4
0
_
aq(

j +

k)
r
2
+
a
2
q
r
2
_
W. Erbsen ELECTRODYNAMICS
Problem 23
Consider a nonconducting sphere of radius R with a uniform charge density , except for a cavity of radius
a, a distance b > a from the center.
a) Find the electric eld at the center of the cavity.
b) Find the voltage potential (V () = 0) at the center of the cavity.
Solution
a) We are given that the dielectric object has a charge density equal to ; and since the geometry of
the problem is simple enough, from this we can readily calculate the total charge Q:
=
CHARGE
VOLUME
=
Q
4
/
3
R
3

4
/
3
a
3
=
Q
4
/
3
(R
3
a
3
)
Q =
4
/
3

_
R
3
a
3
_
(5.2.81)
And now we can apply Gauss Law to nd the eld E:
E4b
2
=
Q
enc

0
E =
1
4b
2

4
3
b
3
Which allows us to say
E =
b
3
0
r
In order to nd the voltage at the center of the cavity, we must imagine that we are dragging a
unit charge to the center of the cavity from innity. We do this in steps, addressing each boundary
individually:
V (r) =
_
a

E ds =
_
R

E
1
ds
_
a+b
R
E
2
ds
_
a
a+b
E
3
ds (5.2.82)
The respective electric elds are
E
1
=
(r
3
a
3
)
3
0
r
2
r =
(R
3
a
3
)
3
0
r
2
r (5.2.83a)
CHAPTER 5: DEPARTMENTAL EXAMINATIONS 435
E
2
=
(r
3
a
3
)
3
0
r
2
r (5.2.83b)
E
3
=
b
3
0
r (5.2.83c)
We now substitute (5.2.83a)-(5.2.83c) into (5.2.82) and integrate. I will do each of the three integrals
in turn and combine them later. The rst is
V
1
(r) =
(R
3
a
3
)
3
0
_
R

1
r
2
dr
=
_
(R
3
a
3
)
3
0
r

=
(R
3
a
3
)
3
0
R
(5.2.84)
And now for the second,
V
2
(r) =

3
0
_
a+b
R
r
3
a
3
r
2
dr
=

3
0
_
_
a+b
R
rdr a
3
_
a+b
R
1
r
2
dr
_
=

3
0
_
_
r
2
2

a+b
R
a
3
_

1
r

a+b
R
_
=

3
0
_
(a +b)
2
2

R
2
2
+
1
a +b

1
R
_
(5.2.85)
And that last little bitch is
V
3
(r) =
b
3
0
_
a
a+b
dr
=
b
3
0
[a (a +b)]
=
b
2
3
0
(5.2.86)
We now take (5.2.84) - (5.2.86) and substitute it into (5.2.82):
V (r) =
(R
3
a
3
)
3
0
R
+

3
0
_
(a + b)
2
2

R
2
2
+
1
a + b

1
R
_
+
b
2
3
0
Which nally leads us to
V (r = 0) =

3
0
_
(R
3
a
3
)
R
+
(a +b)
2
2

R
2
2
+
1
a +b

1
R
+b
2
_
Problem 24
W. Erbsen ELECTRODYNAMICS
A particle of charge +q and mass m has a velocity of V = (0, v
y
, v
z
) when at a position of (x, 0, 0). There is
a magnetic eld of B = (0, 0, B).
a) Find an expression for the cyclotron period, i.e. the time to go once around a circular orbit.
b) Find the position of the charge after one cyclotron period.
Solution
a) The cyclotron period can be found by:
F
C
= F
m
m
2
r = qvB =
qB
m
So the period is
=
2

=
2m
qB
b) Most generally, we would approach the problem like this
F = mr = qv B
Which leads us to
( x x + y y + zz) =
qB
m
[ x( y) + y x] = [ y x x y]
This equation may be separated very easily; the resulting coupled dierential equations are
x = y (5.2.87a)
y = x (5.2.87b)
z =0 (5.2.87c)
We begin by integrating (5.2.87a) with respect to t:
CHAPTER 5: DEPARTMENTAL EXAMINATIONS 437
x = y + C(v
x
) (5.2.88)
Substituting (5.2.88) into (5.2.87b),
y =
2
y C(v
x
) (5.2.89)
Our initial conditions state that at t = 0 then v
x
= 0, so C(v
x
) in (5.2.89) vanishes. We are left
with a second order dierential equation, with solutions of the form
y(t) = Acos t + Bsin t
If we again apply our initial conditions, we note that at t = 0 then y = 0, so we must require that
A = 0. Our solution is then
y(t) = B sin t (5.2.90)
We now substitute (5.2.90) into (5.2.88):
x = yB sin t x(t) = Bcos t +C(x) x(t) = B cos t +x
0
(5.2.91)
Where we note that initial conditions imply that C(x) is none other than the initial position of our
particle. Additionally, we can unleash the true identity of the integration constant B everpresent
in both (5.2.90) and (5.2.91). We do this by taking the derivative of (5.2.90) and setting it equal
to v
y
:
y(t) = B cos 0 = v
y
B =
v
y

And now we can rewrite (5.2.90) and (5.2.91):


x(t) =
v
y

cos t +x
0
(5.2.92a)
y(t) =
v
y

sin t (5.2.92b)
And now, after one full period, t , which implies that = 2, it can be easily (very easily)
seen that (5.2.92a) and (5.2.92b) reveal the location of the particle to be
x() = x
0

v
y

, y() = 0
Problem 25
A long wire lies along the z-axis. A square coil of wire, each side of length a, lies in the xz-plane. The side
nearest the wire is parallel to the wire and located at x = a.
a) Find the vector potential A if the wire is carrying a current I and there is no current in the loop.
b) What is the ux through the loop, using the results from part a)?
c) What is the mutual inductance of the loop and on the wire?
W. Erbsen ELECTRODYNAMICS
d) If the current I is varying at a rate dI/dt = K, what is the induced emf in the coil?
e) If there is a current in the loop varying at a rate dI/dt = C, what is the size of the voltage across the ends
of the long wire?
Solution
a) In order to nd the magnetic vector potential A, we must uncurl it from B = A. We do
this by rst setting up the determinant,

z
A

A
z

Due to the geometry of this particular problem, since I ows along the z direction and the face of
the loop is adjacent aligned to the wire, we can make some shortcuts. We know that A only ows
in the z direction in our case, so the other terms are zero. Additionally, we can see that A will
depend only on , so any derivatives of A that arent with respect to will be zero.

0 0
0 0 A
z

A
z

So, we now have


B = A =

A
z

=

0
I
2
(5.2.93)
The dierential equation in (5.2.93) can be solved straightforwardly,

A
z

=

0
I
2

A
z
=

0
I
2
log
_
a

_
(5.2.94)
b) To nd the ux, we use the typical denitions:

B
=
_
B dA =
_
(A) dA =
_

0
I
2
ddz =

0
I
2
_
2a
a
1

d
_
a
0
dz (5.2.95)
The integration in (5.2.95) is straightforward, and the result is

B
=

0
Ia
2
log [2] (5.2.96)
c) If we recall that the mutual inductance is qualitatively dened as M =
B
/I, then the mutual
inductance is simply:
M =

0
a
2
log [2]
CHAPTER 5: DEPARTMENTAL EXAMINATIONS 439
d) Using (5.2.96), we can straightforwardly nd the emf to be
E =
d
B
dt
=
d
dt

0
Ia
2
log [2] =

0
a
2
log [2]
dI
dt
And using the handy substitution the prompt suggests,
E =
K
0
a
2
log [2]
e) With the given substitution, the voltage across the ends of the long wire is
E =
C
0
a
2
log [2]
Problem 26
A disc carrying a uniform surface charge density has radius a and rotates at an angular frequency . This
disc lies in the xy-plane and rotates along the z-axis. A second identical disc rotating at the same frequency is
located with its center on the z-axis at Z, but this one lies in the yz-plane about an axis parallel to the x-axis.
Find the torque on the second disc. Indicate in a sketch the direction of the torque and the sense of rotation of
the two discs.
Solution
We begin by recanting a rather unfamiliar form of Biot-Sevarts Law:
dB =

0
4
KdS r
[r[
3
(5.2.97)
Where K is known as the surface charge density, and is described by K = v. Similar to what we did in
a previous problem, we dene all the elements needed to compute the magnetic eld from (5.2.97). The
line element and its magnitude are, respectively
r = zz , and [r[ =
_
z
2
+
2
And we can also expand upon the surface charge density:
K = v =

And we also recall that that the volume element for cylindrical coordinates is ds = dd. We can now
calculate,
Kds r =( ) ( dd) (zz )
=
2
dd

(zz )
W. Erbsen ELECTRODYNAMICS
=
2
dd(z +z) (5.2.98)
We can now substitute (5.2.98) into (5.2.97),
dB =

2
(z + z)
(z
2
+
2
)
3
/2
dd
B =

4
__

2
(z + z)
(z
2
+
2
)
3
/2
dd
=

2
_
z
_
a
0

2
(z
2
+
2
)
3
/2
d +
_
a
0

3
(z
2
+
2
)
3
/2
z d
_
(5.2.99)
The rst integral in (5.2.99) is neglected, due to symmetry. The second integral can be found in tables,
and the result is
B =

0

2
_
a
2
+ 2z
2

z
2
+a
2
2z
_
z (5.2.100)
Now, if we can reconcile with the fact that K = v, then we can nd the magnetic moment of the upper
loop as follows
=
_
a
0
I d =
_
a
0
K(
2
) d =
_
a
0
v(
2
) d =
_
a
0
(
2
) d =
_
a
0

3
d =
a
4
4
(5.2.101)
And if we recall that within this context = B, then by using (5.2.100) and (5.2.101) we can nd
the torque:
=

0

2
a
4
8
_
a
2
+ 2z
2

z
2
+a
2
2z
_
( y)
Problem 27
The KSU ultra-short pulse, ultra-high intensity laser in the JRM Lab has = 800 nm, pulse width of 20 femtosec-
onds, pulse energy of 6 mJ, and can be focused to a spot size of 100 m.
a) What is the intensity of the light in the focus?
b) What average electric eld produces this intensity?
c) To how many atomic units of the intensity and electric eld does this correspond?
Solution
CHAPTER 5: DEPARTMENTAL EXAMINATIONS 441
a) Intensity is dened as the amount of energy per unit area per unit time and has units W/m
2
.
Mathematically,
I =
ENERGY
(AREA) (TIME)
=
6 10
3
J

_
10010
3
m
2
_
2
(20 10
13
s)
I 3.8197 W/m
2
(5.2.102)
b) The relationship between the eld and intensity is
I =
1
2
c
0
[E
0
[
2
[E
0
[ =
_
2I
c
0
(5.2.103)
Where we can plug in our value from (5.2.102) and also for the various constants to arrive at a nal
value. It should be noted however that I [E[
2
.
c) Donno
Problem 28
A parallel plate capacitor is being charged with a current I. The capacitor is circular and has an area of a
2
.
a) What is the displacement current between the plates?
b) Referring to the drawing below, what is H in regions 1, 2, and 3 for < a. (Neglect fringing elds)
c) From the boundary condition on H
t
, where H is evaluated at r = (< a), what are the values on the surface
currents K on the face of each of the capacitor plates?
d) Make a sketch of all the current ows and label them.
Solution
a) Most formally, the displacement current is found by dierentiating the electric displacement eld
W. Erbsen ELECTRODYNAMICS
D =
0
E + P

J
d
=
0
E
t
+
P
t
We have no dielectric so the polarization term is neglected in our case. One thing that should
denitely be remembered is that the electric eld between the plates of a parallel plate capacitor
is E = /
0
. We are now in a place to nd the displacement current J
d
:
J
d
=
0
E
t
=
0

t
_

0
_
=

t
=

t
_
Q(t)
a
2
_
(5.2.104)
If we recall that I(t) =

Q(t), then (5.2.104) is
J
d
=
I
a
2
z (5.2.105)
b) Recall that Amperes Law for H elds is
_
H d = I
d
Within region 2, we can use (5.2.105) to nd H:
H
2
(2) =
I
a
2

H
2
=
I
2

(5.2.106)
And in regions 1 and 3, the value for H is simply
H
1,3
=
I
2

(5.2.107)
Problem 29
A spherical conductor A contains two spherical cavities. The total charge on A itself is zero. However, there
is a point charge q
b
at the center of one cavity, and q
c
at the center of the other. A considerable distance r away
is another charge q
d
. What force acts on each of the four objects A, q
b
, q
c
, and q
d
? Which answers, if any, are only
approximate? If q
b
is displaced slightly away from the center of the cavity, will it experience a resistance force?
(If so, what is it?)
Solution
The forces on charges q
b
and q
c
themselves are, of course, zero. To calculate the force of q
d
acting on A,
F =
1
4
0
q
d
(q
b
+q
c
)
r
2
(5.2.108)
CHAPTER 5: DEPARTMENTAL EXAMINATIONS 443
Which is the same as the net force of A acting on q
d
. We must recognize, however that (5.2.108) is an
approximation. To get the exact answer you must employ the method of images with an image charge
inside the sphere. The exact result turns out to be
F =
1
4
0
q
d
(q
b
+ q
c
)
r
2

1
4
0
q
2
Rr
(r
2
R
2
)
2
Problem 30
A uniformily charged rod of length a resides along the z-axis as drawn. The linear charge density is . The
voltage potential at (x, 0, 0) is
V =
1
4
0
_
a
0

z
2
+x
2
dz (5.2.109)
a) Expand the denominator of this expression for x z to two nonzero terms.
b) Substitute this expansion into the expression for V , work with two integrals.
c) Identify the two results as monopole, dipole, quadrupole, octapole or whatever and give an expression for the
two moments.
Solution
W. Erbsen ELECTRODYNAMICS
a) The series expansion around z is
1

z
2
+x
2
=
1
x
_
z
2
/x
2
+ 1
=
1
x

z
2
2x
3
+
3z
4
8x
5

5z
6
16x
7
...
So, expanded to the rst two nonzero terms:
1

z
2
+ x
2
=
1
x

z
2
2x
3
(5.2.110)
b) Substituting (5.2.110) into (5.2.109), we have
V =
1
4
0
_
a
0

_
1
x

z
2
2x
3
_
dz
=
1
4
0

_
1
x
_
a
0
dz
1
2x
3
_
a
0
z
2
dz
_
=
1
4
0

_
a
x

a
3
6x
3
_
So the result is
V =

4
0
a
x


24
0
a
3
x
3
(5.2.111)
c) The rst term in (5.2.111) is monopole, because of the 1/x factor, and the second is quadrupole,
since it has a 1/x
3
term.
Problem 31
a) Find the voltage potential at point P a distance b from the right end of a uniform line charge with length
and total charge Q as drawn.
b) Using this voltage nd the x-component of the electric eld at point P.
Solution
a) We rst note that our object has a charge given by dq = d, and the total charge is Q, and the
total length is , so we know that = Q/. Furthermore, we know that E = V dV = Edx.
We can take this further:
V =

4
0
_
0

1
(b x)
dx
=
1
4
0
Q

[log (b x)[
0

CHAPTER 5: DEPARTMENTAL EXAMINATIONS 445


=
1
4
0
Q

[log (b) log (b + )]


=
1
4
0
Q

log
_
b
b +
_
So that the potential is
V =
1
4
0
Q

log
_
b +
b
_
(5.2.112)
b) E can be calculated rather straightforwardly as follows
E =

x
V x
=
1
4
0
Q

_

b
log
_
b +
b
__
x
=
1
4
0
Q

_
1
(b+)
/
b
b +
_
x
=
1
4
0
Q

1
b
_
b b
b +
_
x
From which we arrive at
E =
1
4
0
Q
b
_
1
b +
_
x
Problem 32
A dipole consistent of equal and opposite charges q separated by x resides a distance d/2 above a conduct-
ing, grounded, innite plane as drawn.
a) On the drawing, accurately draw the image charge(s).
b) Find the force between the dipole and the plane.
c) Use the binomial expansion to expand this force to order (x/d)
2
when d x. With what power does the
force depend on the distance d?
W. Erbsen ELECTRODYNAMICS
Solution
a) Two charges are imagined to be symmetrically placed about the grounded conducting plane; the
magnitudes of opposing charges are opposite.
b) We use the following equation:
F =
3(P r)P

+ 3(P r)P+ 3(P P

)r 15(P r)(P

r)r
r
4
(5.2.113)
Which should probably be memorized. The vectors P and P

represent the dipole moments of


the charged, and uncharged dipoles (respectively). Their values are P = qx x and P

= qx( x).
The vector r represents the distance between the two moments r = 2d( y). We can see that the
majority of the terms in (5.2.113) vanish,
F =
3(P P

) r
r
4
=
3(qx x qx( x)) r
r
4
F =
3q
2
x
2
16d
4
y (5.2.114)
Problem 33
a) Two spheres initially uncharged are connected by a battery of voltage V . After the switch is closed, what is
the charge on the larger sphere?
b) What is the capacitance of the larger sphere?
Solution
a) When the switch is closed, the amount of charge that ows into the larger sphere (of radius r = b)
can be found by
V =
1
4
Q
b
Q = 4bV
CHAPTER 5: DEPARTMENTAL EXAMINATIONS 447
b) To nd the capacitance of the larger sphere we recall that Q = CV , so:
C = 4b
Problem 34
A circular loop of wire with radius a and electrical resistance R lies in the xy-plane. A uniform mgnetic eld
is turned on at time t = 0; for t > 0 the eld is
B(t) =
B
0

2
_

j +

k
_
_
1 e
t

(5.2.115)
a) Determine the current I(t) induced in the loop.
b) Sketch a graph of I(t) versus t.
Solution
a) When the magnetic eld is turned on, ux passes through the loop which causes current to ow.
The amount of current is determined by how much ux passes through. We start from Ohms law,
= IR. Faradays Law says,
E =
d
B
dt
(5.2.116)
Where the magnetic ux,
B
, is proportional to the amount of eld lines that pass through a given
area. In our case, this is

B
=
_
B dA =
a
2
B
0

2
_

j +

k
_
_
1 e
t

(5.2.117)
Substituting (5.2.117) into (5.2.116) we can nd E:
E =
d
dt
_
a
2
B
0

2
_

j +

k
_
_
1 e
t

_
W. Erbsen ELECTRODYNAMICS
=
a
2
B
0

2
_

j +

k
_
e
t
(5.2.118)
Using (5.2.118) and also Ohms Law, we arrive at
I(t) =
a
2
B
0

2 R
_

j +

k
_
e
t
(5.2.119)
b) Looks like an exponentially decaying signal. The negative sign denotes the direction, the coecients
the magnitude.
Problem 35
The solenoid pictured below is long, and has n turns per unit length and cross section A.
a) If the current in the solenoid changes from I
1
to I
2
, how much charge passes through the resistor?
b) If the current in the solenoid, as a function of time t, is
I(t) = I
1
e

t
/
+ I
2
_
1 e

t
/
_
(5.2.120)
what is the current I
R
(t) in the resistor?
c) Sketch separately the I
1
and I
2
terms and the total I vs.
t
/

for I
1
= 1A and I
2
= 2A. On a separate graph
sketch I
R
(t) vs.
t
/

.
Solution
a) For discrete changes in quantities, there are no need for integrals or derivatives, eg dt t. Keeping
this in mind,
E =

B
t
=

t
BA =
B
t
A =

0
n(I
2
I
1
)A
t
(5.2.121)
CHAPTER 5: DEPARTMENTAL EXAMINATIONS 449
We also know Ohms Law, which states that E = IR, where I = Q/(t). With this knowledge,
(5.2.121) becomes
Q
t
R =

0
n(I
2
I
1
)A
t
From which it is clear that the t on the denominator cancels, and we are left with
Q =

0
n(I
2
I
1
)A
R
b) To calculate the current,
E =

t
BA =

0
nIA =
0
nA
I(t)
t
(5.2.122)
Recalling again Ohms Law, I
R
(t) can be found by combining (5.2.122) and (5.2.120):
I
R
(t) =
E
R
=

0
nA
R

t
_
I
1
e

t
/
+ I
2
_
1 e

t
/
__
=

0
nA
R
_

I
1

t
/
+
I
2

t
/
_
From which it is easily seen that
I
R
(t) =

0
nA
R
e

t
/
[I
2
I
1
]
5.3 Modern Physics
Problem 1
Recently, 2000 rubidium atoms (
87
37
Rb), which had been compressed to a density of 10
13
atoms/cm
3
, were ob-
served to undergo a Bose-Einstein condensation as a function of temperature. This occurs when the deBroglie
wavelengths overlap and the system forms a single macroscopic quantum state. Estimate the temperature at which
Bose-Einstein condensation occurs for this system.
Solution
We rst recall that the deBroglie wavelength is given by
=
h
p
(5.3.1)
Where we are also told that
N =2000 atoms
=10
13
atoms cm
3
W. Erbsen MODERN PHYSICS
So, we can say that the spacing is proportional to the deBroglie wavelength, and the spacing can be
found by
2000 atoms
L
3
= 10
13
atoms cm
3
L =
_
2000 atoms
10
1
3 atoms cm
3
_1
/3
(5.3.2)
We now wish to set (5.3.2) equal to the deBroglie wavelength. But rst, we recall that
p =

2mE
E k
B
T
_
p
_
2mk
B
T (5.3.3)
Where putting (5.3.3) into (5.3.1) of course yields

h

2mk
B
T
(5.3.4)
Setting (5.3.4) equal to (5.3.2) yields
_
2000 atoms
10
13
atoms cm
3
_1
/3
=
h

2mk
B
T
Solving for T leaves us with
T
h
1
/2
2mk
B
_
2000 atoms
10
13
atoms cm
3
_1
/6
Problem 2
Rutile, TiO
2
, is a tetragonal crystal a = 4.4923

A. The position of the Ti atoms in the unit cell are 0, 0, 0;


1
/
2
,
1
/
2
,
1
/
2
and the position of the oxygen atoms are 0.31, 0.31, 0; 0.81, 0.19, 0.5; 0.69, 0.69, 0; 0.19, 0.81, 0.5.
a) What would be the d-spacing of the 310 planes of rutile?
b) Using CuK x-radiation, = 1.54

A, what would be the diraction angle from the (310) planes of Rutile?
c) Assuming atomic scattering factors of f
Ti
for the 2 titanium atoms and f
O
for the 4 oxygen atoms, what is
the crystalline scattering factor from the (310) planes of TiO
2
?
Solution
We rst note that:
a =4.4923

A
Ti =
0, 0, 0
1
/
2
,
1
/
2
,
1
/
2
CHAPTER 5: DEPARTMENTAL EXAMINATIONS 451
O =
0.31, 0.31, 0
0.81, 0.19, 0.5
0.69, 0.69, 0
0.19, 0.81, 0.5
a) In order to nd the d-plane spacing, we recall that
1
d
2
=
h
2
a
2
+
k
2
b
2
+

2
c
2
(5.3.5)
Where h, k and correspond to the the indices of the plane (eg h = 3, k = 1, and = 0).
Furthermore, the numbers a, b, and c correspond to the sides of the crystal. In our case a = b =
c a. Taking all this into account, (5.3.5) becomes
1
d
2
=
h
2
a
2
+
k
2
a
2
d = a
_
1
h
+
1
k
_
(5.3.6)
Plugging into (5.3.6) the appropriate values, we have
d = 4.4923

A
_
1
3
2
+
1
1
_
d = 1.42

A (5.3.7)
b) We recall that the diraction equation from a periodic structure is given by
n = 2d sin (5.3.8)
Where integer multiples of yield maxima. Since we are interested in the rst maxima, then n = 1
in our case, and after applying this to (5.3.8) and solving for , we have
= sin
1
_

2d
_
(5.3.9)
Putting in the correct numbers into (5.3.9) leaves us with
= sin
1
_
1.54

A
2
_
1.42

A
_
_
= 32.8
o
c) In order to nd the crystalline scattering factor for our lattice, we employ the following equation
for reasons unknown:
F
hk
=

j
f
j
exp [i
hk
(j)] (5.3.10)
Where

hk
(j) = 2 (hx
j
+ky
j
+ z
j
) (5.3.11)
Substituting (5.3.11) into (5.3.10) yields
F
hk
=

j
f
j
exp [i2 (hx
j
+ ky
j
+z
j
)] (5.3.12)
In our case, is always zero, so we can make our lives easier by ignoring the last term in (5.3.14):
F
hk
=

j
f
j
exp [i2 (hx
j
+ky
j
)]
W. Erbsen MODERN PHYSICS
And we also know what h and k are, so lets go ahead and put them in too:
F
310
=

j
f
j
exp [i2 (3x
j
+y
j
)] (5.3.13)
Completing (5.3.10) and (5.3.11) for Titanium is the easiest, so lets do that sum rst:
F
Ti
310
=f
Ti
exp [i2 (3 0 + 0)] +f
Ti
exp
_
i2
_
3
1
/
2
+
1
/
2
_
=f
Ti
[1 + exp [i4]] (5.3.14)
Now, lets do the same for Oxygen:
F
O
310
=f
O
exp [i2 (3x
j
+ y
j
)]
. .
I
+f
O
exp [i2 (3x
j
+ y
j
)]
. .
II
+f
O
exp [i2 (3x
j
+ y
j
)]
. .
III
+f
O
exp [i2 (3x
j
+ y
j
)]
. .
IV
So lets do these one at a time..
I =f
O
exp [i2 (3 0.31 + 0.31)] f
O
exp [2.48i] (5.3.15a)
II =f
O
exp [i2 (3 0.81 + 0.19)] f
O
exp [5.24i] (5.3.15b)
III =f
O
exp [i2 (3 0.69 + 0.69)] f
O
exp [5.52i] (5.3.15c)
IV =f
O
exp [i2 (3 0.19 + 0.81)] f
O
exp [2.76i] (5.3.15d)
Combining (5.3.15a)-(5.3.15d) back into our original equation,
F
O
310
=f
O
[exp [2.48i] + exp [5.24i] + exp [5.52i] + exp [2.76i]] (5.3.16)
And now, combining (5.3.14) with (5.3.16),
F
310
= F
Ti
310
+ F
O
310
F
310
= f
Ti
[1 + exp [i4]] +f
O
[exp [2.48i] + exp [5.24i] + exp [5.52i] + exp [2.76i]]
Problem 3
Copper is a monovalent metal of density 8 g/cm
3
and atomic weight of 64 g/mole. Using the free electron
approximation, and considering a cubic system of side L,
a) Find a formula for the number of states dN within an energy interval d.
b) Use your expression to evaluate the Fermi energy for copper in eV, at 0 K.
Solution
The Free Electron Approximation assumes that the valence electrons are essentially detached from their
parent ions. Subsequently, the free electrons are thought of as conduction electrons, and are able
to move freely throughout the crystal. This approximation completely ignores all electron-electron and
ion-electron interactions.
CHAPTER 5: DEPARTMENTAL EXAMINATIONS 453
a) In this part we are looking for the density of states for a single valence electron. Using the free
electron approximation, we imagine the electron to be bound within a cube, each side of length L.
The TISE equation says that


2
2m

2
= E

2
2m
_

2
x
2
+

2
y
2
+

2
z
2
_
= E (5.3.17)
Whose corresponding eigenvalues are
E =

2

2
2mL
2
_
n
2
x
+n
2
y
+ n
2
z
_
(5.3.18)
If we imagine the energy levels to be continuous rather than discrete, then the total number of
states can be approximated by the volume of a sphere of radius r =
_
n
2
x
+n
2
y
+ n
2
z
1
/2
. The total
number of states, N, can then be found by integrating this radius within n-space:
N = 2
1
8

4
3
r
3
(5.3.19)
Where in (5.3.19), the factor of 2 is due to spin degeneracy (two dierent spins with the same
energy), while the factor of
1
/
8
is due to the fact that we are only interested in the quadrant where
all ns are positive (n
x
, n
y
, n
z
> 0), which happens only within one quadrant, which corresponds to
1
/
8
of the total volume of the sphere. Substituting r in (5.3.19) for its equivalent form in terms of
n gives
N = 2
1
8

4
3

_
n
2
x
+ n
2
y
+ n
2
z
3
/2
(5.3.20)
Now, we solve (5.3.18) as follows
n
2
x
+n
2
y
+n
2
z
=
2mL
2
E

2
(5.3.21)
Substituting (5.3.21) back into (5.3.20),
N =2
1
8

4
3

_
2mL
2
E

2
_
3
/2
=

3

(2m)
3
/2
L
3

3
E
3
/2
=
(2m)
3
/2
L
3
3
3

2
E
3
/2
(5.3.22)
We now recall that the density of states g(N), is dened as the rst derivative of N with respect
to energy. Using (5.3.22), this becomes
g(N) =
dN
dE
=
d
dE
_
(2m)
3
/2
L
3
3
3

2
E
3
/2
_
=
(2m)
3
/2
L
3
3
3

2
_
d
dE
E
3
/2
_
(5.3.23)
Carrying out the simple derivative in (5.3.23), we have
W. Erbsen MODERN PHYSICS
g(N) =
(2m)
3
/2
L
3
2
3

2
E
1
/2
(5.3.24)
b) In order to nd the Fermi Energy, E
F
, we rst recall that the number of particles is given by
N =
_
EF
0
g(E) f(E) dE (5.3.25)
Where g(E) is the density of states, and f(E) is the Fermi-Dirac distribution function, which is
given by
f(E) =
1
exp [(E ) /k
B
T] + 1
(5.3.26)
At T = 0 K, then (T = 0) = E
F
, where E
F
is the Fermi Energy. If E < , then (5.3.26) becomes
f(E) =
1
exp [(E ) /k
B
T] + 1
1
With this, (5.3.25) becomes
N =
_
EF
0
g(E) dE
=
_
EF
0
_
(2m)
3
/2
L
3
2
3

2
E
1
/2
_
dE
=
(2m)
3
/2
L
3
2
3

2
_
EF
0
E
1
/2
dE
=
(2m)
3
/2
L
3
2
3

2

2
3
E
3
/2
F
=
(2m)
3
/2
L
3
3
3

2
E
3
/2
F
(5.3.27)
From the values given initially, we know that
N
V
=
N
L
3
=8
g
cm
3

1 mol
64 g

(100 cm)
3
(1 m)
3
N
A
=8
g
cm
3

1 mol
64 g

(100 cm)
3
(1 m)
3
6.022 10
22
atoms
mol
=7.528 10
28
atoms
m
3
(5.3.28)
Now, we wish to solve (5.3.27) for E
F
, which yields
E
F
=

2
2m
_
3
2
N
L
3
_2
/3
(5.3.29)
Substituting in (5.3.28) and the other appropriate values into (5.3.29) yields
CHAPTER 5: DEPARTMENTAL EXAMINATIONS 455
E
F
=
_
3
_
6.582 10
16
eV s
_
3

2
(2 0.511 10
6
eV)

_
3 10
8
m/s
_
3
7.528 10
28
atoms
m
3
_
2
/3
E
F
= 6.4 eV
Problem 4
A white dwarf is a star which has used up its nuclear fuel and contracted under its own weight to a state of
high density and arbitrarily low temperature. It is held up by the kinetic energy of its degenerate electrons.
Assume that all electrons in the star behave like free electrons in a metal, that all the electron states are full up
to the Fermi energy, that the star is spherical and made of helium, and that the electrons are non-relativistic.
a) Discuss briey the physical principles needed to nd the equilibrium radius of the star. Write down a set of
equations which, when combined, will determine the radius. Dont worry too much about the factors of 2.
b) Show that this radius is given approximately by
R = C
h
2
(Gm
e
m
p
)
5
/3
M

1
/3
(5.3.30)
Where C is a purely numerical constant, G is the gravitational constant, m
e
and m
p
are the electron and
proton masses, and M is the mass of the star.
Solution
a) The equilibrium radius is dened as the radius which minimizes the total energy of the star. In
our case the energy comes from the gravitational energy, which acts to contract the star, and
also the degenerate energy, which is attributed to the tendency of Fermions to repel one another.
Mathematically,
E
tot
= E
grav
+E
deg
(5.3.31)
b) Lets nd the gravitational energy in (5.3.31) rst. In order to nd the gravitational self-energy of
some spherical mass with a density , we recall that = m/V m = V . In other words,
m =
4
3
r
3

_
recalling that V =
4
3
r
3
_
(5.3.32)
From which we can say that
dm = 4r
2
dr (5.3.33)
We also recall that the potential energy is given by
dU =
Gm
r
dm (5.3.34)
Substituting (5.3.33) into (5.3.34), and recalling that
_
U = E
grav
, we have
E
grav
=
_
R
0
Gm
r
dm
W. Erbsen MODERN PHYSICS
= G
_
R
0
1
r
_
4r
3

3
_
_
4r
2

_
dr (5.3.35)
Where I have substituted m for the value from (5.3.32) and likewise for dm from (5.3.33). Contin-
uing,
E
grav
=
16
2
G
2
3
_
R
0
r
4
dr
=
16
2
G
2
3
_
R
5
5
_
=
16
2
G
2
15
_
R
5

=
16
2
G
2
15
_
3M
4R
3
_
=
3
5
GM
2
R
(5.3.36)
We now recall that the total energy of a degenerate electron gas is given by

E
deg
=
3
5
NE
F
(5.3.37)
Where the Fermi Energy, E
F
, is given by
E
F
=

2
2m
e
_
3
2
N
V
_2
/3
(5.3.38)
Substituting (5.3.38) into (5.3.37),
E
deg
=
3
5
N
_

2
2m
e
_
3
2
N
V
_2
/3
_
=
3
2
10m
e
N
_
3
2
N
V
_2
/3
=
3
2
10m
e
_
3
2
_2
/3

1
V
2
/3
N
5
/3
(5.3.39)
Recalling that V = 4/3R
3
, (5.3.39) becomes
E
deg
=
3
2
10m
e
_
3
2
_2
/3

_
3
4R
3
_2
/3
N
5
/3
=
3
2
10m
e
_
9
4R
3
_2
/3
N
5
/3
=
3
2
10m
e
1
R
2
_
9
4
_2
/3
N
5
/3
(5.3.40)
Substituting (5.3.36) and (5.3.40) into (5.3.31),
E
tot
=
3
5
GM
2
R
+
3
2
10m
e
1
R
2
_
9
4
_2
/3
N
5
/3
(5.3.41)

Recall that E =
R
E d() d
CHAPTER 5: DEPARTMENTAL EXAMINATIONS 457
And now minimizing the total energy with respect to R, we take the rst derivative of (5.3.41) with
respect to R and set it equal to zero:
E
tot
R
=

R
_

3
5
GM
2
R
+
3
2
10m
e
1
R
2
_
9
4
_2
/3
N
5
/3
_
=
3
5
GM
2

R
_
1
R
_
+
3
2
10m
e
_
9
4
_2
/3
N
5
/3

R
_
1
R
2
_
=
3
5
GM
2
_

1
R
2
_
+
3
2
10m
e
_
9
4
_2
/3
N
5
/3
_

2
R
3
_
=
3
5
GM
2
_
1
R
2
_

6
2
10m
e
_
9
4
_2
/3
N
5
/3
_
1
R
3
_
(5.3.42)
Setting (5.3.42) equal to zero and solving for R, we nd that
3
5
GM
2
R =
6
2
10m
e
_
9
4
_2
/3
N
5
/3
R =

2
Gm
e
M
2
_
9
4
_2
/3
N
5
/3
(5.3.43)
Recalling that N = M/m
p
, (5.3.43) becomes
R =

2
G
_
9
4
_2
/3
1
m
e
(m
p
N)
2
N
5
/3
=

2
G
_
9
4
_2
/3
1
m
e
m
2
p
1
N
1
/3
=

2
G
_
9
4
_2
/3
1
m
e
m
2
p
_
m
p
M
_1
/3
=

2
G
_
9
4
_2
/3
1
m
e
m
5
/3
p
1
M
1
/3
(5.3.44)
And now recalling that = h/2, (5.3.44) becomes
R =
h
2
4
2
G
_
9
4
_2
/3
1
m
e
m
5
/3
p
1
M
1
/3
=
1
4
2
_
9
4
_2
/3
h
2
G
1
m
e
m
5
/3
p
1
M
1
/3
(5.3.45)
And we are nally left with
R = C
h
Gm
e
m
5
/3
p
M

1
/3
Where
C =
1
4
2
_
9
4
_2
/3
W. Erbsen MODERN PHYSICS
Problem 5
A particular atom has two energy levels with a transition wavelength of 1064 nm. At 297 K there are 2.5 10
21
atoms in the lower state.
a) How many atoms are in the upper state?
b) Suppose that 1.8 10
21
of the atoms in the lower state are pumped to the upper state. How much energy
could this system release in a single laser pulse? How many photons are emitted in this pulse?
Solution
a) We rst recall that the number of atoms in a particular state is
N =
1
Z
exp
_

E
k
B
T
_
(5.3.46)
Taking the ratio of the two populations, and using (5.3.46), we have
N
U
N
L
= exp
_

(E
U
E
L
)
k
B
T
_
(5.3.47)
Where we note that
E =E
U
E
L
=
hc

=
_
4.136 10
5
eV s
_ _
3 10
8
m s
1
_
(1064 10
9
m)
=1.166 eV (5.3.48)
Substituting (5.3.48) back into (5.3.47) and solving for N
U
yields
N
U
=
_
2.5 10
21
atoms
_
exp
_

(1.166 eV)
_
8.617 10
5
eV K
1
_
(300 K)
_
N
U
= 40.87 atoms (5.3.49)
b) If 1.8 10
21
atoms are pumped to the upper state, and we are interested in how much energy
they could release in a single laser pulse, this is the same as asking what happens if all atoms
simultaneously fall down to the lower state from the upper state. We know that the energy dierence
between the two levels is E = 1.166 eV from (5.3.48), so the total energy that would be released
is
E
pulse
=
_
1.8 10
21
_
(1.166 eV)
_
1.602 10
19
J eV
1
_
E
pulse
= 2.099 10
21
eV
Pulse
E
pulse
= 336.292
Joules
Pulse
CHAPTER 5: DEPARTMENTAL EXAMINATIONS 459
Problem 6
A helium-neon laser emits light with a wavelength of 6328

A in a transition between two states of neon atoms.
What would be the ratio of population of the upper state relative to the lower state of the neon atoms if they were
in thermal equalibrium at 300 K?
Solution
At equilibrium (T = 300 K in this case), the population of the two states are equal. That is,
N
1
= N
2
And the ratio would be
N
2
N
1
= exp
_
(E
2
E
1
)
k
B
T
_
(5.3.50)
Where
E =E
2
E
1
=
hc

=
_
4.136 10
15
eV s
_ _
3 10
8
m s
1
_
(632.8 10
9
m)
=1.961 eV
Applying this to (5.3.50) at T = 300 K yields
N
2
N
1
= exp
_

(1.961 eV)
_
8.617 10
5
eV K
2
_
(300 K)
_

N
2
N
1
= 1.136 10
33
Problem 7
The atomic number of Na is 11.
a) Write down the electronic conguration for the ground state of the Na atom showing in standard notation
the assignment of all electrons to various one-electron states.
b) Give the standard spectroscopic notation for the ground state of the Na atom.
c) The lowest frequency line in the absorption spectrum of Na is a doublet. What are the spectroscopic desig-
nations of the pair of energy levels to which the atom is excited as a result of this absorption process?
d) Discuss the mechanism responsible for the splitting between this pair of energy levels.
e) Which of these levels lies the lowest? Discuss the basis on which you base your choice.
W. Erbsen MODERN PHYSICS
Solution
a) The ground state electronic conguration of Sodium in standard notation is:
1s
2
2s
2
2p
6
3s
1
b) Only the unpaired 3s electron contributes to the total angular momentum J:
L = 0, S =
1
/
2
_
J =
1
/
2
So, the spectroscopic notation is:
2
S1
/2
c) The rst excited state is the 3p state, which means that
L = 1, S =
1
/
2
_
J =
1
/
2
,
3
/
2
And therefore, the spectroscopic notation for the lowest excited states is
2
P1
/2
,
2
P3
/2
d) The levels 3
2
P1
/2
and 3
2
P3
/2
are split when the spin-orbit interaction is taken into account. In the
electrons rest frame, it sees the nucleus orbiting it. This causes the electron to see a magnetic eld
caused by the nucleus orbiting the electron. This is seen as a current loop, and is proportional
to the electrons orbital angular momentum, L
i
. This eld interacts with the electrons magnetic
dipole moment,
i
, which is proportional to its spin angular momentum, s
i
.
This creates a perturbation term which is proportional to L
i
, S
i
for each electron. The pertur-
bation term is given by
H =
1
2m
2
c
2
1
r
i
dV(r
i
)
dr
i
L
i
S
i
e) The 3
2
P1
/2
state lies lower than the 3
2
P3
/2
when spin-orbit is taken into account. The reason is
because of Hunds rules, which states that for any two states with identical L and S values, the one
with the lowest J is more tightly bound with the lowest energy shells that are less than half-full (as
is the case with a single electron in a P-state).
Problem 8
A slab of lead shielding 1.0 cm thick reduces the intensity of 15 MeV rays by a factor of 2.
a) By what factor will 5.0 cm of lead reduce the beam?
b) What is the assumption cross-section for 15 MeV s in lead nuclei? Lead has a density of 11.4 g/cm
3
, and
an atomic weight of 207 g/mol.
Solution
CHAPTER 5: DEPARTMENTAL EXAMINATIONS 461
The decrease in intensity of radiation passing through matter is mathematically given by
I = I
0
exp [d] (5.3.51)
Where = n, and n is the number of atoms per cm
3
, and is the absorption cross section.
a) First, we nd for the initial case:
0.5I
0
= I
0
exp [] = log [2]
So, for d = 5.0 cm, (5.3.51) becomes
I = I
0
exp [log [2] 5] I =
I
0
32
b) Recalling that = n, and that for lead we found that = log [2], we have:
log [2] = n =
log [2]
n
(5.3.52)
Where for lead, we nd that
n =11.4
g
cm
3

1 mol
207 g

N
A
atoms
1mol
=
11.4 N
A
207
atoms
cm
3
(5.3.53)
Substituting (5.3.53) into (5.3.52),
= log [2]
207
11.4 6.022 10
23
cm
3
= 2.090 10
23
cm
3
Problem 9
The potential energies of two diatomic molecules of the same reduced mass are shown. From the graph, determine
which molecule has the larger
a) Inter-nuclear distance
b) Rotational inertia (moment of inertia)
c) Separation between rotational energy levels
d) Binding energy
e) Zero-point energy
f ) Separation between low-lying vibrational states
Solution
W. Erbsen MODERN PHYSICS
a) 1 has a larger inter-nuclear distance
b) 1 has a larger rotational intertia , because I = R
2
and since we have the same and 1 has a
larger inter-nuclear distance, than 1 must also have a higher rotational inertia.
c) 2 has a larger separation between rotational energy levels , because
E
rot

( + 1)
2
2I
And from part b), I
1
> I
2
so we must conclude that E
1
< E
2
.
d) 2 has a larger binding energy , since the well is deeper.
e) 2 has the largest zero-point energy . We recall that the zero-point energy is dened as the lowest
energy possible e.g. the lowest vibrational state. We can approximate, using the simple harmonic
oscillator, that
E
1
=
1
/
2

1
E
2
=
1
/
2

2
_
T =
_
2m

2
(V
i
E)
_1
/2
,
i
=
_
K
i
m
And since V
2
> V
1
, then K
2
> K
1
and
2
>
1
, and therefore E
2
> E
1
.
f ) 2 has the largest separation between the lowest-lying vibrational states , since
E
vib

2
>
1
Problem 10
A K
0
meson (mass 497.7 MeV/c
2
) decays into a
+
,

meson pair with a mean life of 0.89


10
s.
a) Which one of the fundamental interactions is responsible for this decay?
CHAPTER 5: DEPARTMENTAL EXAMINATIONS 463
b) Suppose the K
0
has a kinetic energy of 276 MeV when it decays, and that the two Mesons emerge at equal
angles to the original K
0
direction. Find the kinetic energy of each meson and the opening angle between
them. The mass of a meson is 139.6 MeV/c
2
.
Solution
a) This decay is caused by the weak interaction . This is know because K
o
has strange quarks, while
-particles do not. Only in the weak interaction strangeness is not conserved.
b) At this point, we must employ the wonders of conservation of momentum, which says
p
k
= p

+ +p

(5.3.54)
We now recall the handy formula
p
2
c
2
= E
2

_
mc
2
_
2
(5.3.55)
First it would be nice to combine the two momenta from the Pions in (5.3.54):
p
k
= 2p

(5.3.56)
However, we must be careful! The Pions do not both go in the same directions they y o at
equal angles. Therefore, the momentum we concern ourselves with from with is
p
k
= 2p

cos (5.3.57)
At this point we recognize that p

in (5.3.57) can be rewritten using our good ol pal (5.3.55):


p
2

c
2
= E
2

_
m

c
2
_
2
p

c =
_
E
2

(m

c
2
)
2
(5.3.58)
Now multiplying both sides of (5.3.58) by c,
p
k
c = 2p

c cos (5.3.59)
And substituting (5.3.58) into (5.3.59),
p
k
c = 2
_
E
2

(m

c
2
)
2
cos (5.3.60)
And squaring both sides,
p
2
k
c
2
= 4
_
E
2

_
m

c
2
_
2
_
cos
2
(5.3.61)
And substituting (5.3.55) into (5.3.61) for the LHS,
E
2
k

_
m
k
c
2
_
2
= 4 cos
2

_
E
2

_
m

c
2
_
2
_
(5.3.62)
We know all the constants in this equation, except for E

. From conservation of energy,


E
k
= 2E

=
1
/
2
E
k
Substituting this back into (5.3.65),
E
2
k

_
m
k
c
2
_
2
=4 cos
2

_
E
2
k
4

_
m

c
2
_
2
_
W. Erbsen MODERN PHYSICS
=cos
2

_
E
2
k

_
2m

c
2
_
2
_
(5.3.63)
We can now solve (5.3.63) for ,
= cos
1
_
E
2
k
(m
k
c
2
)
2
E
2
k
(2m

c
2
)
2
_
(5.3.64)
Before we go any further, we must recognize that E
k
represents that total energy of the K
o
, so it
includes both the mass energy term and the kinetic term. Using all the constants applied, (5.3.64)
becomes
= cos
1
_

(497.7 + 276 MeV)


2
(497.7 MeV)
2
(497.7 + 276 MeV)
2
(2 139.6 MeV)
2
_
(5.3.65)
Evaluating (5.3.65), we are left with
= 34.82
o
HOWEVER, if we want the total angle between the two Pions, we must double this result, which
yields
= 69.34
o
The kinetic energy can just be found via Conservation of Energy, which says that
T
k
= 2T

=
T
k
2
T

=
276 eV
2
T

= 138 MeV
Problem 11
A beam of 50 eV electrons is scattered from a cubic crystal with 1.2

A spacing between the crystal planes. The


electron beam is perpendicular to the crystal plane. At what angle will the rst diraction maximum be detected?
Solution
We rst recall that the deBroglie wavelength is given by
=
h
p
(5.3.66)
And the momentum is
T =
p
2
2m
p =

2mT (5.3.67)
Substituting (5.3.67) into (5.3.66) yields
CHAPTER 5: DEPARTMENTAL EXAMINATIONS 465
=
h

2mT
(5.3.68)
We also recall that the equation for diraction from a 3D period structure is
n = 2d sin (5.3.69)
Where we recognize that the path length must be in integer multiples of for completely constructive
interference. For the rst maxima, we have n = 1, and with (5.3.68), we have
h

2mT
= 2d sin = sin
1
_
h
2d
1

2mT
_
(5.3.70)
Substituting in the appropriate values into (5.3.70) yields
= sin
1
_
_
6.626 10
34
J s
2 (1.2 10
10
m)
1
_
2 (9.109 10
31
Kg) (50 eV)
_
1.602 10
19
J eV
1
_
_
_
= 46.3
o
Problem 12
One of the strongest emission lines observed from distant galaxies comes from Hydrogen and has a wavelength of
122 nm (in the ultraviolet region).
a) How fast must a galaxy be moving away from us in order for that line to be observed in the visible region of
366 nm?
b) What would be the wavelength of the line if that galaxy were moving towards us at the same speed?
Solution
a) If the source and receiver are moving away from each other, the Doppler eect says that
f

=
_
1 u
2
/c
2
_
1 + u
2
/c
2
f
o
=
_
1 u
2
/c
2
1 +u
2
/c
2
f
o
(5.3.71)
Recalling that f = c/, we can rewrite (5.3.71) in the following form
1

1 u/c
1 +u/c
1

o
W. Erbsen MODERN PHYSICS
_

_
2
=
1 u/c
1 + u/c
(5.3.72)
=
[1 u/c]
[1 +u/c]
(c/c)
(c/c)
=
c u
c +u
(5.3.73)
So, with (5.3.73) and the values given, we have
c u
c + u
=
_

_
2
=
_
122 nm
366 nm
_
2
=
_
1
3
_
2
=
1
9
(5.3.74)
Rewriting, (5.3.74) becomes
c u
c + u
=
1
9
9 (c ) =c + u
9c 9u =c + u
u + 9u =9c c
From which it is easy to see that
u =
8
10
c
b) If the galaxy is moving towards Earth at the same speed, we simply change the sign of u in (5.3.72):
1

1 +u/c
1 u/c
1

1 u/c
1 +u/c

0
=
_
1 0.8
1 + 0.8
122 nm
=
122 nm
3
Which nally leads us to

= 40.67 nm
Problem 13
Hall eect: Consider a free electron gas with an electric eld E along the x-direction. The electrons are con-
stantly undergoing collisions resulting in some average velocity E is applied.
a) The conductivity is dened using the current density j = E. Find an expression for in terms of the
relaxation time T which is half of the time between successive collisions. Let n be the density of electrons in
CHAPTER 5: DEPARTMENTAL EXAMINATIONS 467
the material and m the electron mass.
b) Now assume the electron gas is conned to a wire lying along the x-direction (E is still applied). Under these
circumstances the equation of motion of an electron is
m
dv
dt
= e (E+ v B) m
v
T
(5.3.75)
Where the rst term in the Lorentz force and the second term takes account of collisions. It is found that
a voltage develops along the y-direction (across the wire). A measure of this voltage is given by the Hall
constant R = E
y
/(j B
z
). Show that R = 1/(ne).
c) Will the Hall constant be larger in a metal or a semiconductor and why?
Solution
a) To nd an expression for the conductivity, in terms of the relaxation time, T, we must employ
several not-so-tedius substitutions. First, we write down the (given) Ohms Law:
J = E (5.3.76)
We also recall that the drift velocity is dened by
v =
J
nq
(5.3.77)
Where v is the velocity vector. We also recall that
v = a T (5.3.78)
Where a is the acceleration of course. Now, from Newtons second law,
F = ma (5.3.79)
And we must also recall that the force on a charged particle is given by
F = qE (5.3.80)
Setting (5.3.79) equal to (5.3.80),
ma = qE a =
qE
m
(5.3.81)
We now take (5.3.81) and substitute it back into (5.3.78):
v =
_

qE
m
_
T (5.3.82)
We now set (5.3.82) equal to (5.3.77), which yields
J
nq
=
qE
m
T J =
nq
2
ET
m
(5.3.83)
And nally, we set (5.3.83) and (5.3.83) equal to one another:
W. Erbsen MODERN PHYSICS
E =
nq
2
ET
m
=
nq
2
T
m
(5.3.84)
b) The very rst thing we need to do in this problem is to solve (5.3.75), which reads (for convienence)
m
dv
dt
= q (E +v B)
mv
T
(5.3.85)
Lets evaluate the cross product rst. We are told that the magnetic eld is along the z-direction,
and so
v B =

i

j z
v
x
v
y
v
z
0 0 B
z
= v
y
B
z

i v
x
B
z

j (5.3.86)
Substituting this in to (5.3.85) and breaking down the components,
m
d(v
x
+ v
y
+ v
z
)
dt
= q
_
E
x
+ E
y
+E
z
+ v
y
B
z

i v
x
B
z

j
_

m(v
x
+v
y
+ v
z
)
T
(5.3.87)
A voltage only exists along the y-direction, so lets get rid of everything not in the y-direction, and
(5.3.88) becomes
m
dv
y
dt
= q
_
E
y
v
x
B
z

j
_

mv
y
T
And now,
0 = q
_
E
y
v
x
B
z

j
_
v
x
=
E
y
B
z

j (5.3.88)
Recalling that J R = v
x
, (5.3.88) becomes
R =
E
y
JB
z

j (5.3.89)
And now, recall that
J R = v
x
, J = q v n
Combining these,
Rqv
x
n = v
x
R =
1
nq
(5.3.90)
c) The hall constant will be larger .
Problem 14
The density of states g(E) is dened as the number of electronic states per unit energy range.
CHAPTER 5: DEPARTMENTAL EXAMINATIONS 469
a) Write down an expression for the density of the electron states in metal.
b) Find an expression for the Fermi energy from a).
c) Find an expression for the fraction of the conduction electrons which are within kT of the Fermi energy.
d) Evaluate this fraction for Copper at T = 300K. [Copper is a monovalent metal with = 8.92g/cm
3
and
atomic weight M = 63.5g/mole.]
Solution
a) In order to nd an expression for the density of states for a 3D electron gas, we start with the
assumption that the metal is in the shape of a cube, each side having a length of L. Assuming
for the time being hat we have one free electron and innite potential at the cube boundaries, the
Eigenenergies are given by
E =

2

2
2mL
2
_
n
2
x
+n
2
y
+ n
2
z

(5.3.91)
For macroscopic systems, we assume that L is large enough that we can assume that the energy
spectrum is continuous. In this approximation, the total number of states (up to an energy E)
is proportional to the volume of a sphere of radius r =
_
n
2
x
+ n
2
y
+ n
2
z
1
/2
. Since we are only
interested in one quadrant, we concern ourselves with only 1/8
th
of the total volume of the sphere.
Furthermore, we also have a 2-fold degeneracy, so we must also include a factor of 2. So, we have
N =2
1
8

4
3

_
n
2
x
+n
2
y
+ n
2
z
2
/2
=

3
_
n
2
x
+ n
2
y
+ n
2
z

3
/2
(5.3.92)
From (5.3.91) we have
_
n
2
x
+ n
2
y
+n
2
z

=
2mL
2

2
E (5.3.93)
Substituting (5.3.93) into (5.3.92),
N =

3
_
2mL
2

2
E
_
3
/2
(5.3.94)
Recalling that the density of states is dened by
g(E) =
dN
dE
(5.3.95)
We then substitute (5.3.94) into (5.3.95), yielding
g(E) =
d
dE
_

3
_
2mL
2

2
E
_
3
/2
_
=
(2m)
3
/2
L
3
3
3

2
d
dE
_
E
3
/2
_
=
(2m)
3
/2
L
3
3
3

2
_
3
2
E
1
/2
_
(5.3.96)
W. Erbsen MODERN PHYSICS
Simplifying, (5.3.96) leads us to
g(E) =
(2m)
3
/2
L
3
2
3

2
E
1
/2
(5.3.97)
b) We rst recall that the number of particles in a system is given by
N =
_
EF
0
g(E) f(E) dE (5.3.98)
Where g(E) is the density of states, and f(E) is the relevant distribution function, which in our
case is the Fermi-Dirac function:
f(E) =
1
exp [(E )/k
B
T] + 1
(5.3.99)
For T = 0, this distribution gives the average number of particles in a state of energy E to be 1 for
E < and 0 for E > . This maximum occupied energy ((T = 0)) is known as the Fermi Energy.
So, letting f(E) 1, and substituting (5.3.97) into (5.3.98), we have
N =
_
EF
0
_
(2m)
3
/2
L
3
2
3

2
E
1
/2
_
dE
=
(2m)
3
/2
L
3
2
3

2
_
EF
0
E
1
/2
dE
=
(2m)
3
/2
L
3
2
3

2
_
2
3
E
3
/2
F
_
=
(3m)
3
/2
L
3
2
3

2
E
3
/2
F
(5.3.100)
Solving (5.3.100) for E
F
,
E
F
=
_
3
3

2
(2m)
3
/2
L
3
N
_2
/3
Which can be rewritten slightly nicer as
E
F
=

2
2m
_
3N
2
L
3
_
2
/3
(5.3.101)
c) In order to nd the fraction of conduction electrons within k
B
T of the conduction band, we need
to make a few assumptions. First, we assume that the density of states is constant over the entire
k
B
T band, and secondly that the Fermi-Dirac distribution is still on the order of 1 for particles in
this region. Then the number of particles within the k
B
T band is given by
N
kBT
=g(E
F
) k
B
T (5.3.102)
Substituting (5.3.97) into (5.3.102),
N
kBT
=
_
(2m)
3
/2
L
3
2
3

2
E
1
/2
_
k
B
T (5.3.103)
CHAPTER 5: DEPARTMENTAL EXAMINATIONS 471
And also from (5.3.94) we know that
N =

3
_
2mL
2

2
E
_
3
/2
(5.3.104)
Taking the ratio of (5.3.103) and (5.3.104) leads us to
N
kBT
N
=
3
2
1
E
F
k
B
T (5.3.105)
d) We now wish to evaluate (5.3.105) for Copper. The rst thing that we want to do is nd the fraction
N/V , which is:
N
V
=
8.92 mol
63.5 cm
3

_
100 cm
1 m
_
3
6.022 10
23
atoms
mol
(5.3.106)
The Fermi Energy as found in (5.3.101) is
E
F
=
_
3
3

2
(2m)
3
/2
L
3
N
_2
/3
=
_
3
3

2
(2m)
3
/2
_2
/3 _
N
V
_2
/3
(5.3.107)
Where I have set L
3
V . Substituting (5.3.107) into (5.3.105),
N
kBT
N
=
3
2
k
B
T
_
_
_
3
3

2
(2m)
3
/2
_2
/3 _
N
V
_2
/3
_
_
1
=
3
2
k
B
T
_
_
_
(2m)
3
/2
3
3

2
_
2
/3 _
V
N
_2
/3
_
_
(5.3.108)
Substituting in to (5.3.108) the appropriate values,
N
kBT
N
=
3
2
_
1.381 10
23
J K
1
_
(300 K)
_
_
_
2 9.109 10
31
Kg
_2
/3
3 1.055 10
34
J s
_
_
2
/3

_
63.5
8.92 10
3
6.022 10
23
_2
/3
Evaluating this, we are left with
N
kBT
N
= 5.5 10
3
Problem 15
A hypothetical atom has only two excited states, at 4.0 and 7.0eV, and has a ground-state ionization energy
W. Erbsen MODERN PHYSICS
of 9.0eV. If we used a vapor of such atoms from the Franck-Hertz experiment, for what voltages would we expect
to see decreases in the current? List all voltages up to 20V.
Solution
We rst recall the principles behind the Franck-Hertz experiment. This seminal experiment demonstrated
the quantization of atomic energy states by means of accelerating negatively charged electrons through a
positively charged grid, surrounded by Mercury vapor. The resulting data takes the form of an increases
oscillating signal, when plotting the net electron current vs the potential dierence of the grid.
The hypothetical atom described in the prompt has three energy levels:
V
3
=9 eV [Continuum]
V
2
=7 eV
V
1
=4 eV
The electron current will drop whenever any multiple of V
1
, V
2
, or V
3
is reached. Therefore,
1V
1
=4 eV
2V
1
=8 eV
3V
1
=12 eV
4V
1
=16 eV
5V
1
=20 eV
1V
2
=7 eV
2V
2
=14 eV
1V
3
=9 eV
2V
3
=18 eV
Also, allowing for combinations of V
1
, V
2
and V
3
,
V
1
+V
2
=11 eV
V
1
+V
3
=13 eV
V
2
+V
3
=16 eV
2V
1
+V
2
=15 eV
2V
1
+V
3
=17 eV
3V
1
+V
2
=19 eV
So, we expect the current to drop at the following values:
V = 4, 7, 8, 9, 11, 12, 13, 14, 15, 16, 17, 18, 19, and 20 eV
Problem 16
CHAPTER 5: DEPARTMENTAL EXAMINATIONS 473
Consider two protons of mass m
p
and electric charge e, separated by a distance r.
a) Compute the ratio of the electromagnetic to gravitational forces acting on the protons.
b) Given your every day experiences with bulk neutral bodies, what do you conclude from the numerical values
that you computed in a)? In particular, what force is more important for small bodies? For very large bodies?
c) Consider a large spherical body, of constant density, made up of N hydrogen atoms. Determine the critical
mass of this body at which the binding energy of the body just balances the gravitational potential energy
of teh body. Use the fact that the atom size is approximately the Bohr radius [= 1/(am
e
)] and the atomic
binding energy is twice the Rydberg energy [=a
2
m
e
/2], where a is the ne structure constant and m
e
is the
electron mass. Also assume that the mass of the hydrogen atom is equal to m
p
.
d) Determine the critical radius of the body, corresponding to the critical mass derived in c).
Solution
a) The electrostatic force between the two protons is
F
e
=
1
4
0
e
2
r
2
K
e
2
r
2
(5.3.109)
While the gravitational force acting on the bodies is
F
G
= G
M
2
p
r
2
(5.3.110)
So that if we were to nd the ratio of electromagnetic and gravitational forces acting on the protons,
we would divide (5.3.109) by (5.3.110), which leads us to
F
e
F
g
=
Ke
2
/r
2
Gm
2
p
/r
2
=
Ke
2
Gm
2
p
(5.3.111)
Plugging into (5.3.111) the appropriate values,
F
e
F
g
=
_
8.988 10
9
N m
2
C
2
_ _
1.6022 10
19
C
_
2
_
6.674 10
11
N m
2
Kg
2
_
(1.673 10
27
Kg)
2

F
e
F
g
10
36
(5.3.112)
b) Rewriting (5.3.111),
F
e
F
g
=
K
g
_
e
m
p
_
2
10
20
_
e
m
p
_
2
(5.3.113)
So, unless the mass of the particle is more than ten orders of magnitude larger than the charge, the
electrostatic force will be more prevalent.
However, in our everyday experiences, we have mostly bulk substances, being neutral in nature,
so for really large objects (stars, planets, etc), the gravitational force wins, whereas on the atomic
level (atoms, molecules), the electrostatic force still dominates.
W. Erbsen MODERN PHYSICS
c) Here we must nd the critical mass, in which the binding energy just balances the gravitational
energy. In the following calculations, we will assume that the Hydrogen atom radius is on the order
of the Bohr radius, and that the binding energy of Hydrogen is twice the Rydberg energy. We rst
recall that
=
m
V
m = V =
4
3
r
3
(5.3.114)
Taking the dierential limit of (5.3.114),
dm = 4r
2
dr (5.3.115)
We also recall that the gravitational self energy, in dierential form is
U
g
=
GM
r
dm (5.3.116)
Substituting (5.3.115) into (5.3.116) and carrying out the integration,
U
g
=
_
R
0
Gm
r
dm
=
_
R
0
G
r
_
4
3
r
3

_
_
4r
2
dr
_
=
16
2
G
3

2
_
R
0
r
4
dr
=
16
2
G
3

2
_
R
5
5
_
=
16
2
G
2
15
R
5
(5.3.117)
And the density is (recall that M = m
p
N):
=
M
4/3R
3
=
3m
p
N
4R
3
(5.3.118)
And according to the prompt, the binding energy of one Hydrogen atom is
U
b
=
_
a
2
m
1
2
_
2 (5.3.119)
For the entire mass, (5.3.119) becomes
U
b
= NA
2
m
e
(5.3.120)
Putting (5.3.118) into (5.3.117),
U
G
=
16
2
GR
5
15

9m
2
p
N
2
16
2
R
6
=
G9m
2
p
R
5
N
2
15R
=
G9m
2
p
N
2
15R
=
G3m
2
p
N
2
5R
CHAPTER 5: DEPARTMENTAL EXAMINATIONS 475
=
3
5
G
m
2
p
R
N
2
(5.3.121)
Recalling now that
U
b
= Na
2
m
e
(5.3.122)
Setting (5.3.121) and (5.3.122) equal to one another,
Na
2
m
e
=
3
5
G
m
2
p
R
N
2
m
p
N =
5
3
a
2
m
e
Gm
p
R (5.3.123)
We now recall that M = m
p
N, so that (5.3.123) becomes
M =
5
3
a
2
m
e
Gm
p
R (5.3.124)
d) The critical radius is related to the critical mass as

R = N
1
/3
R
Bohr
Applying this to (5.3.124), we are left with

R =
_
5
3
a
2
m
e
Gm
2
p
R
2
Bohr
_
1
/3
Problem 17
Use the Bohr theory to answer the following question:
a) Derive the energy levels of the hydrogen atom.
b) The orbiting speed of the electron in the ground state.
c) If an excitation laser pulse has a duration of one picosecond, what is the principal quantum number n for
which the orbiting period of the electron is about one picosecond?
d) If the electron is replaced by a negatively charged muon, what is the ground state energy of this muon? Use
these facts to get your numerical answers: the ground state of atomic hydrogen has a binding energy of 13.6
eV, the mass of the proton is 1837 times that of en electron and teh mass of a muon is 206 times that of an
electron. What is the photon energy for the n = 2 to n = 1 transition in this case?
e) Use the uncertainty principle to estimate the ground state energy of atomic hydrogen and compare the results
from the Bohr model.

Hint: volume of sphere = N volume of hydrogen.

R = N
1
/
3
r
0
W. Erbsen MODERN PHYSICS
Solution
a) The potential (energy) of a Hydrogen atom is given in its most general form by
V =
1
4
0
e
2
r
(5.3.125)
While the total energy of the system (kinetic plus potential) is
E =T +V
=
m
e
v
2
2

1
4
0
e
2
r
(5.3.126)
Assuming that we have a circular orbit, we may use the following formula for centripetal force:
F =
mv
2
r
=
1
4
0
e
2
r
2
. .
Coulomb Force
(5.3.127)
Solving (5.3.127) for v
2
,
v
2
=
1
4
0
e
2
m
e
r
(5.3.128)
And now substituting (5.3.128) back into (5.3.126),
E =
m
e
2
_
1
4
0
e
2
m
e
r
_

1
4
0
e
2
r
=
1
2
_
1
4
0
e
2
r
_

2
2
_
1
4
0
e
2
r
_
=
1
8
0
e
2
r
(5.3.129)
One of Bohrs key assumptions is that circular orbit had quantized angular momentum. Mathe-
matically,
L = mvr = n (5.3.130)
Solving (5.3.130) for r,
r =
n
m
e
v
r
2
=
n
2

2
m
2
e
v
2
(5.3.131)
Substituting our value for v
2
from (5.3.128) back into (5.3.131), we have
r
2
=
n
2

2
m
2
e
4
0
m
e
r
e
2
r =
n
2

2
4
0
m
e
e
2
(5.3.132)
Now substituting (5.3.132) into (5.3.129),
E =
1
8
0
e
2

m
e
e
2
n
2

2
4
0
(5.3.133)
CHAPTER 5: DEPARTMENTAL EXAMINATIONS 477
We can rearrange (5.3.133), which nally leads us to
E =
m
e
e
4
2 (4
0
)
2
n
2

2
(5.3.134)
b) For the ground state, n = 1, and using (5.3.128), and (5.3.132), we have
v =
_
1
4
0
e
2
m
e
m
e
e
2

2
4
0
_
1
/2
=
_
1
(4
0
)
2
e
4

2
_1
/2
=
1
4
0
e
2

(5.3.135)
Plugging in the appropriate values into (5.3.135),
v =
_
98.987 10
9
N m
2
C
2
_
_
1.6023 10
19
C
_
2
(1.055 10
34
J s)
v = 2.187 10
6
m s
1
(5.3.136)
c) By default, the orbital period is given by
T
n
=
2r
n
v
n
(5.3.137)
Substituting our previous functional value of v into (5.3.137), as well as our value for r and also
(5.3.137), we have
T
n
=2
_
n
2

2
4
0
m
e
e
2
__
4
0

n
e
2
_
=
2 (4
0
)
2
n
3

3
m
e
e
4
(5.3.138)
Solving (5.3.138) for n,
n =
_
T
n
m
e
e
4
2 (4
0
)
3

3
_1
/3
=
_
T
n
m
e
e
4
K
2
2
3
_
1
/3
(5.3.139)
Substituting in the appropriate values into (5.3.139),
n =
_
_
10
12
s
_ _
9.109 10
31
Kg
_ _
1.602 10
19
C
_
4
_
8.987 10
9
N m
2
Kg
2
_
2 (1.055 10
34
J s)
3
_
(5.3.140)
Evaluating (5.3.144), we are left with,
n = 18.73 n = 19
d) The energy of a Muon instead of an electron is found by replacing m
e
in (5.3.136) by , which is
the reduced mass of the atom, and is mathematically described by
W. Erbsen MODERN PHYSICS
=
m
p
m

m
p
+m

(5.3.141)
The relative mass of a proton to an electron is m
p
= 1838 m
e
, so (5.3.141) becomes
=
(1836 m
e
) (207 m
e
)
1836 m
e
+ 207 m
e
= 186.0 m
e
(5.3.142)
With this new value of reduced mass, we must re-evaluate our results from (5.3.134), which leads
us to
E =
m
e
e
4
2n
2

2
k
2
=
_
9.109 10
31
Kg
_ _
1.602 10
19
C
_
4
_
8.897 10
9
N m
2
Kg
2
_
2n
2
(1.055 10
34
; J s)
2
=
13.6 eV
n
2
(5.3.143)
Applying our reduced mass from (5.3.145) to (5.3.146),
E =
14.6 eVPn
2
186.0 m
e
m
e
=
13.6 eV 186.0
n
2
(5.3.144)
For the ground state, (5.3.144) becomes
E
gs
() = 2629 eV (5.3.145)
And the photon energy for this case would just be E
2
E
1
evaluated using (5.3.144), which yields
E = E
2
E
1
= 2529 eV
_
1
4
1
_
E = 1896 eV
e) From the uncertainty relation, we recall that
pr p

r
Using these results, the total energy is
E =
p
2
2m
k
e
2
r
=

2
2m(r)
2

ke
2
r
(5.3.146)
We must minimize (5.3.146) in the usual fashion,
E(r)
r
=0

2
2m

r
1
r
2
ke
2

r
1
r
=0
(5.3.147)
Rearranging things a bit, (5.3.147) becomes
CHAPTER 5: DEPARTMENTAL EXAMINATIONS 479
r =

2
2m
2
1
Ke
2
=

2
kme
2
(5.3.148)
Substituting (5.3.148) back in to (5.3.146),
E =

2
2m
_
K
2
m
2
e
4

4
_
ke
2
_
Kme
2

2
_
=
me
4
k
2
2
2

me
4
k
2

2
=
me
4
k
2

2
(5.3.149)
From (5.3.149), we can easily deduce that
E =
me
4
2
2
k
2
(5.3.150)
Which we can see is identical to (5.3.144).
Problem 18
The resistivity of a metal due to scattering of the quasi-free electrons as per the Drude model is given by
=
m
e
v
av
ne
2

(5.3.151)
a) Derive the expression above. What are the quantities n and , and explain qualitatively is inversely
proportional to them?
b) The average electron velocity v
av
is very dierent in the classical and quantum-mechanical treatments. Ex-
plain the origin of the dierence (why the quantum-mechanical velocity is much larger at room temperature,
and is nearly independent of temperature).
c) How is related to the cross-section for scattering of conduction electrons in the metal?
d) Naively, one might take to be the geometrical area of the lattice ions. A more correct treatment gives a
much smaller cross section which depends on temperature. Explain how this arises.
Solution
a) We rst note that n is the number of electrons per unit volume, while is the mean free path. The
electrons will move with a drift velocity, v
d
. In some time interval t all charge that passes through
cross sectional area A will be Q = neAv
d
t. We can nd the current by taking the derivative,
I =
dQ(t)
dt
=
d
dt
[neAv
d
t] (5.3.152)
W. Erbsen MODERN PHYSICS
If we assume that the time interval is appropriately constant, (5.3.152) becomes
I = neAv
d
(5.3.153)
At this point, we must recall a number of seemingly arbitrary denitions. We of course recall Ohms
law as V = IR, and the resistivity is given as
R =
L
A
(5.3.154)
We also recall that the electric eld within the material is given by
E =
V
L
V = EL (5.3.155)
Substituting (5.3.155) into Ohms Law, we have
EL = IR (5.3.156)
We now substitute our expression for resistivity from (5.3.154) into (5.3.156),
EL = I
_
L
A
_
I =
EA

(5.3.157)
We now set (5.3.153) equal to (5.3.157),
neAv
d
=
EA

=
E
nev
d
(5.3.158)
In the same mantra as before, we recall that Newtons second law says F = ma, F = eE, and
v = a, so we can solve for v and arrive at
v
d
=
eE
m
(5.3.159)
Putting (5.3.159) back into (5.3.158),
=
E
ne

m
eE
=
m
ne
2

(5.3.160)
But hold on there jack, is really the average time the particle goes without a collision, and we
want our expression in terms of the mean free path. The formula which connects these two entities
is simple: = /v
avg
. Substituting this back into (5.3.160),
=
m
ne
2

v
avg

=
mv
avg
ne
2

(5.3.161)
b) The average electron velocity, v
avg
is a very dierent animal when investigated in either the classical
or quantum-mechanical regime. Classically,
T =
mv
2
2
=
3
2
k
B
T (5.3.162)
Playing with (5.3.162) a little bit,
v =
_
3k
B
T
m
(5.3.163)
Quantum-mechanically, only electrons that have energies within the k
B
T band contribute, so the
average energy is the Fermi energy: E
avg
= E
F
:
CHAPTER 5: DEPARTMENTAL EXAMINATIONS 481
T = E
F
=
mv
2
2
(5.3.164)
Playing some more,
v
avg
=
_
2E
F
m
(5.3.165)
Where we note that E
F
does not depend on T, so v
avg
does not depend on it either. Freaky! A
qualitative summary,
E
f
= k
B
T
F
k
B
T
room
v
avg
(Quantum) v
avg
(Classical)
c) To answer this we need only consider two elements. Firstly, we recognize that an electron entering
on the left of the cylinder will be scattered before it reaches the right side of the same cylinder. We
know that the volume of the cylinder is , and that the density of scatterers is n = 1/, so, our
answer is then
=
1
n
d) The area swept out by the atomic vibrations perpendicular to the direction of propagating electrons
is given by the cross section, . Increasing the temperature, T increases the vibrations. Therefore,
as they both increase without limit, we have no choice but to say that , T at the same rate.
Problem 19
The band theory of solids can be used to explain how a pure, intrinsic semiconductor (such as germanium)
can become a p-type semiconductor.
a) How do conduction bands arise in a solid?
b) Sketch an energy level diagram for electrons in a pure germanium crystal. Indicate both the conduction and
valence bands and give order of magnitude estimates of the energies in your diagram.
Solution
a) This is a tricky question to answer, in the sense that the full explanation is a science in and of
itself. Irregardless, we start with the idealized scenario of a repeating series of potentials (a Dirac
Comb) that has a xed height (V
0
) and each barrier is a xed distance from one another a. The
Barrier itself has a width, which we call b. These series of potentials go on forever, as to neglect
any edging eects.
At this point, we introduce our old friend the TISE in 1-D:
_


2
2m

x
+V (x)
_
(x) = E(x) (5.3.166)
Whose realizable solutions end up begin
W. Erbsen MODERN PHYSICS
(x) = Ae
ikx
, k =
_
2m(E V (x))

(5.3.167)
Our periodicity boundary condition also satises the TISE, but the solution will be shifted by some
phase factor:
(x +N(a + b)) = e
ik(a+b)
(x) (5.3.168)
Where (5.3.168) is lovingly referred to Blochs Theorem. Solving the TISE for repeating lattices
using boundary conditions (eventually yields a transcidental equation, which can be used to visually
determine the bands and band-gaps within a particular solid.
The size, length, and location of the bands and bandgaps depends on the distance between the
potentials, a, the width of the potentials, b, and the height of the potentials, V
0
. From these three
variables, we can plot the energy versus the wave number, and depending on the combination of
the supplied variables, the plots may indicate a conductor, insulator, or a semiconductor.
For a conductor, the Fermi Energy lies within a band and accordingly, the valence bands and
conduction bands overlap. This implies that at T = 0, some of the electrons will be free to
conduct.
For an insulator, the Fermi energy lies either at the beginning, or the end of the gap (or
somewhere inbetween). Subsequently, at T = 0 all orbitals below E
F
are lled, leaving no free
electrons to conduct.
For the case of a semi-conductor, the gaps are small enough for some electrons to jump across
and become free. These particular characteristics can be controlled by way of introducing impurities
to the pure semiconductor (known as doping).
b) I cant sketch here, but it should be fairly straightforward. Anyway, as far as an order of magnitude
estimate for the gap spacing between the valence and conduction bands in a semi-conductor, the only
way I can think to solve this problem is by experience, which tells me that the gap should be 1 eV
Problem 20
When sodium metal is illuminated with light of wavelength 4.20 10
2
nm, the stopping potential is found to
be 0.64 V; when the wavelength is changed to 3.10 10
2
nm, the stopping potential becomes 1.69 V. Using only
this data and the values of the speed of light and electronic charge, nd the work function of sodium and the value
of Plancks constant.
Solution
We rst recall that the key equation for the photoelectric eect is
E
max
= hf (5.3.169)
The stopping potential is the potential that is required to stop an electron with some maximum kinetic
energy E
max
. Since the potential energy created by a potential dierence V is U = qV , (5.3.169) becomes
qV =hf
CHAPTER 5: DEPARTMENTAL EXAMINATIONS 483
=
hc

(5.3.170)
Given that = 4.20 10
2
nm, and also that the stopping potential is V 0.65 V, (5.3.170) becomes
qV
1
=
hc

_
V
1
= 0.65 V

1
= 420 10
9
m
(5.3.171)
And similarly, for = 3.10 10
2
nm and V = 1.69 V, we have
qV
2
=
hc

_
V
2
= 1.69 V

2
= 310 10
9
m
(5.3.172)
Solving (5.3.171) and (5.3.172) for and setting them equal to one another leads us to
hc

1
qV
1
=
hc

2
qV
2
hc
_
1

2
_
=q (V
1
V
2
)
h =
q
c
(V
1
V
2
)
_
1

2
_
1
(5.3.173)
Substituting in the correct values into (5.3.173) leads us to
h =
1.602 10
19
C
2.998 10
8
m s
1
(0.65 V 1.69 V)
_
1
420 10
9
m

1
310 10
9
m
_
1
h = 6.578 10
34
J s (5.3.174)
Now, using either (5.3.171) or (5.3.172), we can calculate the work function . Using (5.3.172),
=
hc

1
qV
1
Substituting in the appropriate values,
=
_
6.578 10
34
J s
_ _
2.998 10
8
m s
1
_
(420 10
9
m)

= 3.661 10
19
J
= 2.285 eV
Problem 21
Two identical photons are produced when a proton and an antiproton, both at rest, annihilate each other. What
are the frequencies and corresponding wavelengths of the photons? Is it possible to produce a single photon in
proton-antiproton annihilation? Why or why not?
Solution
W. Erbsen MODERN PHYSICS
We rst recall the following energy/momentum relations:
E
2
p
=m
2
c
4
+p
2
c
2
(5.3.175a)
E
2
p
=m
2
c
4
(5.3.175b)
We also recall that E
p
= mc
2
= 938.3 MeV. Now, recall from Einsteins relation that
E

= hf f =
E

h
=
938.3 10
6
eV
4.136 10
15
eV s
=2.296 10
23
Hz (5.3.176)
Now, we can say that
=
c
f
=
3 10
8
m s
1
2.269 10
23
Hz
= 1.322 10
15
m
It is not possible to produce a single photon in this particular interaction, since it would violate con-
servation of momentum (recall that the protons start o at rest )
Problem 22
A dust particle at rest relative to the sun is in equilibrium between the radiation pressure and gravitational
attraction. What is the size of the dust particle? (For simplicity, you can assume that the dust particle in
spherical)
Solution
We rst recall that the radiation pressure is dened by
P
rad
=
S)
c
(5.3.177)
Where S) is the time-averaged Poynting vector, aka the intensity. The power emitted by the sun is
around P 3.8 10
26
W, and so the intensity is
I = S) =
3.8 10
26
W
4r
2
(5.3.178)
Applying (5.3.178) to (5.3.177),
P
rad
=
3.8 10
26
W
4r
2
c
(5.3.179)
Since pressure is dened as P = F/A, then (5.3.179) becomes
CHAPTER 5: DEPARTMENTAL EXAMINATIONS 485
F
rad
=
_
3.8 10
26
W
_ _
4R
2
_
4r
2
=
_
3.8 10
26
W
_
r
2
R
2
(5.3.180)
Now, if we assume that the dust particles are perfectly reective, then (5.3.180) becomes
F
rad
=2
_
3.8 10
26
W
_
r
2
R
2
(5.3.181)
And we also recall that the gravitational force is given by
F
grav
= G
mM
r
2
(5.3.182)
Equating (5.3.181) and (5.3.182),
2
_
3.8 10
26
W
_
r
2
R
2
= G
mM
r
2
Solving for R, this becomes
R =

GmM
2 (3.8 10
26
W)
(5.3.183)
Problem 23
What is the density of a neutron star? Evaluate the size (radius) of a neutron star as massive as the sun,
given that the radius
197
79
Au nucleus is 7 10
14
m, mass of the sun is 2 10
30
kg.
Solution
Density is of course given by = m/v, and given the supplied constants, we have
=
197 1.67 10
27
Kg
4/3 (7 10
14
m)
3
= 2.194 10
14
Kg m
3
(5.3.184)
To nd the radius of the neutron star, we recall that from the density, we can say that
=
M
4/3R
3
R =
_
3M
4
_1
/3
(5.3.185)
Plugging into (5.3.185) the appropriate values,
R =
_
(3)
_
2 10
30
Kg
_
4 (2.194 10
14
Kg m
3
)
_1
/3
R = 129589 m
W. Erbsen MODERN PHYSICS
Problem 24
A typical particle production process is
p + p p +p +
0
where m
p
= 938MeV/c
2
, mass of
0
= 135MeV/c
2
.
a) In a standard accelerator one of the initial protons would be at rest. In this case what is the minimum
laboratory energy for the moving particle so that the reaction listed above occurs?
b) In colliding beam accelerators, such as the one proposed for the superconducting supercollider, the two initial
particles are moving towards each other with equal speeds. Why do colliding beam accelerators have an
energetic advantage over the type of accelerator from a)?
c) What energy would be required of each proton in a colliding beams accelerator to produce a pion?
Solution
We rst recall the following:
E
2
=p
2
c
2
+
_
m
0
c
2
_
2
(5.3.186a)
E =mc
2
(5.3.186b)
And that the invariant quantity is
_
m
0
c
2
_
2
. .
Lab frame invariant before collision
=
Center of mass frame invariant after collision
..
E
2
p
2
c
2
(5.3.187)
Where in the center of mass frame (COM) the momentum is zero, so (5.3.187) becomes
_
m
0
c
2
_
2
= E
2
(5.3.188)
Lets make a table:
Before After

m 2m
p
2m
p
+ m

(5.3.189)
a) If we choose not to use the COM frame, then the momentum is conserved (lets do it this way). We
rst apply conservation of energy to our system:
E
i
= E
f
(5.3.190)
Squaring (5.3.190) and substituting in (5.3.186a),
p
2
i
c
2
+m
2
i
c
4
= p
2
f
c
2
+ m
2
f
c
4
(5.3.191)
Conservation of momentum says that p
i
= p
f
, and so (5.3.191) becomes
m
2
i
c
4
=m
2
f
c
4
CHAPTER 5: DEPARTMENTAL EXAMINATIONS 487
_
2m
p
c
2
_
2
=
_
(2m
p
+m

) c
2

2
4m
2
p
c
4
=4m
2
p
c
4
+ 2m
p
m

c
4
+ 2m
p
m

c
4
+ m
2

c
4
0 =4m
p
m

c
4
+m
2

c
4
0 =m

c
2
_
m

c
2
2m
p
c
2
+ 2
_
0 =m

c
2
_
m

2m
p
+ 2
_
(5.3.192)
We can now say that this is the threshold energy, which is the minimum possible energy that will
allow this reaction to happen. The RHS of (5.3.192) is this so-called threshold energy:
E
T
= m

c
2
_
m

c
2
2m
p
c
2
+ 2
_
Substituting in the appropriate values, we have
E
T
= 135 MeV
_
135 MeV
2 938 MeV
+ 1
_
E
T
= 279.7 MeV (5.3.193)
And to nd the total energy, we combine the results of (5.3.193) plus the rest mass energy of the
proton:
E
tot
= 279.7 MeV + 938 MeV E
tot
= 1217.7 MeV
b) Since in the case of a), all the energy needed to create the
o
is carried by the rst proton, however
if both were to move towards one another, then the energy would need to be much less in order to
reach the threshold energy.
c) If they are moving towards one another, we use the same logic as before - we want the threshold
energy, that is the energy that is just barely enough to complete the reaction. In this case, we care
only about the creation of three particles: two Protons, and one Pion. Therefore, each incident
proton must have an initial energy equal to
E
p
=
1
2
(2m
p
+ m

)
Plugging in the appropriate number,
E
p
=
1
2
[2 (938 MeV) + 135 MeV] E
p
= 1005.5 MeV
Problem 25
In a muonic hydrogen atom, the electron is replaced by a muon, which is 207 times heavier.
a) What is the wavelength of the n = 2 to n = 1 transition in Muonic hydrogen? In what region of the
electromagnetic spectrum does this wavelength belong?
b) How would the ne structure and Zeeman eects dier from those in ordinary atomic hydrogen?
W. Erbsen MODERN PHYSICS
Solution
a) We rst recall the Rydberg formula, which reads
1

= R
_
1
n
2
f

1
n
2
i
_
(5.3.194)
While the Rydberg constant is given by
R =
m
e
4c
3
_
e
2
4
0
_
2
= 1.097 10
7
m
1
(5.3.195)
R C m
e
Where m
e
is the electron mass. For Muonic Hydrogen, we replace m
e
with /m
e
, where is the
reduced mass, where
m

= 207 m
e
, m
p
= 1836 m
e
So, the reduced mass is
=
(207 m
e
) (1836 m
e
)
(207 m
e
) + (1836 m
e
)
= 186.0 m
e
(5.3.196)
Substituting (5.3.196) into (5.3.194),
1

= R

m
e
_
1
n
2
f

1
n
2
i
_
(5.3.197)
Substituting in the appropriate values, (5.3.197) becomes
1

=
_
1.097 10
7
m
1
_
_
186.0
m
e
m
e
__
1
4
1
_
=1.530 10
9
m
1
(5.3.198)
From which it is easy to see that
= 6.535 10
10
m
Which are x-rays.
b) The only appreciable dierence between Hydrogen and Muonic Hydrogen is the dierence in the
masses of the electron and the Muon, respectively. Therefore, we must nd the splitting as a func-
tion of mass for both ne structure and Zeeman eects.
The ne structure is of order:
E
fs

4
me
2
E
fs
m
While the Zeeman eect is of order:
E
z
m
e

e
2m
B E
z

1
m
CHAPTER 5: DEPARTMENTAL EXAMINATIONS 489
Problem 26
For an intrinsic semiconductor, the number of electrons per unit volume in the conduction band and the number
of holes in the valence band are given by
n =N
C
exp
_

(E
C
E
F
)
k
B
T
_
(5.3.199a)
p =N
V
exp
_

(E
F
E
V
)
k
B
T
_
(5.3.199b)
Where
N
C
= N
V
=
2(2mk
B
T)
3
/2
h
3
(5.3.200)
a) Find the position of the Fermi energy in an intrinsic semiconductor.
b) Show that the product of the number of electrons in the conduction band and the number of holes in the
valence band depends only on temperature and the gap energy, E
g
.
c) Discuss how one can use the temperature (T) dependence of the conductivity to approximately measure
the gap energy provided that T is not too high.
Solution
We rst recall that the Fermi Energy is dened as the highest occupied state at T = 0 K. We also recall
that the Intrinsic Fermi Energy (E
i
) is dened as the Fermi Energy in the absence of doping.
a) In an intrinsic semiconductor, the number of electrons is equal to the number of holes (n = p), since
the thermal excitation of an electron leaves behind a hole in the valence band.
The very rst thing that we wish to do is multiply (5.3.199a) and (5.3.199b):
np = N
c
N
v
exp
_

(E
c
E
v
)
k
B
T
_
(5.3.201)
And furthermore, from (5.3.200), we may deduce that
N
C
= 2
_
2m
e
k
B
T
h
2
_3
/2
(5.3.202a)
N
V
= 2
_
2m
h
k
B
T
h
2
_3
/2
(5.3.202b)
Where m
h
is the electron eective mass near the top of the valence band. Using (5.3.202a)-
(5.3.202b), we can now rewrite (5.3.199a) and (5.3.199b), respectively:
n =2
_
2m
e
k
B
T
h
2
_3
/2
exp
_

(E
c
E
F
)
k
B
T
_
(5.3.203a)
p =2
_
2m
h
k
B
T
h
2
_3
/2
exp
_

(E
F
E
v
)
k
B
T
_
(5.3.203b)
Multiplying (5.3.203a) and (5.3.203b) leads us to
W. Erbsen MODERN PHYSICS
np = 4
_
2k
B
T
h
2
_
3
(m
e
m
h
)
3
/2
exp
_

(E
c
E
v
)
k
B
T
_
(5.3.204)
The intrinsic carrier concentration depends exponentially on (E
c
E
V
) /2k
B
T.
At the Fermi Energy, n = p because the number of electrons and the number of holes are created
in pairs for intrinsic semiconductors, as is what we have here. So,
N
c
exp
_

(E
c
E
f
)
k
B
T
_
=N
v
exp
_

(E
F
E
v
)
k
B
T
_
Since N
c
and N
v
are equal in this case, this becomes
(E
c
E
f
)
k
B
T
=
(E
F
E
v
)
k
B
T
E
c
E
f
= E
F
E
v
From which it is easy to gather that
E
F
=
E
v
+E
c
2
In other words, the Fermi Energy is located exactly inbetween the valence band and the conduction
band.
b) From the results of the previous problem, we found that
np = 4
_
2k
B
T
h
2
_
3
(m
h
m
e
)
3
/2
exp
_

E
g
k
B
T
_
(5.3.205)
Where E
g
= E
c
E
v
is the gap energy.
c) We recall from Ohms law that
=
j
E
, V
d
=
j
nq
Combining these, we nd that
=
nqv
E
(5.3.206)
Also recalling that F = qE, (5.3.206) becomes
=
nq
2
v
F
(5.3.207)
Now, we recall that v = a, where is the time between collisions. Combining this with Newtons
second law, F = ma, we have F = vm/, and (5.3.207) becomes
=
nq
2

m
(5.3.208)
Recalling from before that
=
c
+
v
CHAPTER 5: DEPARTMENTAL EXAMINATIONS 491
=
q
2

m
_
2
_
2m
e
k
B
T
h
2
_3
/2
exp
_

(E
c
E
F
)
k
B
T
_
+ 2
_
2m
h
k
B
T
h
2
_3
/2
exp
_

(E
F
E
v
)
k
B
T
_
_
From this we can easily say that
C exp
_

(E
v
+E
c
)
k
B
T
_
So, as long as the temperature is low, the gap will dominate this equation, and we can solve for the
gap as needed.
Problem 27
The potential energy between two atoms in a molecule can often be described rather well by the Lennard-Jones
potential, which can be written
U(r) = U
0
_
_
a
r
_
12
2
_
a
r
_
6
_
(5.3.209)
Where U
0
and a are constants.
a) Find the interatomic separation r
0
in terms of a for which the potential energy is minimum.
b) Find the corresponding value of U
min
.
c) Use the gure below to obtain numerical values for r
0
and U
0
for the H
2
molecule. Express your answer in
nanometers and electron volts.
d) Make a plot of the potential energy U(r) versus the internuclear separation r for the H
2
molecules. Plot each
term separately, together with the total U(r).
Solution
a) To nd the value of the interatomic separation r
0
when the potential energy is at a minimum, we
must minimize (5.3.209). The general prescription for minimizing any function is to take the deriva-
tive with respect to the minimization variable and set it equal to zero, solving for the appropriate
minimized value. In our case,
U(r)
r

r=r0
= 0 (5.3.210)
So, applying (5.3.210) to (5.3.209), we have

r
_
_
a
r
_
12
2
_
a
r
_
6
_
= 0
a
12
_

r
1
r
12
_
2a
6
_

r
1
r
6
_
= 0
W. Erbsen MODERN PHYSICS
a
12
_

12
r
13
_
2a
6
_

6
r
7
_
= 0
a
12
12
r
13
= 12
a
6
r
6
a
6
=r
6
(5.3.211)
Applying r = r
0
at (5.3.211), we have
r
0
= a (5.3.212)
b) The corresponding value of U
min
can be found by applying (5.3.212) to (5.3.209):
U(r
0
) =U
0
_
_
a
r
0
_
12
2
_
a
r
0
_
6
_
=U
0
_
_
a
a
_
12
2
_
a
a
_
6
_
(5.3.213)
Which leads us to
U
min
= U
0
(5.3.214)
c) The values for r
0
and U
0
can be obtained quite simply from the gure. The value of r
0
is the
horizontal position of the deepest point in the potential, while the minimal energy is simply the
depth from the bottom:
CHAPTER 5: DEPARTMENTAL EXAMINATIONS 493
r
0
= 0.074 nm
U
min
= [27.2 eV 4.5eV[ U
min
= 31.7 eV
Problem 28
The measured conductivity of copper at room temperature is 5.88 10
7

1
m
1
, its Fermi energy is 7.03eV,
density is 8.96g m/cm
3
and molar mass is 63.5gm/mole.
a) Calculate the density of free electrons in copper.
b) Calculate the average time between collisions of the conduction electrons.
c) Calculate the mean free path of electrons in copper.
d) Calculate the smallest possible deBroglie wavelength of electrons in copper at T = 0K. How does this value
compare with the atomic separation in copper?
Solution
a) Copper has only one free (valence) electron. According to the Free Electron Approximation, all the
conductivity is due to this one electron per copper atom. So, if we wanted to calculate the density
of free electrons in copper, we recall that = M/V , and since we have only one free electron per
atom, we nd the density as
n =
_

Cu
m
_
N
A
=8.96
g
cm
3

_
63.5
g
mol
_
1
6.022 10
23
atoms
mol
=8.96
g
cm
3

_
1
63.5

mol
g
_
6.022 10
23
atoms
mol
(5.3.215)
From (5.3.215), it is easy to see that we are left simply with
n = 8.497 10
22
atoms
cm
3
(5.3.216)
b) In order to calculate the average time between collisions of the conduction electrons, we rst must
consider the kinetic energy of these electrons, and from that nd the average velocity:
E =
mv
2
2
v =
_
2E
m
(5.3.217)
Where we take E in (5.3.217) to be the Fermi Energy, E
F
, which is given in the prompt. Accordingly,
we can evaluate v in (5.3.217) as
v =

2 (7.03 eV)
(0.511 10
6
eV)
(3 10
8
m/s)
2
v = 1.574 10
6
m
s
(5.3.218)
W. Erbsen MODERN PHYSICS
We now recall that the conductivity is dened as
=
ne
2
d
mv
(5.3.219)
Where we recognize that the relationship between velocity, the time between collisions and the
distance traveled between collisions is v = d/ d/v = . Substituting this into (5.3.219),
=
ne
2
m

_
d
v
_
=
ne
2
m
(5.3.220)
Solving (5.3.220) for yeilds
=
m
ne
2
=
_
9.109 10
31
Kg
_ _
5.880 10
7

1
m
1
_
(8.497 10
22
atoms cm
3
) (100 cm m
1
)
3
(1.602 10
19
C)
2
(5.3.221)
Evaluating all the goodness in (5.3.221) leaves us with
= 2.462 10
14
s (5.3.222)
c) The mean free path is found from the velocity found in part b) as
d =v
=
_
1.574 10
6
m s
1
_ _
2.462 10
14
s
_
d = 38.75 nm (5.3.223)
d) The deBroglie wavelength can be found rather straightforwardly using (5.3.218) as
=
h
p
=
6.626 10
34
J s
(9.109 10
31
Kg) (1.574 10
6
m s
1
)
(5.3.224)
From (5.3.224) we see that
= 4.621 10
10
m (5.3.225)
The atomic separation of copper can be found from the density, , which is given in the problem:
=
m
v
=
m
L
3
L =
_
m

_1
/3
(5.3.226)
Substituting in the appropriate values into (5.3.226) yields
L =
_
_
63.5 g mol
1
_ _
6.022 10
22
atoms mol
1
_
(8.96 g cm
3
) (100 cm m
1
)
3
_
L = 2.275 10
10
m
Problem 29
The proper mean lifetime of Mesons (Pions) is 2.6 10
8
s. If a beam of such particles has a speed of 0.9 c:
CHAPTER 5: DEPARTMENTAL EXAMINATIONS 495
a) What would their mean life be as measured in the laboratory?
b) How far would they travel (on the average) before they decay?
c) What would your answer be to part b) if you neglect time dilation?
d) What is the interval in space-time between creation of a typical pion and its decay?
Solution
We rst recall that the equation for time-dilation is
T = T
0
(5.3.227)
Where the Lorentz factor, , is equal to
=
1
_
1 v
2
/c
2
(5.3.228)
a) In the lab frame, using (5.3.227)-(5.3.228) we have
T =
2.6 10
8
s
_
1 (0.9)
2
T = 5.965 10
8
s (5.3.229)
b) In the lab frame, we know that x = v T, and using (5.3.229), we have
x = (0.9)
_
3 10
8
m s
1
_ _
5.965 10
8
s
_
x = 16.105 m (5.3.230)
c) If time-dilation is neglected, we can nd x as
x = (0.9)
_
3 10
8
m s
1
_ _
2.6 10
8
s
_
x = 7.020 m
d) The Space-time interval is dened as
s
2
= x
2
+c
2
T
2
(5.3.231)
Substituting in the appropriate values into (5.3.231),
s
2
=
_
3 10
8
m s
1
_ _
5.965 10
8
s
_
2
(16.105 m)
2
s
2
= 60.86 m
2
So, the space-time interval is
s =

60.86 m
2
s = 7.801 m
Problem 30
Four identical, non-interacting particles are placed in a one dimensional box of length L. Find the lowest to-
tal energy of the system and list the quantum numbers of all occupied states (including m
s
) if:
a) The particles are electrons
W. Erbsen MODERN PHYSICS
b) The particles are neutral Pions,
0
.
c) Two of the particles are protons and the other two are neutrons.
Solution
We rst recall that the 1-D, time independent SE reads

2
2m

2
x
2
(x) = E(x) (5.3.232)
Solving (5.3.232) with the appropriate boundary conditions, we nd that the corresponding Eigenfunction
and Eigenvalues are, respectively
(x) =
_
2
L
sin
_
nx
L
_
(5.3.233a)
E
n
=
n
2

2
2mL
2
(5.3.233b)
a) Since electrons are Fermions, no two electrons can be described by the same set of quantum numbers,
as necessitated by the Pauli Exclusion Principle. Therefore, the lowest occupied states are:
[n, m
s
) = [1,
1
/
2
), [1,
1
/
2
), [2,
1
/
2
), [2,
1
/
2
)
And from (5.3.233b), the lowest energy corresponding to this state is
E
gs
=

2

2
2mL
2
[1 + 1 + 4 + 4] E
gs
=
5
2

2
m
e
L
2
b) The neutral pion has a spin of zero, making it a Boson. Since the Pauli Exclusion principle does
not apply to Bosons, all the particles may lie in the lowest state:
[n, m
s
) = [1, 0), [1, 0), [1, 0), [1, 0)
While the energy is, following (5.3.233b),
E
gs
=

2

2
2mL
2
[1 + 1 + 1 + 1] E
gs
=
2
2

2
m

L
2
c) Both Protons and Neutrons are Fermions, but they are distinguishable from one another. Therefore,
the lowest state is then
[n, m
s
) = [1,
1
/
2
), [1,
1
/
2
), [1,
1
/
2
), [1,
1
/
2
)
And the Eigenenergies are
E
gs
=

2

2
2m
p
L
2
[1 + 1] +

2

2
2m
n
L
2
[1 + 1] E
gs
=

2

2
L
2
_
1
m
p
+
1
m
n
_
CHAPTER 5: DEPARTMENTAL EXAMINATIONS 497
5.4 Statistical Mechanics
Problem 1
a) Dene the chemical potential . Show that two systems are in diusive equilibrium if
1
=
2
. You may
start with F = F
1
+ F
2
(free energy) and use the fact that
1
=
2
should minimize F.
b) Now consider a 1D gas of length L and number of distinguishable and non-interacting particles N. Assume
that temperature T is high enough that we are in the classical regime. Find the chemical potential in terms
of T and other parameters of the problem.
c) Now nd the free energy F and the specic heat at constant volume C
V
.
Solution
a) The chemical potential, , can be thought of as a number that allows us to quantify whether or not
two (or more) systems are in diusive equilibrium.
If they are equal, then there will be no (net) ux of particles moving from one system to another.
If they are not equal, then particles will tend to ow from a region of high chemical potential to a
region of low chemical potential.
We also recall that the Helmholtz Free Energy (F) measures the amount of useful work that
is extractable from a system at constant temperature and volume. The Free Energy has several
important characteristics:
i) At equilibrium, the Free Energy is at a minimum
ii) An important form of the second law of thermodynamics is
U = F + TS
Where U is the internal energy, and S is the nal entropy.
At diusive equilibrium, we know that dF = 0, and the number of particles in each system remains
constant. From this, we may also deduce that the total number of particles remains constant.
Quantitatively,
N = N
1
+N
2
In fact, whether we are in diusive equilibrium or not, the total number of particles must remain
constant! That is,
0 = dN
1
+ dN
2
dN
1
= dN
2
(5.4.1)
We can also think of the free energy of each system, as we did with the particle number previously.
The total free energy would be described by
F = F
1
+ F
2
(5.4.2)
And when we are in diusive equilibrium, according to our rst stipulation, F must be minimized
with respect to the particle number:
W. Erbsen STATISTICAL MECHANICS
dF
dN
= 0 (5.4.3)
Combining (5.4.1) and (5.4.2),
dF =
F
1
N
1
dN
1
+
F
2
N
2
dN
2
(5.4.4)
Setting (5.4.4) equal to zero, as demonstrated in (5.4.3), we have
F
1
N
1
dN
1
=
F
2
N
2
dN
2
(5.4.5)
Applying (5.4.1) once again to (5.4.5), we are left with
F
1
N
1
dN
1
=
F
2
N
2
(dN
1
)
F
1
N
1
=
F
2
N
2
(5.4.6)
We now recall that the chemical potential is dened by
=
_
F
N
_
V,T
(5.4.7)
Applying the denition of chemical potential from (5.4.7) to our result (5.4.6), it is easy to see that
at equilibrium,

1
=
2
b) We are tasked with nding the chemical potential for distinguishable, non-interacting particles in
the classical regime for a 1-D gas. As we do in all of statistical mechanics, we begin by nding the
(single particle) partition function:
Z
1
=

i
exp
_

E
i
k
B
T
_
(5.4.8)
For a classical gas, the energy will come in the form of the kinetic energy, which is E = p
2
/(2m).
Furthermore, within the classical regime, we may take our energy distribution to be continuous,
and as such may be integrated in the following way:
Z
1
=
1
h
3
_
exp
_

p
2
2mk
B
T
_
d
3
p
_
d
3
x (5.4.9)
For a 1-D gas, as is the case here, (5.4.9) becomes
Z
1
=
1
h
_

exp
_

p
2
2mk
B
T
_
dp
_
L
0
dx
=
1
h

_
_
2mk
B
T
_
[L]
=
L
h
_
2mk
B
T (5.4.10)
Within the classical approximation, the full partition function can be approximated by
Z
N

= Z
N
1
Applying this to the single particle partition function of (5.4.10), we have
CHAPTER 5: DEPARTMENTAL EXAMINATIONS 499
Z
N
=
_
L
h
_
2mk
B
T
_
N
(5.4.11)
We now recall that the Free energy takes the form
F = k
B
T log [Z
N
] (5.4.12)
Substituting (5.4.11) into (5.4.12),
F = k
B
T log
_
_
L
h
_
2mk
B
T
_
N
_
= Nk
B
T log
_
L
h
_
2mk
B
T
_
(5.4.13)
Now, we recall that the chemical potential, given by (5.4.7), can be calculated from (5.4.13) as
=

N
_
Nk
B
T log
_
L
h
_
2mk
B
T
__
(5.4.14)
The answer is then
= k
B
T log
_
L
h
_
2mk
B
T
_
c) From (5.4.13), F was found to be
F = Nk
B
T log
_
L
h
_
2mk
B
T
_
To nd the specic heat capacity at constant volume, we have to get back to work. Recall that this
quantity is dened as
C
V
=
_
U
T
_
V
(5.4.15)
We also recall that the internal energy is given by
U =

log (Z
N
) (5.4.16)
Now substituting (5.4.11) into (5.4.16),
U =

log
_
_
L
h
_
2mk
B
T
_
N
_
= N

log
_
L
h
_
2mk
B
T
_
= N

log
_

2mL
2
h
2
_
= N

log
_
_
2mL
2
h
2
_
1
/2
_
W. Erbsen STATISTICAL MECHANICS
=
N
2

log
_
2mL
2
h
2
_
=
N
2

log
_
2mL
2
h
2
. .
Garbage
1

_
(5.4.17)
The derivative of (5.4.17) comes out being very simple,
U =
N
2

1

=
Nk
B
T
2
(5.4.18)
Putting our cleaned up expression back into (5.4.15), we nd C
V
:
C
V
=

T
_
Nk
B
T
2
_
C
V
=
Nk
B
2
Problem 2
Consider a particle in a 1-dimensional box of side L with available energy levels (in the presence of a magnetic
eld B along z)

n
=
2
n
2
+m B;
2
=

2
2m
_

L
_
2
(5.4.19)
Where n is an integer greater than zero and m = m
z

k.
a) What is the partition function for this particle?
b) What is the partition function for N classical, identical, noninteracting particles in the same box in the
presence of a magnetic eld? Hence nd an expression for the internal energy U, and the Helmholtz free
energy F.
c) What is the equilibrium pressure and magnetization of this gas?
Solution
From the information supplied,
E
n
=
2
n
2
+ m B

2
=

2
2m
_

L
_
2
_

_
E
n
=
n
2

2
2mL
2
+ m B (5.4.20)
a) The rst step in this problem, as is in most problems in this eld, is to nd the partition function.
Lets ride this pony:
Z
1
=

n
exp
_

E
n
k
B
T
_
=

n
exp [E
n
] (5.4.21)
CHAPTER 5: DEPARTMENTAL EXAMINATIONS 501
Substituting (5.4.20) into (5.4.21),
Z
1
=

n
exp
_

_
n
2

2
2mL
2
+ m B
_

_
=

n
exp
_

n
2

2
2mL
2
m B
_
=

n
exp
_

n
2

2
2mL
2
m
z
B
0

_
(5.4.22)
I changed m B m
z
B
0
. Because of the in front of the spin quantum number, we need to split
(5.4.22) into two distinct sums, each addressing its respective parity. With this, (5.4.22) is now
Z
1
=

n
_
exp
_

n
2

2
2mL
2
m
z
B
0

_
+ exp
_

n
2

2
2mL
2
+m
z
B
0

__
(5.4.23)
At this point, we turn to our calculus loving friends and with a tote on their pipe I decide to go the
way of the integral. And it was so, the sum was transferred into an integral, and (5.4.23) becomes
Z
1
=
_

0
_
exp
_

n
2

2
2mL
2
m
z
B
0

_
+ exp
_

n
2

2
2mL
2
+m
z
B
0

__
dn
=
_

0
_
exp
_

n
2

2
2mL
2

_
exp [m
z
B
0
] + exp
_

n
2

2
2mL
2

_
exp [m
z
B
0
]
_
dn
=exp [m
z
B
0
]
_

0
exp
_

n
2

2
2mL
2

_
dn + exp [m
z
B
0
]
_

0
exp
_

n
2

2
2mL
2

_
dn
=exp [m
z
B
0
]
_

0
exp
_

2
2mL
2
n
2
_
dn + exp [m
z
B
0
]
_

0
exp
_

2
2mL
2
n
2
_
dn
=exp [m
z
B
0
] + exp [m
z
B
0
]
_

0
exp
_

2
2mL
2
n
2
_
dn
=2 cosh [m
z
B
0
]
_

0
exp
_

2
2mL
2
n
2
_
dn
=2 cosh [m
z
B
0
]
_
1
2

2mL
2

2
_
(5.4.24)
From (5.4.24), it is easy to see that the partition function becomes
Z
1
= cosh [m
z
B
0
]

2mL
2

2
(5.4.25)
b) But what if the partition function is to be applied to a classical, indistinguishable, non-interacting
system? From this we must nd the internal energy U, as well as the free energy F.
We rst recall that since we have indistinguishable particles, the full partition function can be
found from the single-particle partition function as
Z
N

=
Z
N
1
N!
(5.4.26)
Rearranging the single particle partition function (5.4.25) and applying (5.4.26), we have
W. Erbsen STATISTICAL MECHANICS
Z
N
=
1
N!
_
cosh [m
z
B
0
]

2mL
2

2
_N
(5.4.27)
Now, the free energy is
F = k
B
T log [Z
N
] (5.4.28)
And plugging (5.4.27) into (5.4.28),
F = k
B
T
_
_
_
log
_
_
_
cosh [m
z
B
0
]

2mL
2

2
_N
_
_
log [N!]
_
_
_
(5.4.29)
At this point, we recall Stirlings Approximation:
log [N!]

= N log [N] N
Applying this to (5.4.29),
F = k
B
T
_
_
_
log
_
_
_
cosh [m
z
B
0
]

2mL
2

2
_N
_
_
(N log [N] N)
_
_
_
= k
B
T
_
N log
_
cosh [m
z
B
0
]

2mL
2

2
_
N log [N] + N
_
= Nk
B
T
_
log
_
cosh [m
z
B
0
]

2mL
2

2
_
log [N] + 1
_
(5.4.30)
Simplifying (5.4.30) further leads us to
F = Nk
B
T
_
log
_
cosh [m
z
B
0
]
N

2mL
2

2
_
+ 1
_
(5.4.31)
To nd the internal energy, U, we rst recall that
U =

log [Z
N
] (5.4.32)
Substituting the partition function from (5.4.27) into (5.4.32),
U =

log
_
_
1
N!
_
cosh [m
z
B
0
]

2mL
2

2
_N
_
_
= N

log
_
1
N!
1
/N
_
cosh [m
z
B
0
]

2mL
2

2
__
= N

log
_
1
N!
1
/N
_
2mL
2

2
. .
Garbage
cosh [m
z
B
0
]

_
Proceeding,
CHAPTER 5: DEPARTMENTAL EXAMINATIONS 503
U = N

log
_
cosh [m
z
B
0
]

_
= N

_
log [cosh [m
z
B
0
]] log
_
_

__
= N
_

log [cosh [m
z
B
0
]]
1
2

log []
_
(5.4.33)
Playing around with (5.4.33) a bit we can see that we have our answer:
U = N
_
m
z
B
0
tanh [m
z
B
0
]
1
2
_
(5.4.34)
c) In order to nd the equilibrium pressure, we rst recall that
P =
F
V

F
L
(5.4.35)
Where I took V L since we are only in 1-D. Anyway, we can substitute our value for the Free
Energy from (5.4.31) into (5.4.35):
P =

L
_
Nk
B
T
_
log
_
cosh [m
z
B
0
]
N

2mL
2

2
_
+ 1
__
=Nk
B
T

L
_
log
_
cosh [m
z
B
0
]
N

2mL
2

2
_
+ 1
_
=Nk
B
T

L
_
log
_
L
cosh [m
z
B
0
]
N
_
2m

2
_
+ 1
. .
Garbage
_
(5.4.36)
Eww smelly garbage yuck yuck. Anyway, ignoring the icky garbage, (5.4.36) becomes
P =Nk
B
T

L
[L] P =
Nk
B
T
L
(5.4.37)
The Magnetization, however, is a dierent story entirely. The relationship we are interested in is
M =
F
B
(5.4.38)
Substituting our previous value of F into (5.4.38) yields
M =

B
_
Nk
B
T
_
log
_
cosh [m
z
B
0
]
N

2mL
2

2
+ 1
___
=Nk
B
T

B
log
_
cosh [m
z
B
0
]
N

2mL
2

2
+ 1
_
=Nk
B
T

B
log
_
cosh [m
z
B
0
]
1
N

2mL
2

2
+ 1
_
M =Nk
B
T

B
log
_
cosh [m
z
B
0
]
_
W. Erbsen STATISTICAL MECHANICS
=Nk
B
T [m tanh [m
z
B
0
]]
Which nally leaves us with
M = Nm
z
tanh [m
z
B
0
]
Problem 4
a) From the condition that two coexisting phases of a given chemical substance must have the same chemical
ptential
1
=
2
, derive the Claussius-Clayperon equation
dP
dT
=
S
2
S
1
V
2
V
1
(5.4.39)
b) Drawn below is the phase diagram for He
4
. Carefully explain why the slope of the liquid-solid phase boundary
is zero at T = 0.
Solution
a) We rst recall that a common form of the rst law of thermodynamics is
dU = T dS P dV (5.4.40)
While the Helmholtz and Gibbs free energy are given, respectively, by
F =U TS (5.4.41a)
G =F +PV (5.4.41b)
CHAPTER 5: DEPARTMENTAL EXAMINATIONS 505
If we make the substitution (5.4.41a) into (5.4.41b), we have
G = U TS +PV (5.4.42)
Taking (5.4.42) to the dierential limit, we have
dG =dU T dS S dT + P dV + V dP
=dU (T dS P dV ) s dT + V dP
=dU dU S dT +V dP
= S dT + V dP (5.4.43)
Where I substituted in (5.4.40) in the second to last step arriving at (5.4.43). We now recall that
the chemical potential is given by
=
_
F
N
_
V,T

G
N
(5.4.44)
We can now play around with (5.4.44) as follows
=
G
N
d =
G
N
dG = Nd (5.4.45)
Applying (5.4.45) to (5.4.43),
n d =V dP S dT
d =
V
N
dP
S
N
dT (5.4.46)
We now temporarily recall that since we are at diusive equilibrium, then
1
=
2
and furthermore,
d
1
= d
2
. Ergo, (5.4.46) is transformed into
V
1
N
dP
S
1
N
dT =
V
2
N
dP
S
2
N
dT
V
1
N
dP
V
2
N
dP =
S
1
N
dT
S
2
N
dT
dP (V
1
V
2
) =dT (S
1
S
2
) (5.4.47)
From (5.4.47), it is most easily seen that
dP
dT
=
S
1
S
2
V
1
V
2
(5.4.48)
b) We are given a phase diagram for He
4
, which we immediately recognize as a Boson. This is especially
interesting because Bosons do not obey the Pauli Exclusion Principle, and can therefore all occupy
the same state at an arbitrarily low temperature. The multiplicity, that is, the number of states
available to a system without changing anything, is only 1. e.g., at low temperatures, there is only
1 way that the system can be congured (all in the ground state), and subsequently the multiplicity
is = 1.
This is particularly interesting, since He
4
has a multiplicity of 1, then the entropy is 0:
S = k
B
T log [] S = k
B
T log [1] S = 0
The Claussius-Clayperon equation that we derived earlier in (5.4.48) describes the slope of the phase
boundaries. In the case that we have low temperatures (approaching zero), where the multiplicity,
W. Erbsen STATISTICAL MECHANICS
, is 1, while the entropy, S, is 0. Plugging this in to (5.4.48), we get
dP
dT
=
0 0
V
1
V
2

dP
dT
= 0
The Claussius-Clayperon equation can be thought of as the slope equation for phase diagrams.
When the result is zero, that implies that there is no slope, and so you get a straight line.
Problem 5
Consider an ideal gas of diatomic molecules. The rotational motion is quantized according to
j
= j(j +1)
0
, j =
0, 1, 2, ... The multiplicity of each rotational level is g
j
= 2j + 1.
a) Find the rotational part of the partition function Z for one molecule.
b) Convert the sum (for Z) to an integral and evaluate the integral for kT
0
. From this nd an expression
for the specic heat capacity at constant volume at low temperatures.
Solution
a) For a system consisting of degeneracy, the form of the partition function that we will employ is
Z
1
=

j
g
j
exp
_

E
j
k
B
T
_
(5.4.49)
In our case, we have
g
j
= 2j + 1 ,
j
= j(j + 1)
0
Applying both of these to our single-particle partition function in (5.4.49), we have
Z
1
=

j
(2j + 1) exp
_

j(j + 1)
0
k
B
T
_
(5.4.50)
b) Converting the sum in (5.4.50) into an integral:
Z
1
=
_

0
(2j + 1) exp
_

j(j + 1)
0
k
B
T
_
dj (5.4.51)
u = j(j + 1) = j
2
+j
du = 2j + 1 dj
And using this du substitution, we can transform (5.4.51) into
Z
1
=
_

0
exp
_

u
0
k
B
T
_
du
=
k
B
T

0
_
exp
_

u
0
k
B
T
_

0
CHAPTER 5: DEPARTMENTAL EXAMINATIONS 507
From which it is easy to deduce that
Z
1
=
k
B
T

0
(5.4.52)
We need to move away from the single-particle partition function, and since we are dealing indis-
tinguishable entities:
Z
N

=
Z
N
1
N!
(5.4.53)
Substituting the single particle partition function from (5.4.52) into (5.4.53),
Z
N
=
1
N!
_
k
B
T

0
_
N
(5.4.54)
Now hold on to yer britches, and recall that
C
V
=
_
U
T
_
V
, U =

log [Z
N
]
First, lets evaluate the internal energy U, by substituting into (5.4.54) this expression:
U =

log
_
1
N!
_
1

0
_
N
_
= N

log
_
1
N!
1
/N
_
1

0
__
= N

log
_
1
N!
1
/N
_
1

0
_
. .
1

_
(5.4.55)
And (5.4.55) becomes
U = N

log
_
1

_
(5.4.56)
Carrying out the simple derivative in (5.4.56) leads us to
U =
N

u = Nk
B
T (5.4.57)
To nd the specic heat, we substitute the internal energy (5.4.57) into the appropriate formula
C
V
=

T
(nk
B
T) C
V
= Nk
B
For an ideal gas with three degrees of freedom, we nd that
U =
3
2
Nk
B
T
And so the specic heat at constant volume becomes
C
V
=
5
2
Nk
B
W. Erbsen STATISTICAL MECHANICS
Problem 6
a) Write down the Grand Partition Function (Gibbs sum), . Dene all quantities used and explain what the
summations are over.
b) Show that the average number of particles in a Grand Canonical Ensemble is given by
N) =

log (5.4.58)
where = kT and is the chemical potential.
c) Now consider a dilute gas of hydrogen atoms. Assume each atom may have the following states:
State No. of Electrons Energy
Ground 1 0
Positive ion 0
Negative ion 2 +
Find the condition that the average number of electrons per atom is 1.
Solution
a) We recall that within the framework of the Grand Canonical Ensemble, the system in question is
in both thermal and diusive equilibrium. The sum itself takes the form
(, T) =

N=0

s(N)
exp
_
N E
s,N
k
B
T
_
(5.4.59)
= The Gibbs Sum
= The Chemical Potential
N = The Number of Particles
E
s,N
= The energy of N-particles in S-states
b) We recall that, in general, the average number of particles is
N) =

s,N
N P (N, E
s,N
) (5.4.60)
Where the probability function is given by
P (N, E
s,N
) =
1
(, T)
exp
_
N E
s,N
k
B
T
_
(5.4.61)
And substituting the probability function (5.4.61) back into (5.4.60) to nd the average number of
particles,
N) =

s,N
N
1
(, T)
exp
_
NU E
s,N
k
B
T
_
(5.4.62)
For the time being, lets take the following (seemingly) arbitrary derivative:
CHAPTER 5: DEPARTMENTAL EXAMINATIONS 509
(, T)

_
_

N,s(N)
exp
_
N E
s,N
k
B
T
_
_
_
=

N,s(N)
N
k
B
T
exp
_
N E
s,N
k
B
T
_
=
1
k
B
T

N,s(N)
N exp
_
N E
s,N
k
B
T
_
(5.4.63)
Solving (5.4.63) for the portion that resembles a part of (5.4.62),
k
B
T

(, T) =

N,s(N)
N exp
_
N E
s,N
k
B
T
_
(5.4.64)
We can substitute the RHS of (5.4.64) into (5.4.62):
N) =

s,N
k
B
T
(, T)

(, T) (5.4.65)
Lets rewrite (5.4.65):
N) =

s,N
k
B
T
1
(, T)

(, T)
. .
(5.4.66)
Lets look a little closer at the Gibbs function from (5.4.59) (ignore sums momentarily):
(, T) = exp
_
N E
s,N
k
B
T
_
Lets take the rst derivative with respect to :

(, T) =

exp
_
N E
s,N
k
B
T
_
=
N
k
B
T
exp
_
N E
s,N
k
B
T
_
(5.4.67)
If we take the rst derivative of the Gibbs function with respect to , we get some coecient times
the original function. Now, theres two ways of getting rid of the original function: assuming that
you just want to play around with the coecients for some reason. You could a) simply divide your
answer by the original function, or b), you could take the same derivative you took before, but had
the natural log of the function:
1
(, T)

(, T) =

log [(, T)] (5.4.68)


We note that the LHS of (5.4.68) is identical to the underbraced term in (5.4.66). Substituting the
RHS of (5.4.68) into (5.4.66), replacing the underbraced term, we have:
N) =

s,N
k
B
T

log [(, T)] (5.4.69)


Rearranging, (5.4.69) becomes
W. Erbsen STATISTICAL MECHANICS
N) = k
B
T

log [(, T)] (5.4.70)


Where we note that the summation is implicit in the Gibbs function.
c) The rst thing that we have to do is nd the Gibbs sum. Using the probability function (5.4.61) as
our outline, the sum itself takes the form
(, T) =

N=0

s(N)
exp
_
N E
s,N
k
B
T
_
(5.4.71)
There are only three unique states, so the second sum in (5.4.71) only goes to 3. We also know that
we have three states of electrons, of occupancy 0, 1, or 2 as given in the provided table. Accordingly,
(5.4.71) becomes
(, T) =exp
_
(1) 0
k
B
T
_
+ exp
_
(0) ()
k
B
T
_
+ exp
_
(2) ()
k
B
T
_
=exp
_

k
B
T
_
+ exp
_

k
B
T
_
+ exp
_
2
k
B
T
_
=exp [] + exp [] + exp [(2 ) ] (5.4.72)
Substituting (5.4.72) into the equation for average number of particles from (5.4.70),
N) =
1

log [exp [] + exp [] + exp [(2 ) ]]


=
1

_
exp [] + 2 exp [(2 )]
exp [] + exp [] + exp [(2 )]
_
=
exp [] + 2 exp [(2 )]
exp [] + exp [] + exp [(2 )]
(5.4.73)
We are actually looking for a condition where N) = 1, so setting this in (5.4.73) yields
exp [] + exp [] + exp [(2 )] =exp [] + 2 exp [(2 )]
exp [] = exp [(2 )]
= (2 ) (5.4.74)
From (5.4.74), one of the conditions that might be met is in terms of the chemical potential, which
reads
=
+
2
Problem 7
A system of N distinguishable noninteracting particles has one-particle energy levels of E
n
= n, where the
degeneracy of the n
th
level is equal to n, and n = 1, 2, 3, ...
CHAPTER 5: DEPARTMENTAL EXAMINATIONS 511
a) Show that the partition function of the system is
Z =
_
e

(e

1)
2
_
N
(5.4.75)
where = (k
B
T)
1
.
b) Calculate the entropy of the system of particles.
c) Determine the behavior of the entropy at high temperature.
d) Determine the behavior of the specic heat at high temperatures.
Solution
We rst recognize that since we have distinguishable particles, then the partition function as a function
of the single-particle partition function would be Z
N

= Z
N
1
.
a) The standard form of the partition function (with degeneracy) is
Z
1
=

n
n exp
_

E
n
k
B
T
_
(5.4.76)
We now substitute E
n
= n into (5.4.76),
Z
1
=

n
n exp
_

n
k
B
T
_
(5.4.77)
We can rewrite (5.4.77) slightly,
Z
1
=

n
n exp
_


k
B
T
_
n
(5.4.78)
And now, we see that if we were to take some arbitrary derivative of some function then the answer
could be (5.4.78). One such form is
Z
1
=
1

n
n exp []
n
(5.4.79)
Where I have changed form k
B
T to
1
. We now recall the following summation identity:

r
n
=
r
1 r
Applying this to (5.4.79),
Z
1
=
1

_
exp []
1 exp []
_
=
1

_
exp []
1 exp []
__
=
1

_
exp []
_

1
1 exp []
_
+
1
1 exp []
_

exp []
__
W. Erbsen STATISTICAL MECHANICS
=
1

_
exp []
_
exp []
(1 exp [])
2
_
+
1
1 exp []
[ exp []]
_
=
exp [2]
(1 exp [])
2
+
exp []
1 exp []
=
exp [2]
(1 exp [])
2
+
exp []
1 exp []

1 exp []
1 exp []
=
exp [2] + exp [] exp [2]
(1 exp [])
2
=
exp []
(1 exp [])
2
=
exp []
1 2 exp [] + exp [2]

exp [2]]
exp [2]]
=
exp []
exp [2] 2 exp [] + 1
(5.4.80)
We an nally say that (5.4.80) can be reduced to
Z
1
=
exp []
(exp[] 1)
2
(5.4.81)
Converting (5.4.81) into the full partition function,
Z
N
=
_
exp []
(exp[] 1)
2
_
N
(5.4.82)
b) To calculate the entropy for a system of particles, we rst recall that
S =
F
T
(5.4.83)
So, we rst need to nd F:
F = k
B
T log [Z
N
] (5.4.84)
Substituting the partition function (5.4.82) into (5.4.84),
F = k
B
T log
_
_
_
_
exp []
(exp[] 1)
2
_
N
_
_
_
= Nk
B
T log
_
exp []
(exp[] 1)
2
_
= Nk
B
T
_
log [exp []] log
_
(exp [] 1)
2
__
= Nk
B
T log [exp []] 2 log [(exp [] 1)] (5.4.85)
Substituting (5.4.85) back into (5.4.83), we can nd S,
S =

T
[Nk
B
T log [exp []] 2 log [(exp [] 1)]]
CHAPTER 5: DEPARTMENTAL EXAMINATIONS 513
=Nk
B

T
_
T log
_
exp
_

k
B
T
__
+ 2 log
__
exp
_

k
B
T
_
1
___
=Nk
B

T
_

k
B
..
Garbage
+2 log
_
exp
_

k
B
T
_
1
..
Dont matter
__
(5.4.86)
Rewriting (5.4.86):
S =2Nk
B

T
log
_
exp
_

k
B
T
_
1
_
=2Nk
B
_
_


k
B
T
exp
_

kBT
_
exp
_

kBT
_
1
_
_
(5.4.87)
Rearranging (5.4.87),
S =
2N
T
exp
_

kBT
_
exp
_

kBT
_
1
(5.4.88)
Problem 8
In a mono-atomic crystalline solid each atom can occupy either a regular lattice site or an interstitial site. The
energy of an atom at an interstitial site exceeds the energy of an atom at a lattice site by . Assume that the
number of atoms, lattice sites, and interstitial sites are all equal in number.
a) Calculate the entropy of the crystal in the state where exactly n of the N atoms are at interstitial sites.
b) What is the temperature of the crystal in this state, if the crystal is at thermal equilibrium?
c) If = 0.5 eV and teh temperature of the crystal is 273 K, what is the fraction of atoms at interstitial sites?
Hint: You have two choices to make in this problem - which atoms to put at interstitial sites, and which interstitial
sites to put them in.
Solution
a) To calculate the entropy, we recall that
S =
F
T
(5.4.89)
And also that the Free energy is
F = k
B
T log [Z
N
] (5.4.90)
While the single-particle partition function in the case of degeneracy is
W. Erbsen STATISTICAL MECHANICS
Z
1
=

i
g
i
exp
_

E
i
k
B
T
_
(5.4.91)
Now, there are two dierent sources of degeneracy in this problem. There is one source coming
from the n interstitial lattices, and then there is N for the regular lattices. So, combining these
into (5.4.91), yields
Z
1
=

n
nexp
_

E
n
k
B
T
_
+

N
N exp
_

E
N
k
B
T
_
(5.4.92)
Depending on where we are on the lattice we have a dierent energy, such that we should rewrite
(5.4.92) as
Z
1
=

n
nexp
_

(E
0
+)
k
B
T
_
+

N
N exp
_

E
0
k
B
T
_
=exp
_

E
0
k
B
T
_
_

n
nexp
_


k
B
T
_
+

N
N
_
(5.4.93)
At this point, we say that the degeneracy for this particular two-level case can be expressed in the
following more compact form:
g(N, n) =
N!
(N n)!n!
(5.4.94)
Using this equation, we can completely transform (5.4.93), which now reads
Z
N
=
N!
(N n)!n!
exp
_

E
0
k
B
T
_ _
exp
_


k
B
T
_
+ 1
_
(5.4.95)
Using (5.4.90), we can now nd the Free Energy by way of (5.4.95) as follows
F = k
B
T log
_
N!
(N n)!n!
exp
_

E
0
k
B
T
_ _
exp
_


k
B
T
_
+ 1
__
= k
B
T
_
log
_
N!
(N n)!n!
_
+ log
_
exp
_

E
0
k
B
T
__
+ log
_
exp
_


k
B
T
_
+ 1
__
= k
B
T
_
log
_
N!
(N n)!n!
_

E
0
k
B
T
+ log
_
exp
_


k
B
T
_
+ 1
__
(5.4.96)
And to calculate the entropy, we now substitute (5.4.96) back in to (5.4.89):
S =

T
_
k
B
T
_
log
_
N!
(N n)!n!
_

E
0
k
B
T
+ log
_
exp
_


k
B
T
_
+ 1
___
=k
B

T
_
T log
_
N!
(N n)!n!
_

E
0
k
B
+ T log
_
exp
_


k
B
T
_
+ 1
__
=k
B
_
log
_
N!
(N n)!n!
_
+ log
_
exp
_


k
B
T
_
+ 1
_
+T

T
log
_
exp
_


k
B
T
_
+ 1
__
=k
B
_
log
_
N!
(N n)!n!
_
+ log
_
exp
_


k
B
T
_
+ 1
_
+T
_

T
2
exp [/(k
B
T)]
exp [/(k
B
T)] + 1
__
(5.4.97)
We can simplify (5.4.97) again:
CHAPTER 5: DEPARTMENTAL EXAMINATIONS 515
S = k
B
log
_
N!
(N n)!n!
_
+ k
B
log
_
exp
_


k
B
T
_
+ 1
_
+k
B

T
exp [/(k
B
T)]
exp [/(k
B
T)] + 1
(5.4.98)
b) We recall that from the rst law of thermodynamics,
dU = T dS P dV (5.4.99)
At constant volume, (5.4.99) becomes
dU = T dS
dS
dU
=
1
T
(5.4.100)
Applying the chain rule to (5.4.100),
dS
dU
=
dS
dU

dU
dn

dn
dU
=
dS
dn

dn
dU
=
1
T
And now,
U =n(E +) + (N n) E
=nE +n +NE nE
=n + NE
Taking the dierential limit,
dU = dn
dn
d
=
1

Substituting this back in to our chain rule,


dS
dn

1

=
1
T

dS
dn
=

T
(5.4.101)
At this point, we apply (5.4.101) to (5.4.98):
dS
dn
=
d
dn
_
k
B
log
_
N!
(N n)!n!
_
+k
B
log
_
exp
_


k
B
T
_
+ 1
_
+ k
B

T
exp [/(k
B
T)]
exp [/(k
B
T)] + 1
_
=k
B
d
dn
log
_
N!
(N n)!n!
_
(5.4.102)
We can further rewrite (5.4.102),
dS
dn
=k
B
d
dn
log [N!] log [(N n)!] + log [n!] (5.4.103)
Recall the identity
log [N!] = N log [N] N (5.4.104)
Using (5.4.104), we can rewrite (5.4.103) as:
dS
dn
=k
B
d
dn
log [(N n)!] +nlog [n] n (5.4.105)
We can apply the same logic to the mixed term in (5.4.105):
log [(N n)!] =(N n) log [N n] (N n)
W. Erbsen STATISTICAL MECHANICS
=N log [N n] nlog [N n] N + n (5.4.106)
Substituting (5.4.106) back into (5.4.105),
dS
dn
=k
B
d
dn
N log [N n] nlog [N n] N + n + nlog [n] n
=k
B
d
dn
N log [N n] nlog [N n] +nlog [n]
=k
B
_
N
N n
log [N n]
n
N n
+ log [n] + 1
_
=k
B
_
log
_
N n
n
_
+
N
N n

n
N n
+ 1
_
=k
B
_
log
_
N n
n
_
+ 2
_
(5.4.107)
We now set (5.4.107) to /T:
k
B
_
log
_
N n
n
_
+ 2
_
=

T
(5.4.108)
Solving (5.4.108) for T,
T =

k
B
_
log
_
N n
n
_
+ 2
_
1
(5.4.109)
c) To nd the fraction of atoms in the interstitial sites, we simply solve (5.4.109) for N/n:

k
B
T
= log
_
N n
n
_
+ 2
exp
_

k
B
T
1
_
=
N n
n
(5.4.110)
From (5.4.110) we can see that
N
n
= exp
_

k
B
T
1
_
+ 1 (5.4.111)
Problem 9
Calculate the magnetic susceptibility
=
_
M
H

H=0
(5.4.112)
Where M is the magnetic moment of the sample and H is the applied eld. Calculate as a function of T of a
gas of N permanent dipoles, each of moment ,
a) If any direction is allowed (classical spines).
b) If the dipole is only allowed to assume 2 directions, parallel and opposite to the applied eld (Ising spins).
CHAPTER 5: DEPARTMENTAL EXAMINATIONS 517
Solution
a) The rst thing that we recall is the energy of a dipole in an external eld, which is given by
U = H (5.4.113)
And the magnetization is given by
M =
F
H
(5.4.114)
Now, if we have a whole bunch of dipoles, then they are not all perfectly aligned (or antialigned),
but rather at some random angle
i
. Then the total energy from the entirety of the molecules can
be found as
U = H

i
cos
i
= H

i
cos
i
(5.4.115)
Since the particles in this case are distinguishable, the full partition function can be found from the
single particle partition function as Z
N

= Z
N
1
. The single-particle partition function is
Z
1
=

i
exp
_
H cos
i
k
B
T
_
(5.4.116)
Turning the sum in (5.4.116) into an integral, we have
Z
1
=
_
2
0
_

0
exp
_
H cos
k
B
T
_
sin dd (5.4.117)
Letting u = cos and du = sin d, (5.4.117) becomes
Z
1
= 2
_
1
1
exp
_
Hu
k
B
T
_
du
= 2
k
B
T
H
_
exp
_
Hu
k
B
T
_

1
1
= 2
k
B
T
H
_
exp
_
H
k
B
T
_
exp
_
H
k
B
T
__
=4
k
B
T
H
sinh
_
H
k
B
T
_
(5.4.118)
The full partition function can be found from (5.4.118) easily as
Z
N
=
_
4
k
B
T
H
sinh
_
H
k
B
T
__
N
(5.4.119)
We now use (5.4.119) to nd the Free Energy, F:
F = k
B
T log [Z
N
]
= k
B
T log
_
_
4
k
B
T
H
sinh
_
H
k
B
T
__
N
_
W. Erbsen STATISTICAL MECHANICS
= Nk
B
T log
_
4
k
B
T
H
sinh
_
H
k
B
T
__
(5.4.120)
We now substitute F from (5.4.120) into (5.4.114), the equation for the Magnetization, M:
M =

H
_
Nk
B
T log
_
4
k
B
T
H
sinh
_
H
k
B
T
___
=Nk
B
T

H
log
_
4
k
B
T
H
sinh
_
H
k
B
T
__
=Nk
B
T

H
_
log
_
4k
B
T

_
log [H] + log
_
sinh
_
H
k
B
T
___
=Nk
B
T
_

1
H
+

H
log
_
sinh
_
H
k
B
T
___
=Nk
B
T
_

1
H
+

k
B
T
cosh
_
H
k
B
T
__
=N
_

k
B
T
H
+cosh
_
H
k
B
T
__
(5.4.121)
Now, according to the prompt, to nd the susceptibility , all we must do is dierentiate M with
respect to H. Doing this, we take the derivative of (5.4.121) with respect to H:
=
M
H
=

H
_
N
_

k
B
T
H
+ cosh
_
H
k
B
T
___
=N
_
k
B
T
_

H
1
H
_
+
_

H
cosh
_
H
k
B
T
___
=N
_
k
B
T
_

1
H
2
_


k
B
T
csch
2
_
H
k
B
T
___
=N
_
k
B
T
H
2
+

2
k
B
T
csch
2
_
H
k
B
T
__
(5.4.122)
While (5.4.122) is the exact answer, and I believe that we were supposed to use an approximation.
We make this approximation in (5.4.121), where we expand the cosh term in a series expansion:

H=0
cosh
_
H
k
B
T
_
=
k
B
T
H
+
H
3k
B
T


3
H
3
45 (k
3
B
T
3
)
+...
If we assume that the eld is weak, then we can approximate this function by its rst two expansion
terms. Substituting them back into the Magnetization from (5.4.121),
M =N
_

k
B
T
H
+
_
k
B
T
H
+
H
3k
B
T
__
M =
N
2
H
3k
B
T
It is trivial then, to nd the susceptibility in the weak-eld regime:
=

H
_
N
2
H
3k
B
T
_
=
N
2
3k
B
T
b) We now need to see how things change if the dipoles are only allowed to assume 2 directions,
parallel and anti-parallel. The only dierence is that here our energies are discrete, so our problem
CHAPTER 5: DEPARTMENTAL EXAMINATIONS 519
is actually easier. I will y through it rather quickly. Firstly, the single particle partition function
is
Z
1
=exp
_

H
k
B
T
_
+ exp
_
H
k
B
T
_
Whose full counterpart is then
Z
N
=
_
exp
_

H
k
B
T
_
+ exp
_
H
k
B
T
__
N
(5.4.123)
And (5.4.123), we can nd the Fermi energy
F = k
B
T log
_
_
exp
_

H
k
B
T
_
+ exp
_
H
k
B
T
__
N
_
= Nk
B
T log
_
exp
_

H
k
B
T
_
+ exp
_
H
k
B
T
__
= Nk
B
T log
_
2 cosh
_
H
k
B
T
__
(5.4.124)
Finding the magnetization of (5.4.124),
M =Nk
B
T
_

H
log
_
2 cosh
_
H
k
B
T
___
=Nk
B
T
_

k
B
T
tanh
_
H
k
B
T
__
(5.4.125)
Approximating the rst two terms in tanh in very much the same way we did before,

H=0
tanh
_
H
k
B
T
_
=
H
k
B
T


3
H
3
3 (k
3
B
T
3
)
+
2
5
H
5
15k
5
B
T
5
+ ...
Substituting in the rst two terms back into the Magnetization equation (5.4.125),
M =Nk
B
T
_

k
B
T
_
H
k
B
T


3
H
3
3 (k
3
B
T
3
)
__
=N
_
H
k
B
T


3
H
3
3 (k
3
B
T
3
)
_
(5.4.126)
And nally, to nd the susceptibility we dierentiate (5.4.126) with respect to H:
=

H
_
N
_
H
k
B
T


3
H
3
3 (k
3
B
T
3
)
__
Playing around with this a little, we are nally left with
= N
_

k
B
T


3
H
2
(k
3
B
T
3
)
_
W. Erbsen STATISTICAL MECHANICS
Problem 10
Similar to the van der Waals equation of state is the Dieterici equation of state,
p(V b) = RTe
a/RTV
(5.4.127)
Find the critical constants pc, V c and Tc in this model of a weakly interacting gas. This equation of state was
proposed to account for the interaction of gas atoms with walls.
Solution
We rst recall what a typical phase diagram looks like, which is of P vs. T. The critical point is where
the phase boundaries cease to exist, and all phases coexist simultaneously. This point has the coordinates
P
C
and T
C
.
We can also plot a phase diagram of P vs. V , plotting a variety of dierent isotherms (lines of con-
stant temperature). We nd that in plotting one particular isotherm, that has a point which coincides
with P
c
and V
C
from the PT diagram plot. At this point, the slope is zero.
The isotherms that lie inside the shaded area are in stable equilibrium of gas and liquid. At the critical
point, on the isotherm at the critical temperature, the rst (and therefore, the second) derivative with
respect to V must be zero:
_
P
V
_
Tc
=
_

2
P
V
2
_
Tc
= 0
Solving the equation of state given from (5.4.127) for P yields
P =
RT
V b
exp
_

a
RTV
_
(5.4.128)
Taking the rst derivative of (5.4.128) yields
_
P
V
_
Tc
=

V
_
RT
V b
exp
_

a
RTV
_
_
=
RT
V b
_

V
exp
_

a
RTV
_
_
+ exp
_

a
RTV
_

V
RT
V b
=
RT
V b
_
a
RTV
2
exp
_

a
RTV
__
+ exp
_

a
RTV
_
_

RT
(V b)
2
_
=
a
V
2
(V b)
exp
_

a
RTV
_

RT
(V b)
2
exp
_

a
RTV
_
=
exp
_

a
RTV

V b
_
a
V
2

RT
V b
_
(5.4.129)
And setting (5.4.129) equal to zero,
_
P
V
_
Tc
=0
a
V
2
=
RT
V b
V
2
RT aV + ab = 0 (5.4.130)
Taking the second derivative of P is the same as taking the rst derivative of (5.4.129), so lets do that:
CHAPTER 5: DEPARTMENTAL EXAMINATIONS 521
_

2
P
V
2
_
Tc
=

V
_
exp
_

a
RTV

V b
_
a
V
2

RT
V b
_
_
=
exp
_

a
RTV

RTV
4
(V b)
3
_
a
2
(b V )
2
2aRTV (b 2V )(b V ) + 2R
2
T
2
V
4

(5.4.131)
Setting (5.4.131) equal to zero,
_

2
P
V
2
_
Tc
= 0 a
2
(b V )
2
= 2aRTV (b 2V )(b V ) + 2R
2
T
2
V
4
(5.4.132)
We now have two equations, and two unknowns. Lets now solve (5.4.130) for T
T =
a(V b)
RV
2
(5.4.133)
And substituting this into our second equation (5.4.132),
a
2
(b V )
2
=2aRV
_
a(V b)
RV
2
_
(b 2V )(b V ) + 2R
2
_
a(V b)
RV
2
_
2
V
4
a
2
(V b)
2
=2a
2
(V b)
2
(b 2V )
V
+ 2a
2
(V b)
2
1 =2
(b 2V )
V
+ 2
3 =
2(2V b)
V
(5.4.134)
From (5.4.134), we can see that V
c
= 2b . Plugging in this value into (5.4.133) yields
T =
a(2b b)
R(2b)
2
T
c
=
a
4bR
Substituting both V
c
and T
c
into P from (5.4.128),
P =
R
2b b
_
a
4bR
_
exp
_

a
R

4bR
a

1
2b
_
P
c
=
a
4b
2
exp [2]
Problem 11
For a gas of molecules with diameter d, number density n and at temperature T, nd
a) The mean free path
b) Their average speed
c) The pressure of the gas using kinetic arguments
W. Erbsen STATISTICAL MECHANICS
Solution
a) The mean free path is dened as the average distance that a gas particle transverses between
collisions. If we imagine two gas particles, each of diameter d, passing close to one another, the
question is: what is the eective collisional diameter? In other words, if two individual particles
happen to collide at one point in space, then it is easy to see that if any particle approaches any
other particle, then the net new particle diameter would have to be the sum of the two collided
molecules, or 2d.
This illustration works particularly well if we make a number of critical assumptions:
i) Only one particle is moving, while all other molecules (target molecules) are sta-
tionary. If the moving particle comes within a distance of 2d of a stationary
particle, then a collision occurs.
The eective collision area is then A = d
2
.
If the particle travels through space some distance, and in its wake it sweeps out a volume propor-
tional to its cross sectional area and the distance that it traveled. The volume of such a sphere is
V = A = d
2
, where is the length of the swept out cylinder.
We must also recognize that we must harness the time that passes while the particle snakes its path
through space. Recalling that velocity is dened as v = x/t, and in our case x = and t is
just plain old t. Then we can say that v is the average velocity, and t is the time between successive
collisions.
When the volume of this imaginary cylinder equals the average volume per particle, then we are
likely to get a collision.
The mean free path () is the length of the cylinder when this condition is met
Quantitatively, this condition is
Volume of Cylinder =Avg Volume per Particle
d
2
=
V
N
(5.4.135)
We can solve (5.4.135) for the mean free path, :
=
1
d
2

V
N
(5.4.136)
It is important that we recognize that (5.4.136) is only an approximation, since according to stipu-
lation i), we have assumed that the target molecules are stationary, which is most certainly never
true.
To remedy this, we can rewrite from (5.4.136) in terms of the average velocity, v:
= v t =
1
d
2

V
N
(5.4.137)
In truth, what we are interested in is the relative velocity between the various particles, not the
average velocity. Luckily, this question has been asked before, and a quantitative link has been
dened relating the two quantities. The savior comes in the form of the Maxwell-Boltzmann Speed
Distribution Function, which I shall derive later. The punch-line is
v
rel
=

2 v v =
1

2
v
rel
(5.4.138)
CHAPTER 5: DEPARTMENTAL EXAMINATIONS 523
Substituting (5.4.138) into (5.4.137),
=
1

2
v
rel
t =
1

2 d
2

V
N
(5.4.139)
b) We recall that any probability can be obtained by integrating the probability distribution function
over a particular range:
P(v) =
_
v2
v1
f(v) dv
Our goal is to nd some distribution function that allows us to integrate and nd a value for speed.
We also note that there are many dierent speeds that can correspond to any particular velocity.
In general,
f(v)
_
Probability of having
a velocity v
_

_
Number of velocity vectors
corresponding to the same speed
_
(5.4.140)
The rst factor in (5.4.140) is simply the Boltzmann factor, so we can begin building our distribution
function:
f(v) exp
_

mv
2
2mk
B
T
_
(5.4.141)
And for the second term in (5.4.140), we need to get a little more abstract.. we imagine a 3-D
coordinate system in which the axes correspond to dierent velocity components v
x
, v
y
, and v
z
.
Some speed then, will act to mark an invisible sphere around the origin of our coordinate system,
such that if one end of the vector is planted at the origin, it can trace out the sphere in any direction
it pleases.
This velocity space analogy convinces us that the second factor in (5.4.140) is only a propor-
tionality factor, equal to the area that is swept out in v-space. This area is A = 4v
2
. Therefore,
from (5.4.141), our distribution function is now
f(v) exp
_

mv
2
2mk
B
T
_
4v
2
C (5.4.142)
Where C normalizes our distribution function. Now,
_

0
f(v) dv = 1
4C
_

0
v
2
exp
_

mv
2
2mk
B
T
_
dv = 1
4C
_
_
_
1
4
_

_
2k
B
T
m
_
3
_1
/2
_
_
_
= 1
C
_
2k
B
T
m
_3
/2
= 1
C =
_
m
2k
B
T
_3
/2
(5.4.143)
Substituting (5.4.143) back into our function (5.4.142)
W. Erbsen STATISTICAL MECHANICS
f(v) = 4v
2

_
m
2k
B
T
_3
/2
exp
_

mv
2
2mk
B
T
_
(5.4.144)
This, (5.4.144), is the famous Maxwell-Boltzmann Distribution Function. We will now use it. But
rst, going back to the fundamentals: an average quantity can be found by
x) =
_
x
0
x f(x) dx
Following this example, we now wish to nd the average speed, v, using our distribution function
from (5.4.144):
v =
_

0
v f(v) dv
=4
_
m
2k
B
T
_3
/2
_

0
v
3
exp
_

mv
2
2mk
B
T
_
dv (5.4.145)
The integral in (5.4.145) is tedious , and I would recommend doing it in Mathematica or possibly
a table. Anyway, the result is
v =
_
8k
B
T
m
(5.4.146)
c) What is pressure? Well, kinetically speaking, pressure is the average force exerted on the walls of
some container. Mathematically,
F
avg
= N
mv
2
x
L
(5.4.147)
Now, if the speed is the same in all directions, then (5.4.147) becomes
F
avg
= 3N
mv
2
L
(5.4.148)
And we recall that the pressure is dened as P = F/A, in our case, the force is the average force
applied to the sides of the container. Using (5.4.148), the pressure becomes
P =
F
avg
Area
P = N
3mv
2
L A
Problem 12
Consider a large reservoir energy U
0
in thermal contact with a system with energy .
a) Give an argument based on the multiplicity (number of distinguishable ways a state with a given energy may
be obtained) which leads to the result
P() exp
_


kT
_
(5.4.149)
Where P() is the probability of nding the system in a particular state with energy . Hint: Recall that the
entropy S = k log (multiplicity).
CHAPTER 5: DEPARTMENTAL EXAMINATIONS 525
b) A system of N magnetic moments ( ) may orient themselves parallel or antiparallel to an applied eld.
Deduce from a) the total magnetization M

1

i
as a function of H and T.
Solution
a) Multiplicity refers to the number of congurations that a system has available to it while staying
essentially the same. The entropy can be linked to the multiplicity as
S = k
B
log [] (5.4.150)
And the probability of being in one particular state is
P =
1

(5.4.151)
And we recall that the rst law says that
dU = T dS P dV (5.4.152)
Since the volume in our case is constant, (5.4.152) reduces to
dU = T dS
dS
dU
=
1
T
(5.4.153)
Applying the chain rule to (5.4.153),
dS
dU

dU
d

d
dU
=
dS
d

d
dU
=
1
T
(5.4.154)
Sweet. Now, substitute our relation for S from (5.4.150) into (5.4.154) and lets see what happens:
dS
d

d
dU
=
_
d
d
k
B
log []
_

d
dU
=
k
B


d
dU
(5.4.155)
And setting (5.4.155) equal to 1/T, as originally intended,
k
B


d
dU
=
1
T

1

d =
1
k
B
T
dU (5.4.156)
Integrating both sides of (5.4.156), we nd that
log [] =
E
k
B
T
=exp
_
E
k
B
T
_
(5.4.157)
And applying (5.4.151) to (5.4.157), we have
P = exp
_

E
k
B
T
_
b) We rst recall that for a two-state paramagnet, we can either be in the direction of the eld, or
anti-parallel to it:
W. Erbsen STATISTICAL MECHANICS
E = B (5.4.158)
While the total number of domains is given by
N = N

+N

Each with respective energies


E

=B
E

= B
We also recall that the magnetization is dened as the density of magnetic moments. In our case,
M = (N

+ N

) (5.4.159)
The single-particle partition function is then given by
Z
1
=exp
_
B
k
B
T
_
+ exp
_

B
k
B
T
_
=2 cosh
_
B
k
B
T
_
And assuming that our particles are distinguishable, then the full partition function can be found
easily:
Z
N
=
_
2 cosh
_
B
k
B
T
__
N
(5.4.160)
Now nding the Helmholtz Free Energy, we use the partition function from (5.4.160), and so:
F = k
B
T log
_
_
2 cosh
_
B
k
B
T
__
N
_
= Nk
B
T log
_
2 cosh
_
B
k
B
T
__
(5.4.161)
The magnetization, M, is then found using (5.4.161) as
M =

B
_
Nk
B
T log
_
2 cosh
_
B
k
B
T
___
=Nk
B
T
_

B
log
_
2 cosh
_
B
k
B
T
___
=Nk
B
T
_

k
B
T
tanh
_
B
k
B
T
__
Simplifying things a bit, the magnetization becomes
M = Ntanh
_
B
k
B
T
_
CHAPTER 5: DEPARTMENTAL EXAMINATIONS 527
Problem 13
Consider a two-state system with energies 0 and .
a) Calculate the partition function
b) Find the average energy at temperature T
c) Find the heat capacity and graph it versus temperature
Solution
a) We rst recall that the plain old partition function is given in its most basic form as
Z =

s
exp
_

E
s
k
B
T
_
(5.4.162)
We only have a two-level system, so it is easy to expand (5.4.162) to suit our needs:
Z = exp [0] + exp
_


k
B
T
_
Which can be rewritten as:
Z = 1 + exp
_


k
B
T
_
(5.4.163)
b) In order to nd the average energy, we recall that
U =
1
Z

s
E
s
exp [E
s
/(k
B
T)] (5.4.164)
Evaluating the internal energy from (5.4.164) out to the two allowed terms, much as we did before:
U =
0 exp [0] + exp [/(k
B
T)]
1 + exp [/(k
B
T)]
(5.4.165)
Getting rid of the null terms in (5.4.165) leads us to
U =
exp [/(k
B
T)]
1 + exp [/(k
B
T)]
(5.4.166)
We can actually nicen (5.4.166) up a bit (for reasons that will become clear momentarily) by letting
= 1/(k
B
T):
U =
exp []
1 + exp []
(5.4.167)
c) To nd the heat capacity at constant volume, we rst recall:
C
V
=
_
U

_
V
(5.4.168)
Substituting the internal energy from (5.4.167) into (5.4.168),
W. Erbsen STATISTICAL MECHANICS
C
V
=

_
exp []
1 + exp []
_
= exp []

_
1
1 + exp []
_
+
_
1
1 + exp []
_

[ exp []]
= exp []
_
exp []
(1 + exp [])
2
_
+
_
1
1 + exp []
_
_

2
exp []

= exp []
_
exp []
(1 + exp [])
2
_
+
_

2
exp []
1 + exp []
_
=
2

__
exp [2]
(1 + exp [])
2
_
+
(exp [])
(1 + exp [])
(1 + exp [])
(1 + exp [])
_
=
2

_
exp [2] exp [] exp [2]
(1 + exp [])
2
_
We can now deduce that
C
V
=
2

_
exp []
(1 + exp [])
2
_
Problem 14
An ideal diatomic gas has rotational energy levels given by
E
j
=
h
2
8
2
I
j(j + 1), with degeneracies g
j
= 2j + 1 (5.4.169)
a) For Oxygen, what fraction of the molecules is in the lowest rotational energy state at T = 50K? Recall that

rot
=
h
2
8
2
Ik
B
= 2K (5.4.170)
b) Repeat this for Hydrogen, with
rot
= 85K.
Solution
a) The single particle partition function (with degeneracy) is by default given by
Z
1
=

j
g
j
exp
_

E
j
k
B
T
_
(5.4.171)
Substituting in the appropriate values into (5.4.171),
Z
1
=

j
(2j + 1) exp
_

1
k
B
T

h
2
8
2
I
j(j + 1)
_
(5.4.172)
CHAPTER 5: DEPARTMENTAL EXAMINATIONS 529
At this point, we can rewrite E
j
and
rot
as follows
E
j
=
h
2
8
2
I

k
B
k
B
j(j + 1)
=
rot
k
B
j(j + 1) (5.4.173)
Substituting (5.4.173) back into our partition function from (5.4.172),
Z
1
=

j
(2j + 1) exp
_

1
k
B
T

rot
k
B
j(j + 1)
_
=

j
(2j + 1) exp
_

rot
T
j(j + 1)
_
(5.4.174)
We now wish to convert the sum in (5.4.174) into an integral:
Z
1
=
_
(2j + 1) exp
_

rot
T
j(j + 1)
_
dj (5.4.175)
Using u = j(j + 1) = j
2
+ 1 and du = (2j + 1) du, (5.4.175) becomes
Z
1
=
_

0
exp
_

rot
T
u
_
du
=
T

rot
_
exp
_

rot
T
u
_

0
=
T

rot
(5.4.176)
From the provided information, we have T = 50 K, and (5.4.176) becomes
Z
1
=
T

rot

1
Z
1
=

rot
T

1
Z
1
=
1
25
(5.4.177)
b) We recall the single particle partition function from (5.4.175) and using the fact that
rot
= 85 K,
and T = 50 K, (5.4.175) becomes
Z
1
=

j
(2j + 1) exp
_

85
50
j(j + 1)
_
(5.4.178)
The sum in (5.4.178) seems to converge very quickly. Lets look at the rst few terms:
Z
1
(0) =1
Z
1
(1) =3 exp
_

85
50
2
_
= 0.11
Z
1
(2) =5 exp
_

85
50
6
_
= 0.00
So, taking only the rst two terms, we have Z
1
1.100, and so the inverse of this yields
1
Z
1
0.909
W. Erbsen STATISTICAL MECHANICS
Problem 15
On a glass surface, water molecules can hydrogen bond to either 1 or 2 OH groups as shown below with en-
ergies or 2, respectively. The molecule may also be weakly bound to the surface with energy
1
() (not
shown).
a) Find an expression for the partition function for a water molecule on the surface.
b) Find the limiting behavior of the internal energy U and the specic heat C at i) high temperatures and ii)
low temperatures.
c) Hence, sketch the behavior of U and C as a function of temperature.
Solution
a) The single-particle partition function is easily found to be
Z
1
= exp [] + exp [2] + exp [
1
] (5.4.179)
b) The relevant expression for the internal energy is
U =k
B
T
2

T
log [Z
n
] =

log [Z
N
] (5.4.180)
Since our particles in this case are distinguishable, the partition function in (5.4.179) becomes
Z
N
= Z
N
1
= [exp [] + exp [2] + exp [
1
]]
N
(5.4.181)
Substituting (5.4.181) into (5.4.180),
U =

log
_
[exp [] + exp [2] + exp [
1
]]
N
_
= N

log [exp [] + exp [2] + exp [


1
]]
= N
_
exp [] 2 exp [2]
1
exp [
1
]
exp [] + exp [2] + exp [
1
]
_
(5.4.182)
We now recall that the specic heat at constant volume is given by
CHAPTER 5: DEPARTMENTAL EXAMINATIONS 531
C
V
=
_
U
T
_
V
(5.4.183)
Rewriting (5.4.182) and substituting into (5.4.183),
C
V
= N

T
_
_
_
exp
_

kBT
_
2 exp
_

2
kBT
_

1
exp
_

1
kBT
_
exp
_

kBT
_
+ exp
_

2
kBT
_
+ exp
_

1
kBT
2
_
_
_
_
=
N
k
B
T
2
_

_
exp
_
2+1
kBT
_ _

2
_
4 + exp
_
3
kBT
_
+ 9 exp
_
+1
kBT
__
+ 2
1
_
exp
_
3
kBT
_
1
_
+
2
1
_
exp
_
3
kBT
___
_
exp
_
2
kBT
_
+ exp
_
1
kBT
_
+ exp
_
3+
kBT
__
2
_

_
(5.4.184)
Here, (5.4.184) represents the full expression for the specic heat. See attached plots of U vs. T as
well as C
V
vs. T below.
20 40 60 80 100
T
0.25
0.30
0.35
U
2 4 6 8 10
T
0.1
0.2
0.3
0.4
0.5
Cv
Problem 16
The Diesel engine cycle consists of several processes
Suppose the cycle is applied to n moles of an ideal diatomic gas. In terms of the given pressures and volumes and
the gas constant R, give expressions for
a) The heat QH given to the gas during combustion.
b) The heat QC removed from the gas during the cooling process.
c) The entropy change of the gas in the process BC.
d) The entropy change of the gas in process CD.
AB: Adiabatic compression from volume V
0
to volume V
1
, where pressure = p
1
. The compression ratio rc =
V
0
/V
1
is high enough that it causes ignition of the air-fuel mixture without needing a spark.
BC: Expansion at constant pressure p
1
from volume V
1
to V
2
< V
0
, during the burning of the fuel.
CD: Adiabatic expansion from V
2
to V
0
, with expansion ration = V
0
/V
2
.
W. Erbsen STATISTICAL MECHANICS
DA: Constant volume (V
0
) cooling back to the original temperature at point A.
Treat the gas as an ideal gas, and suppose that the constant pressure (C
P
) and constant volume (C
V
) heat
capacities are known.
a) Calculate the eciency of the cycle, dened as the ratio of the work output to the heat absorbed per cycle,
= W/Q
H
.
b) Express as a function of = C
P
/C
V
, r
e
, and r
c
.
c) Calculate the entropy changes in each process of the cycle, and sketch the cycle in a S-T diagram.
Solution
a) To nd the heat given o during ignition, QH, we are looking at the BC isobar. We recall that
the heat is given by
Q = nCT (5.4.185)
Applying this to the BC isobar,
Q
BC
= nC
P
[T
C
T
B
] (5.4.186)
We now recall that the ideal gas law says that
PV = nRT T =
PV
nR
(5.4.187)
Substituting this equation for T into (5.4.186),
Q
BC
=nC
P
__
P
1
V
1
nR
_

_
P
1
V
2
nR
__
Q
BC
=
C
P
P
1
R
[V
1
V
2
] (5.4.188)
CHAPTER 5: DEPARTMENTAL EXAMINATIONS 533
b) We now wish to nd the heat QC removed during the cooling process, which along the DA
isochor. The process is very much the same as in the last part, adopting (5.4.186) for our purposes,
Q
DA
= nC
V
[T
A
T
D
] (5.4.189)
And substituting (5.4.187) into (5.4.189),
Q
DA
= nC
V
__
P
D
V
0
nR
_

_
P
A
V
0
nR
__
Q
DA
=
C
V
V
0
R
[P
D
P
A
] (5.4.190)
c) Now we want to calculate the change in entropy along BC. We rst recall that the entropy is
dened by
S =
Q
T
(5.4.191)
Substituting (5.4.188) into (5.4.191),
S =
C
P
P
1
RT
[V
1
V
2
] (5.4.192)
And also that the specic heat at constant pressure is
C
P
= T
S
T
(5.4.193)
Treating (5.4.193) like a dierential equation,
1
C
P
dS =
1
T
dT
S
C
P
= log
_
V
2
V
1
_
(5.4.194)
Solving (5.4.194) for S,
S = C
P
log
_
V
2
V
1
_
(5.4.195)
d) To nd the change in entropy along the isochore DA, we can go ahead and use (5.4.195) and say
that
S = C
V
log
_
V
0
V
0
_
S = 0 (5.4.196)
e) We are now asked to nd the eciency of the entire cycle, which is given as = W/Q
H
. We also
recall that W = Q
in
Q
out
, so that in our case, the eciency is given by
=
Q
BC
Q
DA
Q
BC
=
_
CP P1
R
[V
1
V
2
]
_

_
CV V0
R
[P
D
P
A
]
_
_
CP P1
R
[V
1
V
2
]
_
Which leads us to
=
C
P
P
1
[V
1
V
2
] C
V
V
0
[P
D
P
A
]
C
P
P
1
[V
1
V
2
]
W. Erbsen STATISTICAL MECHANICS
f ) We now wish to rewrite in terms of some new variables, which are
=
C
P
C
V
, r
e
=
V
0
V
2
, r
c
=
V
0
V
1
The result is
= 1
1
P
1

_
P
D
P
A
1
/
rc

1
/
re
_
= 1
1
P
1

_
P
D
P
A
r
c
r
e
_
r
c
r
e
g) The entropy for both cycles are
S
BC
= S
DA
= C
P
log
_
V
2
V
1
_
, S
AB
= S
CD
= 0
And the net change in entropy is of course
S
net
= 0
Problem 17
A classical monatomic ideal gas of N particles (each of mass m) is conned to a cylinder of radius r and in-
nite height. A gravitational eld points along the axis of the cylinder downwards.
a) Determine the Helmholtz energy A.
b) Find the internal energy U and the specic heat C
V
.
c) Why is C
V
,=
3
/
2
Nk?
Solution
a) There are two components of energy in this system: kinetic (p
2
/2m) and potential (mgh). The
Hamiltonian for this system is then
H =
p
2
2m
+ mgz
=
p
2
x
+p
2
y
+p
2
z
2m
+ mgz (5.4.197)
While the single-particle partition function takes the form
Z
1
=
1
h
3
_ _
exp
_

H
k
B
T
_
d
3
pd
3
r
(5.4.198)
Substituting our Hamiltonian from (5.4.197) into the partition function from (5.4.198) yields
CHAPTER 5: DEPARTMENTAL EXAMINATIONS 535
Z
1
=
1
h
3
_ _
exp
_

p
2
x
+ p
2
y
+p
2
z
2mk
B
T

mgz
k
B
T
_
d
3
pd
3
r
=
1
h
3
_

exp
_

p
2
x
+p
2
y
+ p
2
z
2mk
B
T
_
dp
x
dp
y
dp
z
_

0
exp
_

mgz
k
B
T
_
dz
=
1
h
3
_

exp
_

p
2
x
2mk
B
T
_
dp
x
_

exp
_

p
2
y
2mk
B
T
_
dp
y
_

exp
_

p
2
z
2mk
B
T
_
dp
z
_

0
exp
_

mgz
k
B
T
_
dz
=
1
h
3

_
_
2mk
B
T
_

_
_
2mk
B
T
_

_
_
2mk
B
T
_
_

0
exp
_

mgz
k
B
T
_
dz
=
[2mk
B
T]
3
/2
h
3
_

0
exp
_

mgz
k
B
T
_
dz
. .
(5.4.199)
We actually want to solve this last integral in polar coordinates:
_

0
exp
_

mgz
k
B
T
_
dz =
_

0
_
2
0
_
R
0
exp
_

mgz
k
B
T
_
rdrddz
=
_

0
exp
_

mgz
k
B
T
_
dz
_
2
0
d
_
R
0
r dr
=
k
B
T
mg
_
exp
_

mgz
k
B
T
_

0
[2]
_
R
2
2
_
=
k
B
T
mg
2
R
2
2
=
R
2
k
B
T
mg
(5.4.200)
Substituting (5.4.200) back into the original integral of our function (5.4.199),
Z
1
=
[2mk
B
T]
3
/2
h
3

R
2
k
B
T
mg
(5.4.201)
And for classical particles, we recall that Z
N

= Z
N
1
, so applying this to the single particle partition
function (5.4.201) yields
Z
N
=
_
[2mk
B
T]
3
/2
h
3

R
2
k
B
T
mg
_
N
=
_
_
2mk
B
T
h
2
_3
/2

R
2
k
B
T
mg
_
N
=
_
_
_
_
_
2mk
B
T
h
2
_

_
R
2
k
B
T
mg
_
2
/3
_
3
/2
_
_
_
N
(5.4.202)
We now recall that the internal energy is dened by
U =

log [Z
n
] (5.4.203)
W. Erbsen STATISTICAL MECHANICS
Substituting the partition function (5.4.202) into the internal energy (5.4.203),
U =

log
_

_
_
_
_
_
_
2mk
B
T
h
2
_

_
R
2
k
B
T
mg
_
2
/3
_
3
/2
_
_
_
N
_

_
=
3N
2

log
_
_
2m
h
2
_

_
R
2

mg
_
2
/3
_
=
3N
2

log
_
_
_
2
3
/2
m
1
/2

5
/2

5
/2
gh
3
_2
/3
_
_
= N

log
_
2
3
/2
m
1
/2

5
/2
gh
3
. .
Garbage
1

5
/2
_
(5.4.204)
Carrying out the derivative in (5.4.204),
U = N

log
_

5
/2
_
=
5N
2

log [] U =
5N
2
k
B
T (5.4.205)
Recalling that the specic heat at constant volume is none other than the rst derivative of internal
energy with respect to temperature, we have from (5.4.205)
C
V
=

T
_
5N
2
k
B
T
_
C
V
=
5N
2
k
B
b) We recognize that C
V
,=
3
/
2
Nk
B
because we have more degrees of freedom in this particular system,
which stems from our original Hamiltonian.
Problem 18
A thin-walled vessel of volume V is kept at a constant temperature T. A gas leaks slowly out of the vessel
through a small hole of area A into surrounding vacuum. Find the time required for the pressure in the vessel to
drop to half of its original value.
Solution
We rst note that
P
i
= P
0
N
i
= N
P
f
=
1
/
2
P
0
N
f
=
1
/
2
N
The number of particles leaving the container over a time interval dt is
dN = nAv dt (5.4.206)
CHAPTER 5: DEPARTMENTAL EXAMINATIONS 537
Where we recall that n = N/V . Furthermore, the average velocity is given by
v =
_
8k
B
T
m
(5.4.207)
Substituting v from (5.4.207) into (5.4.206), and solving for dt, yields
dN = nA
_
8k
B
T
m
dt
dt =
V
N

1
A
_
m
8k
B
T
dN (5.4.208)
Integrating (5.4.208),
t =
V
A
_
m
8k
B
T
_ N
/2
N
1
N
; dN
=
V
A
_
m
8k
B
T
_
log
_
N
/
2

log [N]

(5.4.209)
Simplifying (5.4.209) gives us a neat expression for t:
t =
V
A
_
m
8k
B
T
log [2]
Problem 19
Consider a paramagnetic substance with the equation of state M = AH/(T T
0
). Here M is the magnetiza-
tion, H is the applied magnetic eld, A and T
0
are constants, and T is the temperature. The equation of state is
valid only for T > T
0
. Show that C
M
, the heat capacity at constant magnetization, is independent of M.
Solution
We rst recall that specic heat (in general) is dened as
C =
Q
T
(5.4.210a)
C =T
S
T
(5.4.210b)
From the 2
nd
Law, if the heat capacity at constant M is indeed independent of M itself, then taking the
derivative with respect to it should be equal to zero:
C
M
M
= 0 (5.4.211)
This is the condition that we are trying to prove. One possible way to do this is to work backwards,
hoping to arrive at the same result. Plugging (5.4.210b) into (5.4.211), we have
W. Erbsen STATISTICAL MECHANICS
C
M
M
=

M
_
T
S
T
_
=T

2
S
MT
(5.4.212)
Now, we switch the order of the derivatives in (5.4.212), yielding
C
M
M
=T

M
S
T
=T

T

S
M
(5.4.213)
And now, we recall a few seemingly unrelated identities:
dU =T dS + HM (5.4.214a)
dU =T dS + dF (5.4.214b)
dF = SdT + HdM (5.4.214c)
Now, dont forget the function that was given in the prompt:
M =
AH
T T
0
H = (T T
0
)
M
A
(5.4.215)
We now recall one of the least-studied areas within the realm of statistical mechanics. One of them is
particularly useful for this problem, and that is
_
S
M
_
T
=
_
H
T
_
M
(5.4.216)
We now substitute H from (5.4.215) into the RHS of (5.4.216)
_
S
M
_
T
=

T
_
(T T
0
)
M
A
_
=
M
A
(5.4.217)
We now wish to substitute the LHS of (5.4.216) into the RHS of (5.4.213):
C
M
M
=T

T

S
M
=T

T

M
A
(5.4.218)
And since neither M nor A has any time dependence, (5.4.218) is clearly null, and the result could not
be more clear:
C
M
M
= 0
CHAPTER 5: DEPARTMENTAL EXAMINATIONS 539
Problem 20
A sample of ideal gas is taken through the cyclic process abca shown in the gure. At point a, T = 300K.
a) What are the temperatures of the gas at points b and c?
b) Complete the table by inserting a plus sign, a minus sign, or a zero in each indicated cell. Note that Q is
positive when heat is absorbed by the gas and W is positive when work is done by the gas. E is the change
in internal energy of the gas.
Q W E
a b
b c
c a
Solution
a) We recall from the ideal gas law that
PV = Nk
B
T (5.4.219)
From the gure, we are able to deduce that both P and V strictly by visual inspection, while T is
given only at point a. We can now solve for Nk
B
in (5.4.219):
Nk
B
=
PV
T
(5.4.220)
Throughout the cycle, the fraction in (5.4.220) is constant, so to nd the temperature at some other
point when the other variables are given,
P
a
V
a
T
a
=
P
b
V
b
T
b
T
b
=
P
b
V
b
T
a
P
a
V
a
(5.4.221)
W. Erbsen STATISTICAL MECHANICS
Plugging in the appropriate values into (5.4.221),
T
b
=
_
7.5 10
3
N m
2
_ _
3.0 m
3
_
(300 K)
(2.5 10
3
N m
2
) (1.0 m
3
)
T
b
= 2700 K
We can repeat the same process to nd T
c
by making a change of variables from b c:
T
c
=
P
c
V
c
T
a
P
a
V
a
(5.4.222)
And substituting in the correct values into (5.4.222), we have
T
c
=
_
2.5 10
3
N m
2
_ _
3.0 m
3
_
(300 K)
(2.5 10
3
N m
2
) (1.0 m
3
)
T
c
= 900 K
b) We note that Q is positive when heat is absorbed, while W is positive when work is done by the
gas. Accordingly,
From t Q W
a b
b c =
c a
Problem 21
A sample of helium gas inside a cylinder terminated with a piston doubles its volume from V
i
= 1m
3
to V
f
= 2m
3
.
During this process the pressure and volume are related by PV
6
/5
= A = constant. Assume that the product PV
always equals
2
/
3
U, where U is the internal energy.
a) What is the change in energy of the gas?
b) What is the change in entropy of the gas?
c) How much heat was added to or removed from the gas?
Solution
a) Given that the original formula PV
6
/5
= A, we can solve for P before and after this expansion:
P
i
=
A
V
6
/5
i
, P
f
=
A
V
6
/5
f
And the change in energy can be found using the second given expression,
PV =
2
3
U U =
3
2
PV (5.4.223)
And so the change in energy, U, can be found by
U =
3
2
(P
f
V
f
P
i
V
i
)
CHAPTER 5: DEPARTMENTAL EXAMINATIONS 541
=
3
2
_
A
V
6
/5
f
V
f

A
V
6
/5
i
V
i
_
=
3
2
A
_
1
V
1
/5
f

1
V
1
/5
i
_
(5.4.224)
Substituting in the appropriate values into (5.4.224),
U =
3
2
A
_
1
(2 m
3
)
1
/5

1
(1 m
3
)
1
/5
_
(5.4.225)
b) To nd the change in entropy of the gas, we rst recall that a common form of the rst law of
thermodynamics is
dU = T dS P dV (5.4.226)
Solving (5.4.226) for dS,
dS =
1
T
dU +
P
T
dV (5.4.227)
We now recall from the ideal gas law, and also from the information given in the problem, that
PV = Nk
B
T =
2
3
U U =
3
2
Nk
B
T (5.4.228a)
T =
2
3
U
Nk
B
(5.4.228b)
P =
Nk
B
T
V
=
2
3
U
V
(5.4.228c)
Substituting (5.4.228a)-(5.4.228c) into (5.4.227),
dS =
_
3
2
Nk
B
U
_
dU +
1
T
_
Nk
B
T
V
_
dV
=
3
2
Nk
B
_
1
U
dU
_
+nk
B
_
1
V
dV
_
(5.4.229)
Integrating, (5.4.229) becomes
S =
3
2
Nk
B
[log U[
Uf
Ui
+ nk
B
[logV [
V
f
Vi
=
3
2
Nk
B
log
_
U
f
U
i
_
+ Nk
B
log
_
V
f
V
i
_
(5.4.230)
And we now innocently recall that
U
f
=
3
2
A
1
(2 m
3
)
1
/5
=
3
2
A
1
V
1
/5
f
(5.4.231a)
U
i
=
3
2
A
1
(1 m
3
)
1
/5
=
3
2
A
1
V
1
/5
i
(5.4.231b)
And substituting (5.4.231a) and (5.4.231b) into S from (5.4.230),
W. Erbsen STATISTICAL MECHANICS
S =
3
2
Nk
B
log
_
_
V

1
/5
f
V

1
/5
i
_
_
+ Nk
B
log
_
V
f
V
i
_
=
3
10
Nk
B
log
_
V
i
V
f
_
+Nk
B
log
_
V
f
V
i
_
=
7
10
Nk
B
log
_
V
f
V
i
_
(5.4.232)
Substituting the appropriate values into (5.4.232), we are left with
S =
7
10
Nk
B
log [2]
c) We rst recall that
dQ = dW + dU (5.4.233)
To nd out what W is in (5.4.233), we recall that work is dened as W =
_
PdV . Also remembering
that PV
6
/5
= A P = AV

6
/5
, so we have
W =A
_
Vf
Vi
V

6
/5
dV
= A 5
_
1
V
1
/5

Vf
Vi
= A 5
_
1
V
1
/5
f

1
V
1
/5
i
_
(5.4.234)
Now, substituting U from (5.4.225) and W from (5.4.234) into (5.4.233), we have
Q = A 5
_
1
V
1
/5
f

1
V
1
/5
i
_
+
3
2
A
_
1
V
1
/5
f

1
V
1
/5
i
_
=
7
2
A
_
1
V
1
/5
f

1
V
1
/5
i
_
And with the given values, this becomes
Q =
7
2
A
_
1
(1 m
3
)
1
/5

1
(2 m
3
)
1
/5
_
Problem 24
An arbitrary network of resistors, capacitors, and inductors is in thermal equilibrium at temperature T. There are
no sources in the network.
CHAPTER 5: DEPARTMENTAL EXAMINATIONS 543
a) By considering a capacitor C as a subsystem, nd the probability density P(V ) that a voltage V exists across
it. Find the mean and the root-mean-square voltages on a 10 pF capacitor when T = 300 K and when
T = 77 K.
b) Similarly, nd the probability density P(I) that a current I ows through an inductor L at temperature T.
What is the rms current in a 1 mH inductor when T = 300 K and when T = 77 K?
Solution
a) We recall that the energy stored in a capacitor is given by
E(V ) =
CV
2
2
(5.4.235)
While the probability function is most generally given by
P(
s
) =
1
Z
exp
_


s
k
B
T
_
(5.4.236)
Where the partition function in this case is
Z =

s
exp
_


s
k
B
T
_
(5.4.237)
And substituting (5.4.235) into (5.4.237) yields
Z =

V
exp
_

CV
2
2k
B
T
_
(5.4.238)
Passing this to the integral limit,(5.4.238) becomes
Z =
_

exp
_

CV
2
2k
B
T
_
dV Z =
_
2k
B
T
C
(5.4.239)
Substituting (5.4.235) and (5.4.239) into (5.4.236),
P(V ) =
_
C
2k
B
T
exp
_

CV
2
2k
B
T
_
To nd the mean voltage, V ), we recall that a common way to dene this quantity is
x) =
1
Z
_
x exp
_

E(x)
k
B
T
_
dx (5.4.240)
In our case, (5.4.240) becomes
V ) =
_
C
2k
B
T
_

V exp
_

CV
2
2k
B
T
_
dV V ) = 0
This is because we were trying to integrate an even function multiplied by an odd function over a
symmetric domain. The RMS, however, does not t this criteria, and as such we would expect a
non-zero result:
W. Erbsen STATISTICAL MECHANICS
V
2
) =
_
C
2k
B
T
_

V
2
exp
_

CV
2
2k
B
T
_
dV
=
_
C
2k
B
T
_
_
1
2


_
2k
B
T
C
_
3
_
_
=

4

C
2k
B
T

8k
3
B
T
3
C
3
=
_
k
2
B
T
2
C
2
(5.4.241)
From (5.4.241) it is easy to deduce that
V
2
) =
k
B
T
C
b) Finding the probability density for current owing through an inductor is identical to the process
just underwent, therefore I will only summarize the results:
P(I) =
_
I
2k
B
T
exp
_

IL
2
2k
B
T
_
I) = 0
I
2
) =
k
B
T
L
Problem 25
a) Compare the 4 level systems shown below. More than one particle may occupy a level. Which system has
the
Highest temperature Lowest temperature
Lowest specic heat Highest entropy
CHAPTER 5: DEPARTMENTAL EXAMINATIONS 545
b) Consider the system shown below. Explain why it can be thought of as having a negative temperature. (Hint:
Consider the Boltzmann factor e
E/kT
).
c) If this system (from part b)) is brought into contact with a large reservoir at temperature TR (positive) draw
a graph indicating how its temperature will change as a function of time as it comes to equilibrium with the
reservoir. (Take the initial temperature of the system to be TS a negative number).
d) Explain why no problems with the third law are encountered in part c).
Solution
a)
Highest temp: (ii)
Lowest temp: (iii)
Lowest specic heat: (ii)
Highest entropy: (ii)
b) If the temperature is increased slightly, then it is possible that the total energy will decrease, which
is the opposite of the typical behavior.
c) The third law states that as the entropy in a perfect crystal approaches zero, then the temperature
approaches zero as well.
This example does not violate the third law of thermodynamics because system at a negative tem-
perature is further away from absolute zero than a system at positive temperature (from the point
of view of energy, anyway).
W. Erbsen STATISTICAL MECHANICS
Problem 26
Equipartition. A classical harmonic oscillator
H =
p
2
2m
+
Kq
2
2
(5.4.242)
is in thermal contact with a heat bath at temperature T. Calculate the partition function for the oscillator in the
canonical ensemble and show explicitly that
E) = k
B
T, and (E E))
2
) = (k
B
T)
2
(5.4.243)
Solution
We rst recall that the single-particle partition function is given by
Z
1
=
1
h
3
_ _
exp [H] d
3
pd
3
q (5.4.244)
However, in our case we are assuming that the Hamiltonian is only in 1-D, so that substituting our
Hamiltonian from (5.4.242) into (5.4.244), we have
Z
1
=
1
h
_ _
exp
_

_
p
2
2m
+
kq
2
2
__
dpdq
=
1
h
_

exp
_

p
2
2mk
B
T
_
dp
_

exp
_
Kq
2
2
_
dq
=
1
h
_
_
2mk
B
T
_
_
_
2k
B
T
K
_
=
2k
B
T
h
_
m
K
(5.4.245)
We now recall that the internal energy, E), is dened by
E) =

log Z
1
(5.4.246)
Rearranging (5.4.245) and substituting into (5.4.246), we have
E) =

log
_
2
h
_
m
K
_
E) = k
B
T (5.4.247)
Now, we recall that
(E E))
2
) = E
2
) E)
2
(5.4.248)
And we can also express E) in yet another way
E) =
1
Z
1
_ _
E exp [H] dpdq E) = k
B
T (5.4.249)
CHAPTER 5: DEPARTMENTAL EXAMINATIONS 547
As before, We can extend this to nd E
2
), however, and (5.4.249) becomes
E
2
) =
1
Z
1
_ _
E
2
exp [H] dpdq (5.4.250)
And now, seemingly arbitrarily, we take the rst derivative of E) with respect to in (5.4.247):

E) =

=
1

2
(5.4.251)
And now, stay with me here, we wish to do the same thing for the primary expression in (5.4.249):

E) =

_
1
Z
1
_ _
E exp [H] dpdq
_
(5.4.252)
Recognizing that Z
1
has a dependence on , we continue with (5.4.252), yielding

E) =
1
Z
1
_

_ _
E exp [H] dpdq
_
+
_ _
E exp [H] dpdq
_

1
Z
1
_
=
1
Z
1
_ _
E
2
exp [H] dpdq +
_ _
E exp [H] dpdq
_

1
Z
1
. .
_
(5.4.253)
Lets take a closer look at the underbraced term in (5.4.253), using the value of Z
1
found in (5.4.245):

1
Z
1
=

_
h
2
_
K
m
_

h
2
_
K
m
(5.4.254)
Substituting (5.4.254) back in to (5.4.253),

E) =
1
Z
1
_ _
E
2
exp [H] dpdq +
h
2
_
K
m
_ _
E exp [H] dpdq (5.4.255)
Inserting in to (5.4.255) our expression for Z
1
found in (5.4.245),

E) =
h
2k
B
T
_
K
m
_ _
E
2
exp [H] dpdq +
h
2
_
K
m
_ _
E exp [H] dpdq
. .
(5.4.256)
Let us know rewrite the underbraced term in (5.4.256):
h
2
_
K
m
_ _
E exp [H] dpdq =
h
2
_
K
m
Z
1
E)
=
h
2
_
K
m

_
2k
B
T
h
_
m
K
_
E)
=
1

E) (5.4.257)
Plugging (5.4.257) back into (5.4.256),

E) =
1
Z
1
_ _
E
2
exp [H] dpdq
. .
+
1

E) (5.4.258)
W. Erbsen STATISTICAL MECHANICS
We immediately recognize the underbraced term to be none other than our good friend E
2
). Using this
knowledge, we can now rewrite (5.4.258),

E) =E
2
) +
1

E)
Rearranging this a bit,
E
2
) =
1

E)

E)
=
1

_
1

_
1

_
=
1

2
+
1

2
=
2

2
(5.4.259)
Substituting (5.4.247) and (5.4.259) into (5.4.248),
E
2
) E)
2
=
2

2

1

2
E
2
) E)
2
= (k
B
T)
2
Problem 27
Consider two single-particle states and two particles. Calculate the entropy of this system when the particles
have the following statistics:
a) Maxwell-Boltzmann (i.e. classical)
b) Bose-Einstein
c) Fermi-Dirac
d) If two particles of mass m are conned to a box of volume V , approximately what temperature do classical
statistics lose their validity?
Solution
a) We rst recall how the entropy depends on the multiplicity
S = k
B
log [] (5.4.260)
We rst need to know what the multiplicity, is before we can use this tool though. The multi-
plicity refers to the number of available states in the system. Its as simple as that.
For Maxwell-Boltzmann statistics, we recognize that this is a classical system, and both particles
may occupy any of the states available with no restrictions what-so-ever:
CHAPTER 5: DEPARTMENTAL EXAMINATIONS 549


_
_
_

() = 4
So, the entropy in this case is
S = k
B
log [4]
b) Within the framework of hte Bose-Einsein universe, we recognize that the particles may still occupy
the same orbital, however being indistinguishable particles, there is no dierence between states
with similar occupancies. Eg,


_
_
_

() = 3
And the entropy ends up being
S = k
B
log [3]
c) For the Fermi-Direac distribution, we recognize that we are constrained clenched st of Paulis
Exclusion Principle. According to Mr Pauli, there is only one possible conguration:

_
_
_

() = 1
And, as expected, the entropy, which is given as
S = k
B
log [1] S = 0
d) Within the classical regime, the single-particle partition function is given by
Z
1
=
1
h
_ _
exp
_

_
p
2
x
+p
2
y
+ p
2
z
_
2mk
B
T
_
d
3
p d
3
r
=
V
h
_
exp
_

_
p
2
x
+p
2
y
+ p
2
z
_
2mk
B
T
_
d
3
p
=
V
h
(2mk
B
T)
3
/2
(5.4.261)
And the classical limit is dened as
Z
1
N
1 (5.4.262)
Substituting (5.4.261) into (5.4.262),
V
h
(2mk
B
T)
3
/2
N
1
(2mk
B
T)
3
/2

Nh
V
2mk
B
T
_
Nh
V
_2
/3
(5.4.263)
W. Erbsen STATISTICAL MECHANICS
From (5.4.263), it is easy to see that the condition is
T
1
2mk
B
_
Nh
V
_2
/3
Problem 28
A zipper has N links; each link has a state in which it is closed with energy 0 and a state in which it is open with
energy . We require, however, that the zipper can only unzip from the left end, and that the link number, s, can
only open if all links to the left (1, 2, ..., s 1) are already open.
a) Show that the partition function can be summed in the form:
Q
N
=
1 exp [(N + 1)]
1 exp[]
(5.4.264)
b) In the limit kT, nd the average number of open links.
The above model is a very simplied model of the unwinding of two-stranded DNA molecules.
Solution
Ooooh the zipper problem. I zip you so much.
a) The standard form of the single-particle partition function is
Z
1
=

s=0
exp
_


s
k
B
T
_
(5.4.265)
Because the particles are distinguishable, then we can gain the full partition from (5.4.265) quite
easily
Z
s
=

s=0
exp
_


s
k
B
T
_
s
(5.4.266)
We now take a eld trip. Lets take a close look at the following series:
N

s=0
x
s
= 1 +x + x
2
+ x
3
+ ... +x
N
(5.4.267)
We now multiply both sides of (5.4.267) by x. This looks like
x
N

s=0
x
s
= x + x
2
+ x
3
+ x
4
+ ... +x
N+1
(5.4.268)
We now take (5.4.267) and subtract from it (5.4.268). This yields
CHAPTER 5: DEPARTMENTAL EXAMINATIONS 551
(1 x)
N

s=0
x
s
= 1 x
N+1
(5.4.269)
If we were to rearrange (5.4.269), then it would look like
N

s=0
x
s
=
1 x
N+1
1 x
(5.4.270)
This exact sum is what we need in order to evaluate (5.4.266)!
Z
s
=
1 exp [(N + 1) ]
1 exp []
(5.4.271)
b) In general, the way to nd the average number of open links, we use the form
s) =
1
Z
s
N

s=0
s exp[s] (5.4.272)
Now, notice that we can arrive at (5.4.272) if we are very tricky and take a derivative of the following
form:
s) =
1
Z
s

()
N

s=0
exp [s]
. .
Zs
(5.4.273)
Rewriting (5.4.273) once more,
s) =
1
Z
s

x
Z
s
(5.4.274)
Where I have made the temporary substitution x = for convenience. Carrying out the rst
derivative in (5.4.274),

x
Z
s
=

x
_
1 exp [(N + 1) x]
1 exp [x]
_
=
1
1 exp [x]
_

x
[1 exp [(N + 1) x]]
_
+ [1 exp [(N + 1) x]]
_

x
1
1 exp [x]
_
=
1
1 exp [x]
[(N + 1) exp [(N + 1)x]] [1 exp [(N + 1)x]]
_
exp [x]
(1 exp [x])
2
_
=(N + 1)
exp [(N + 1)x]
1 exp [x]
exp [x]
_
1 exp [(N + 1)x]
(1 exp [x])
2
_
=(N + 1)
exp [(N + 1)x]
1 exp [x]

exp [x]
(1 exp [x])
2
+
exp [x] exp [(N + 1)x]
(1 exp [x])
2
=(N + 1)
exp [(N + 1)x]
1 exp [x]

1
1 exp [x]
_

exp [x]
1 exp [x]
+
exp [(N + 1)x] exp [x]
1 exp [x]
_
=
1
1 exp [x]
_
(N + 1) exp [(N + 1)x]
exp [x]
1 exp [x]
+
exp [(N + 1)x] exp [x]
1 exp [x]
_
W. Erbsen STATISTICAL MECHANICS
=
1
1 exp [x]
_
(N + 1) exp [(N + 1)x] exp [x]
_
1 exp [(N + 1)x]
1 exp [x]
__
(5.4.275)
Where we note that the expression in brackets in (5.4.275) is none other than Z
s
. Rewriting,

x
Z
s
=
1
1 exp [x]
[(N + 1) exp [(N + 1)x] exp [x] Z
s
] (5.4.276)
At this point we are ready to substitute (5.4.276) and (5.4.271) into (5.4.274):
s) =
1
Z
s

1
1 exp [x]
[(N + 1) exp [(N + 1)x] exp [x] Z
s
]
=
(N + 1) exp [(N + 1)x] exp [x]
1 exp [x]
=
(N + 1) exp [(N + 1)x]
1 exp [x]

exp [x]
1 exp [x]
(5.4.277)
Putting the numbers back in to (5.4.277) leads us to
s) =
(N + 1) exp [(N + 1)]
1 exp []

exp []
1 exp []
(5.4.278)
We are asked to evaluate the average number of open links from (5.4.278) given the following
stipulation:
k
B
T 1
If 1, then all the exponential terms approach zero, and so the average number of open links is
s) = 0
Problem 29
a) Obtain the Van der Waals equation of state
_
p +
N
2
a
V
_
(V Nb) = NkT (5.4.279)
by making the following two corrections to the ideal gas:
i) Use an eective volume, instead of the usual volume
ii) Include interactions in a mean eld form. State clearly where the mean eld assumptions come in.
b) What is the work done by the gas if it expands isothermally from V
1
to V
2
at some temperature T?
Solution
CHAPTER 5: DEPARTMENTAL EXAMINATIONS 553
a) The ideal gas law is, of course, given by
PV = Nk
B
T (5.4.280)
Lets apply ii) rst to (5.4.280), where we are including interactions from the mass eld form.
Mean eld theory essentially consists of replacing all interactions attributed to any one body, with
an average, or eective interaction.

This reduces a complicated multi-body problem to an eectively single-body problem, whose


solutions are easier to nd.
But what is the probability that two particles will be in the same place at the same time?

The density ( = m/v) is sort of like a probability, in the sense that if the density is large, the
probability that two particles will try to be in the same space at the same time increases (this
probability actually goes like P
2
).

Think of two coins: if the probability that one is heads up is , then the probability that they
will both be heads up is
2
.
This adds an eective pressure term to (5.4.280). The addition of a new term indicates that
due to the chance of spatially co-existing molecules, the pressure is actually more, proportional to
several constants. Anyway, with this correction, (5.4.280) becomes
_
P + a
2
_
V = Nk
B
T (5.4.281)
Where the constant a is a proportionality constant, which depends on how much the particles in-
teract with one another.
We now recall how we dene the particle density:
=
N
V
Substituting this in to (5.4.281) yields
_
P + a
N
2
V
2
_
V = Nk
B
T (5.4.282)
Next, we wish to address i), which is attributed to the eective volume, which basically means
that the two particles cannot coexist at the same place at the same time.

Therefore, the amount of space available for these molecules ying around is actually less then
the total volume of the container, by an amount proportional to the number of molecules present:
V
e
= V Nb (5.4.283)
Where b is the volume of the individual gas particle, and N is the number of particles in the volume.
Therefore, substituting (5.4.283) into (5.4.282), we have
_
P +a
N
2
V
2
_
(V Nb) = Nk
B
T (5.4.284)
W. Erbsen STATISTICAL MECHANICS
b) To nd the work done by a gas which expands isothermally (constant temperature), from some
initial volume V
i
to some nal volume V
f
, we must rst recall that
W =
_
Vf
Vi
P dV (5.4.285)
At this point, we solve (5.4.284) for P, which will subsequently be substituted into (5.4.285). But
rst things rst:
P =
Nk
B
T
V Nb

aN
2
V
2
(5.4.286)
And now substituting (5.4.286) back into (5.4.285),
W =
_
Vf
Vi
_
Nk
B
T
V Nb

aN
2
V
2
_
dV
=Nk
B
T
_
Vf
Vi
1
V Nb
dV aN
2
_
Vf
Vi
1
V
2
dV
=Nk
B
T [log [V Nb][
Vf
Vi
aN
2
_

1
V

Vf
Vi
=Nk
B
T [log [V
f
Nb] log [V
i
Nb]] + aN
2
_
1
V
f

1
V
i
_
(5.4.287)
We can simplify (5.4.287) slightly, leaving us with
W = Nk
B
T log
_
V
f
Nb
V
i
Nb
_
+aN
2
_
1
V
f

1
V
i
_
Problem 30
A quantum harmonic oscillator has energy levels E
n
=
_
n +
1
/
2
_

0
, where n = 0, 1, 2, ... Treat this single oscilla-
tor to be a small system coupled to a heat bath at temperature T. What is the probability of nding the oscillator
in its n
th
quantum state?
Solution
The Quantum Harmonic Oscillator (or simply QHO, as we lovingly call it) has the following eigenenergies:
E
n
=
_
n +
1
/
2
_
(5.4.288)
And if we dutifully recall, the partition function is given by
Z =

n=0
exp
_

E
n
k
B
T
_
(5.4.289)
CHAPTER 5: DEPARTMENTAL EXAMINATIONS 555
Now, we are interested in nding the QHO in some other state, lets call it n

. How do we nd this out?


Well, we rst recall that
P(E
n
) =
exp [E
n
/(k
B
T)]
Z
(5.4.290)
Now lets tailor it to our system in need:
P(E

n
) =
exp [E

n
/(k
B
T)]
Z
(5.4.291)
Now, we must nd Z:
Z =

n=0
exp
_

_
n +
1
/
2
_

0
k
B
T
_
(5.4.292)
Now, the n
th
excited state is n

, and (5.4.291) becomes


P(E

n
) =
exp
_

_
n

+
1
/
2
_

/(k
B
t)

n=0
exp [(n +
1
/
2
)
0
/(k
B
T)]
=
exp
_

_
n

+
1
/
2
_

n=0
exp [(n +
1
/
2
)
0
]
=exp
_

_
n

+
1
/
2
_

n=0
exp
__
n +
1
/
2
_

=exp
_
n

0

1
/
2

exp
_
1
/
2

n=0
exp [n
0
]
=exp
_
n

0

1
/
2

0
+
1
/
2

n=0
exp [n
0
]
=exp [n

0
]

n=0
exp [n
0
] (5.4.293)
At this point we recall the following arcane summation identity

n=0
e
an
=
1
1 e
a
Applying this to (5.4.293),
P(E

n
) =
exp [n

0
]
1 exp [
0
]
Problem 31
The surface temperature of the sun is T
0
(= 5800
o
K); its radius is R(= 7 10
8
m) while the radius of the
W. Erbsen STATISTICAL MECHANICS
Earth is r(= 6.37 10
6
m). The mean distance between the sun and Earth is L(= 1.5 10
13
m). In rst approx-
imation, one can assume that both the sun and the Earth absorb all electromagnetic radiation incident upon them.
Assume that Earth has reached a steady state at some temperature T.
a) Find an approximate expression for the temperature T of Earth in terms of the astronomical parameters
mentioned above.
b) Calculate the temperature T numerically.
Solution
a) The Stefan-Boltzmann Law gives the total power that the Sun is emitting, and is given by
P
S
= 4R
2
T
4
0
(5.4.294)
Since the sun emits radiation in all directions, the Earth only absorbs a small fraction of the total
output, and depends on the cross sectional area of the Earth and the distance it is from the emitter.
Quantitatively,
P
E
=P
S
A
4L
2
=P
S
r
2
4L
2
=P
S
r
2
4L
2
(5.4.295)
As suggested in the prompt, if we assume in the rst approximation that the Earth is a perfect
absorber, then (5.4.295) holds, and we can use (5.4.294) to come up with another expression which
describes the power absorbed by the Earth.
We start by slightly rewriting (5.4.294)
P
E
= 4r
2
T
4
(5.4.296)
At this point, we substitute both (5.4.294) and (5.4.296) into (5.4.295):
4r
2
T
4
= 4R
2
T
4
0

r
2
4L
2
T
4
= T
4
0
R
2

1
4L
2
(5.4.297)
From (5.4.297) it is easy to see that
T = T
0
_
R
2L
(5.4.298)
b) Substituting in the appropriate values into (5.4.298), we solve
T = 5800 K

7 10
8
m
2 (1.5 10
15
) m
T = 28.017 K
CHAPTER 5: DEPARTMENTAL EXAMINATIONS 557
Problem 32
A 1.00 gram drop of water is supercooled to 5.00
o
C (remains in liquid state). Then suddenly and irreversibly
it freezes and becomes solid ice at the temperature of the surrounding air, 5.00
o
C. The specic heat of ice is
2.220 J g
1
K, and of water is 4.136 J g
1
K, and the heat of fusion of water is 333 J g
1
.
a) How much heat leaves the drop as it freezes?
b) What is the change in entropy of the drop as it freezes?
Solution
m = 1.00 g
T
1
= 5.00
o
C
w
= 2.220 J g
1
K
T
2
= 5.00
o
C
i
= 4.186 J g
1
K
_
L
f
= 3.333 J g
1
a) Two equations that have been since long forgotten are
Q =mCT (5.4.299a)
Q =mL
f
(5.4.299b)
Where we recall that the heat, Q is the key quantity in all this. If a cup of ice is sitting in room
temperature, then heat is entering the vessel, causing it to melt. If, however you have the same cup
of ice cooled to exactly 32
o
F (remember, ice doesnt normally freeze lower than this), and it starts
to turn to water of the same temperature. The heat required to make this phase transition may
not contribute towards a change in T, however energy is being applied because it takes energy to
undergo a phase change.
In our case, the sample does not undergo any change in temperature, and the amount of heat
that leaves the drop is due only to the change of phase. Therefore, we only need to use (5.4.299b),
and applying the appropriate constants, yields
Q = (1.00 g)
_
333 J g
1
_
Q = 333 J (5.4.300)
b) A common form of entropy is
dS =
Q
T
(5.4.301)
And substituting in the appropriate values into (5.4.301), we have
S =
333 J
273 5 K
S = 1.243 J K
1

You might also like